Thanks to visit codestin.com
Credit goes to www.scribd.com

100% found this document useful (1 vote)
732 views470 pages

Lsat Logic Games

Copyright
© © All Rights Reserved
We take content rights seriously. If you suspect this is your content, claim it here.
Available Formats
Download as DOCX, PDF, TXT or read online on Scribd
100% found this document useful (1 vote)
732 views470 pages

Lsat Logic Games

Copyright
© © All Rights Reserved
We take content rights seriously. If you suspect this is your content, claim it here.
Available Formats
Download as DOCX, PDF, TXT or read online on Scribd
You are on page 1/ 470

logic

games
for the
lsat

NewYork
Copyright © 2011 LearningExpress, LLC.

All rights reserved under International and Pan American Copyright Conventions.
Published in the United States by LearningExpress, LLC, New York.

Library of Congress Cataloging-in-Publication Data


Logic Games for the LSAT.
p. cm.
ISBN-13: 978-1-57685-779-3 (pbk.)
ISBN-10: 1-57685-779-4 (pbk)
1. Law School Admission Test—Study guides. 2. Law schools—United States—Entrance examina- tions
—Study guides. 3. Logic—Problems, exercises, etc. I. LearningExpress (Organization)
KF285.Z9L645 2011
340.076—dc22
2011006976

Printed in the United States of

America 9 8 7 6 5 4 3 2 1

First Edition

ISBN 978-1-57685-779-3

For more information or to place an order, contact LearningExpress


at: 2 Rector Street
26th Floor
New York, NY 10006

Or visit us at:
www.learnatest.com
coNteNts

Introduction.....................................................................................................................................................v

Chapter 1: LSAT Logic, Game Types, and General Strategies........................................................1

Chapter 2: The LearningExpress Test Preparation System...........................................................21

Chapter 3: Sequencing Games Review............................................................................................39

Chapter 4: Selection Games Review.................................................................................................71

Chapter 5: Distribution Games Review.........................................................................................101

Chapter 6: Matching Games Review..............................................................................................129

Chapter 7: Hybrid Games Review.................................................................................................167

Chapter 8: Practice Test 1.................................................................................................................195

Chapter 9: Practice Test 2.................................................................................................................227

iii
iNtroductioN

The Law School Admission Test (LSAT) is the standardized test used by American Bar Association–
approved law schools in assessing and admitting law school applicants. If you’re planning to go to law
school in the United States, taking the LSAT is a must. The test includes five 35-minute multiple-choice
sections followed by a 35-minute writing sample. It’s a long test; including breaks, distribution and
collection of tests, and other test- day procedures, it lasts over four hours.
The LSAT is administered by the Law School Admission Council (LSAC), a nonprofit corporation
that assists law schools and applicants with nearly every aspect of the admissions process, from the
LSAT to let- ters of recommendation. (LSAC will be responsible for collecting your application
information if you sign up for their Law School Data Assembly Service.) LSAC usually administers the
test four times a year, in February, June, September or October, and December. For law school admission
in the fall, you’re often required to take the LSAT by the previous December, but applicants who are on
the ball often the take one of the earlier June or September/October tests—that is, a year or more
before they intend to enroll in law school.
The LSAT is administered at one of many test centers worldwide. Make sure to register early
(at www.lsac.org) to ensure that you get your choice of testing locations, as they have limited availability
and are filled on a first-come, first-served basis.

what’s on the lsat

According to LSAC, the purpose of the LSAT is to “measure skills that are considered essential for
success in law school: the reading and comprehension of complex texts with accuracy and insight; the
organization and management of information and the ability to draw reasonable inferences from it; the
ability to think critically; and the analysis and evaluation of the reasoning and arguments of others.”1

1 See “What the Test Measures” at www.lsac.org/JD/LSAT/about-the-LSAT.asp

v
–i NtroductioN –

Essentially, the LSAT tests reading comprehension, information management and analysis, and
argu- ment analysis. These areas correspond to the three sections on the LSAT. LSAC calls these sections
Reading Comprehension, Analytical Reasoning, and Logical Reasoning. Among test-takers and test prep
literature, the Analytical Reasoning section is known as the logic games section, and the Logical Reasoning
section is known as the arguments section.
The five multiple-choice sections are broken down as follows, but can appear on the actual test in
any order (except for the Writing Sample, which is always last):

■■ One Reading Comprehension section


■■ One Analytical Reasoning (i.e., logic games) section
■■ Two Logical Reasoning (i.e., arguments) sections

■■ One experimental section—an unscored section resembling one of these section-types for the sake of

future LSAT test creation (you won’t know which is the experimental section)
■■ Writing Sample

The sequence of sections and breaks at a typical LSAT administration might be as follows:

1. Analytical Reasoning (35 minutes)


2. Logical Reasoning (35 minutes)
3. Reading Comprehension (35 minutes)
4. Break (10–15 minutes)
5. Analytical Reasoning, experimental (35 minutes)
6. Logical Reasoning (35 minutes)
7. Break (5 minutes)
8. Writing Sample (35 minutes)

So what are these section types all about anyway?

Reading Comprehension
The Reading Comprehension section on the LSAT should be familiar to you already, as it is essentially a
spiced- up version of similar sections on the SAT and the ACT. You are given four passages on various
subjects in the natural sciences, social sciences, and humanities, each accompanied by five to eight
questions (for a total of 26 to 28 questions). These questions test your ability to quickly absorb essential
information from the passage, un- derstand and interpret the passage, and reason about the ideas presented.
You could be asked about the author’s attitude, details in the passage, implications of statements made,
main ideas, the function of various parts of the passage, or a host of other questions designed to test your
level of comprehension. The passages are gener- ally 55–60 lines—about what you’re used to on
standardized tests you took in high school—but they are more complex than those on the SAT and
ACT, drawn from sources ranging from Scientific American to the ABA Journal, and seemingly
chosen to make you sweat.

vi
–i NtroductioN –

Logical Reasoning
The Logical Reasoning section of the LSAT may not look like anything you’ve seen on other
standardized tests you have taken. Nonetheless, it tests your ability to carefully read and analyze arguments
—skills that the LSAC deems essential for success in law school. Beyond that, it is the only scored section
that appears twice on the LSAT. The sorts of ideas and skills tested in this section are truly the bread and
butter of legal education and practice: as LSAC puts it, the skills of “drawing well-supported conclusions,
reasoning by analogy, determining how additional evidence affects an argument, applying principles or
rules, and identifying argument flaws.”1
A Logical Reasoning section consists of 20–25 short passages, each followed by one or two
questions (for a total of 24 to 26 questions). The passages, if followed by one question, are usually three to
four sentences long, and comprise an argument of some sort—that is, a presentation of a viewpoint or
thesis along with supporting claims (although often not explicitly so). If followed by two questions, the
passages will usually consist of two short opposing arguments, often presented as an argument followed
by a rebuttal. The vast majority of ques- tions are based on a single argument, rather than the
argument-rebuttal format.
The questions test precisely the concepts LSAC says they will. They will include asking you to
provide miss- ing assumptions in the argument, identify logical flaws, identify statements that would
undermine or strengthen an argument, and sometimes even complete an argument (when a part has
been left out of a passage).

Analytical Reasoning (i.e., Logic Games)


The Analytical Reasoning section will be the least familiar section to the majority of test-takers. It
consists of four logic games, each accompanied by five to seven questions, for a total of 22 to 24
questions. Each game con- sists of a setup paragraph, which provide the general parameters of the game,
followed by a set of conditions. For example:

1. Over the course of seven days, exactly seven different Shakespeare plays will be presented in a
theater.
a. Othello must follow Romeo and Juliet
b. If Hamlet is shown on Wednesday, Macbeth won’t be shown on any subsequent day
c. Either The Taming of the Shrew or Hamlet must be shown on Wednesday
d. A Midsummer Night’s Dream will be shown on Saturday

Following this game setup, a series of questions are presented that test your ability to understand
the me- chanics of the game and work with new information presented. For example, one question might
ask: if The Taming of the Shrew is not shown at all, which play cannot be shown on Friday? A likely answer
will be Macbeth, since if The Taming of the Shrew is not shown, Hamlet must be shown on Wednesday,
and if Hamlet is shown on Wednesday, Macbeth can’t be shown on any later day.
Logic games are designed to present information concerning relationships among entities and then
test your ability to both manage the information and reason logically about those relationships. The
relationships can be spatial, temporal, or logical, and the entities can be persons, places, or things. The
example here concerns temporal relationships between things (e.g., that Othello follows Romeo and
Juliet).
When in the midst of trying to solve a particularly cumbersome or difficult logic game, LSAT test-
takers often find themselves wondering how on earth this could be relevant to law school. LSAC says
these highly
1 See “What the Test Measures” at www.lsac.org/JD/LSAT/about-the-LSAT.asp

vii
–i NtroductioN –

unusual mental exercises “reflect the kinds of complex analyses that a law student performs in the course of
legal problem solving.”1 Now, this may or may not be true—and many have difficulty seeing how it
could be—but in the end, that’s beside the point. LSAC thinks logic games are important, and until you
finish the last question on the LSAT, it’s in your interest to think they’re important, too!
Altogether, the three section types on the LSAT will call on your powers of rigorous analytical
reasoning. This book will help you to master the art of manipulating rules and conditions and to develop
expertise with the logic games section of the LSAT.

how the lsat is scored

Four of the five multiple-choice sections are scored, and they will have a considerable impact on your law
school application. The writing sample is not scored, but it is sent to law schools along with your
application.

The Raw Score


While you will end up answering between 120 and 130 questions (including those on the experimental
section) on any given LSAT administration, the four scored multiple-choice sections comprise only 100 to
101 questions. There is no penalty for wrong answers, and each question is weighted equally. Your raw score
is simply the num- ber of questions (of the 100 to 101 scored questions) you answer correctly.

The Scaled Score


Your raw score out of 100 or 101 is translated into a scaled score between 120 and 180. This occurs
through a statistical process called equating, which attempts to correct for minor differences in difficulty
between tests. Every LSAT question is pretested (partially through the experimental sections), and data
collected from these tests enable equating. The original test that the score is equated against was given in
June 1991, and the process effectively ensures that you are competing for your scaled score against a
much larger pool of LSAT takers from various test administrations. So don’t worry if all the geniuses you
know are taking the test on your test date, and the test seemed harder than your practice LSATs—that
won’t prevent you from getting your best score.
On the high end, a 180 doesn’t necessarily reflect a perfect raw score. You can miss up to three
questions and often still get a 180. On the low end, to get past a 120, you generally need to get 14 to
17 questions right. Beyond that, for every three raw points you get (every three questions answered
correctly), your scaled score will generally go up about two points—but that is a rough rule and varies
from test to test.

The Percentile Rank


The score report LSAC sends you will include your percentile rank, which indicates the percent of test-
takers over a recent three-year period with scaled scores lower than your scaled score. So if you got a
scaled score of 166 and your reported percentile rank is 93%, it means that 93% of all LSAT test-takers
over some specified period of time scored lower than 166. Each LSAT will have a table, slightly different
from test to test that correlates each

1 See “What the Test Measures” at www.lsac.org/JD/LSAT/about-the-LSAT.asp


viii
–i NtroductioN –

scaled score to a percentile rank; for example, a score of 180 put you in the 99.9th percentile, while a score
of 151 put you in the 50th percentile. To score better than 90% of test-takers, you need to get around a 164
or 165. In the extreme upper and lower ranges, differences in raw and scaled scores don’t translate to major
percen-
tile rank changes: someone who misses one question to get a 180 might be in the 99.9th percentile, while
some- one who misses 11 questions to get a 173 might be in the 99.0th percentile. But in the middle of the
field—for the majority of law school applicants—LSAT percentile ranks can be very sensitive to changes in
raw and scaled scores, and test-takers would be wise to keep this in mind as they look for motivation to
keep practicing. Say someone misses 40 questions to get a score of 153 and a rank in the 59th percentile.
Another person might miss 30 questions, and get a score of 159 and a rank in the 79th percentile. That is
a 20-point percentile difference!
The following table gives a sense of the approximate relationship between raw scores, scaled scores,
and percentile ranks. Please note that this is merely an example meant to illustrate how LSAT score
correlations generally work—actual score correlations will vary from test to test.

Raw SCoRe SCaled SCoRe PeRCentIle Rank


(how many questions you (your actual lsat (the percent of test-takers during
answered correctly) score, reported to law a recent three-year period with a
schools) lower scaled score)

101 180 99.9


90 173 99.0
80 166 94.0
70 159 79.0
60 153 59.0
50 147 36.0
40 141 17.0
30 134 5.5
20 125 0.5
0–15 120 0.0

The LSAT is constructed so that the majority of test-takers won’t finish every section and every
problem. Keep this in mind both as you consider this table and as you practice for the test. To maximize
your scaled score and percentile rank, you are better off ensuring that you correctly answer as many
questions as you can, rather than rushing through the test attempting to answer every question and
ultimately getting fewer correct answers. If you only get to 3/4 of the questions on the test, answer 4/5 of
those correctly, and randomly guess on the rest, you are looking at raw score of about 65, which might
translate to a scaled score of about 156 and percentile ranking of about 70. If you manage to correctly
answer all of those carefully analyzed questions, your decision to focus energies on that select set will
result in a raw score of about 80, which might translate to a scaled score of about 166 and percentile
rank of about 94!
ix
–i NtroductioN –

how the lsat is used

Virtually every law school uses the LSAT as a major factor in the assessment of their applicant pool.
According to validity studies conducted by LSAC, the LSAT is a statistically significant numerical predictor of
performance in law school (better than undergraduate GPA alone).1 The median statistical correlation
between LSAT scores and first-year performance is 0.33 (a value of 0 indicates a random relationship,
and a value of 1 indicates per- fect correlation). The idea is that the LSAT directly tests a large portion of
the skills necessary for success in the first year of law school and beyond. But that claim is largely
speculative, and is exceedingly difficult to validate. Students should realize that although the test is a
predictor (statistically speaking), it might not be a very strong one—and more importantly, that the
LSAC validity study results certainly don’t mean that the test will predict your performance in law
school, let alone your success in a legal career. LSAC itself wisely notes that the LSAT is not a perfect
predictor, especially because it varies from one school to another in its predictive ability.
The LSAT and undergraduate GPA together provide a slightly better predictor of first-year grades
than either one taken alone (with a median statistical correlation of 0.46). For this reason, and for the sake
of con- venience, many law schools combine the GPA and LSAT into one index score with which they
rank their ap- plicants. Law schools often devise their own formulas for calculating this index score,
based on the relative importance they attach to the LSAT score versus GPA. Applications are sorted for
further review based on this index. However, the same caveats apply to the index, so no deterministic
conclusions about the future perfor- mance of any individual in law school or the legal profession
should be made based on that measure!

how to use this Book

This book is designed to help you master the art and science of logic games. The Analytical Reasoning
(logic games) section of the LSAT is one of the most unusual standardized test sections out there, and
most test-takers will be unfamiliar and unpracticed with the skills it requires. Fortunately, there are a
limited number of prin- ciples behind both the mechanics of the games and the proper approach to the
games, which when studied and practiced, will make the Analytical Reasoning section just as
approachable and tractable as any other section. There are a limited number of game types, question
types, and specific strategies and skills tested—and once you learn them you will become a confident
logic gamer.
This book breaks down the LSAT logic games into five types—sequencing games, selection games,
distri- bution games, matching games, and hybrid games—with a chapter devoted to each. Each chapter
breaks down the essential components of that game type, and provides specific tips and strategies
for mastering it.
First, familiarize yourself with the LearningExpress Test Preparation System in Chapter 2. There, you
will find suggested schedules for working through the rest of the book. Make sure to read the general
strategies in Chapter 1. Beyond that, you will be advised that if you have the time, you should work
through each chapter se- quentially, carefully reading the tips and strategies at the beginning of the
chapter, working through each prac- tice game and the answer explanations to understand how the
strategies should be applied to each, and making sure to apply them to each new practice game as you
go along. If you are crunched for time, you will be advised
1 See www.lsac.org/jd/pdfs/LSAT-Score-Predictors-of-Performance.pdf

x
–i NtroductioN –

to start by taking one of the practice tests at the end of the book to assess which games you need the
most help with. Then go to those chapters, and work through the tips, strategies, and practice games.
Either way, make sure to take both practice tests, at least one of which you should do after working
through the strategy chapters, to see your progress!
When working through practice games, whether in the strategy chapters or on the practice tests,
make sure to mix timed and untimed trials. There are eight practice problems in each chapter, so
consider doing four games untimed, and then four games in a row with a 35-minute total time limit
(since you will have to do four games in 35 minutes on the actual LSAT). The untimed games will help
you to practice the thought processes and diagramming techniques recommended, and to carefully and
deliberately apply the tips and strategies you have learned. The timed games will help you practice
efficiency with those processes and learn to quickly and efficiently apply the proper techniques. Over
time, this approach will build speed, efficiency, and accuracy.
After every practice game or group of four games, make sure to review the detailed answer
explanations, even for answers you got right. Your main concern, especially in the initial practice games
for each game type, is to ensure that you are tackling the game and questions with the right thought
processes and approaches, and reasoning in a sound, logical manner. This comes from reviewing the answer
explanations, comparing your ap- proach to that presented in the explanation, and (if needed) determining
how you can improve your approach the next time around. Don’t get into a logic game rut: stay active
and engaged with the practice and explana- tions by reflecting on what your thought processes were when
working through a practice game, and constantly strive to amend those processes so that you won’t miss
similar question types, deductions, or connections in the future. The book is carefully designed to help
you master logic games, but your success in doing so depends on how much you put into it!

xi
1
lsat logic, game
chapt e r

types, and general


strategies

LSAT logic games are ultimately about logical reasoning. They require general logical reasoning, such as
real- izing that if A comes before B, then B cannot be first in a sequence and A cannot be last in a
sequence. This kind of logical reasoning is ubiquitous in logic games.
Logic games also require that we translate sentences into formal logical constructions using “logical
con- nectives” (in particular, “and,” “or,” “not,” and “if-then”), and that we manipulate the resulting
constructions. This kind of logical reasoning is more complex and may be unfamiliar to many
students.

know Your lsat formal logic

In the following discussion, we describe how statements are formed using logical connectives and how to
reason with such statements. Our building blocks are two “simple” statements, which we call statement
“A” and state- ment “B.” “A” and “B” can stand for any declarative statement, but on the LSAT, these
statements will often be bits of concrete information presented in the logic game setup, such as “Abigail
is in the morning section” or “Brian is in the morning section.”

1
lsat logic, game tYpes, aNd geNeral strategies

Conjunctions (“A and B”)


A conjunction uses the word “and.” It means that both statements are true. If you’re told that A and B are
true, then the conjunction is “A and B.”
An example would be “Abigail and Brian are both in the morning section.”

Disjunctions (“A or B”)


A disjunction uses the connective “or.” It means that either one or the other statement is true, or both are
true. The disjunction “A or B” means that either A or B is true, or both are true. An example is the
statement “Either Abigail or Brian is in the morning section.” Note that this disjunction is inclusive—
both A and B can be true. An exclusive disjunction is where only one statement, not both, can be
true.

Negations (“not A”)


You can also negate a single statement with the connective “not” (it’s called a connective even though it
operates on just one statement, rather than connecting two statements). This is very straightforward: if
you want to say that A is not true, you say “not A.” It can be symbolized with an equal sign with a
line through it: ≠ A
Note that if we negate the negation of statement A—that is, if we say “not A” is not true—we end
up with “not (not A),” which is the same things as just saying “A.” Think about it this way: if you say that
it is not true that A is not true, you are really saying that A is true. A double negative in English
grammar works the same way. If you say that you are not unhappy, that means you are happy.

Negating Conjunctions and Disjunctions (“not (A and B),” “not (A or B)”)


We have considered the connectives “and,” “or,” and “not.” Before turning to the other connectives, we must
take a look at how to apply connectives to one another—in particular, how to negate conjunctions and
disjunctions. First, keep in mind that what results from using a connective on any statement is itself a
new statement, which can be a part of yet another statement using a connective. So if we create the
conjunction “A and B,” this
new statement “A and B” can be negated to create “not (A and B).”
What does “not (A and B)” mean? It means that it’s not the case that both A is true and B is
true. In other words, one of the two statements (or both) must be false. Using the previous example, the
statement “not (A and B)” tells us that either Abigail is not in the morning section or Brian is not in
the morning section, or neither are in the morning section. Notice that you can also say “either A is
not the case or B is not the case.” This is simply the disjunction “not A or not B.”
In fact, these two statements are equivalent: not (A and B) = not A or not B.
Saying A and B can’t both be true (“not (A and B)”) is the same as saying at least one isn’t true
(“not A or not B”).
It is important to use parentheses to avoid confusion. For example, “not (A and B)” isn’t the same
state- ment as “not A and B.” The first one, “not (A and B),” says that A and B can’t both be true, so B
could be true, as long as A is not true. The second one, “not A and B,” says “A is not true” and “B is
true.” These two are clearly not equivalent statements.
So far we have seen how to negate a conjunction “A and B.” What about negating the disjunction “A
or B”? The disjunction “A or B” tells us that at least one statement is true. So “not (A or B)” means neither
statement is true. This is the same as both not A and not B are true (“not A and not B”).

2
lsat logic, game tYpes, aNd geNeral strategies

Therefore, these two statements are equivalent: not (A or B) = not A and not B.
Saying that it can’t be true and that at least one statement is true is the same as saying that
both A is not true and B is not true.

Conditionals (“if A, then B”)


The most important logical concept on the LSAT is the conditional statement, which takes the form “if A,
then B.” It is symbolized with an arrow: A S B.
This conditional means that if statement A is true, then statement B is true. Before we can
understand how to work with conditional statements, it’s worth pausing to figure out what this means
exactly in the logical sense, which is similar to, but slightly different from the everyday sense.

the logical Meaning of Conditionals


When people make “if A, then B” statements in everyday English speech, they usually have some sort of
causal connection between A and B in mind. A common example of an everyday English “if-then”
statement is “if it is raining, then I will bring my umbrella.” The idea is that a causal connection exists
between the first part of the statement and the second part—you bring your umbrella because it is raining.
However, this statement is false when it is raining but I do not bring my umbrella, because the causal
connection clearly does not hold (the rain did not cause me to bring my umbrella). Therefore, we do not
have a clear understanding of the circumstances that make the conditional statement true. For example,
is the conditional statement true when I bring my um- brella even though it is not raining? Further, this
example doesn’t address the confusion students often feel when they encounter if-then statements on the
LSAT that do not have a clear causal connection, like “if car A has a spoiler, then car B has performance
tires.” What does car A’s spoiler have to do with car B’s performance tires? How is that the same as me
bringing an umbrella because it is raining?
In short, in the logical meaning of conditionals, there is no causal connection between A and B. It
sim- ply means that when A is true, B is also true. Think of it as “if A is true, then B must be true.”
We don’t know why the truth of A guarantees the truth of B (why car A having a spoiler guarantees that
car B has performance tires), but from a logical perspective (which is the only perspective of the LSAT
logic games!), we don’t care. All we care about is that fact: when car A has a spoiler, car B must have
performance tires.
Further, the conditional statement must be true in all circumstances except when A is true and B is
false. So, a conditional statement is true when A is true and B is true, when A is false and B is true,
and when A is false and B is false.
This answers our question about what happens when it isn’t raining, but I bring my umbrella
anyway: What happens is that the statement is true. You might wonder how this could be.
Remember that there is no causal connection in the strictly logical “if-then” statement. Even still, you
might have issues with conditionals that are true whenever A is false but B is true—that’s ok, but for the
purpose of the LSAT, you need to accept it. Why does this even matter for the LSAT? Well, let’s take a
look at an example. Suppose that a logic game gives us the following set of restrictions:

If Brian is in the afternoon section, then Charles is in the evening section.


If Danielle is in the morning section, then Elizabeth is also in the morning section.

3
lsat logic, game tYpes, aNd geNeral strategies

Now suppose that in the course of trying to answer a question, we want to check whether the following
is ac- ceptable for Brian and Elizabeth, without even knowing where Charles and Danielle are
assigned.

Morning: Elizabeth
Afternoon:
Evening: Brian

We need to see whether this assignment violates either of our two restrictions. Given our understanding
of the logic of conditionals, we can quickly check the assignment against each of the conditional
statements—and as long as either the first part of the conditional statement is false or the second part of
the conditional statement is true (or both), we know that that conditional statement is true (that is, the
restriction is not violated).
For the first conditional statement, we see that the first part of the conditional (that Brian is in the
after- noon section) is false (because according to this assignment, Brian is in the evening section). So the
conditional statement as a whole must be true (regardless of whether Charles ends up in the
evening section or not).
If you’re still having trouble with the logic of conditionals, another way to think about this is that
since Brian is not in the afternoon section, the first restriction can’t be violated because the first restriction
“does not apply” to this scenario—it would only apply if Brian were in fact in the afternoon section
(which would then require Charles to be in the evening section).
For the second conditional statement, the second part (that that Elizabeth is in the morning section)
is true, so (again) the conditional statement as a whole must be true (regardless of whether Danielle ends
up in the morning section or not). Since Elizabeth is in the morning section, the second restriction can’t be
violated because the only way for it to be violated would be for the first part to be true (Danielle in the
morning section) and the second part false (Elizabeth not in the morning section). Since both restrictions
are true with this (par- tial) assignment of students to sections—that is, since neither restriction is violated—
this would be an acceptable assignment. The point here is that checking to see whether the conditional
restrictions are violated becomes a relatively straightforward matter once you understand the logic of
conditionals.

the Basic operation of Conditionals


To make sure we understand the basics of how conditionals work, let’s use another logic game example.
Suppose that a game gives us the following set of restrictions (each a conditional statement):

If book A is selected for the reading list, then book B is selected for the
reading list. If book B is selected for the reading list, then book C is selected
for the reading list. If book D is selected for the reading list, then book E is
selected for the reading list.

Now, we can symbolize these conditional statements in the following manner.

First, let A = A is selected


B = B is selected
C = C is
selected D = D
is selected E =
E is selected

4
lsat logic, game tYpes, aNd geNeral strategies

Then, we can rewrite the conditional statements as:

If A, then
B If B,
then C If
D, then E

Or we can rewrite them as:

A S
BBS
C D
SE

Suppose we have a question which begins, “if A is selected for the reading list…”. What deductions can
be made from this information?
Well, we know that “A S B”—if A is true, then B must be true—and we know that A is true (A is
selected), so we can deduce that B is true as well (B is selected). And we know that “B S C”—if B is
true, then C must be true—and we know that B is true, so we can deduce that C is true as well. This is
the most basic way in which conditional statements operate.
Now, suppose we are considering a reading list and trying to figure out whether it is acceptable
given the set of restrictions for the game. Suppose A, B, C, and D are selected for the list, but E is not
selected. To figure out whether that reading list is acceptable, we need to see whether it violates any of
the three conditional state- ments (the three restrictions).
The first statement (A S B) is true (not violated) because B is true (B is selected). The second
statement (B S C) is true (not violated) because C is true (C is selected). But the third statement (D
S E) is not true (the restriction is violated) because while D is true (D is selected), E is not true (E is not
selected). That is, this read- ing list, which includes only A, B, C, and D, makes the conditional
statement “D S E” false.

Common mistakes with Conditionals


Finally, an important word of caution concerning two common mistakes students make with
conditionals. Suppose we have a question which begins, “if book B is selected for the reading list…”.
What deductions can be made from this information? Many students are tempted to try to apply the
conditional statement “A S B” to conclude that since B is selected, A must have been selected. This is
a mistake!
The conditional statement “A S B” means that if A is true, B must be true. But that does not
mean that if B is true, then A must be true! It might be that B is true even though A is not true—we just
don’t know. A likely reason that students sometimes make this mistake is that they rely on the causal
sense of the conditional, like when the rain causes me to bring my umbrella. However, “if I bring my
umbrella, then it must be raining” is flawed logical reasoning. The only information conveyed by the
statement “if A, then B” is that if A is true, then B must be true. To reiterate, knowing that book B is
selected for the reading list doesn’t give us any information about whether book A is selected, despite
the fact that we know that “if A is selected, then B is selected.”
Suppose we have a question that begins, “If book A is not selected for the reading list...”. What
deductions can be made from this information? Many students are tempted to conclude that B is also not
selected. This is also a mistake!

5
lsat logic, game tYpes, aNd geNeral strategies

Again, the conditional statement that “if A, then B” does not mean that B is true only when A is
true! In other words, knowing that book A is not selected for the reading list doesn’t give us any
information about book B. To make sure that you avoid these common mistakes, keep in mind that “if A,
then B” does not mean “if
B, then A,” and does not mean “if not A, then not B.”

Conditionals as Disjunctions
Recall that we said that a conditional statement (such as “if A, then B”) is true when either A is false or B
is true, or both are true. Notice that we can write this as a disjunction: either not A or B. In fact, these two
statements are equivalent:

If A, then B = not A or B

Or, using the arrow symbol: A S B = not A or B

It is especially useful to keep in mind that conditionals can be written as disjunctions because
occasionally the LSAT will give you a restriction that looks like this:

If book A is not selected for the reading list, book B is selected.

We can translate this as: (≠ A) S B

But we know that this is equivalent to the disjunction: ≠ (≠ A) or B

And we know that “not (not A)” is the same thing as A, so we end up with “A or B.”

So the conditional “(≠ A) S B” is really the disguised disjunction “A or B,” and that’s sometimes
help- ful to know in a logic game.

The Wording of Conditional Statements


The wording of conditional statements is usually fairly straightforward. However, sometimes “if A, then
B” is worded “B, if A.” Using the reading list example, the conditional could be worded “book B is
selected for the reading list if book A is selected for the reading list.”
Sometimes you might encounter the phrase “only if.” If you see a statement “A only if B,” just
rewrite this to as “If A, then B.” To understand this, think of them as both saying that if A is true,
then B must be true (the only circumstances in which A is true are those in which B is also true). For
example, the condition “book A is selected only if book B is selected” could be reworded “If book A
is selected, then book B is selected.”
You might also sometimes encounter the word “unless.” This has a special meaning in a strictly
logical sense. “Unless” means “if not.” So if you see the statement “A unless B,” just rewrite this as “A if
not B,” or “if not B, then A,” or “(≠ B) S A).”
For example, the condition “book A is selected unless book B is selected” can be reworded as “book
A is selected if book B is not selected” or “if book B is not selected, then book A is selected.”
Note that this does not mean that if book B is selected, book A is not selected! This is often
confusing for
6
lsat logic, game tYpes, aNd geNeral strategies

students who might think that the wording of “A unless B” intuitively means that A is true except when
B is true, in which case A is false. It’s an interesting interpretation, but you need to forget it. The word
“unless” has a specific logical meaning (“if not”), which you just need to accept for the LSAT. You can
blame logicians for giving “unless” a less-than-intuitive logical meaning.

Contrapositives
Every conditional statement has an equivalent “contrapositive” form. The conditional (if-then) statement
and its contrapositive say exactly the same thing, only in different words.
The contrapositive is: If not B, then not A.
Pause here, and memorize this. The procedure for producing the contrapositive form of any
conditional statement is to flip the order of the two parts of the statement and negate each part: from
“A S B” to “≠ B S ≠ A.” How does the contrapositive follow from the original conditional? Suppose B is
not true. Well then, A couldn’t be true because according the original statement, if A were in fact true, B
would have to be true. It might help to make this a bit more concrete. Suppose I have the conditional
statement: “if book A is selected for the reading list, then book B is also selected for the reading list.”
Now, suppose I know that book B is not selected for the reading list. Then I know that book A couldn’t
possibly have been selected for the reading list, since if it were, book B would also have to be selected. In
other words, the following conditional is true: “if book B is not
selected for the reading list, then book A is not selected for the reading list.”
The contrapositive is equivalent to the original conditional, regardless of any implied causal
connection: “if A, then B” and “if not B, then not A” is the same thing, even if A and B have nothing to
do with one another. Basically, any time you see a statement of the form “if A, then B” (in symbols, “A S
B”), you should im- mediately write down the equivalent (but seemingly different) piece of information
that “if not B, then not A” (in symbols, “≠ B S ≠ A”). As we will see, this contrapositive form of the
original statement will often allow
for connections and deductions that you otherwise might have missed.
Because contrapositives are so important to the LSAT, let’s take a look at some more complex examples
where we use the rules for negating disjunctions and conjunctions.

conditional: contrapositive:
If A, then B.example of conditional:
corresponding contrapositive:
If not B, then not A.
comments: If book A is selected, then book B is selected.
If book B is not selected, then book A is not selected. This is the simple, basic form of the contrapositive. Apply this genera
complex constructions.

conditional: If A, then (B and C).


contrapositive: If not (B and C), then not A.
equivalent contrapositive: If not B or not C, then not A.
example of conditional: If book A is selected, then books B and C are
selected.

7
lsat logic, game tYpes, aNd geNeral strategies

corresponding contrapositive: If either book B is not selected or book C is not se-


lected, then book A will not be selected.
comments: We have negated a conjunction (“B and C”) in pro-
ducing this contrapositive.

: contrapositive: equivalent contrapositive: example of conditional:


If A, then (B or C).
If not (B or C), then not A.
ing contrapositive: If not B and not C, then not A.
If book A is selected, then either book B is selected or book C is selected.
If neither book B nor book C is selected, then book A is not selected.
We have negated a disjunction (“B or C”) in produc-
ing this contrapositive.

conditional: If A and B, then C.


contrapositive: If not C, then not (A and B).
equivalent contrapositive: If not C, then not A or not B.
example of conditional: If book A and book B are both selected for the read-
ing list, then book C will also be selected.
corresponding contrapositive: If book C is not selected for the reading list, then
either book A will not be selected or book B will not
be selected.
comments: We have negated a conjunction (“A and B”) in pro-
ducing this contrapositive.

: contrapositive: equivalent contrapositive: example ofIf conditional:


A or B, then C.
If not C, then not (A or B).
ing contrapositive: If not C, then not A and not B.
If either book A or book B is selected, then book C is selected.
If book C is not selected, then book A is not selected and book B is not selected.
We have negated a disjunction (“A or B”) in produc-
ing this contrapositive.

conditional: If not A, then B.


contrapositive: If not B, then A.
example of conditional: If book A is not selected, then book B is selected.
corresponding contrapositive: If book B is not selected, then book A is selected.

8
lsat logic, game tYpes, aNd geNeral strategies

comments: We have negated a negation in producing this con-


trapositive; we negated “not A” to produce “A.” Also,
note that if we use the disjunctive form of this
condi- tional, we get “either not (not A) or B,” which
is the same as “either A or B.”

conditional: contrapositive: example of conditional:


If A, then not B. If B, then not A.
corresponding contrapositive: If book A is selected, then book B is not selected. If book B is selected, then book A is not sele
comments: We have negated a negation in producing this con-
trapositive; we negated “not B” to produce “B.”

conditional: If not B, then not A.


contrapositive: If A, then B.
example of conditional: If book B is not selected, then book A is not selected.
corresponding contrapositive: If book A is selected, then book B is selected.
comments: We have negated two negations in producing this
contrapositive; we negated “not A” to produce “A”
and negated “not B” to produce “B.” Note that this
example just shows the equivalence of the basic
conditional and its contrapositive—that is, that “if
A, then B” is the same thing as “if not B, then not A.”

These are just a few examples of conditionals and their contrapositives, and they are meant to illustrate
how to negate the two parts of a conditional statement and flip them to produce the contrapositive. Keep
in mind that you can produce a contrapositive for any conditional, no matter the form given to you in an
LSAT game. For example, suppose you are given a conditional of the form: “If A, then B and not C.”

Negate the second part to produce: B and not C = not B or not (not C) = not B or C
And negate the first part to produce: not A
Now switch the first and second parts to produce the contrapositive: If not B or C, then not A
In other words, if either B is not true or C is true, then A is not true.

Biconditionals
A biconditional statement can be understood as a conjunction of two conditional statements. The
statement “if A then B, and if B then A” is a biconditional statement. If the LSAT uses the phrase “if
and only if,” then it is giving you a biconditional statement. For example, if you are told “book A is
selected if and only if book B is selected,” you are being given a biconditional, which is the equivalent of
“If book A is selected, then book B is selected, and if book B is selected, then book A is selected.” You can
see this by breaking down the statement “A if and only if B” into two parts:

9
lsat logic, game tYpes, aNd geNeral strategies

A if B and A only if B

Which is equivalent to:

If B, then A and If A, then B

One thing to note about a biconditional statement such as “A if and only if B” is that it is true in exactly
two cir- cumstances: when statements A and B are both true, and when statements A and B are both
false. If one of the two statements is true and the other is false, then at least one of the two conditional
statements (If A, then B or If B, then A) will be false, making the whole biconditional statement false.
Also note that you can symbolize a biconditional statement with a double-arrow: A 4 B.

know Your logic game types

Before you can understand strategies for engaging your adversary, it helps to know your adversary. In this
section, we briefly describe the five types of logic games: sequencing, distribution, selection, matching, and
hybrid games.

Sequencing Games
The sequencing game is a staple of the Analytical Reasoning section. In a sequencing game, you are
given a set of entities that must be put in a sequential order. The entities can be persons, places, or things,
and the order can be on a spatial, temporal, or some other metric level. For example, it may be a list of
cities (the entities) that are to be visited on a trip (temporal order). Or it may be a set of friends (the
entities) who are to be seated around a table (spatial order). Or it may be a list of television shows (the
entities) which are to be ranked according to popularity (a nonspatial/nontemporal metric).
A typical sequencing game will present a set of conditions that will determine what the sequence
must, can, and can’t look like. For example, concerning a list of cities to be visited on a trip, the game
might specify that Boston must be visited after New York, that Chicago must be the second stop, and that
if New York is vis- ited after Chicago, Seattle can’t be the last city visited.

Distribution Games
In a distribution game, you are given a group of entities that are to be distributed into two or more
subgroups. The entities can be persons, places, or things, and the division into subgroups can be based
on just about any characteristic of those entities. For example, it may be a set of eight movies that are to
be classified into drama, comedy, and thriller genres. Or it may be a set of ten books that are to be placed
onto three different shelves. Or it may be a group of college students who are to be split into three
sections.
A typical distribution game will present a set of conditions that will determine how the distribution
must, can, or can’t occur. For example, concerning college students who are to be split into sections, the
game might specify that Anne is only available for the first and third section times, that Barbara must be
in the second sec- tion, and that Carlos and Anne cannot be in the same section.
It’s helpful to note that distribution games are sometimes called “grouping” games. Some
distribution games can be treated as selection games. This is covered in depth in Chapter 5.

10
lsat logic, game tYpes, aNd geNeral strategies

Selection Games
In a selection game, you are given a group of entities from which some are to be selected. The entities can
be persons, places, or things, and the selection can be based on just about any characteristic of those
entities. For example, it may be a group of astronauts to be selected for a space flight. Or it may be a set of
novels to be selected for a reading list. Or it may be a list of philosophy lectures that a student must
choose to attend.
A typical selection game will present a set of conditions that will determine how the selection must,
can, or can’t occur. For example, concerning choosing from a list of lectures, the game might specify that
the student must attend the Aristotle lecture; that if the Kant lecture is attended, the Mill lecture will not
be attended; and that the Hume lecture and Mill lecture meet at the same time.
The selection game can be thought of as a special case of the distribution game: you are given a
group of entities that are to be distributed into two subgroups; the “in” subgroup and the “out” subgroup. For
this reason, many of the strategies specific to distribution games will also apply to selection games and,
keep in mind, some distribution games can in fact be treated as selection games.

Matching Games
In a matching game, you are given a group of entities that are to be matched with another group of
entities or with some set of characteristics. The entities can be persons, places, or things, and the
characteristics can be anything from shape to color to location. For example, it may be pilots (one group
of entities) who must fly (another group of entities). Or it may be supermarket clerks (one group of
entities) who must operate checkout aisles (another group of entities). Or it may be a league of soccer
teams (a group of entities) that must be assigned a different jersey color (a set of characteristics).
A typical matching game will present a set of conditions that will determine how the matching
must, can, or can’t occur. For example, concerning the assignment of pilots to routes, the game might
specify that Diane cannot fly to Sydney, Gerald will fly either to New York or Boston, and if Francis takes
the flight to New York, Edgar will fly to Tokyo.

Hybrid Games
The Analytical Reasoning section will often present logic games that combine two or more of the four
game types just discussed: sequencing, distribution, selection, and matching. These games are generally more
complex than games that represent just one type. A hybrid game might, for example, present a group of
people who are to attend a baseball game. They are to sit in two rows and are to sit in a particular
sequence of numbered seats in each row. This game combines a distribution element (of the attendees
into one of two rows) and a sequenc- ing element (of attendees into a sequence within each row). The
types of conditions will be similar to those in simple game types, but may involve two condition types
in a single condition; for example, if Ingrid sits in the front row, Jackie must sit adjacent to Kevin
(combining a distribution condition and a sequencing condition).
Keep in mind, even though we go into detail with these explanations, it’s not so important to know
the names of the game types. The most important thing is to be confident in how you approach the
games.
11
lsat logic, game tYpes, aNd geNeral strategies

know Your logic game strategies

The general strategies for approaching logic games can be ordered into a five-step technique:

1. Order Your Battles


2. Distill/Collate the Information and Rules
3. Digest the Game
4. Order and Answer the Questions
5. Keep Your Head

1) Order Your Battles


All questions are equally weighted, and the four games in an Analytical Reasoning section do not come
in any strict order of difficulty, so you should consider “ordering your battles” before jumping in. Take a
brief look at each game and rank them by how approachable each looks. Which game looks familiar
based on your practice? Which game or game type are you best at, based on your experience with
practice games?
Hybrid games are often more complex than single question type games. Games that don’t have one-
to-one mapping of entities to entities, sequence slots, or characteristics are usually more difficult. For
example, a game in which five dishes will be served over seven days—such that some dishes will have to
make a second or third appearance and some dishes may not be served at all—will be more difficult
than a game in which seven dishes appear exactly once over the course of seven days.
Look at the list of conditions as well. A lack of restrictive and concrete conditions will be relatively
more difficult. Also keep an eye out for the type of questions asked in a game—questions that make additional
supposi- tions (e.g., if Diane operates aisle three, who cannot operate aisle five?) are generally easier and more
straightfor- ward than other questions. These are just some of the general characteristics that might affect
a game’s difficulty level, but as you go through practice games, you should gain a better sense of what
makes a logic game easier or harder for you.
Once you get a better sense of the things that make a game more or less attractive, you’ll have to
tinker with your ordering strategy and develop your sense for ordering through practice.
Based on your assessment of difficulty, you may wish to do the easiest game first to relieve stress
during the section and boost your confidence, and leave the hardest for last—by then, you will have
gotten the logic juices flowing, and if you don’t get to it, you will have maximized your raw points by not
sinking your time into the most difficult game.
However, how you order the games may depend on whether you generally finish all four logic
games or not. For example, completing an Analytical Reasoning section in time then saving the hardest
game for last may not be the best strategy, since you will be fatigued by that point. If, on the other hand,
you usually complete only three of the four games, then doing the three easiest first is probably the right
strategy. You will have to experi- ment with ordering your battles to figure out what the best strategy
is for you.
Whatever your strategy, be flexible. It might turn out that in some instances, you may not want to
skim all four games in a section and order them before turning to any of the questions. For example, you
might read the first game, see that it has only a few supposition questions and is your weakest game
type, and skip to the second game. You might then see that the second game has only a moderate
number of supposition questions,

12
lsat logic, game tYpes, aNd geNeral strategies

is your best game type, is structurally similar to a game you’ve practiced, and contains a lot of highly
restrictive or concrete clues. At this point, you might decide to jump in here while the game prompt is
fresh in your head. You would answer the questions for the second game and finally return to your
difficulty-assessment phase for the next two games.
Just make sure you don’t spend too much time ordering your battles. If after a lot of practice, you
still have trouble quickly assessing the games, then consider picking an order ahead of time and
sticking to it.

2) Distill/Collate the Information and Rules


According to LSAC, logic games are designed to “measure the ability to understand a structure of
relationships and to draw logical conclusions about that structure You are asked to reason deductively
from a set of state-
ments and rules or principles that describe relationships among persons, things, or events.”1 Logic games
are essentially about logical relationships, and logical relationships are generally more easily understood and
ma- nipulated in a symbolic, visual form, rather than in a verbal form (either written or in your head). So,
to mini- mize the time spent referring to the mess of words presented in game setup, you need to gather
your information symbolically and visually in one place. If you properly symbolize and diagram the
information you’re reading, the game will become understandable and amenable to manipulation. Its
once-hidden mechanics will become transparent, and you’ll start playing the game rather than letting
the game play you.
There are three elements here: the symbolic, the visual, and being in one place.

Symbolization
You want to distill the information in the setup using shorthand and logical symbols. Let’s take an
example mini-game, matching pilots to flights.
Let’s suppose the setup tells you that there are five pilots: Diane, Gerald, Francis, Edgar, and Chris.
And suppose there are five flights: to New York, Boston, Sydney, Tokyo, and London. The first thing to
do is write out a list of shorthand symbols for the entities involved. You might separate the two entity
types by using low- ercase versus uppercase symbols: c, d, e, f, g for the pilots and N, B, S, T, L for
the flights.
Next, turn to the conditions. One condition is that “if Francis takes the flight to New York, Edgar
will pilot the flight to Tokyo.” The verbal form of this condition is cumbersome, so you might distill
this as f = N S e = T, where the arrow sign indicates an if-then statement, and the equal sign indicates
the piloting of a flight (or in general, the matching of entities from two groups—in this case, pilots
and flights).
This is just one example of symbolizing a rule based on a conditional statement. We will encounter
many different kinds of rules in this book, and ways to symbolize them. The conditional rule deserves
special atten- tion, however, because whenever you symbolize a conditional statement, you should
always symbolize its con- trapositive. As we will see below, this will allow you to make deductions you might
have otherwise missed. With this example, the symbolization of the contrapositive would be:

e≠TS f≠N

Throughout this book, we present suggestions for shorthand symbolizations, but you can develop your
own shorthand as you go along. Just remember two very important things: 1) make sure shorthand is
unambiguous,
1 See “What the Test Measures” at http://www.lsac.org/JD/LSAT/about-the-LSAT.asp

13
lsat logic, game tYpes, aNd geNeral strategies

so that you are not making mistakes on account of bad shorthand, and 2) keep it consistent, so you aren’t
get- ting confused on test day.
Also, make sure to take each symbolized piece of information and check it against the original to
ensure that you aren’t misreading the symbolization and that you haven’t overlooked any
information.

Visualization (diagramming)
Logic games almost always require a diagram. In subsequent chapters, we will see the kinds of diagrams
that are best suited for each game. In general, sequencing games require slots in sequential order,
matching games require a grid, and distribution and selection games require columns, lists of entities, or
slots. As you practice, you will become better at figuring out what kind of diagram to use. This is
probably the most crucial element of approaching any logic game, because the diagram is meant to visually
capture and clarify the basic underlying mechanism of the game, giving you a crucial crutch for otherwise very
difficult mental calculations. So, pay close attention to both the use of diagrams in this book and what sorts
of diagrams work best for you as you practice. Whatever you do, as with the symbolization, be careful! A
slight misreading or misrepresentation of in-
formation can have disastrous results for your gaming ability.
Suppose you have a game setup that says “one of seven possible movies is shown each day of a cable
channel’s movie week, from Wednesday to Tuesday.” You might immediately diagram a series of seven slots in
sequential order, and congratulate yourself for noting that the setup didn’t say that movies can’t be
shown twice (you won’t make the mistake of assuming each movie is show exactly once!). But as you’re
diagramming, you proceed to label the slots Monday through Sunday, from left to right, forgetting that
this movie week starts on Wednesday and ends on Tuesday. But one of the conditions is that The Matrix
will be shown sometime before Terminator, and Terminator will be shown on Thursday, which means The
Matrix must be shown on Wednesday. Missing this crucial piece of information, you get stuck on every
question you try, since they all assume you realize that no matter what, The Matrix is shown on
Wednesday. As you review the symbolized conditions you’ve collected next to the diagram, you don’t
realize that the root of the problem is in the diagram itself! So, take the time you need to collect and
represent carefully the information given to you.
Once you have the diagram in place, start filling it in, both in this Distill/Collate the Information and
Rules phase, and in the Digest the Game phase (following). As you encounter or deduce concrete bits of
information (e.g., that Diane does not pilot the flight to Sydney), write them into your diagram to the
extent possible (e.g., cross out the box representing the intersection of Diane and Sydney on your
matching grid.) Even less concrete bits of information and conditions can sometimes be drawn into the
diagram; for example, the information provided by the condition that Gerald will either pilot the flight
to New York or Boston can be entered into the diagram by crossing out every box representing the
intersection of Gerald with various cities, except for the in- tersections with New York and Boston.
As we consider each game type in turn, we will see further ways of incorporating rules into the
diagrams specific to those games.

all in one Place


Make sure that the symbolic forms of the conditions and setup information, the list of entity and
characteristic symbols, and the diagram all appear in one place on your scratch paper. This will be your
central place of ref- erence as you work through the questions, and you would be surprised by what
jumps out at you when you see
14
lsat logic, game tYpes, aNd geNeral strategies

pieces of information encoded next to one another. Also, make sure to write down any bit of information
that isn’t blatantly obvious so that you don’t have to rely on memory—it’s easy to forget details from the
game setup when stressed and under time constraints.
The techniques for distilling and collating the information and rules are highly recommended. But
you should use some practice games and tests to tinker and try out different shorthand and
diagramming tech- niques, then settle on your own brand of symbolization and visualization.
Afterwards, practice that technique so it becomes like a second language to you.

3) Digest the Game


On some tests, it makes sense to jump straight to the questions; for example, on some Reading
Comprehension sections, savvy test-takers will read the questions first, so they can look for specific
things as they read the pas- sage. This is obviously not the case with the Analytical Reasoning section of
the LSAT—we have discussed the necessity of symbolizing and diagramming the game setup and
conditions, but that’s just the start. It’s usually worth investing a bit more time in understanding the
mechanics of the game before engaging the questions. Taking the time to digest the game will pay off
once you start on the questions.
How do you do this? First and foremost, you need to make deductions or inferences from the
information and conditions presented to you. Look for links between stated game conditions, which
usually means looking for entities involved in two or more conditions. Sometimes, the best you can do is
just keep in mind the fact that there is some sort of immediate link between two conditions. But you will
often be able to generate new condi- tions or inferences from what’s presented. There are a number of
ways to make deductions in logic games, but for the sake of illustration, here’s a simple example,
based on chains of if-then statements.

If-then Statement Chains


Suppose the game conditions for the game matching pilots to flights (discussed previously) include the following:

Diane does not fly to Sydney.


Gerald will fly either to New York or Boston.
If Francis flies to New York, Edgar will fly to
Tokyo. If Edgar flies to Tokyo, Chris does not fly
to London.

Now look at the last two conditions. They are conditional statements (if-then statements), with an
element in common: Edgar flying to Tokyo. So you can make a new deduction that “f Francis flies to
New York, Chris does not fly to London” (since Edgar must fly to Tokyo).
You might symbolize the last two given conditions as:

f=NS e=T
e=TSc≠L

And then you could symbolize the new deduction as:

f=NS c≠L
15
lsat logic, game tYpes, aNd geNeral strategies

Or, if you feel comfortable doing so, you might cross out the two given conditions and just write in
a chain of symbolized if-then statements to concisely capture all the information given and deduced here
in one line:

f = N S e = T S not c = L

Sometimes it’s possible to make a number of different if-then chains. In these cases, it often makes
more sense to just note the possible connections between the if-then statements and write out the chains
as needed when you get to the questions, rather than write out all the possible chains ahead of time.

Considering Contrapositives
Further, make sure that you always consider the contrapositive of any if-then statement. The
contrapositive form of the original statement might allow for connections and deductions that you
otherwise might not have seen. Let’s look at an example.
Suppose that an additional condition to those presented previously for the game matching pilots to
flights is that “if Edgar does not fly to Tokyo, then Francis does not fly to Sydney.”
As stated, this condition does not seem to yield any new deductions. But now consider the
contrapositive: If it’s not the case that Francis does not pilot the flight to Sydney, then it’s not the case that
Edgar does not pilot the flight to Tokyo. The double negatives are a bit confusing, so restate the
contrapositive without them: If Francis flies to Sydney, Edgar flies to Tokyo. This is much simpler, and now
we have another inference chain that can be deduced: If Francis flies to Sydney, then Chris does not fly to
London (since Edgar must fly to Tokyo).
In symbols:

f=SS c≠L

Or, the full chain:

f=SS e=TS c≠ L

Let’s consider a second example in which the contrapositive would prove useful. Suppose that we encounter
the following question: If Chris flies to London, then each of the following statements could be true
EXCEPT:
We start by looking for any conditional statements that start with Chris flying to London, to see if
we can make any deductions. Unfortunately, there aren’t any and so it seems we are stuck: although
Chris and London appear in the last conditional, they appear in the form of Chris not flying to London.
But if we had written down the contrapositive forms, we would have noticed that the contrapositive of
the last conditional statement is that if Chris flies to London, then Edgar does not fly to Tokyo.
Bingo. We can now deduce that when Chris flies to London (according to the supposition of this
question), Edgar does not fly to Tokyo. In fact, we can deduce even more. The contrapositive of the third
conditional is if Edgar does not fly to Tokyo, then Francis does not fly to New York.
So we have yet another inference that could be made—since Edgar does not fly to Tokyo, Francis
does not fly to New York.
These are concrete bits of information that can provide answers. If one of the choices is “Francis
flies to New York,” we have found our answer. But without the contrapositive forms, we would be
lost.
16
lsat logic, game tYpes, aNd geNeral strategies

So always consider the contrapositive forms of the given conditional rules when digesting a
game!

General Connections
There are hosts of other ways to generate new conditions or inferences from what’s presented that don’t
just rely on if-then statement chains or contrapositives. When looking for links between conditions of the
pilots/flights game, you might notice that the second and third conditions have an element in common—
New York. Once you notice this, you might realize that if Francis takes the flight to New York, not only
will Edgar fly to Tokyo (as stated), but also Gerald will fly to Boston (since he must fly either to New
York or Boston, and New York is taken). You might symbolize this realization as:

f=NS g=B

There are many other kinds of deductions specific to different game types that LSAT authors often
require of the test taker. We will discuss these in subsequent chapters so you can be on the lookout for
those deductions as you tackle any logic game.

Using Scenarios
One important strategy for digesting a game is to see if you can sketch out the various possibilities for the
game. But only do this if you can quickly come up with two or three scenarios that exhaust the
possibilities inherent in the game, and that provide a decent amount of concrete information (or at least a
lot of information that dif- fers between the two scenarios).
Let’s consider a mini-logic game to illustrate (we will learn about selection games and how to
diagram them in Chapter 4, but for now we’ll keep it simple enough to understand the general
mechanics of creating scenarios). Suppose we need to select four items from a group of six items—A, B,
C, D, E, and F—and we are given the following rules:

If A is not selected, B is selected.


If A is selected, both C and D are
selected. If E is not selected, B is not
selected.

You might notice that whether A is selected makes a big difference to the how the selection goes,
and that we can divide the game into two scenarios: one where A is selected, and in another where
A is not:

Scenario 1
A

Scenario 2
A

In Scenario 1, since A is selected, C and D must be selected. In Scenario 2, since A is not selected, B
must be selected, and since B is selected, E must be selected. All of sudden, we have an information-rich
set of sce- narios that are exhaustive.
17
lsat logic, game tYpes, aNd geNeral strategies

Scenario 1
ACD

Scenario 2
AB E

This kind of scenario list might be useful for answering the questions in the game. And notice that the list
com- pletely captures the information in the first two rules, so that as you tackle the questions, you can
just pay at- tention to the scenarios and the third rule. You could make further deductions that fully
incorporate the third rule by realizing that B cannot be selected in Scenario 1, since if it were, E would
have to be selected, leading to five entities selected.
If you can quickly create an exhaustive list of two to three scenarios, you will have a clear, visual
repre- sentation of how the logic game works and what the possibilities there are. You will not have to
do much more diagramming or deduction-making since you can simply check question stems or answer
choices against your scenarios. In general, look for rules that severely restrict or drive the mechanics of a
game, and that allow for at least two possible scenarios. And look for disjunctions (such as “either A is
selected or B is selected,” or “X is ei- ther first or second”) that set off a chain of deductions (e.g., if we
also have the rule that “if A is selected, then C and D are selected,” or if we also have the rule that “if X is
second, then Y and Z are the last two.”) When faced with a logic game, knowing when to create scenarios
and when to avoid them is ultimately a judgment call. It is largely a matter of practice.

knowing when to Move on


Your objective in the digesting phase of any logic game is to gain a solid understanding of its particular
mechanics.
But the warning that applies in creating scenarios also applies to the digestion phase in general.
Investing time upfront is valuable, but only to a certain degree. If you can make deductions quickly, it
might pay off to do them up front; but if they generally take a bit longer for you, cut off the initial
deduction process earlier and just make deductions as the questions require (besides, tackling the questions
themselves will help you understand the game as well, especially “Test-the-Rules” questions, discussed
below).
As you practice, you will both gain efficiency with initial deductions/diagramming and gain a better
sense of when you are starting to waste time on this phase and need to move on to the questions.

4) Order and Answer the Questions


Now that you’ve wrestled with the mechanics of the game, it’s finally time to turn to the questions. The
time you’ve invested should pay off now. As you turn to the questions, remember that they are all
weighted equally and appear in no strict order of difficulty. So briefly look for the easiest questions and
plan to answer them first. Your assessment of which questions are easiest will partly depend on all your
practice. As you do practice logic games, take note of which question types are naturally easy for you,
and which require more concentration.

test-the-Rules Questions
Often a logic game will include a question that requires only the simple application of each given
condition indi- vidually. Every one of the wrong answer choices will violate a rule, while the correct answer
choice won’t violate
18
lsat logic, game tYpes, aNd geNeral strategies

any rule. These questions are great to tackle first, for two reasons. First, they are relatively easy; you
don’t have to do any complicated deductions or game-playing, you just have to check each answer choice
against the rules. And second, they force you to review the game conditions once more, a process that
helps you to crystallize the rules in your head even further and digest the game in preparation for the
subsequent questions. These ques- tions are often the first questions in a set.
When approaching a “Test-the-Rules” question, you could either take each answer choice in turn and
check to see if it violates any of the rules, or take each rule in turn and see which answer choices
violate it. The sec- ond method is often more efficient, partly because sometimes a rule will rule out
more than one answer choice.

Supposition Questions
Questions that give additional suppositions are usually easier than the rest because they set into motion
a chain of deductions or inferences (not necessarily straightforward if-then inferences) that lead to the
correct answer. It’s like getting an additional condition in the condition set, which allows for even more
deductions and concrete bits of information. Here’s an example based on the game matching pilots to
flights:

1. If Francis flies to New York, which of the following MUST be true?


a. Edgar will fly to London
b. Chris will fly to Sydney
c. Diane will fly to Sydney
d. Gerald will fly to Boston
e. Gerald will fly to Tokyo

The supposition that Francis flies to New York adds an additional bit of information to the game,
which when added to the second condition (that Gerald flies either to New York or Boston) immediately
yields a new concrete bit of information—that Gerald must fly to Boston. It’s a very simple example, but
it illustrates what a supposition question looks like, and why, in general, they are relatively easy
questions to tackle.
In addition, supposition questions often generate acceptable scenarios that can help with other
questions.
For example, sometimes you may be given a question that reads:

2. Each of the following statements could be true EXCEPT:

If you don’t immediately know which answer choice can’t be true, then at the very least you can use
the scenario generated by a supposition question to rule out statements that are true in that
scenario.
As usual, keep in mind that however you attack the question, do not spend too much time trying to
figure out which questions to do first. Figuring that out should be a relatively quick process based on
skills you have developed through practice, so that you are not wasting valuable time that could be
spent actually answering a question.

5) Keep Your Head


Although you should be able to master the Analytical Reasoning section after working through this
book, logic games require intense focus, concentration, and mental application. Don’t let it get to you.
After you’ve circled

19
lsat logic, game tYpes, aNd geNeral strategies

the answer choices for all the questions accompanying a given game, transfer that block of answers to your
score sheet. That way you don’t find out at the end of the section that you misaligned your answers and
now have just two minutes to transfer 20 answers. And before moving to the next game, take a brief
mental break: 15 seconds of deep breaths to clear your head.
If you approach the Analytical Reasoning section with these five steps and strategies in mind, you
will have half the battle won. The other half involves deploying the tips and strategies specific to each
game, which will be covered in Chapters 3 through 7.

develop Your own general strategy and Be ready to adapt

The strategies and rules presented here are not to be followed blindly. As you practice, change the five
steps and strategies around if you need to. For example, if you find yourself always finishing the games
in the allot- ted time for a given section, but missing a few here or there, consider skipping the Order
Your Battles phase to save some time.
Sometimes the LSAT will give you a game that isn’t readily amenable to the diagramming or
general tech- niques discussed here, or even the specific diagramming methods you have developed on
your own. At this point, one of a number of things could happen: 1) you digest the game and perceive its
structural similarity to other games you have solved or diagrammed, 2) you realize it really is different from
what you’ve seen and needs a different type of diagram you must develop on the spot, or 3) you realize
the game really is different but needs to be solved without a diagram (this is rare, but it happens).

Know What to Do If You Get Stuck


Don’t be a deer in the Logic Game headlights! If you get stuck, overcome your paralysis by doing any
one of a number of things: re-read the setup and conditions to make sure you didn’t miss anything,
double- or triple- check that your symbolizations are accurate, and try to find any deductions or connections
you may have missed. If you keep engaging the game, you have a much better chance of figuring out
what you’re missing.

Realize That There Are Objectively Correct Answers


When choosing answers in Logical Reasoning and Reading Comprehension passages, students are often
advised to choose the best answer from a set of answers of which more than one might arguably be
correct. Not so with logic games, which are more like the math in this respect. There is one true right
answer and the other answers are provably wrong. There is no arguing and no ambiguity.
This confidence means two things. First, you can save precious time on this exam. You don’t have to
weigh the merits of different answer choices and check every answer choice; as soon you know an answer is
right, move on with confidence. Second, you can maximize the impact of process of elimination; when
you eliminate an answer, you can do so with complete confidence.
Process of Elimination will be discussed in more detail in Chapter 2.

20
the

2
chapt e r

learningexpress
test
preparation
system

Taking any test can be tough, let alone one that tests unfamiliar logic game skills, but don’t let the LSAT
scare you! If you prepare ahead of time, you can achieve a top score. The LearningExpress Test
Preparation System, developed exclusively for LearningExpress by leading test experts, gives you the
discipline and attitude you need to be a winner.
First, the bad news: Getting ready for any test takes work. If you plan on entering law or another
career that benefits from a legal education, you will either be required to or be strongly encouraged to
take the LSAT. The LSAT has three sections, but this book focuses on the logic games, which many find
to be the most difficult material on the test. By honing your logic game skills and mastering that section
of the test, you will take your first step toward achieving the career of your dreams. However, there are
all sorts of pitfalls that can prevent you from doing your best on exams in general, whether logic-
based or not.
Here are some obstacles that can stand in the way of your success:

■■ Being unfamiliar with the format of the exam


■■ Being paralyzed by test anxiety
■■ Leaving your preparation to the last minute

■■ Not preparing at all

■■ Not knowing vital test-taking skills like:

■■ How to pace yourself through the exam

■■ How to use the process of elimination


■■ When to guess

21
the learNi NgeXpress test preparatioN s Ystem

■■ Not being in tip-top mental and physical shape


■■ Forgetting to eat breakfast and having to take the test on an empty stomach
■■ Forgetting a sweater or jacket and shivering through the exam

What’s the common denominator in all these test-taking pitfalls? One word: control. Who’s in
control, you or the exam?
Now the good news: The LearningExpress Test Preparation System puts you in control. In just nine
easy- to-follow steps, you will learn everything you need to know to make sure you are in charge of
your preparation and performance on the exam. Other test-takers may let the test get the better of them;
other test-takers may be unprepared or out of shape, but not you. You will have taken all the steps you
need to take to maximize your score. Here’s how the LearningExpress Test Preparation System works: Nine
easy steps lead you through every- thing you need to know and do to get ready to master your exam.
Each of the steps listed below gives you tips and activities to help you prepare for any exam. It’s
important that you follow the advice and do the activities,
or you won’t be getting the full benefit of the system. Each step gives you an approximate time estimate.

Step 1. Get Information 30 minutes


Step 2. Conquer Test Anxiety 20 minutes
Step 3. Make a Plan 50 minutes
Step 4. Learn to Manage Your Time 10
minutes Step 5. Learn to Use the Process of Elimination
20 minutes Step 6. Know When to Guess 20
minutes Step 7. Reach Your Peak Performance Zone 10
minutes Step 8. Get Your Act Together 10
minutes
Step 9. Do It! 10 minutes

Total time for complete system 180 minutes—3 hours

Estimate that working through the entire system will take you approximately three hours, though
it’s per- fectly okay if you work faster or slower than the time estimates say. If you can take a whole
afternoon or evening, you can work through the entire LearningExpress Test Preparation System in one
sitting. Otherwise, you can break it up and do just one or two steps a day for the next several days. It’s up
to you—remember, you’re in control.

step 1: get information

Time to complete: 30 minutes


Activities: Read the Introduction.
If you haven’t already done so, stop here and read the introduction of this book. There you’ll learn all
about the LSAT, such as the length of the test, the number and type of questions, and the way that the
test is scored.
Knowledge is power. The first step in the LearningExpress Test Preparation System is finding out
every- thing you can about the types of logic game and questions that you will encounter on the exam.
The intro- duction will give general information about the LSAT. Chapter 1 will give you a general idea
of how each of

22
the learNi NgeXpress test preparatioN s Ystem

the five types of logic games (sequencing, selection, distribution, matching, and hybrid) work, as well as
im- portant general strategies, but the chapters dedicated to each type will go into depth, providing very
specific game-solving techniques, as well as practice games and detailed solutions which show how to
use the recom- mended techniques.
Ideally, after completing the LearningExpress Test Preparation System, you will begin to apply the
test- taking strategies you learn as you work through the practice games in these chapters game-type
specific chap- ters (Chapters 3 through 7). You can see how well your training paid off in the practice
tests at the end of the book, which are also accompanied by detailed game solutions against which you
can compare your approach.

step 2: conquer test anxiety

Time to complete: 20 minutes


Activity: Take the Test Stress
Test
Having complete information about the exam is the first step in getting control of the exam. Next, you
have to overcome one of the biggest obstacles to test success: test anxiety. Test anxiety not only impairs
your perfor- mance on the exam, but it can even keep you from preparing!
In Step 2, you’ll learn stress management techniques that will help you succeed on your exam.
Learn these strategies now, and practice them as you work through the practice games in this book, so
they’ll be second na- ture to you by exam day.

Combating Test Anxiety


The first thing you need to know is that a little test anxiety is a good thing. Everyone gets nervous before
a big exam—and if that nervousness motivates you to prepare thoroughly, so much the better. It’s said that Sir
Laurence Olivier, one of the world’s most celebrated actors, was ill before every performance. His stage
fright didn’t im- pair his performance; in fact, it probably gave him a little extra edge—just the kind of
edge you need to do well, whether on a stage or in an exam room.
On page 25 is the Test Stress Test. Stop here and answer the questions on that page to find out
whether your level of test anxiety is something you should worry about.

Stress Management before the Test


If you feel your level of anxiety getting the best of you in the weeks before the test, here is what you need
to do to bring the level down again:

■■ Get prepared. There’s nothing like knowing what to expect. Being prepared will put you in control of
test anxiety. That’s why you’re reading this book. Use it faithfully, and remind yourself that you’re
better prepared than most of the people taking the test.
■■ Practice self-confidence. A positive attitude is a great way to combat test anxiety. This is no time to be

humble or shy. Stand in front of the mirror and say to your reflection, “I’m prepared. I’m full
of self- confidence. I’m going to ace this test. I know I can do it.” Say it into a recorder and play it
back once a day. If you hear it often enough, you’ll believe it.
■■ Fight negative messages. Every time someone starts telling you how hard the exam is, start telling them

23
the learNi NgeXpress test preparatioN s Ystem

your self-confidence messages above. If you are the one telling yourself that you don’t do well on
exams and you just can’t do this, don’t listen. Turn on your recorder and listen to your self-
confidence messages.
■■ Visualize. Imagine yourself reporting for your first day on the job. Visualizing success can help make it

happen—and it reminds you why you’re preparing for the exam so diligently.
■■ Exercise. Physical activity helps calm down your body and focus your mind. Besides, being in good

physical shape can actually help you do well on the exam. Go for a run, lift weights, go swimming—and
do it regularly.

Stress Management on Test Day


There are several ways you can bring down your level of test anxiety on test day. To find a comfort level,
experi- ment with the following exercises in the weeks before the test, and use the ones that work
best for you.

■■ Breathe deeply. Take a deep breath while you count to five. Hold it for a count of one, then let it out on
a count of five. Repeat several times.
■■ Move your body. Try rolling your head in a circle. Rotate your shoulders. Shake your hands from the

wrist. Many people find these movements very relaxing.


■■ Visualize again. Think of the place where you are most relaxed: lying on the beach in the sun, walking

through the park, or sipping a cup of hot tea. Now close your eyes and imagine you’re actually
there. If you practice in advance, you’ll find that you need only a few seconds of this exercise to
experience a significant increase in your sense of well-being.

When anxiety threatens to overwhelm you right there during the exam, there are still things you can do
to man- age your stress level:

■■ Repeat your self-confidence messages. You should have them memorized by now. Say them quietly to
yourself, and believe them!
■■ Visualize one more time. This time, visualize yourself moving smoothly and quickly through the test

answering every question right and finishing just before time is up. Like most visualization
techniques, this one works best if you’ve practiced it ahead of time.
■■ Find an easy question. Skim over the questions until you find an easy question, and then answer it.

Filling in even one circle gets you into the test-taking groove.
■■ Take a mental break. Everyone loses concentration once in a while during a long test. It’s normal, so

you shouldn’t worry about it. Instead, accept what has happened. Say to yourself, “Hey, I lost it
there for a min- ute. My brain is taking a break.” Put down your pencil, close your eyes, and do
some deep breathing for a few seconds. Then you’re ready to go back to work.

Try these techniques ahead of time, and see if they work for you!
24
the learNi NgeXpress test preparatioN s Ystem

Test Stress Test


You only need to worry about test anxiety if it is extreme enough to impair your performance. The
following questionnaire will provide a diagnosis of your level of test anxiety. In the blank before each
statement, write the number that most accurately describes your experience.

0 = never
1 = once or
twice 2 =
sometimes
3 = often

I have gotten so nervous before an exam that I simply put down the books and didn’t study for it.
I have experienced disabling physical symptoms such as vomiting and severe headaches because I was
ner- vous about an exam.
I have simply not showed up for an exam because I was scared to take it.
I have experienced dizziness and disorientation while taking an exam.
I have had trouble filling in the circles because my hands were shaking too hard.
I have failed an exam because I was too nervous to complete it.
Total: Add up the numbers in the blanks above.

Your test Stress Score


Here are the steps you should take, depending on your score. If you scored:

■■ 0–3: Your level of test anxiety is nothing to worry about; it’s probably just enough to give you the
motiva- tion to excel.
■■ 3–6: Your test anxiety may be enough to impair your performance, and you should practice the stress

management techniques listed in this section to try to bring your test anxiety down to a more
manage- able level.
■■ 6+: Your level of test anxiety is a serious concern. In addition to practicing the stress management tech-

niques listed in this section, you may want to seek additional, professional help. Call your college,
university, or community college and ask for the academic counselor. Tell the counselor that you
have a level of test anxiety that sometimes keeps you from being able to take an exam. The
counselor may be willing to help you or may suggest someone else you should talk to.

step 3: make a plan

Time to complete: 50
minutes Activity: Construct a
study plan
Maybe the most important thing you can do to get control of yourself and your exam is to make a study
plan. Too many people fail to prepare simply because they fail to plan. Spending hours on the day before
the exam poring over sample test questions not only raises your level of test anxiety, it is also no
substitute for careful preparation and practice.

25
the learNi NgeXpress test preparatioN s Ystem

Don’t fall into the cram trap. Take control of your preparation time by mapping out a study
schedule. If you’re the kind of person who needs deadlines and assignments to motivate you for a
project, here they are. If you’re the kind of person who doesn’t like to follow other people’s plans, you
can use the suggested schedules here to construct your own.
Even more important than making a plan is making a commitment. You can’t develop the skills
you need to do well on the LSAT in one night. You have to set aside some time every day for study and
practice. Try for at least 20 minutes a day. Twenty minutes daily will do you much more good than two
hours the day before the exam. Start now. Even ten minutes a day (just one practice game), with half an
hour or more on weekends, can make a big difference in your score—and in your chances of
obtaining the score you want!

Schedule A: The 30-Day Plan


If you have at least one month before you take your test, you have plenty of time to prepare—as long as
you don’t procrastinate! If you have less than a month, turn to Schedule B. Of course, keep in mind that
this book will get you ready for the logic games section of the LSAT, not the entire test. You might
consider creating similar, concurrent schedules to prepare for the exam as a whole.

day 1
Skim over any written materials you may have about the LSAT. Learn the specific content and skills that
you need to brush up on to prepare for the test. Read the introduction of this book. Look over relevant
information on the LSAC website and review actual LSAT example questions there.

day 2
Read Chapter 1, LSAT Logic, Game Types, and General Strategies. Learn the five-step technique for
approach- ing the whole logic games section, and general strategies for solving logic games. Consider
writing down the five steps, along with key elements from each step, and reviewing them before each
logic game or logic game section you practice. Take the time to make sure you understand the formal
logical rules that are essential to cracking the games.

days 3–5
Read Chapter 3, “Sequencing Games Review.” Experiment and practice with the diagramming and
problem- solving techniques presented for sequencing games. Make sure to apply these techniques as
you work through the practice games! After each game, compare your approach to the suggested
approach and see if you can im- prove your accuracy or efficiency in the next game.

day 6
Review any Chapter 3 concepts or techniques that you feel are necessary for you to brush up on.

days 7–9
Read Chapter 4, “Selection Games Review.” Experiment and practice with the diagramming and problem-
solving techniques presented for selection games. Make sure to apply these techniques as you work through
the practice games! After each game, compare your approach to the suggested approach, and see if you
can improve your accuracy or efficiency in the next game.

26
the learNi NgeXpress test preparatioN s Ystem

day 10
Review any Chapter 4 concepts or techniques that you feel are necessary for you to brush up on.

days 11–13
Read Chapter 5, “Distribution Games Review.” Experiment and practice with the diagramming and
problem- solving techniques presented for distribution games. Make sure to apply these techniques as you
work through the practice games! After each game, compare your approach to the suggested approach,
and see if you can im- prove your accuracy or efficiency in the next game.

day 14
Review any Chapter 5 concepts or techniques you feel are necessary for you to brush up on.

days 15–17
Read Chapter 6, “Matching Games Review.” Experiment and practice with the diagramming and
problem- solving techniques presented for matching games. Make sure to apply these techniques as you work
through the practice games! After each game, compare your approach to the suggested approach, and see
if you can improve your accuracy or efficiency in the next game.

day 18
Review any Chapter 6 concepts or techniques you feel are necessary for you to brush up on.

days 19–21
Read Chapter 7, “Hybrid Games Review.” Experiment and practice with the diagramming and problem-
solving techniques presented for hybrid games. Make sure to apply these techniques as you work
through the practice games! After each game, compare your approach to the suggested approach, and
see if you can improve your accuracy or efficiency in the next game.

day 22
Review any Chapter 7 concepts or techniques you feel are necessary for you to brush up on.

days 23–25
Take the two practice logic games sections in Chapter 8 under timed conditions—allow yourself 35
minutes, as you would on the actual exam. Apply the five-step technique to the section, and apply the
game-type specific strategies you have learned. After each game, review the answer explanations to see how
you can improve your game-solving technique for the next practice section. If you had difficulty with a
game, review the techniques and tips in the chapter specific to that game type.

days 26–28
Take the two practice logic games sections in Chapter 9 under timed conditions—allow 35 minutes, as you
would on the actual exam. Apply the five-step technique to the section, and apply the game-type specific
strategies you have learned. After each game, review the answer explanations to see how you can improve
your game-solving technique for the next practice section. If you had difficulty with a game, review the
techniques and tips in the chapter specific to that game type.

27
the learNi NgeXpress test preparatioN s Ystem

day 29
Quickly refresh yourself on all the suggested game-solving techniques presented in the book. Do one
final re- view of the concepts and techniques related to any game-types you discovered you were weak
on while taking the practice logic sections the day before the exam

Relax
Do something unrelated to the exam and go to bed at a reasonable hour.

Schedule B: The 14-Day Plan


If you have two weeks or less before the exam, you may have your work cut out for you. Use this 14-day
schedule to help you make the most of your time.

day 1
Read the introduction and strategies in Chapter 1.

day 2
Take the practice section in Chapter 8 to identify game types you need help with. Rank order the five
game types based on your ability to solve them quickly.

days 3–5
Read the chapter on the game type you need the most help with. Complete the practice games, and
review the explanations.

days 6–8
Read the chapter on the second game type on your list. Complete the practice games, and review the
explanations.

days 9–10
Read the chapter on the third game type on your list. Complete the practice games, and review the
explanations.

day 11
Read the chapter on the fourth game type on your list. Complete the practice games, and review the
explanations.

day 12
Read the chapter on the fifth game type on your list. Complete the practice games, and review the
explanations.

day 13
Take the practice section in Chapter 9, review the answer explanations, and review concepts and
techniques with which you were weakest.

day before the exam


Relax. Do something unrelated to the exam and go to bed at a reasonable hour.
28
the learNi NgeXpress test preparatioN s Ystem

step 4: learn to manage Your time

Time to complete: 10 minutes to read, many hours of practice!


Activities: Use these strategies as you take the practice games and sections in this book
Steps 4, 5, and 6 of the LearningExpress Test Preparation System put you in charge of your exam by
showing you some test-taking strategies that work. Practice these strategies as you take the sample test in
this book, and then you’ll be ready to use them on test day.
First, take control of your time on the exam. The LSAT Analytical Reasoning (Logic Games) section
has a time limit of 35 minutes, which may give you more than enough time to complete all the questions
—or not enough time. It’s a terrible feeling to hear the examiner say, “five minutes left,” when you’re
only three-quarters of the way through the test. Here are some tips to keep that from happening to
you.

■■ Follow directions. Carefully read the LSAT Analytical Reasoning directions ahead of time and know
them well, so you don’t have to re-read them after the exam begins. Ask questions before the exam
begins if there is anything you don’t understand. Write down the beginning time and ending time of the
exam in your booklet.
■■ Pace yourself. Glance at your watch every few minutes, and compare the time to how far you’ve gotten

in the test. Each game should take you about 8 minutes. If you’re falling behind, pick up the
pace a bit.
■■ Keep moving. Don’t waste time on one question. If you don’t know the answer, skip the question and

move on. Circle the number of the question in your test booklet in case you have time to come back
to it later. Do not let one question on one game bog you down!
■■ Keep track of your place on the answer sheet. If you skip a question, make sure you skip it on the

answer sheet too. Check yourself every few questions to make sure the question number and the
answer sheet number are still the same.
■■ Don’t rush. Although you should keep moving, rushing won’t help. Try to keep calm and work

methodi- cally and quickly.

step 5: learn to use the process of elimination

Time to complete: 20 minutes


After time management, your most important tool for taking control of your exam is using the process of
elim- ination wisely. As discussed in Chapter 1, the proper Process of Elimination will be a bit different
for LSAT Analytical Reasoning than it is for other sections of the LSAT. It’s standard test-taking wisdom
that you should always read all the answer choices before choosing your answer, since this will ensure
that you are choosing the best answer. That standard wisdom does NOT apply to logic games. For every
question, there is one objectively and provably correct answer choice, and four incorrect choices.
The LSAT, and especially the Analytical Reasoning section, is a test of efficiency as much as it is a
test of problem-solving or logical abilities, so you need to shave off unnecssary time expenditures as much as
possible. Reviewing other answer choices is simply a waste of time once you have found the correct
answer choice. The key, of course, is knowing that your answer choice is correct. As long as you diagram
correctly and make no logical mistakes in your deductions, you will be in a position to mark the correct
answer choice with confidence and move on to the next question. By working through this book, you
will practice these techniques and gain the analytical strength to get the right answer and fearlessly
circle it.

29
the learNi NgeXpress test preparatioN s Ystem

So what use does the Process of Elimination (POE) have on the LSAT? Plenty. First, some questions,
such as “Test-the-Rules” questions (which appear in almost every logic game), are better approached by
elimating all the incorrect answer choices rather than finding the correct answer choice.
Second, you may not always know how to apply your diagramming techniques and deductions to find
the correct answer right away. In these cases, the POE will work at least as well here as on any other test
—in fact, sometimes it works better, because you might know that an answer choice is incorrect.
Let’s review how to find the right answer by POE, when you can’t easily find it directly.

Understand the Question


As you read a question, you may find it helpful to underline important information or make some notes
about what you’re reading. When you get to the heart of the question, circle it and make sure you
understand what it is asking. Keep an eye out for the words ‘NOT’ and ‘EXCEPT’—they make a big
difference to the question. If you miss these words when reading a question, wrong answer choices will
seem right, and right answer choices will seem wrong. If you’re not sure of what’s being asked, you’ll never
know whether you’ve chosen the right answer. If after reading the question, you really have no idea what it
is asking, it sometimes helps to take a quick look at the answer choices for some clues.
For example, if the question asks “which one of the following could be a partial, accurate list of the
people who are on the flight,” you might not know what is being asked right away. But if you look at the
answer choices, you may have a better idea of what the question is asking:

a. Paul, Theresa
b. Paul, Ryan
c. Paul, Sam, Theresa
d. Ryan, Sam, Theresa
e. Paul, Ryan, Sam, Theresa

The answer choices might help you realize that the question is looking for a subset of a larger list,
where that subset only includes people on the flight and where that subset could be any size.
What you do next depends on the type of question you’re answering.

Eliminating Wrong Answers in Logic Games


When using POE on a typical test, you might start by marking the answers that seem unreasonable. POE
for logic games will be a bit different. There are certainly reasons to think that an answer will probably
be incorrect, and we will discuss these reasons in the next section. Moreover, it will be helpful to use
these reasons as a guide when figuring out which answers to try to eliminate first. But when it comes to
actually eliminating answers, the best thing you can do is test the answer and make logical inferences to
see that the answer is wrong. There are a number of ways in which you can know an answer is
wrong. Here are a few important ones.

Rule Violations
The most straightforward way to eliminate an answer choice is to see that it violates one of the restricions
in the game setup. For example, in a game that asks you to place seven flights in the order of their
departure from an airport, one of the restrictions may be that flight A departs after flight D. Then if you
have a question that asks for an acceptable departure order, you can eliminate any answer choice in
which flight A does not depart after

30
the learNi NgeXpress test preparatioN s Ystem

flight D. For example, the question might be “if flight C departs before flight F, which one of the
following must be the order of departures, from first to last?” Whatever the right answer is, you can
surely eliminate the answer choice ‘C, B, A, E, D, F, G,’ since it violates the restriction that flight A
departs after flight D.

Use Prior work


Each logic game will have five to seven questions accompanying it. Sometimes, diagrams you created,
deductions you made, and answers you found for a previous question will help eliminate answer choices on a
later question. When trying to use POE, remember to look at previous work or answers to see if they
can help.
For example, suppose a question asks “which one of the following CANNOT be the departures
sequence, in order from first to last?” Suppose you find that the correct answer choice is ‘C, B, A, E,
D, F, G.’ Then sup- pose another question asks “if E departs fourth, which one of the following MUST
be the departure sequence, in order from first to last?” If ’ ‘C, B, A, E, D, F, G’ appears as one of your
answer choices, you can immediately eliminate it, because you know from the earlier question that it
can’t be the departure sequence.

Interchangeables
Sometimes two entities will be indistinguishable with respect to their role in the game. These entities are
‘in- terchangeables.’ If a set of answer choices includes an answer choice for both members of an
interchangeable duo, then both of those answer choices must be wrong. This is because each question
demands one and only one answer—and if two essentially indistinguishable answers appear, then
neither one of them can be the one and only answer.
Let’s look at an example. Suppose in the game setup, there are no restrictions that mention the
entities B and G. And suppose the question reads “if E departs fourth, which one of the following MUST
depart sixth?” And suppose the you have the following answer choices:

a. A
b. B
c. D
d. F
e. G

Since B and G are indistinguishable with respect to the roles they play in the game, and are not
mentioned in the question, neither of them can be the one that must depart sixth; if B departs sixth, and
it’s pretty much the same as G, then G must depart sixth as well. So you can immediately eliminate
both of these choices.
Be careful with this technique. Sometimes B and G will appear indistinguishable, but might actually
be indirectly distinguishable. For example, B might be included on a list of flights to Chicago given in the
setup, and G might be included on a list of flights to Detroit given in the setup. If a restriction involves
all flights to Chicago (e.g., “the last three flights are flights to Chicago”), then the two flights might play
different roles in the game, and you cannot apply this technique.
There are other ways to eliminate wrong answers in logic games without knowing what the right answer
is, and these will become apparent as you work through the practice games and answer explanations in
this book. The basic idea, though, is that when you don’t know what the right answer is, do what you
can to eliminate an- swer choices you can fully determine are incorrect. Whatever you do, don’t waste
too much time with any one answer choice. If it’s the right answer, you may be able to eliminate all the
others, and if it’s a wrong answer,

31
the learNi NgeXpress test preparatioN s Ystem

working with the other answer choices may help you make inferences that allow you to eliminate that
answer as well. By quickly eliminating at least some wrong answer choices, you put yourself in a
position to move to the guessing phase and increase your probability of getting the right answer without
losing overall efficiency.
If you haven’t eliminated any answers at all, skip the question temporarily, but don’t forget to mark
the question so you can come back to it later if you have time. If you’re certain that you could never
answer this question in a million years, pick an answer and move on. The test has no penalty for
wrong answers.

Guess on Every Question


You will not be penalized for wrong answers on the LSAT. This is very good news. It means you should
absolutely answer every single question on the test. If you’re hopelessly lost and can’t even cross off one answer
choice, make sure that you don’t leave it blank. Even if you only have 30 seconds left and 10 questions to
answer, you should just guess on all of those last questions.
Of course, if you can eliminate even one of the choices, you improve your odds of guessing
correctly. If you can identify three of the choices as definitely wrong, you have a 50% chance of
answering the question correctly.

If You Finish Early


Use any time you have left to do the following:

■■ Go back to questions you marked to return to later, and try them again.
■■ Check your work on all the other questions. If you have a good reason for thinking your reasoning was
incorrect and your response is wrong, change it.
■■ Review your answer sheet. Most tests are scored in such a way that questions with more than one answer

are marked wrong.


■■ Make sure you’ve put the answers in the right places and you’ve marked only one answer for each question.

■■ If you’ve erased an answer, make sure you’ve done a good job of it.

■■ Check for stray marks on your answer sheet that could distort your score.

Whatever you do, don’t waste time when you’ve finished a test section. Make every second count by
check- ing your work over and over again until time is up.

step 6: know when to guess

Time to complete: 20 minutes


Activity: Take the Guessing Quiz
On the LSAT, the number of questions you answer correctly yields your raw score. So you have nothing
to lose and everything to gain by guessing. Frankly, even if you’re a play-it-safe kind of person with
terrible intuition, you’re still safe in guessing every time because the exam has no guessing penalty. The
best thing would be if you could overcome your anxieties and go ahead and mark an answer. Second, if
you apply Step 5 and learned how to eliminate wrong answers even when you don’t know the right
answer, your chances of guessing correctly are significantly improved. Third, even if you have a bad
LSAT guessing intution, there are a few ways you can im- prove that intuition, which you will see
after the guessing quiz.
So if you don’t know the answer, go ahead and guess!

32
the learNi NgeXpress test preparatioN s Ystem

Your Guessing Ability


The following are a few logic game questions, some without any game setup. You’re not supposed to
know the answers (they are modified from actual logic games). Rather, this is an assessment of your
ability to guess when you don’t have a clue. Read each question carefully, just as if you did expect to
answer it. If the question has a game setup and you think that you might be able to deduce the right
answer by playing the game, don’t! This is meant to test your guessing intuition for situations in
which you don’t know how to play the game.

1. Which one of the following statements cannot be true?


a. Exactly two nurses are chosen.
b. Exactly two foreign professionals are chosen.
c. Exactly three foreign professionals are chosen.
d. Exactly two local professionals are chosen.
e. Exactly three local professionals are chosen.

Five books are to be chosen from seven—A, B, C, D, E, F and G—according to the


following restrictions:
If either A is not chosen or B is not chosen, C is
chosen. If C is chosen, D is not chosen.

2. If D is chosen, which one of the following must be chosen?


a. A
b. C
c. E
d. F
e. G

Five books are to be chosen from seven—A, B, C, D, E, F and G—according to the following

restrictions: If either A is not chosen or B is not chosen, C is chosen.


If C is chosen, D is not
chosen. If F or E are chosen,
D is chosen
If F is not chosen, B is not chosen.

3. If A is not chosen, which one of the following could be chosen?


a. B
b. D
c. E
d. F
e. G
33
the learNi NgeXpress test preparatioN s Ystem

Answers
1. C
2. A
3. E

How Did You Do?


You may have not been able to avoid reasoning through the last two questions in your head, and getting
the an- swers. But if you managed to refrain from reasoning through the deductions, then you will have
a better sense of how you guess. If you got even one answer correct, then you’re doing pretty well. If you
got two or three, then you’re a terrific LSAT logic games guesser. Regardless, you can learn how to guess
when you absolutely have to on these kinds of questions.

Guessing on Logic Games


The three questions on the guessing quiz reperesent three sorts of questions for which educated guessing
might work:

numerical Restrictions
The answer to the first question is that ‘exactly three foreign professionals are chosen.’ When you
encounter ‘cannot-be-true’ questions for logic games that involve numerical restrictions, the answer choices
with the high- est numbers involved are often correct. This is because a larger block of entities (three
entities vs. two) will have less freedom in the mechanics of a numbers-driven game. For example, if the
game specifies that only five pro- fessionals can be chosen, and three of them must be domestic rather
than foreign, then although zero, one, or two foreign professionals could be chosen, three could not.

Must-Be Questions
The correct answer for the second question is that entity A must be chosen. You could have guessed A by
noting that A is mentioned in the game setup. Most of the other entities presented in the answer choices,
except for C, are not involved in any of the restrictions. So there is no reason that they must be chosen.
Something that must be chosen is probably involved in some restrictions.

Could-Be Questions
The opposite logic applies for “could-be” questions. The correct answer for the third question is that
entity G could be chosen. G is the only entity of those presented in the answer choices that is not
involved in any of the restrictions. So there probably can’t be anything preventing it from being
chosen.
These are just some of the question types for which noticing patterns in the answer choices can help
you guess. As you work through the sample questions in this book, you will see more instances where
you might have guessed an answer. Take note of them for those moments on the LSAT when you
need to guess.

34
the learNi NgeXpress test preparatioN s Ystem

step 7: reach Your peak performance Zone

Time to complete: 10 minutes to read; weeks to


complete! Activity: Complete the Physical Preparation
Checklist
To get ready for a challenge like a big exam, you have to take control of your physical, as well as your
mental state. Exercise, proper diet, and rest will ensure that your body works with, rather than against,
your mind on test day, as well as during your preparation.

Exercise
If you don’t already have a regular exercise program going, this is actually an excellent time to start one.
If you’re already keeping fit—or trying to get that way—don’t let the pressure of preparing for an exam
fool you into quitting now. Exercise helps reduce stress by pumping wonderful feel-good hormones
called endorphins into your system. It also increases the oxygen supply throughout your body and your
brain, so you’ll be at peak performance on test day.
A half hour of vigorous activity—enough to break a sweat—every day should be your aim. If you’re
really pressed for time, every other day is OK. Choose an activity you like and get out there and do it.
Jogging with a friend always makes the time go faster as does listening to music.
But don’t overdo it. You don’t want to exhaust yourself. Moderation is the key.

Diet
First of all, cut out the junk. Go easy on caffeine and nicotine, and eliminate alcohol and any other drugs
from your system at least two weeks before the exam.
What your body needs for peak performance is a balanced diet. Eat plenty of fruits and vegetables,
along with protein and complex carbohydrates. Foods that are high in lecithin (an amino acid), such as
fish and beans, are especially good “brain foods.”

Rest
You probably know how much sleep you need every night to be at your best, even if you don’t always get
it. Make sure you do get that much sleep, though, for at least a week before the exam. Moderation is
important here, too. Extra sleep will just make you groggy.
If you’re not a morning person and your exam will be given in the morning, you should reset your
internal clock so that your body doesn’t think you’re taking an exam at 3 a.m. You have to start this
process well before the exam. The way it works is to get up half an hour earlier each morning, and then
go to bed half an hour ear- lier that night. Don’t try it the other way around; you’ll just toss and turn if
you go to bed early without getting up early. The next morning, get up another half an hour earlier, and
so on. How long you will have to do this depends on how late you’re used to getting up. Use the
“Physical Preparation Checklist” on page 37 to make sure you’re in tip-top form.

35
the learNi NgeXpress test preparatioN s Ystem

step 8: get Your act together

Time to complete: 10 minutes to read; time to complete will


vary Activity: Complete Final Preparations worksheet
Once you feel in control of your mind and body, you’re in charge of test anxiety, test preparation, and
test-taking strategies. Now it’s time to make charts and gather the materials you need to take to the
exam.

Gather Your Materials


The night before the exam, lay out the clothes you will wear and the materials you need to bring with
you to the exam. Plan on dressing in layers because you won’t have any control over the temperature of
the exam room. Have a sweater or jacket you can take off if it’s warm. Use the checklist on the worksheet
entitled “Final Preparations” on page 38 to help you pull together what you’ll need.

Follow Your Routine


If you usually have coffee and toast every morning, then you should have coffee and toast before the
test. If you don’t usually eat breakfast, don’t start changing your habits on exam morning. Do whatever
you normally do so that your body will be used to it. If you’re not used to it, a cup of coffee can really
disrupt your stomach. Doughnuts or other sweet foods can give you a stomache ache, too. When
deciding what to have for breakfast, remember that a sugar high will leave you with a sugar low in the
middle of the exam. A mix of protein and carbohydrates is best: Cereal with milk or eggs with toast
will do your body a world of good.
36
Physical Preparation Checklist

For the week before the test, write down what physical exercise you engaged in and for how long and what
you ate for each meal. Remember, you’re trying for at least half an hour of exercise every other day
(preferably every day) and a balanced diet that’s light on junk food.

Exam minus 7 days Exam minus 3 days


Exercise: for minutes
Exercise: for minutes
Breakfast:
Breakfast:
Lunch:
Lunch:
Dinner:
Dinner:
Snacks:
Snacks:

Exam minus 6 days Exam minus 2 days


Exercise: for minutes
Exercise: for minutes
Breakfast:
Breakfast:
Lunch:
Lunch:
Dinner:
Dinner:
Snacks:
Snacks:

Exam minus 5 days Exam minus 1 day


Exercise: for minutes
Exercise: for minutes
Breakfast:
Breakfast:
Lunch:
Lunch:
Dinner:
Dinner:
Snacks:
Snacks:

Exam minus 4 days


Exercise: for minutes
Breakfast:
Lunch:
Dinner:
Snacks:

37
the learNi NgeXpress test preparatioN s Ystem

step 9: do it!

Time to complete: 10 minutes, plus test-taking


time Activity: Ace Your Test!
Fast-forward to exam day. You’re ready. You made a study plan and followed through. You practiced
your test- taking strategies while working through this book. You’re in control of your physical, mental,
and emotional state. You know when and where to show up and what to bring with you.
In other words, you’re better prepared than most of the other people taking the test with you.
You’re psyched!
Just one more thing. When you’re done with the exam, you will have earned a reward. Plan a night
out. Call your friends and plan a party, or have a nice dinner for two—whatever your heart desires. Give
yourself something to look forward to.
And then do it. Go into the exam, full of confidence, armed with the test-taking strategies you’ve
practiced until they’re second nature. You’re in control of yourself, your environment, and your
performance on exam day. You’re ready to succeed.
So do it. Go in there and ace the LSAT! And then, look forward to your new career.

Final Preparations

Date: Departure time: Do I

38
3
chapt e r

sequencing games
review

In a sequencing game, you are given a set of entities to put in sequential order. The entities can be persons,
places, or things, and the order can be spatial, temporal, or some other metric. For example, it may be a list
of cities (the entities) to visit on a trip (temporal order), a set of friends (the entities) to seat around a table
(spatial order) or a list of television shows (the entities) to rank according to popularity (a non-
spatial/temporal metric). The key to mastering sequencing games is to understand the types of conditions that
govern the sequencing,
and to symbolize those conditions. These conditions, either indirectly or directly, result in a set of
sequencing “players.” The ultimate aim of symbolizing these players is to produce a relative ordering of some
of the entities, which can then be mapped onto a diagram of slots representing the number of places in
the sequence (e.g., five slots for a sequence from first to fifth).
To make the examples more concrete, let’s pretend that we have a sequencing game in which five
flights— A, B, C, D, and E—depart from an airport one at a time. To make the sequencing visual, we
diagram five slots, labeled 1 through 5 from left to right.

12345

In general, entities to the left will be “before” or “of a lower number” in the sequence than entities
to the right. The general strategy for diagramming sequencing games is straightforward: to put entities
into the slots. This process is relatively self-explanatory and will become apparent as we work through
practice games.

39
–se QueNciNg games reView–

There are eight common players in


sequencing games. In this chapter, we will look at
40
the game condi- tions that often produce them, and
recommended meth- ods for symbolizing them.
These common players are:

■■ Ordered Blocks
■■ Unordered Blocks

■■ Restricted Blocks

■■ Numbered Sequence Blocks

■■ Sequence Strings

■■ Partially Unordered Sequence Strings

■■ Joint Sequence Strings

■■ Floaters

the eight common


sequencing players

As we will see, symbolizing restrictions will often


re- sult in large, consolidated symbolizations (the
“Joint Sequence Strings”) that are effectively their
own di- agrams. As the practice games will
illustrate, some- times it will be useful to refer to
these symbolizations independently of the slots-
diagram when answering a question, and other
times it will be useful to refer to those
symbolizations in order to help place those enti-
ties into the slots-diagram.

Ordered Blocks
We might be given the information that two
entities are immediately next to one another in a
sequence. For example, we might be told that
flight A departs immediately after f light B.
This could mean that flight B is first and flight
A is second, or that flight B is third and flight A
is fourth. We call it an ordered block because
we know that A and B will be placed on the
diagram next to one another, and know the
order in which they will be placed. We symbolize
the block by just placing the letters next to each
other, in the proper order:

BA
We might also symbolize bigger blocks, if we
know that A immediately follows B, which immedi-
ately follows D:

DBA

The blocks work visually by immediately telling us


where a set of entities might fit in an ordering. For
example, if flight C departs second, then we know that
the BA block must fit somewhere into slots 3–5, and
that therefore neither A nor B can depart first.
Here are some examples of conditions that would
give ordered blocks:

Flight A departs immediately after Flight B.


Adam has his interview the day immediately
after Raja’s interview.
Mona sits just to the left of Lillja.
Wells sits immediately to the left of Anand.

Unordered Blocks
We might be given the information that two entities
are immediately next to one another in a sequence
without knowing which one comes first. For example,
we might be told that Flight A departs either imme-
diately before or after flight B. This could mean that
flight A is first and flight B is second, or that flight
B is first and flight A is second.
We call this an unordered block because we
know that A and B will be placed on the diagram next
to one another, but we do not know their relative
order. We symbolize the block by just placing the
letters next to each other, with a slash in between
them to remind ourselves that we do not know the
order, and that A could come before B, or B could
come before A:

A/B

We might also symbolize bigger blocks. For ex-


ample, if we know that there are no other flights that
separate flights A, B, and D, we could write:

A/B/D
–se QueNciNg games reView–

If we know that A immediately follows B,


and that D is either immediately before B or Numbered Sequence Block
after A, we could combine ordered and unordered Sometimes we might be given the number of
blocks to write: places in the sequence between two entities. For
example, we might be told that there are exactly
(BA)/D two flights that depart between Flight A and B.
So flight A might be first and flight B might be
As with ordered blocks, these unordered blocks fourth, or flight B third and flight A sixth. We
work visually by immediately telling us where a set could symbolize this numbered sequence block
of entities might fit in an ordering. For example, if with two place holders in between A and B, and a
flight C departs second, then we know that slash to indicate that A and B could be reversed
neither A nor B can depart first. in order:
Here are some examples of conditions that
would give unordered blocks: A_/_B

Flight A departs either immediately before or Here are some examples of conditions that
after flight B. would yield numbered sequence blocks (in
Adam has his interview either the day particular, with exact numbers):
immedi- ately before or after Raja’s
interview. There are exactly two flights that depart
Mona sits either just to the left or right of between flight A and flight B.
Lillja. Wells sits next to Anand. There are exactly three interviews in
between Adam’s interview and Raja’s
Restricted Blocks interview.
We might be given the information that certain enti- There are exactly three seats in between
ties are not placed next to one another. For example, Wells and Anand.
we might be told that Flight A does not depart
imme- diately before or after Flight B. Flight A might We might also be told that there are at
be first and flight B third, or flight B second and least a certain number of places in the sequence
flight A fifth. between two entities. For example, we might be
We call it a restricted block because it told that there are at least two departures in
restricts a certain block from existing. We can between flight A and flight
symbolize it by drawing a non-sequenced block, B. So flight A might be first and flight B is
and crossing it out, as follows: fourth, or flight B first, and flight A fifth; as long
as two or more departures separate the flights. We
A/B can symbolize this numbered sequence block by
adding a ‘+’ sign, to indi- cate that the exact
Here are some examples of conditions that number of spaces in the symboliza- tion—or more
would yield restricted blocks: —must exist between the two entities:

Flight A neither departs immediately before A_/_+B


nor after flight B.
Adam does not have his interview on the Here are some examples of conditions that
day immediately before or after Raja’s would yield these kinds of numbered sequence
interview. blocks:
Wells does not sit next to Anand.
There are at least two flights that depart between flight A and flight B.

41
–se QueNciNg games reView–

There are at least two interviews either after 42


Adam’s interview and before Raja’s inter-
view, or after Raja’s interview and before
Adam’s interview.
There is at least one seat between Mona
and Lillja.

Sequence Strings
We might be given information that provides a
rela- tive ordering for some of the entities. For
example, we might be told that flight A departs
after flight B. This differs from the Ordered Block,
because we are not told that flight A is immediately
after flight B. Rather, we are just told that flight
A departs at some point after flight B. Flight B
might be first and flight A third, or flight B
second and flight A fifth.
We call it a sequence string because we
string the entities together by a line, and can
string multiple entities together if we know their
relative order. We would symbolize the condition
that flight A departs after flight B as:

B——–A

We can also get bigger strings. Suppose we


are told that flight A departs after flight B, but
before flight D. Then we can symbolize this
condition:

B——–A——–D

These sequence strings work visually by


telling us where the strings might fit in a
sequence. For ex- ample, suppose we are told that
flight B does not de- part first or second. Since
there are two flights to the right of B on this
string, flight B must depart third. Otherwise, if
it departed fourth or fifth, there would not be
enough room for the rest of the string to the
right of flight B.
Here are some examples of conditions that
would give sequence strings:

Flight A departs after flight B, but before


flight D.
Adam has his interview before Raja has his
interview.
Mona sits to the right of Lillja, but to the left
of Justin.
Wells sits to the right of Anand.

Partially Unordered Sequence String


As a variation on the sequence string, we might be
given information that places two entities in relative
order with respect to a third, but not with respect to
each other. For example, we might be told that flights
F and G depart after flight D. So we might have D de-
part first, F second, and G third; or D departs second,
G third, and F fifth.
We call it partially unordered because unlike se-
quence strings, in which all the entities involved are
ordered with respect to one another (B before A before
D), some of the entities are not ordered with respect to
one another (F might be before G or after G, as long as
both are after D). We can symbolize the partially unor-
dered sequence string by using two slashes to indicate
not only that the order of F relative to G is unknown,
but also that they may or may not depart one right
after the other (like the slash for the unordered
block):

D——–F//G

This symbolization means that F departs some-


time after D, and G departs sometime after D, but that
G might be before F or after F, and either immediately
so or not.
Here are some examples of conditions that would
give us partially unordered sequence strings:

Flights F and G depart after flight D.


Both Adam’s and Raja’s interviews take place
sometime before Cory’s interview.
Mona sits to the left of both Lillja and Justin.

Joint Sequence Strings


Perhaps the most important element of many sequenc-
ing games is the joint sequence string. These are not
given to you in the restrictions, but are rather produced
–se QueNciNg games reView–

by you as you piece together the information It is crucial for sequencing games to
given in different restrictions. In particular, if you understand what this symbolization means, or to
have two sep- arate sequence strings and the two come up with your own method of symbolizing
strings have at least one element in common, then how to join differ- ent relative sequences (B before
you can join them. A before D before C, and E before D). This
For example, suppose you are given the symbolization indicates that E occurs to the left
infor- mation that flight A departs after flight B, of D (before D). The arrow helps to remind us
but before flight D: that it’s not the case that E could only come in
between A and D, but rather that E could occur
B——–A——–D anywhere to the left of D. E could occur first
(furthest left, indicating that E departs first), or in
And suppose you are given the information be- tween B and A, or in between A and D. It
that flight C departs after flight D: just could not occur after D.
Although we use this joint sequence
D——–C string symbolizing technique throughout the
rest of this book, it is only a suggestion. Many
Since the two strings have the entity D in find it intuitive to use vertical lines and arrows
com- mon, you can join them at D: (as we do here), but some find it more intuitive to
use greater than and less than signs, or
B——–A——–D——–C diagonal lines (see figure 1). If the technique
presented here is not intuitive for you, try your
This combines two pieces of information into own method, but make sure that it doesn’t
one, and is far more helpful than the two confuse you into thinking that elements are
disjointed strings. more restricted than they actually are, like
This is a relatively simple string joining; there thinking that E must occur in between A and
are more complicated connections that might be D, rather than any- where to the left of D.
made as well. For example, suppose we are also
told that flight E departs before flight D:

E——–D

Then we again have a point of connection


D, but that point of connection is embedded in the Let’s try a more complicated example now, along with
first string, so we can’t simply produce a a more involved symbolization.
longer, linear string as we did before. Instead, we
can draw a line down from the point of Eight flights—A, B, C, D, E, F, G, and H—
connection, and an arrow with the other entity depart one at a time, according to the fol-
involved, indicating that E must de- part lowing restrictions:
sometime before D:
Flight A departs after flight B, but before
flight D.
B——–A——–D——–C
‹——–E——–|
Flight C departs after flight D.
Flight E departs before flight D.
Flights F and G depart after flight D.
Flight H departs after flight E.
43
–se QueNciNg games reView–

The symbolizations for the restrictions taken in


iso- lation are: B———A———D———C
‹———E———|—F//G–›
Flight A departs after flight B, but
before flight D. What about the fifth restriction, that H is
after E? We can again join this to the
B———A———D
symbolization we have by drawing a line under E,
with H in an arrow point- ing to the right, as
Flight C departs after flight D. follows:

D———C
B———A———D———C
‹———E———|—F//G–›
Flight E departs before flight D. |—H——————›

E———D
This simply means that H occurs somewhere
to the right of E.
Flights F and G depart after flight D. To get a sense of how these symbolizations
help with sequencing games, let’s take the following
D———F//G
question:

Flight H departs after flight E. If flight H departs before flight B, then which
one of the following must be true?
E———H
a. E departs first
b. E departs second
Now, how do we join these sequence strings? c. F departs sixth
We already joined the first two: d. G departs sixth
e. C departs eighth
B———A———D———C
The symbolization shows us that E must
We can also join the third and the fourth be to the left of H. If H is to the left of B, then E
restric- tions (the fourth being a partially unordered must be to the left of B, because H departs after E.
sequence string, discussed above): That is, we can imagine shifting H to the left of B,
forcing E to move left of B as well, and E stays to
E———D———F//G the left of H).

Now, we can join these two sequence strings B——A——D——C E


at the point of connection—that is, by D, the entity ———————–|—F//G–›
that both have in common. To do this, we use |—H–›
one string as a primary string, and draw a line
under the point of connection D, from which we So now we can symbolize the ordering as: E———
draw the remaining elements of the secondary
string. H———B———A———D———C
|—F//G–›

Since the only ‘unsettled’ elements are F and


G, which are somewhere to the right of D,
everything to

44
–se QueNciNg games reView–

the left of D is settled—, and in particular, E must The important point to keep in mind is
de- part first. The correct answer, then, is A—, that that these various players can all be integrated
E de- parts first. together. Very rarely, the symbolizations will not
We might have also arrived at this answer easily allow for integration; in these instances, you
with- out having first produced the master joint must adapt your symbolization to the
sequence string by joining sequence strings given by circumstance to make sure you capture the
the restric- tions as needed. That is, we could note information in one place, or be content to leave
that H depart- ing before B joins the string given the information visually separated and try to
by the first and fifth restrictions, and then join the mentally integrate it or reason with it as
other strings. The point of the initial, master joint needed.
sequence symbolization, however, is to compile Finally, a note about answering sequencing
all the information we can into one diagram, so ques- tions: some questions will introduce new
that we can just refer to that one dia- gram as we sequencing information. Often, you will want to
work through all the questions, rather than modify your se- quencing diagram—or create a
construct a new diagram from scratch for every new diagram using the original one as a reference
ques- tion and have to check each restriction —to incorporate new information. This will
individually. usually mean creating a new primary string. You
Two more points should be made about will gain practice with this tech- nique by
sequence strings in general. First, ordered blocks working through the practice logic games.
can be ele- ments on a joint sequence string. For
example, sup- pose we are told that: Floaters
The final major common player to pay attention to
Flight A departs immediately after Flight B in sequence games is the floater. Floaters are entities
Flight A departs after flight D. that have absolutely no restrictions on them. That
is, they do not appear in any of the restrictions,
Then we have an ordered block (BA) and a and also do not have restrictions placed on them
se- quence string (D———A) which can be indirectly. Always keep in mind which entities can
integrated into a joint sequence string: float into any position when attempting to
sequence.
D———BA
A Final Note about Sequencing Games
Second, unordered blocks can be elements It is important to realize that sequencing games
on a joint sequence string. Suppose we are told will not always allow for the creation of the
that: players we just described, or master joint
sequence strings, or any sequence strings at all up
Flight A either departs immediately before front. Some sequenc- ing games—especially ones
flight B or immediately after flight B. that are heavy on condi- tional statements (e.g., ‘If
Flight A departs after flight D. Wells goes first, then Anand is third’)—will not
lend themselves to such techniques. We have
Then we have an unordered block (A/B) and discussed these techniques because they are
a sequence string (D———A) which can be unique to sequencing games.
integrated into a joint sequence string:

D———A/B
45
–se QueNciNg games reView–

practice game 1

Seven jobs—painting, window washing, caulking, landscaping, insulating, roofing, and brick-work—are to be
performed to improve a house, one at a time, according to the following conditions:

The window washing happens before the painting.


The caulking happens after the insulating, but before the window
washing. Both the roofing and landscaping happen after the
caulking.
The brickwork happens before the roofing.

1. Which one of the following could be the d. Fifth


order of the jobs, from first to last? e. Sixth
a. Landscaping, insulating, caulking, brick-
work, window washing, painting,
roofing
b. Brickwork, insulating, painting,
caulking, landscaping, window
washing, roofing
c. Insulating, brickwork, caulking,
roofing, window washing, landscaping,
painting
d. Brickwork, caulking, landscaping,
roofing, window washing, insulating,
painting
e. Insulating, caulking, landscaping,
roofing, window washing, brickwork,
painting

2. Which one of the following is a complete


and accurate list of the last jobs
performed?
a. Painting
b. Painting, roofing
c. Roofing, landscaping
d. Roofing, landscaping, window washing
e. Painting, roofing, landscaping

3. If the brickwork happens after the


window washing, then which one of the
following is the earliest that the roofing
could occur?
a. Second
b. Third
c. Fourth
4. If the roofing happens fourth, then which one
of the following is the least number of 5. Which one of the following is a complete
tasks separating the brickwork from the and accurate list of the jobs any one of
painting? which could be the second task
a. One performed?
b. Two a. Caulking
c. Three b. Brickwork, caulking
d. Four c. Insulating, brickwork, caulking
e. Five d. Insulating, brickwork, roofing
e. Insulating, caulking, brickwork, landscaping

46
–se QueNciNg games reView–

practice game 2

A job applicant has to interview with a company’s top six executives—Emmett, Fernandez, Garg, Hinton, Isaac,
and Jimenez—one at a time, according to the following conditions:

Emmett is either the first or second person to interview the applicant.


If Garg is the second to interview the applicant, then Jimenez is the fourth to interview the
applicant. If Hinton is the sixth to interview the applicant, then Garg is either the second or third
person to inter- view the applicant.
Fernandez interviews the applicant immediately after Garg interviews the applicant.

1. Which one of the following could be the d. Hinton interviews the


sequence in which the executives interview applicant fifth.
the applicant, from first to last? e. Isaac interviews the applicant
a. Emmett, Hinton, Garg, Fernandez, sixth.
Jimenez,
Isaac
b. Garg, Fernandez, Hinton, Emmett,
Isaac,
Jimenez
c. Emmett, Jimenez, Isaac, Garg, Fernandez,
Hinton
d. Emmett, Fernandez, Jimenez, Garg,
Hinton,
Isaac
e. Emmett, Garg, Fernandez, Isaac, Jimenez,
Hinton

2. Which one of the following statements


CANNOT be true?
a. Isaac interviews the applicant first.
b. Hinton interviews the applicant second.
c. Fernandez interviews the applicant
second.
d. Jimenez interviews the applicant fifth.
e. Hinton interviews the applicant sixth.

3. If Isaac interviews the applicant after


Fernandez interviews the applicant, and
exactly two execu- tives interview the
applicant in between Isaac and Fernandez,
then which one of the following statements
CANNOT be true?
a. Emmett interviews the applicant first.
b. Garg interviews the applicant second.
c. Jimenez interviews the applicant third.
4. If Emmett interviews the applicant either immediately
before or immediately after Garg interviews the
applicant, then which one of the following is a complete
and accurate list of the executives any one of which
could be the fourth executive to interview the
applicant?
a. Isaac
b. Jimenez
c. Fernandez, Hinton
d. Fernandez, Jimenez
e. Fernandez, Hinton, Isaac, Jimenez

5. If Hinton is the last executive to interview the applicant,


then which one of the following is a complete and
accurate list of the spots in which Isaac could
interview the applicant?
a. Fifth
b. First, second
c. First, second, fifth
d. First, third, fifth
e. First, second, fourth, fifth

47
–se QueNciNg games reView–

practice game 3

Six friends—Alan, David, Marcello, Pedro, Rahul, and Sanjay—attend a baseball game. They sit in
Section 5, Row H, seats 1 to 6. Seat 1 is on the left, and the seats increase in number as they go to the
right. The friends sit according to the following restrictions:

Pedro does not sit immediately next to Sanjay.


Rahul sits either immediately to the left or immediately to the right of
Sanjay. Marcello sits to the left of David.
Both Alan and Pedro sit to the left of
Marcello. Rahul sits to the right of Pedro.

1. Which one of the following could be the 4. If Rahul sits in seat 2, then for how many of
order in which the friends sit, from left to the six seats is the occupant determined?
right? a. Two
a. Alan, Marcello, Pedro, Rahul, Sanjay, b. Three
David c. Four
b. Alan, Rahul, Pedro, Marcello, Sanjay, David d. Five
c. Alan, David, Pedro, Marcello, Sanjay, Rahul e. Six
d. Alan, Pedro, Rahul, Sanjay, Marcello, David
e. Alan, Sanjay, Rahul, Pedro, Marcello, David 5. If Sanjay sits to the left of Marcello, then
which one of the following is a complete and
2. Which one of the following is a complete accurate list of the friends any one of which
and accurate list of the friends any one of could be the friend sitting in seat 4?
which could sit in seat 1? a. Alan
a. Alan b. Alan, Sanjay
b. Marcello c. Rahul, Sanjay
c. Alan, Pedro d. Rahul, Sanjay, Marcello
d. Sanjay, Pedro e. Sanjay, Rahul, Alan
e. Alan, Pedro, Sanjay

3. Which one of the following is a complete


and accurate list of the seats any one of
which could be the seat in which Sanjay
sits?
a. Seat 3, seat 5
b. Seat 4, seat 5, seat 6
c. Seat 2, seat 4, seat 5, seat 6
d. Seat 3, seat 4, seat 5, seat 6
e. Seat 2, seat 3, seat 4, seat 5, seat 6
48
–se QueNciNg games reView–

practice game 4

An admissions committee ranks six students—A, B, C, D, E, and F—for further consideration. The
highest- ranked student ranks first. The ranking is consistent with the following conditions:

Student A is ranked the second highest.


Both student C and student E are ranked higher than B.
Student D and student F have consecutive ranks, not necessarily in that order.

1. Which one of the following could be the 4. Which one of the following is a complete
rank- ing of students, from lowest to and accurate list of all the students any one
highest? of which could be ranked fourth?
a. B, A, E, D, F, C a. Student D, student F
b. B, C, F, D, A, E b. Student C, student E
c. B, D, E, C, A, F c. Student C, student D, student F
d. C, D, F, B, A, E d. Student B, student D, student F
e. E, F, D, C, A, B e. Student B, student C, student D, student
F
2. What which one of the following is a
complete and accurate list of the ranks any 5. If student E is ranked third, then which one
one of which could be student B’s rank? of the following statements CANNOT be
a. Sixth true?
b. Third, fifth a. Student D is ranked sixth.
c. Fourth, sixth b. Student C is ranked fifth.
d. Second, fifth, sixth c. Student F is ranked fifth.
e. Fourth, fifth, sixth d. Student F is ranked fourth.
e. Student B is ranked fourth.
3. If student D is ranked higher than student
C, then which one of the following
statements could be true and which one
could be false?
a. Student B is ranked first.
b. Student C is ranked second.
c. Student D is ranked third.
d. Student A is ranked fourth.
e. Student E is ranked fifth.
49
–se QueNciNg games reView–

practice game 5

Seven songs—H,I, J, K, L, M and N—are selected on a jukebox. They are played one at a time, according to the
following conditions:

H is played second.
J is either played third or played
fifth. K is played before J.
N is either played immediate after M or immediately
before L. At least one song is played in between song J
and song K.

1. Which one of the following could be a 4. If song N is played third, then the sixth song
list of the songs played in order from first to be played must be:
to last? a. Either song L or song M.
a. K, L, H, I, J, M, N b. Either song M or song I.
b. M, H, N, K, J, L, I c. Either song L or song I.
c. L, H, J, M, N, K, I d. Either song M or song K.
d. K, H, I, J, N, L, M e. Either song L or song K.
e. K, H, M, N, J, I, L
5. If exactly three songs are played in
2. Song K could be played: between song L and song J, then which
a. Third one of the fol- lowing is a complete and
b. Fourth accurate list of all the songs that could be
c. Fifth played fifth?
d. Sixth a. Song N
e. Seventh b. Song M, song J
c. Song M, song N, song J
3. If song K is played after song H is played, d. Song M, song N, song I
then which one of the following songs e. Song M, song N, song J, song I
must be played after J is played?
a. Song I
b. Song K
c. Song L
d. Song M
e. Song N
50
–se QueNciNg games reView–

practice game 6

An outdoors club picks seven checkpoints for an orienteering course. The checkpoints—L, M, N, O, P, Q, R
—are to be arranged in order according to the following conditions:

Checkpoints M and P both occur before checkpoint


R. Checkpoint P occurs before checkpoint Q.
Checkpoint O occurs before checkpoint M.
Either checkpoint R occurs immediately after checkpoint L, or checkpoint L occurs immediately after
checkpoint R.
Checkpoint P is not the first checkpoint.

1. Which one of the following could be a 4. If checkpoint O is the fourth checkpoint,


list of the checkpoints, in order from first then each of the following statements must
to last? be true EXCEPT:
a. M, N, O, P, L, R, Q a. Checkpoint N is first.
b. N, O, P, M, Q, R, L b. Checkpoint P is second
c. N, Q, O, P, M, L, R c. Checkpoint Q is third.
d. O, N, M, R, L, Q, P d. Checkpoint M is fifth.
e. O, P, M, Q, R, N, L e. Checkpoint L is sixth.

2. Which one of the following checkpoints 5. If checkpoint M occurs before checkpoint


CAN- NOT be the third checkpoint? P, then what is the earliest checkpoint Q
a. Checkpoint M could appear in the course?
b. Checkpoint N a. First
c. Checkpoint P b. Second
d. Checkpoint Q c. Third
e. Checkpoint R d. Fourth
e. Fifth
3. Which one of the following statements
CAN- NOT be true?
a. Checkpoint Q is second.
b. Checkpoint O is second.
c. Checkpoint M is second.
d. Checkpoint N is third.
e. Checkpoint M is third.

51
–se QueNciNg games reView–

practice game 7

Seven parts—Q, R, S, T, U, V, and X—are added to a car chassis on a Mustang assembly line, one at a time.
The order in which they are added is consistent with the following conditions:

The sixth part to be added is either part Q or part S.


If part T is added before part S, then part U is added
second. Part U is added immediately after part V.
Part V is added before both part T and part X.
Part R must be added either immediately before or immediately after part S.

1. Which one of the following could be a list 4. If part R is added last, then which one of
of the parts in the order in which they are the following statements must be true?
added? a. Part V is added first.
a. V, U, T, X, Q, R, S b. Part Q is added third.
b. V, U, S, T, R, Q, X c. Part T is added fourth.
c. U, V, S, R, X, Q, T d. Part X is added fifth.
d. Q, V, U, T, R, S, X e. Part T is added fifth.
e. V, U, X, S, R, Q, T
5. If part T is added after part X but before
2. If part Q is added last, then each of the part S, then which one of the following is a
follow- ing statements must be true complete and accurate list of all the parts
EXCEPT: any one of which could be added fifth in the
a. V is added first. assembly line?
b. U is added second. a. part Q
c. T is added third. b. part R, part Q
d. R is added fifth. c. part S, part T
e. S is added sixth. d. part Q, part R, part X
e. part Q, part R, part T
3. Each of the following parts could be added
last EXCEPT:
a. Part Q
b. Part R
c. Part S
d. Part T
e. Part X
52
–se QueNciNg games reView–

practice game 8

Six speakers—A, B, C, D, E, and F—are to speak at a rally, one at a time. Their line-up is determined by
the fol- lowing conditions:

D speaks either second or


third. A speaks before C.
C either speaks immediately before or immediately after E.
F speaks after E.
B does not speak immediately before or immediately after F.

1. Which one of the following could be 4. If speaker B immediately precedes speaker


the speaker list, in order from first C in the line-up, then how many people
to last? speak in between speaker A and speaker
a. B, D, A,C, E, F F?
b. D, B, A, C, E, F a. Zero
c. C, D, E, F, A, B b. One
d. A, F, D, E, C, B c. Two
e. E, C, D, F, A, B d. Three
e. Four
2. Which one of the following is a complete
and accurate list of the spot speaker E could 5. Which one of the following is a complete
have in the line-up? and accurate list of the speakers any one of
a. Third spot, fourth spot which could speak second?
b. Fourth spot, fifth spot a. B
c. Fifth spot, sixth spot b. A, B
d. Fourth spot, fifth spot, sixth spot c. A, B, D
e. Third spot, fourth spot, fifth spot d. B, D, C
e. A, D, E
3. If there is exactly one speaker in between
speaker A and speaker D, then all of the
follow- ing statements must be true
EXCEPT:
a. A speaks first.
b. B speaks second.
c. D speaks third.
d. C speaks fourth.
e. F speaks sixth.
53
–se QueNciNg games reView–

setup, answers, and


explanations Question 1
The correct answer is choice c. This question is
a “Test-the-Rules” question. The first restriction
Practice Game 1 rules out choice b, since the window washing
All the restrictions in this game provide some sort occurs after the painting. The second restriction
of sequence string. rules out choice d, since both the caulking and the
window washing occur before the insulating. The
The first restriction: W———P third restriction rules out choice a, since the
The second restriction: I———C——— landscaping occurs before the caulking. The
W The third restriction: C———R//L (Note fourth restriction rules out choice e, since the
that this is a partially unordered sequence roofing occurs before the brickwork.
string, so the unknown order of R
relative to L is symbolized with a Question 2
double-dash.) The correct answer is choice e. Consult the sequence
The fourth restriction: B———R dia- gram to see which entities could be the right-
most on the diagram. P does not need to have
Now try to combine these sequence strings anything to its right. R and L are on an arrow
into a joint sequence string. The first and second pointing to the right, and are not ordered with
strings can be straightforwardly combined into: respect to one another, so either of those could be
the right-most entity as well. So each of P, R, and
I———C———W———P L could be the last job performed—this is choice e.

Combine the third string with this string Question 3


by joining them at the common entity C: The correct answer is choice d. Construct a new
se- quence diagram with the new information
I—–C—–W—–P that W—–B. You will want to create a new
|—–R//L–› string with W and B integrated, so the primary
string will now be:
Now combine the fourth string with this
string by joining the two at the common entity I—–C—–W—–B—–P
R:
The rules tell us that roofing must occur after
I—–C—–W—–P the caulking and brickwork. The L and R will
|—–R//L–› have to be separated and added as secondary
‹–B—| strings:

This will be the larger joint sequence string I———C———W———B———P


that will provide a jumping-off point for
answering the questions. |—R——›

|–—L—————————›

Although L must occur after C, there is no


re- striction that keeps R from occurring before L.
So now we can see that the left-most spot for R
could be fifth— choice d.

54
–se QueNciNg games reView–

Question 4 Practice Game 2


The correct answer is choice b. Construct a new The first thing to do is to symbolize the
se- quencing diagram that is based off the initial restrictions, and see if any deductions or joint
diagram and the information that the roofing sequence strings can be made from them. It will
occurs fourth. The roofing can happen fourth if turn out that there won’t be any master sequence
exactly three jobs happen before it (three entities string, and this is largely because the restrictions of
are to the left of R on the diagram). By consulting this sequencing game are largely conditional
the initial sequencing dia- gram, we see that those statements.
three jobs must be I, C, and B. L, W, and P must The first restriction can be symbolized with a
then be pushed to the right of R. We can then diagram of six slots:
create a new primary string relating I, C, R, W,
and P, leaving B and L as secondary strings: dES

I—C—R—W—P 123456
‹—B—|—L—›
The second restriction can be symbolized:
We might just as easily have created a
primary string relating I, C, R, and L, leaving W G = 2 S J = 4 [and the contrapositive, J ≠ 4
—–P and B as secondary strings: S G ≠ 2]

I—C—R—L The third restriction can be symbolized:


‹—B—|—W—P—›
H = 6 S G = 2 or 3 [and the
B must be to the left of R, so the closest we can get contrapositive, (G ≠ 2 and G ≠ 3) S H
it to the right side is just to the left of R. To ≠6 ]
ensure that P is as far left as possible, we push L
as far right as possible, past P. We then have two The fourth restriction gives us an ordered
entities in between B and P, namely, R and W.
(The sequence looks like: I-C-B-R- W-P-L.) So the block: GF
correct answer is b.
Although we don’t have a sequence string to
Question 5 work with, a couple of observations can be made.
The correct answer is choice c. Consult the One, the GF block entails that F cannot be first.
sequenc- ing diagram to see which could be the Two, if F were second, then the GF block would
second task performed. Consider each entity in take up spots 1 and 2, leaving no room for E; so F
turn. I could be second if B was pushed to its left. cannot be second. Three, the GF block means that
C could be second if B was to the right of C. B G cannot be last (sixth). Four, I has no restrictions
could be second if it fell between I and C. on it, and so is a floater.
Neither W nor P could be second, as they are We can capture most of this information in
preceded by two and three entities, respec- tively. the diagram:
Neither R nor L could be second, since R is pre-
ceded by at least three entities (I, C, and B), and dES
L is
preceded by at least two entities (I and C). So I, C, and
B could be second, and no others could be second 1 2 3 4 56
— this is answer choice c. F F G
55
–se QueNciNg games reView–

Question 1 Now check each answer choice against this


The correct answer is choice a. This question is dia- gram. The diagram makes choices a, b, d,
a “Test-the-Rules” question. The first restriction and e all true. Choice c is the correct answer
rules out b, since Emmett is neither first nor because according to the diagram, Jimenez does
second. The second restriction rules out e, since not interview the appli- cant third, but fourth.
Garg is second but Jimenez is not fourth. The third
restriction rules out c, since Hinton is sixth, but Question 4
Garg is not second or third. The fourth restriction
The correct answer is choice d. First, assess what
rules out d, since Fernandez does not go directly
the new information means. The new restriction
after Garg.
that Emmett interviews the applicant either
immediately before or immediately after Garg
Question 2 amounts to an un- ordered block, E/G. We can
The correct answer is choice c. Check to see if combine this block with the ordered block GF to
any of our up-front observations are being stated. create the ordered block EGF. Since E must either
Choice c states that F cannot be second, which we be in the first spot or second spot, we get two
noted in the up-front observations, and so this is possible scenarios:
the correct an- swer. Notice that choices b and d
represent situations that were already determined S1
to be possible in the an- swer to question 1 (in
E G F
choice a of question 1, Hinton is second and
123456
Jimenez is fifth).
S2
Question 3
The correct answer is choice c. If I goes three spots E G F (I, H, J)
after F, then the fact that there are only six spots 123456
lim- its the position of F to either the first, second,
or third spot. But we know from the up-front We write (I, H, J) in the second scenario to
observations that F cannot be in the first or remind ourselves that these three entities are to be
second spot. So F must be in the third spot, and distributed into the remaining slots (and don’t
I in the sixth spot: write this in the first scenario because we are about
to distribute them). In the first scenario, G is
dES second, so J must be fourth, leaving I and H to
F I be distributed into slots 5 and 6:
1 2 3 4 5 6
F F G S1
E G F J I/H
But our ordered block ‘GF’ indicates that G 12345 6
must be in spot 2. And that forces E into spot 1.
By the sec- ond restriction, since G is in spot 2, J Now we consult these two diagrams to see
must in be in spot 4, leaving H for spot 5: which entities could be in the fourth spot. In S1,
J is in the fourth spot. In S2, F is in the fourth
E G F J H I spot. So the list of entities that could be in the
123456 fourth spot includes Jimenez and Fernandez, and
no other entities—that is, choice d.
56
–se QueNciNg games reView–

Question 5 The fifth restriction gives us a sequence


The correct answer is choice c. If H is sixth, then
G must be in spot 2 or spot 3. Consider each string: P—–R
option as a separate scenario.
Scenario 1: If G is in spot 2, then J is in spot
We can combine the sequence strings making
4. And since G is in spot 2, E is forced into spot
up the third and fourth restrictions:
1. And since G is in spot 2, F must be in spot 3.
This leaves I for spot 5. So we have:
A//P—–M—–D

E G F J I H
We can add in the fifth restriction as a
123456
second- ary string:

Scenario 2: If G is in spot 3, then F must be


A//P—–M—–D
in spot 4. So we have:
|—R–›
dES
G F H
Finally, we can add the replace R with the
123456
unor- dered block from the second restriction:

So where can Isaac go? In Scenario 1, Isaac


A//P—–M—–D
is in spot 5. In Scenario 2, Isaac can go into
spots 1, 2 and
|—S/R–›
5. So Isaac could go first, second or fifth, and
couldn’t go in any other spots—choice c.
This sequence diagram captures all the informa-
tion in the second through fifth restrictions. The
Practice Game 3
only other piece of information to keep in mind is
The first restriction gives us a restricted
the first restriction, the restricted block.
block: P/S
Question 1
The correct answer is choice d. This question is
The second restriction gives us an unordered
a “Test-the-Rules” question. The second restriction
block:
rules out choice b, since Sanjay is not sitting
immediately next to Rahul. The third restriction
S/R
rules out choice c, since Marcello is not to the left
of David. The fourth restriction rules out choice
The third restriction gives us a sequence
a, since Pedro is not to the left of Marcello. The
fifth restriction rules out choice e, since Rahul is
string: M—–D
not to the right of Pedro.

The fourth restriction gives us a partially or-


Question 2
dered sequence string:
The correct answer is choice c. Consult the
sequence diagram to see who could sit in seat 1—
A//P—–M
that is, who could be the left-most in the
diagram. Only A and
57
–se QueNciNg games reView–

P, since they are unordered relative to one Question 5


another, could be first. S and R must be to the The correct answer is choice e. Consult the
right of P, and so must M and D. So the correct diagram to see what happens with the new
answer includes Alan and Pedro and no others, information that
choice c. Double-check that the first restriction,
which is not captured in the di- agram, doesn’t
rule out either of these options—it doesn’t since
it only says that P and S cannot be ad- jacent,
which could be the case whether A is first or
whether P is first (just place S to the right of,
say, M).

Question 3
The correct answer is choice d. We want all the
seats that Sanjay could be in. Consult the diagram
and see where S can go, considering each seat in
turn. S can’t be in 1, because at the very least P
will be to his left. Can S be in 2? If S is in 2, then
P must be in 1, since P must be to S’s left. But
then we will violate the re- strictive block,
prohibiting P from being immediately next to S. So S
can’t be in seat 2. So we have eliminated choices c
and e. What about seat 3? S could be 3rd, with
P-A to his left. What about seat 4? S could be
4th, with A-P-R to his left. S could be 5th, with
A-P- M-R to his left. S could be sixth, with A-P-
M-D-R to his left. So S could be in seats 3, 4, 5
and 6—choice d.

Question 4
The correct answer is choice e. Consult the diagram
to see what happens when Rahul is in seat 2. If R is
in 2, then P must be in 1 (since P must be to the
left of R). S must then be in 3, to maintain the S/R
unordered block. This leaves A, M, and D, for seats
4 through 6—which must be in that order,
according to the diagram. So we have:

PRSAM
D12345
6

So every one of the six seats has a


determinate occupant, and the answer is choice
e.
S—–M. Since S/R is a block, this simply cuts off the S1: A-P-R-S-M-D
arrow allowing S/R to be moved to the right:
S2 can be subdivided yet again, with the
A//P——M—–D S/R block in between P and A, or in between A
|–S/R–| and M:

To see more concretely what this means, divide S2.1: P—–S/R—–A-M-D


up this sequencing diagram into two scenarios, based on
the unordered sequence pair A//P. In one scenario, A S2.2: P-A—–S/R—–M-D
will be the left of P, and in another scenario, A will be
to the right of P. Note again that S2.1 is restricted by the
restricted block preventing P and S from
S1: A—–P——–M—–D immediately adjacent, and so just becomes: P-R-
|–S/R–| S-A-M-D.

S2: P—A—––M—–D To summarize, our three scenarios


|——S/R—–|
are: S1: A-P-R-S-M-D
Note that S1 really allows for only one possibility,
since P and S cannot be next to one another: S2.1: P-R-S-A-M-D

S2.2: P-A—–S/R—–M-D

58
–se QueNciNg games reView–

Now look at seat 4 for each. In S1, S sits in


seat 4. In S2.1, A sits in seat 4. In S2.2, S or R B—–C//E
could sit in seat
4. So seat 4 could be occupied by S, R, or A, and by The third restriction gives us an unordered se-
no other entity. This is choice e. quence block:
This is admittedly a fairly complicated way
to answer this question. Another approach would D/F
be to use the information you have gained from
previous questions. You can see from the answer Since there are no entities in common, the
to Question 1 that S has to be on the list, which strings cannot be joined. We can make a couple
rules out choice immediate observations, however. B cannot be
a. You can see from the diagram produced to first or third (be- cause they would be no room for
answer Question 4 that A has to be on the list, C and E to its right), and neither C nor E can be
which rules out choice c. You can rule out any sixth. We could proceed to the questions, or we
answer choice which includes M since at least four could quickly see if breaking down B’s position
letters have to come be- fore M (we know A//P into three scenarios (sixth, fifth, and fourth) will
must precede M from the initial joint sequence be useful. Again, be cautious about sinking too
string, and given the question stem, S/R must also much time into creating scenarios, espe- cially if
precede M)—that is, we can rule out choice you’re looking to produce three (rather than two)
d. This leaves only choices b and e, so you just scenarios. In general, if you find that scenarios are
have to check whether R could be fourth without getting too confusing, or eating too much of your
violating any of the restrictions. time, just proceed to the questions and figure out
sce- narios as needed. In this set of answer
Practice Game 4 explanations, we make use of the three scenarios
This game asks for a ranking of six students—and that depend on B’s placement:
the first question asks for the ranking from lowest
to high- est—so draw six slots, labeled 1 to 6, with S1
6 on the left and 1 on the right. (If you started by B A
drawing the slots in reverse order, you would 654321
simply have to flip your diagram once you got to
the first question and real- ized it would be easier S2
to draw it in the opposite order for the sake of
that question).
BA
654321
654321
S3
The first restriction can be directly entered into B A
the diagram: 654321

A 6 In S3, C and E must fill spots 3 and 1,


54321 leaving D and F to fill spots 6 and 5:

The second restriction gives us a partially S3


unor- dered sequence string: D/F B C/E A E/C
6 5 4 3 2 1
59
–se QueNciNg games reView–

(Keep in mind that the slash between E and E are ranked lower than B. The third restriction
C in spots 3 and 1 do not indicate an unordered rules out choice c, since D is not adjacent to F.
block, as it does for D/F, but rather the two
possibilities for spot 3 and the two possibilities Question 2
for spot 1.) The correct answer is choice c. Consult the three
For S2, the unordered block D/F can only se- quence diagrams to see where B could go. In
fit in the two-spot block consisting of spots 4 and S1.1 and S1.2, B is sixth. In S3, B is fourth. Since
3; leaving E and C to be distributed between spots these exhaust our scenarios, B could be fourth or
6 and 1. But this would violate our sequence string
sixth, and no other rank—choice c.
B—–C//E. So S2 is not a possibility, and B cannot
be fifth Question 3
For S1, this is not an issue, since B is in The correct answer is choice c. Start with the new
spot 6. There are two possibilities for placing the
in- formation that C—–D. The only scenario in
D/F block, each way leaving C and E for the
which C can appear before D is S1.2.
remaining spots:
S1.2
S1.1
B C/E D/F A C/E
6 5 43 2 1
B D/F C/E A C/E
6 54 3 2 1
So C must appear in spot 5, forcing E into spot 1:

S1.2
B C D/F A E
B C/E D/F A C/E
65432 1
6 5 4 3 2 1
Now we consider each answer choice, and
So our three scenarios are:
see whether the statement could be both true and
false. Choice a is incorrect because B is in fact
S1.1 ranked sixth— it can’t be true that B is ranked
B D/F C/E A C/E first. Choice b is incor- rect because C is in fact
6 5 4 3 2 1 ranked fifth—it can’t be true that C is ranked
second. Choice c is correct because D could be
S1.2 ranked third; but it could also be ranked first, such
B C/E D/F A C/E that it would be false that D is ranked third.
6 5 4 3 2 1 Choice d is incorrect because A is in fact ranked
sec- ond, so it couldn’t be the case that it’s ranked
S3 fourth. Choice e is incorrect because E is in fact
D/F B C/E A E/C ranked first, so it couldn’t be the case that it’s
6 5 4 3 2 1 ranked fifth.

Question 1 Question 4
The correct answer is choice b. This question is The correct answer is choice d. Consult the
a “Test-the-Rules” question. The first restriction diagram to see which students could be in spot
rules out choice a, since A is not ranked second 4. In S1.1 and S1.2, students D and F could be in
highest. The second restriction rules out choices d spot 4. In S3, B is in spot 4. So B, D, and F are
and e, since in d, C is ranked lower than B, and the students that could
in choice e, both C and
60
–se QueNciNg games reView–

be in spot 4, and no other students could be—choice S1


d. Also, you know from question 1 that the list has to HJ
include F, and you know from question 2 that the 1234567
list has to include B, so this narrows the options
down to choices d and e. The only difference S2
between the
choices is C, and if you try to put C fourth you will see HJ
that it doesn’t work. 1234567

Question 5 The third restriction places K before J. For


The correct answer is choice b. If E is ranked now, symbolize it as:
third, then we must be in scenario S1.1 or S3. In
either case, C is forced into spot 1: K—–J

S1.1 The fourth restriction says that there must


B D/F E A C be one of two ordered blocks:
654321
MN or NL
S3
D/F B E A C The fifth restriction gives a numbered
65432 1 sequence block:

Now check each answer choice against these J/_+K


two diagrams, looking for a statement that is not
accom- modated by one or both of these diagrams. Remember that the slash indicates the
Choice a is incorrect because D is ranked sixth in positions of J and K can be reversed, and the one
S3. Choice b is correct because C is not ranked space and plus sign that at least one spot must be
fifth in either sce- nario. Choices c and d are in between. Since we already have a restriction
incorrect because F could be ranked fifth or fourth ordering J and K, we can combine the two to
in S1.1. Choice e is incorrect because B is ranked produce:
fourth in S3.
K_+J
Practice Game 5
Start with a seven slots, with the first song at the This indicates that there must be at least
left and seventh song at the right: one space in the sequence string K—–J.

Now see how the third through fifth


1234567 restrictions might play into the two scenarios. In
S1, since K must precede J, we have:
Now incorporate and symbolize the
restrictions. Put H in slot 2. Since H must either S1
be third or fifth, see if creating two scenarios will K H J
be useful, one with J third, and one with J fifth: 1234567

61
–se QueNciNg games reView–

In S2, K must precede J, but must have at Question 1


least one space separating K from J; so K must The correct answer is choice e. This question is
either be first or third. So subdivide S2: a “Test-the-Rules” question. The first restriction
rules out choice a, since H is not second. The
S2.1 second re- striction rules out choice d, since J is
K H J neither third nor fifth. The third restriction rules
1234567 out choice c, since K is not before J. The fourth
restriction rules out choice b, since N does not
S2.2 immediately follow M, nor does L immediately
H K J follow N.
1234567
Question 2
These three scenarios, S1, S2.1, and S2.2, now The correct answer is choice a. Consult the
in- corporate all the information given in the first, diagrams to see when K could be played. K is
second, third, and fifth restrictions. So we can played first in S1 and S2.1, and third in S2.2. So the
ignore those re- strictions going forward, and only correct answer is a, third. It could be helpful to
keep in mind one restriction, that MN or NL. By note for future questions that K can’t be fourth,
considering the dia- grams spatially, we can fifth, sixth, or seventh.
observe that for S2.2, the slot block 6–7 must
contain either MN or NL: Question 3
The correct answer is choice e. If K follows H, then
S2.2 we must be in S2.2. According to the diagram for
(MN scenario 2.2, the space after J—namely, spots 6 and
H K J or NL 7—must be filled either by MN or NL. Either way,
1 2 3 4 5 67 N appears in that space. So choice e is correct.

So in summary, our three scenarios Question 4


The correct answer is choice b. If N is third,
are: S1 then we must be in S2.1 (since S1 has J in spot 3,
KHJ and S2.2 has K in spot 3). So we have:
1234567
S2.1
S2.1 K H N J
K H J 1234567
1234567
Now, we consider our restriction that MN or
S2.2 NL. Since MN can clearly not occur, we must
(MN have NL. That means that L is in spot 4,
H K J or NL leaving M and I for spots 6 and 7:
1 2 3 4 5 67
K H N L J M/I
1234 567
62
–se QueNciNg games reView–

Now look at the sixth spot. It could be filled S1.4


by M or I, and not by anything else. So b is the K H J M I N L
correct answer. 1 2 3 4 5 6 7

Question 5 So much for scenario 1. What about fitting


The correct answer is choice e. If there are the block L_/_ _J into scenarios 2.1 and 2.2?
exactly three songs in between L and J, we have a A brief glance at the diagram for 2.1 shows us that
numbered sequence block: this block cannot fit into that scenario.
For 2.2, L would have to be spot 1. That
L_/_ _J would force MN into spots 6 and 7, leaving I for
spot 4:
Consider the diagrams to see in scenarios
this block could fit, and in how many ways. In L H K I J M N
S1, since J is third, we would have to fit the 1234567
block J _ _ _ L into the scenario, and we would
have: Now, look to see which songs appear in spot 5. In
S1.1, it’s N, in S1.2 it’s M, in S1.3 it’s M, and in
S1 S1.5 it’s I. In S2.2, it’s J. So M, N, J and I, and
KH J L nothing else, can ap- pear in spot 5. That’s
1234567 choice e.
This is admittedly a complicated way to
We now have to accommodate MN or NL. If answer this question. You could also approach
we have MN, there are two ways to accommodate this question by first noting that J must either be
it, ei- ther in the 4–5 space, or 5–6 space, leaving third or fifth, and that each of these options will
I for the remaining space: determine where L has to be. If J is third, then L
must be seventh, and if J is fifth, then L must be
S1.1 second. So J can be fifth, and we can rule out any
K H J M N I L answer choice that does not include J—namely,
1234567 choices a and d. Now look at the remain- ing
answer choices: b, c, and e each include M, so we
S1.2 don’t have to worry about M. We only need to
K H J I M N L decide if N can be fifth and if I can be fifth to
1234567 determine which of the three remaining answer
choices is correct. You need to place N fifth, and
If we use NL, we must have N in spot 6, see if you can construct an acceptable scenario, in
leaving I and M to be distributed into spots 4 and which three spaces separate L and J. You can:
5, either with I in 4 and M in 5, or vice versa:
K H J M N I L
S1.3 1234567
K H J I M N L
1234567 Same with I:
K H J M I N L
1234567
63
–se QueNciNg games reView–

So the correct answer choice is e. The trick


to this approach is to use commonalities among M//P—–R/L
the an- swer choices (that M appears in b, c, and
e) to focus your efforts. You will still have to |—Q–›
construct scenarios (in which N is fifth and in
which I is fifth), but if you see that J’s presence Finally, add the third restriction (O—–M).
on the list rules out two answer choices, and that This is a bit tricky. If you add it directly to the
the remaining three have an element in common, primary string, you get:
then you might save some time over using the
initial scenarios. O—–M//P—–R/L

Practice Game 6 |—Q–›


First, symbolize the restrictions, and then see how
to put them together. As long as you are clear on what this
symboliza- tion means, go ahead and use it. In
The first restriction is a partially unordered particular, keep in mind that you could have
se- quence string: the following order: P—–O—–M--etc.
If, however, this symbolization is confusing,
M//P—–R mod- ify the primary string by making O—–M
part of the primary string (connected directly in
The second restriction: relative order with R), and add P on a secondary
string. We can do this be- cause M//P—–R simply
P—–Q tells us that both M and P are to the left of R, but
not about the order of M and P:
The third restriction:
O—–M—–R/L
O—–M
‹–P—|
The fourth restriction is an unordered
Of course, we need to keep Q attached to
block: R/L P, as yet another secondary string:

The fifth restriction: O—–M—–R/L

P≠1 ‹–P—|

First, consolidate the fourth and first restrictions: |—Q–›

M//P—–R/L This symbolization captures all the information


in the first through fourth restriction, so just keep
Next, add the second restriction (P—–Q) in mind the fifth restriction (that P ≠ 1) when
as a secondary string: consult- ing the diagram. Also keep in mind that
N is a floater, since there are no restrictions on it.

64
–se QueNciNg games reView–

Question 1 Question 3
The correct answer is choice b. This question is The correct answer is choice a. Consult the
a “Test-the-Rules” question. The first restriction diagram to see which letter cannot appear second.
rules out choice d, since P is not before R. The R and L could not appear second, since there are
second re- striction rules out choice c, since P is two letters before them—but neither R nor L
not before Q. The third restriction rules out appear in the answer choices. So take each answer
choice a, since O is not be- fore M. The fourth choice in turn. Could Q appear second? If Q
restriction rules out choice e, since R and L are were second, then according to the diagram, P
not adjacent to one another. would have to be first. But, keeping in mind the
fifth restriction, P cannot be first. So Q can-
Question 2 not be second—choice a.
The correct answer is choice e. Consult the
Question 4
diagram: O—–M—–R/L The correct answer is choice e. If O is the fourth
checkpoint, then we have:
‹–P—|
O
|—Q–› 1234567

Your ideal approach to this question should


be to look at the diagram and try to see which Consulting our diagram, we see that the
letters could not be third—R and L, because they string M—–R/L must fill the spaces to the right, as
are preceded by three letters—and then check the there are only three slots:
answer choices for one of these letters, without
testing each choice. R ap-
pears in choice e. O M R/L
If you don’t do this, you could check each 1234567
answer choice against the diagram, keeping in
mind that N can be used as a floater. Try to see This forces P—–Q to the left of O. Keeping
if the letter in each answer choice could be third in mind that P cannot be first, we see that P—–
(if letters can be moved in a way consistent with Q must take up spots 2 and 3, leaving N for
the diagram so that the let- ter in that answer spot 1:
choice is third). M could be third, if O and P
preceded it, so choice a is incorrect. N could be N P Q O M R/L
third, if O and N preceded it, so b is incorrect. P 1234 567
could be third, if O and M preceded it, so c is
incor- rect. Q could be third if preceded by O Consulting this diagram, we see that N is in
and P, so d is incorrect. R cannot be third, fact first, so a is incorrect; P is in fact second, so
because it has at least three checkpoints b is in- correct; Q is in fact third, so c is
preceding it (O, M, and R), so e is the correct incorrect; M is in fact fifth, so d is incorrect; but L
choice. is not necessarily sixth (it could be seventh), so e
is the correct answer.
65
–se QueNciNg games reView–

Question 5 66
The correct answer is choice d. If M occurs
before P, then by consulting the diagram we see
that P must be in between M and R/L. So we can
modify the dia- gram to:

O—–M—–P—–Q—–R/L

We can see now that the farthest left we can


push Q (the earliest it can occur) is the fourth spot,
after O, M and P. This is choice d.

Practice Game 7
First, draw out seven spots for the seven parts:

1234567

Then, symbolize the

restrictions. The first

restriction:

6 = Q or S

The second restriction:

T—–S –›U = 2

U ≠ 2 –›S—–T [contrapositive]

The third restriction:

VU

The fourth restriction:

V—–T//X

The fifth restriction:

R/S
Now see if any of the sequence strings can be
joined. Combining the third and fourth restrictions,
we have:

VU—–T//X

We can also add the information from the third


and fifth restrictions into the second restriction:

T—–S/R –›V = 1 and U = 2

Note, however, that since this is a conditional


statement, the information that R/S and VU have not
been captured by this modified restriction, and the
fifth restriction must still be considered.
All in all, we have: 6

= Q or S

T—–S/R –›V = 1 and U = 2

VU—–T//X
R/S

Now we can turn to the questions.

Question 1
The correct answer is choice e. This question is a
“Test-the-Rules” question. The first restriction rules
out choice a, since neither Q nor S are in spot 6. The
second restriction rules out choice d, since T precedes
S, but U is not second. The third restriction rules out
choice c, since V does not precede U. The fifth restric-
tion rules out choice b, since R and S are not adjacent
to one another.

Question 2
The correct answer is choice c. Write out the 7 spots,
and start filling them in. It will be obvious how to
apply the rules as you proceed. If Q is in the spot 7,
then by the first restriction, S must be in spot 6. If S
is in spot 6, R can only be in spot 5. Since S and Q
take up the last two spots, T must occur before S,
and U
–se QueNciNg games reView–

must be in spot 2. If U is in spot 2, then V must Waiting to draw on work you have done for
be in spot 1. This leaves T and X for spots 3 and other questions to answer questions like this can
4: save time.

V U T/X R S Q Question 4
123456 7 The correct answer is choice a. Write out the 7
spots, and start filling them in by applying the
Given this diagram, the only statement that restrictions. If R is in spot 7, then S must be in
doesn’t have to be true is that T is added third, spot 6. Since there is no room to the right of S, T
since T could also be in the fourth spot. So c is must be to the left of S, and therefore U must be
the correct answer. in spot 2, carrying V with it into spot 1. This
leaves Q, T, and X to fill spots 3, 4, and 5:
Question 3
The correct answer is choice c. First, consult the V U Q/T/X S R
dia- gram to see which parts can obviously not be 1234567
added last. Since V and U have parts to their
right, neither can be added last. But V and U do Take each answer choice in turn. Since V is
not appear in the answer choices. This is a in- deed added first, choice a is correct.
difficult question, and one approach is to take
each part in turn and seeing if it could be added Question 5
last. We are looking for definitive proof that a The correct answer is choice e. We have the new
part cannot be added last. We saw in the last
infor- mation that X—–T—–S. We can incorporate
question that Q could be added last, so a is this with our existing sequence strings to
incorrect. If R is added last, then S would have to produce:
be in spot 6. Since T would be before S, U
would have to be in spot 2, and V in spot 1. Q,
VU—–X—–T—–S/R
T, and X could then fill spots 3, 4, and 5. So it
seems R could be last, and we can move on to
This gives a relative order to six of the seven parts,
the next choice. If S were in spot 7, then Q
so try to map the string onto the seven slots. The
would have to be in spot 6 to satisfy the first
first re- striction says that Q or S must be in spot
condition. But then we would break the R/S block
6. See what would happen if Q were in spot 6:
from the fifth condition. So S definitely can-
not be in spot 7, and c is the correct answer choice. Q
Since taking each choice in turn can often to test, and if we test choice c as above,
turn out to be time-consuming, you might instead we see that S cannot be last.
save this question for last and use whatever
information you can from other questions to rule
out answer choices— that is, to see which parts
are added last in the ac- ceptable scenarios you
come across or produce for the other questions.
Part T is last in the correct answer to Question 1,
so choice d is ruled out, and part Q is last in the
acceptable scenario produced for Question 2, so
choice a can be ruled out. Part R is last in the
accept- able scenario produced for Question 4, so
choice b is incorrect. That leaves just two choices
1234567 left of Q—but there are only 5 spots to the left of
Q. So Q cannot be in spot 6, and S must instead
VU—–X—–T—–S/R by spot 6. Since T cannot be to the right of S (given
the string VU—–X—–T—–S/R), it must be to the
Since the S/R block cannot be broken up, the en- tire left of S, which implies (via the second restriction,
string of 6 letters VU—–X—–T—–S/R would have to be to the T—–S/R -> V = 1 and U = 2) that V and U take up
spots 1 and 2 respectively:

67
–se QueNciNg games reView–

The second restriction:


V U S
1234567 A—–C

Now, R must be either to the immediate The third restriction:


right of S or immediate left of S, so we can
produce two scenarios. In scenario 1, with R to E/C
the left of S, X and T must be forced into spots 3
and 4, leaving Q for spot 7: The fourth restriction:

S1 E—–F

V U X T R S Q The fifth restriction:


1234567
F/B
In scenario 2, R is to the right of S, leaving X
—–T and Q to fill spots 3, 4, and 5. X—–T and Q The second, third, and fourth restrictions
can be ordered in three ways (Q before X—–T, pro- duce sequence strings that can be joined
in between X—–T, or after X—–T): QXT, XQT, or together into:
XTQ.
So the second scenario looks like: A—–E/C—–F

S2 We have four of the six speakers in a


Q X sequence string, so the sequence possibilities will
T X probably be fairly limited. From the first
Q T restriction, we know that D must be either
X T second or third, so let’s graph our string A—–
Q
E/C—–F onto two scenarios, one scenario in which
V U S R
D is second and one scenario is which D is
1234567 third:

Now see which parts could appear in spot 5.


S1
In S1, R appears in spot 5. In S2, T or Q appear
in spot 5.
Since Q, R, and T, and no other parts can appear D
in spot 5, the correct answer 123456
is e.
S2
Practice Game 8
Six speakers have to be assigned to six spots: D
123456

123456 D = 2 or 3

The first restriction:


Keep in mind S1, E/C—–F will have to take spots 4–6, since E/C
that B cannot be cannot fit into spot 1, and since if E/C took spots 3
next to F, as per the and 4, that would leave B and F next to one
fifth restriction. In another in spots 5 and 6 (A must be in spot 1,

68
–se QueNciNg games reView–

so B cannot be there). So if E/C—–F takes spots


4–6, A and B are distributed into spots 1 and 3: Since A is first, B is second, D is third, and
F is sixth, choices a, b, c, and e all must be true.
S1 Since C could speak fifth, it isn’t the case that C
A/B D B/A E/C F must speak fourth; and so d is the correct
1 2 3 456 answer.

What about S2? Given our string A—–E/C—– Question 4


F, we can see that E/C cannot fit into spots 1 and The correct answer is choice e. If B is just before
2, be- cause that would leave no room for A. So C, then we cannot be in S2 (since E/C is
E/C—–F must again squeeze into spots 4–6, immediately pre- ceded by D), and so we must
leaving A and B to be distributed into spots 1 be in S1, with B in spot 3 and A in spot 1:
and 2:
A D B E/C F
S2 123456
A/B B/A D E/C F
1 2 345 6 We can now see that since A speaks first
and F sixth, four people must speak in between
Question 1 them—an- swer choice e.
The correct answer is choice a. This question is
a “Test-the-Rules” question. The first restriction Question 5
rules out choice b, since D is neither second nor The correct answer is choice c. Consult the
third. The second restriction rules out choices c scenario diagrams to see which speakers could be
and e, since A does not precede C. (The third in spot 2. In S1, spot 2 is taken by D. In S2, spot 2
restriction also rules out choice c, since C and E is taken by A or
are not adjacent to one an- other). The fourth B. So A, B, and D, and no other speakers, could
restriction rules out choice d, since E does not speak second. This is choice c. You could also use
precede F. the answer from question 1 to eliminate choices a
and b and then go from there. It’s easy to show
Question 2 that neither E nor C can be second.
The correct answer is choice b. Consult the
scenario diagrams to see where speaker E could
be. In both S1 and S2, speaker E could be in
either spot 4 or spot 5, and no other spot. This is
choice b.

Question 3
The correct answer is choice d. If there is one
space in between A and D, we cannot be in S1
(since A must either be to D’s immediate right
or left there). So we must be in S2, and A must
be in spot 1, meaning that B is in spot 2:

A B D E/C F
123456
69
4
chapt e r

selection games
review

In a selection game, you are given a group of entities from which some are to be selected. The entities
can be people, places, or things. The selection is always done according to a certain set of rules (for
example, that if en- tity A is selected, entity B is not selected), and these rules are sometimes based on
certain characteristics of those entities (for example, that if one of the green-colored entities is selected,
one of the red-colored entities must be selected). Some selection games specify how many entities are to
be selected (information which is often crucial to understanding the game-play and answering the
questions), while some leave that open-ended.
There are two things you should understand before approaching selection games: the different
types of domains (i.e., the groups of entities from which you are selecting), and the different types of
typical rules.
We consider both here, as well as methods for diagramming the actual selection of entities.

know the domain types

The group of entities from which you are selecting (let’s call this the domain) can take on different levels
of complexity, which will dictate the difficulty of the game and the types of conditions given and questions
asked.
71
selectioN games reView

your diagram of
Simple Domain
The simplest domain is a plain list of some
number of entities, with no further distinction
between them. For example, you may be asked to
select five students for a class section from a
group of seven students labeled A, B, C, D, E, F,
and G. The list of seven students is your simple
domain.

Subgroups Domain
Some domains consist of subgroups. For example,
you may be asked to select five students from a
group of seven students, some of whom are
female (A, B, and
C) and some of whom are male (D, E, F, and G).
The two subgroups are the females and males.

Cross-Group Domain
Finally, there is the “cross-group” domain, in
which the domain is broken down along two or
more dimen- sions. For example, you may be
asked to select five stu- dents from a group of
seven students, some of whom are female and
domestic A, some of whom are female and
international (B and C), some of whom are male
and domestic (D, E, and F), and some of whom
are male and international (G). In this case, there
are two ways to divide the domain into two
groups, resulting in four groups.
You may also be given subgroups and
cross- groups for which specific members are not
specified. For example, you may be asked to
select from a group of seven students, three of
whom are female and four of whom are male;
or to select from a group of seven students, one
of whom is female and domestic, two of whom
are female and international, three of whom are
male and domestic, and one of whom is male
and international. Usually, LSAT does not give
you a cross-group without a breakdown, such as
three female, four male, three domestic, four
international.

Diagramming the Domain


The kind of domain you are given (simple,
subgroup, or cross-group) should dictate what
the entity group will look like. This diagram may be
all there is to diagramming a selection game. If you
have a simple domain, just list the entities in that
domain:

ABCDEF

If you have a group consisting of subgroups, use


capitalization to distinguish them, and perhaps label
them as well (e.g., we label the Female group “F” and
the male group “M” below). If the entities are named
A, B, C, etc., you might write:

F: A B C
M: d e f g

If the entities are not named or otherwise speci-


fied, you might write:

FFFMMMM

If you are given cross-groups, there are a few op-


tions, and you should see what works best for you as
you go through the practice games. The most visual
way to capture cross-group information is with a grid.
The advantage of the grid is that you can visualize
how to apply conditions involving two different
character- istics of a domain (in the example we have
been using, male/female and
domestic/international).
If the entities are specified or named:

d i
f A BC
m DEF G

If the entities are not specified, you can represent


them by dots:
d i
f • • •
m • • • •
72
selectioN games reView

This allows you to visually grasp the effects each


of the rules. For example, if a condition tells
you that at least two females are chosen, a quick
glance at the upper row of the grid (representing
all females) tells you that you can circle a dot in
the upper-right box, since at least one in that box
will have to be selected. If another condition tells
you that at least three domestic are chosen, a quick
glance at the left column of the grid (representing
all domestic students) tells you that you can circle
two in the lower left-hand box (since there is only
one female domestic student, at least two male
domestic students must be chosen). These kinds
of inferences are often easier with the aid of a grid
such as this, since it allows you to be flexible in
how you visualize the relevant groups (for the
female students, look at the top row; for the
domestic students, look at the left column; for the
male international students, look at the bottom-
right box; etc).
Another method for symbolizing the cross-
group entities is to use capitalization to capture
one of the dimensions. For example, domestic
could be represented by lowercase letters and
international by upper-case letters:

fFFmmmM

The advantage of this sort of representation


is that it takes less time to write out. The
disadvantages are that one of the dimensions is
only represented by capitalization (lower-case for
domestic vs. upper-case for international), while
the other is represented by particular letters (“f”
for female and “m” for male), which might lead to
confusion (e.g., forgetting which case represents
domestic and which international).
Also, the inferences may not be as visually
appar- ent (e.g., the condition that three domestic
are chosen requires you to look at the “f” and
three “m” across two “F”, rather than down the
“d” column in the grid, perhaps making the
implication of the condition—that two “m” must be
chosen—less obvious).
You might try to avoid the first problem
by using two different sets of two letters for
dimension—i.e., d and i for domestic and international and m ABCDE
and f for male and female:
Another example:
df if if dm dm dm im
Group 1: A B C
But this notation still doesn’t allow for an easy visual Group 2: D E F
grasp of the rules. Group 3: G H I
You should experiment with different approaches to see
which is more efficient for you—if you are good at grasping You may also wish to keep a separate space
deductions without visual aids, you might save time forgoing for the entities definitely not selected (the “out”
the grids. group). You can do this by writing:

OUT: D, F, H
diagramming the selection
This information was captured by crossing enti-
If a simple domain or subgroup game does not ties off the entity list, but some students find it
specify how many entities are to be selected, it is usually helpful to specifically create an “out” group as
most efficient to just circle and cross out symbols directly well.
on your entity list as you learn information (for entities Even if you use the entity grid for cross-
you know are definitely selected or definitely not selected). group domains as discussed above, then whether
For example: or not the game specifies how many entities are to
be selected,

73
selectioN games reView

it usually helps to circle and cross out symbols on (e.g., a diagram showing that a certain selection
the grid itself as you learn new information. list is acceptable). The downside is that you will
If a simple or subgroup domain specifies spend a lit- tle extra time drawing diagrams (both
how many entities are to be selected, then the entity grid and entity list/columns). You will
consider using slots to keep track of what’s have to experiment with diagramming to see
been selected, and what entities are still what works best for you.
available to select from. For example:

A C ABCDE know the rule types


This is particularly useful for subgroup A typical selection game will present a set of rules
domains, since the restrictions and questions can that will determine how the selection must, can, or
use a mix of information about specific entities and can’t occur. One thing nearly all selection games
their subgroup types. For this reason, keep have in common is that they tend to be logic-heavy:
information about spe- cific entities above the the rules are often conditional statements, sometimes
spaces, and information about subgroup types involv- ing disjunctions (“or” clauses) and conjunctions
below the spaces. For example, if we know that (“and” clauses). Understanding, manipulating, and
five entities are to be selected, two from group connect- ing these statements will require a basic
1, two from group 2, and one from group 3, we command of symbolic logic, which will allow you to
might write: draw inferences.
There are six basic types of rules that are
B G typical in selection games.
1122 3
Concrete Rules
Group 1: A b C Concrete rules tell you which entities
Group 2: D E F are definitely selected.
Group 3: G H I
B is selected.
Finally, a note about replicating your diagrams: C and D are in the section.
For cross-group domain entity grids, it is often
help- ful to leave space underneath your letters or Positive Conditionals
slots so that you can put a check or x-mark or Simple positive conditionals tell you that if one
dot (or some other mark) to indicate selection or entity is selected, another entity will be selected.
exclusion (rather than crossing out or circling).
This allows you to use the same diagram or If A is selected, B is selected.
entity-list multiple times (by making it easy to If C is in the section, D is in the section.
erase your marks), rather than have to redraw the
grid. If you want to keep the informa- tion from
More complicated positive conditionals involve
each question, you should also draw an en- tity
disjunction and conjunction clauses.
list next to each question. Or, you could just have
an “in” and “out” column next to each question
If either A or B are selected, D will be
into which you place entities as you select or exclude
selected.
them. The upside is that you retain information as
If C is in the section, both F and G will be in
you move from question to question that could be
the section.
time-saving
74
selectioN games reView

Negative Conditionals “out” group. If you prefer pure symbolic logic, a


A simple negative conditional tells you that if one nega- tive conditional such as A S ≠ B (that one
en- tity is selected, another entity will not be entity pre- cludes another) is equivalent to:
selected; or that if one entity is not selected,
another entity will be selected. 1. B S  A (the contrapositive, that the
selection of the latter entity precludes the
If A is selected, B will not be selected. former)
If C is not in the section, D must be in 2.  B or  A (one of the two entities
the section. involved must not be selected)
3.  (B and A) (the two entities cannot be
selected together)
More complicated negative conditionals involved
disjunction and conjunction clauses.
This information can also be symbolized by
If either A or B is selected, D will not writing that either A or B must be in the “out”
be selected. group, rather than using pure symbolic logic:
If C is not selected, E and F will both be
selected. OUT: A or B

Positive vs. negative Conditionals Biconditional Rules


We have separated positive and negative Biconditional rules are positive conditionals
condition- als because they usually operate that also work in reverse. For example,
differently in logic games—especially when the suppose we have a positive conditional
conditionals are simple rather than complex. saying that if A is se- lected, B must be
Positive conditionals tell you how selecting selected. As we know, this does not mean that
one entity will pull in another entity—if you if B is selected, A must be selected.
choose A, you must also choose B. And their Biconditionals, however, add this extra bit of
contrapositives tell you that if one entity isn’t in- formation in. So they will say: If A is
chosen, another cannot be—if you don’t choose selected, B is selected, and if B is selected, A is
B, you can’t choose A. selected. This es- sentially means that either A
By contrast, negative conditionals tell you and B must be chosen together, or neither can be
how selecting one entity will preclude the chosen. In more formal terms, the statement
selection of an- other. If you select A, you cannot that “if A, then B, and if B, then A” is
select B. Notice that the contrapositive says that if equivalent to the statement “if not B, then not
you select B, you cannot select A. This means that A, and if not A, then not B.” The latter is what
such negative conditionals really amount to gives us the idea that if any one of them isn’t
statements that two entities cannot be selected chosen, then neither can be chosen. These rules
together, symbolized by our box with a line through are equivalent to this selection block:
it (a selection anti-block):
AB
AB
Remember that biconditionals are often
Further, a negative conditional tells you that stated using the phrase “if and only if.” For
at least one of the two entities involved must not example, the rule discussed here might have been
be se- lected; that is, one of the two entities will stated: “A is selected if and only if B is selected.”
be in the
75
selectioN games reView

Disjunction Rules contrapositive says; so we must select one or the


Disjunction rules are like diluted concrete rules. other, or both.
They tell us that one or both of two entities must Disjunction rules are sometimes useful
be selected (but don’t tell us which one). for breaking up a diagram of possible selections
into two scenarios. This creation of two scenarios
Examples: should usually come after all other bits of the
diagram have been filled out and other deductions
Either A or B must be selected. have been made.
Either A is in the section or B is in
the section. Number Rules
Perhaps the most difficult rules are number
Remember that “or” statements are not rules. They are also often the most important rules,
exclusive by default—that is, if we say that “either playing a large role in driving the mechanics of a
A or B must be selected,” we are not ruling out game. Number rules always tell us something
the possibility that both A and B are selected. about the number, or relative number, of entities
Also note a strange fact about disjunction to be selected.
rules: they can be disguised as conditional
statements. Take the rule “if A is not selected, At least three domestic students are selected.
then B is selected.” This seems like a negative More males are selected than females.
conditional. But what it really says is that either A If exactly two females are chosen, one
must be selected or B must be selected. If we interna- tional student is chosen.
don’t select A, we must select B; and if we If A is selected, at most two domestic stu-
don’t select B, then must select A, as the dents are selected.

76
selectioN games reView

practice game 1

A dish contains one or more of six spices—salt, pepper, chili, basil, garlic, and oregano—according to the
fol- lowing conditions:

If it contains salt, it contains pepper.


If it contains chili, it does not contain
basil. If it does not contain salt, it
contains garlic.
If it contains either chili or pepper, it contains garlic.

1. Which one of the following could be a 4. If the dish does not contain pepper, what is
complete and accurate list of the spices in the largest number of total spices the dish
the dish? could contain?
a. Basil, chili, garlic, pepper a. One
b. Chili, oregano, pepper, salt b. Two
c. Basil, oregano c. Three
d. Garlic, oregano, pepper, salt d. Four
e. Basil, garlic, oregano, salt e. Five

2. Which one of the following must be true? 5. If the dish contains salt, what is the least
a. The dish contains salt. num- ber of total spices the dish could
b. The dish contains garlic. contain?
c. The dish contains either chili or basil. a. One
d. The dish contains pepper. b. Two
e. The dish contains either chili or pepper. c. Three
d. Four
3. If the dish contains exactly five spices, all of e. Five
the following must be true EXCEPT:
a. The dish contains chili.
b. The dish contains salt.
c. The dish contains pepper.
d. The dish contains garlic.
e. The dish contains oregano.
77
selectioN games reView

practice game 2

Five books are selected for a syllabus from three history books (A, B, and C), four philosophy books (D, E,
F, and G), and two literature books (H and I). At least one of each kind is selected, according to the
following conditions:

If more than one philosophy book is chosen, both literature books are
chosen. If D is chosen, I is not chosen.
Either C or B must be chosen.
If B is chosen, then either D or F (or both) must be chosen.

1. Which one of the following could be a 4. If book I is the only literature book
complete and accurate list of the books selected, then the syllabus must include
chosen for the syllabus? which one of the following pair of books?
a. A, B, E, F, I a. F and D
b. A, E, F, H, I b. B and H
c. A, B, C, D, I c. A and F
d. C, F, G, H, I d. C and G
e. A, B, D, F, G e. A and E

2. If book E is selected, then which one of the 5. Suppose we replace the first restriction (if
fol- lowing cannot be true? more than one philosophy book is chosen,
a. H and I are both selected. at least two literature books are chosen)
b. B and G are both selected. with the restriction that if more than one
c. B and H are both selected. philosophy book is chosen, at most one
d. A and C are both selected. literature book is chosen. Now, if E and G
e. C and G are both selected. are the only philoso- phy books selected,
which one of the following could be true?
3. If book A is selected, then which one of the a. C and I are both selected.
fol- lowing must be true? b. A and B are both selected.
a. Exactly one philosophy book is selected. c. D and I are both selected.
b. Exactly one history book is selected. d. H and I are both selected.
c. Exactly two history books are selected. e. B and G are both selected.
d. Exactly two philosophy books are selected.
e. Exactly two literature books are selected.

78
selectioN games reView

practice game 3

A section is comprised of at least four students chosen from among three male students (A, B, and C), and
four female students (D, E, F, G). The students are chosen according to the following restrictions:

At least two female students are


chosen. D cannot be in the section
without E.
If either student A or E is in the section, G cannot be.
The section must have either student B or student D (or both).

1. Which one of the following could be a 4. Which one of the following statements
complete and accurate list of the students in cannot be true?
the section? a. Exactly four females are in the section.
a. A, B, C, F b. Exactly three females are in the section.
b. B, C, E, F c. Exactly three males are in the section.
c. A, C, F, G d. Exactly two males are in the section.
d. A, C, E, F e. Exactly two females are in the section.
e. A, B, D, F, G
5. If student B is not in the section, which one
2. If student G is in the section, which two of the following could be true?
stu- dents must also be in the section? a. Neither A nor D is in the section.
a. A and B b. Neither C nor E is in the section.
b. B and C c. Neither C nor F is in the section.
c. E and C d. Neither C nor G is in the section.
d. D and G e. Neither E nor F is in the section
e. E and B

3. If student A is in the section, which one of


the following cannot be true?
a. B is not in the section.
b. E is not in the section.
c. D is in the section.
d. C is not in the section.
e. F is in the section.
79
selectioN games reView

practice game 4

Four species of trees are selected to populate a new park from among four deciduous species (A, B, D,
and G) and five evergreen species (C, E, F, H, I). Each species is available for planting in exactly one of
two possible ways, ei- ther as a sapling or fully-grown tree. The species available as saplings are A, F, H,
and I, and the species available fully-grown are B, C, D, E, and G. The trees are selected according to
the following conditions:

Exactly two deciduous trees and exactly two evergreen trees are selected.
Exactly two saplings and exactly two fully-grown trees are selected.
The park must include either species I or species C or both.

1. Which one of the following could be a 4. If species E is selected, which one of the
complete and accurate list of the trees selected follow- ing cannot be selected?
for the park? a. A
a. A, D, E, H b. B
b. D, F, G, I c. D
c. A, C, E, I d. G
d. F, G, H, I e. H
e. B, C, D, E
5. If species C is selected, which one of the
2. Which two species cannot be both follow- ing must also be selected?
selected? a. A
a. B and D b. B
b. F and H c. D
c. C and A d. E
d. H and I e. I
e. G and A

3. If species A and E are selected, which other


spe- cies must be selected?
a. C
b. B
c. F
d. G
e. I
80
selectioN games reView

practice game 5

At least four and at most five bands are selected from eight entrants for a music competition. The
entrants in- clude two folk bands from Chicago, two jazz bands from Chicago, three folk bands from
Milwaukee, and one jazz band from Milwaukee. The bands are chosen according to the following
restrictions:

At least two jazz bands are selected.


At least one band from Milwaukee is
selected. At least one band from Chicago
is selected.
If the Milwaukee jazz band is selected, at least two Milwaukee folk bands must be selected.

1. Which one of the following could be a 3. Which one of the following cannot be true?
complete and accurate list of the bands a. No Milwaukee folk band is selected.
selected for the competition? b. Exactly one Milwaukee folk band is
a. One Milwaukee jazz band, one selected.
Milwaukee folk band, two Chicago folk c. Exactly one Chicago folk band is selected.
bands, two Chi- cago jazz bands d. Exactly two Chicago folk bands are
b. Two Chicago folk bands, two Chicago selected.
jazz bands e. Exactly one Chicago jazz band is selected.
c. One Chicago jazz band, two Chicago
folk bands, two Milwaukee folk 4. If exactly one Chicago jazz band is
bands selected, which one of the following
d. One Chicago jazz band, one Milwaukee must be true?
jazz band, two Milwaukee folk bands a. At least two Chicago bands are selected
e. Three Milwaukee folk bands, one Milwaukee b. At least three Chicago bands are selected
jazz band.
c. At least three folk bands are selected
d. At least three Milwaukee bands are
2. If both Chicago folk banks are selected,
selected.
which one of the following must be true?
e. At most three Milwaukee bands are
a. Exactly one Milwaukee jazz band is
selected.
selected.
b. Exactly one Chicago jazz band is
5. If exactly two Milwaukee bands are selected,
selected.
which one of the following statements must
c. No Milwaukee folk band is selected.
be true?
d. Exactly one Milwaukee folk band is
a. Exactly one Chicago folk band is selected.
selected.
b. Exactly two Chicago folk bands are
e. Exactly two Milwaukee folk bands
selected.
are selected.
c. Exactly two Chicago jazz bands are selected.
d. Exactly one Milwaukee jazz band is selected.
e. Exactly one Milwaukee folk band is
selected.
81
selectioN games reView

practice game 6

Four members are selected for a medical expedition from a group of nine medical professionals,
including six nurses (A, B, D, F, G, and H) and three doctors (C, E, and I), six of whom are local (A,
E, F, G, H, and I) and three of whom are foreign (B, C, and D), according to the following
conditions:

At least two nurses are


chosen. At least one doctor
is chosen.
If C is chosen, then at least two local nurses must be
chosen. If either E or I are chosen, then B must be
chosen.

1. Which one of the following could be a 4. If C is not selected, which one of the
complete and accurate list of the expedition following could NOT be an accurate,
members? partial list of the professionals chosen for
a. A, B, C, D the expedition?
b. A, D, F, G a. A, F, G
c. A, B, C, F b. A, B, E
d. A, C, E, I c. G, H, I
e. A, E, F, G d. A, B, H
e. D, E, F
2. If exactly one local nurse joins the
expedition, then which one of the 5. Which one of the following statements
following statements must be true? cannot be true?
a. A is selected. a. Exactly three nurses are chosen.
b. B is selected. b. Exactly two foreign professionals are chosen.
c. D is selected. c. Exactly three foreign professionals
d. E is selected. are chosen.
e. F is selected. d. Exactly two local professionals are chosen.
e. Exactly three local professionals are chosen.
3. Which two professionals cannot both be
chosen for the expedition?
a. A and C
b. E and I
c. B and C
d. C and E
e. B and F
82
selectioN games reView

practice game 7

A student applies to at most five of eight colleges—A, B, C, D, E, F, G, and H—according to the


following restrictions:

She applies to at most three among A, B, C,


and D. She applies to at least two among E,
F, and G.
If she applies to C, then she does not apply to D.
If she applies to A, then she applies to both B and
C. If she does not apply to A, then she applies
to H.

1. Which one of the following could be a 4. If the student applies to college D, then
complete and accurate list of the colleges to which one of the following must be true?
which she applies? a. She applies to A.
a. A, B, E, F b. She applies to B.
b. B, C, F, G c. She applies to E.
c. A, B, C, E d. She applies to G.
d. C, D, E, F, H e. She applies to H.
e. A, B, C, F, G
5. If the student applies to college A, then
2. If the student applies to college H, then which one of the following could NOT be a
which one of the following statements partial, accurate list of the colleges to which
must be true? she applies?
a. She does not apply to A. a. E, C
b. She does not apply to B. b. C, G
c. She does not apply to C. c. E, F, G
d. She does not apply to E. d. B, E, F
e. She does not apply to F. e. B, F, G

3. The student cannot apply to both of which


two schools?
a. Both H and B
b. Both D and A
c. Both A and C
d. Both F and B
e. Both D and E
83
selectioN games reView

practice game 8

A textbook is to be translated into at least one of seven languages—Arabic, Chinese, French, German, Hindi,
Russian, and Spanish—according to the following conditions:

If it is translated into French, it is translated into German.


If it translated into Arabic or Chinese or both, it will not be translated into
Russian. If it is not translated into Arabic, it is translated into Hindi.
If it is translated into Spanish, it will not be translated into German.

1. Which one of the following could be a 4. If the book is not translated into Hindi,
complete and accurate list of the languages which one of the following must be true?
into which the book is translated? a. It is translated into Spanish.
a. Chinese, German b. It is translated into German.
b. Arabic, French, Spanish c. It is not translated into French.
c. French, German, Hindi, Chinese d. It is not translated into German.
d. Arabic, French, German, Spanish e. It is not translated into Russian.
e. Arabic, Chinese, Russian, Spanish
5. If the book is neither translated into Hindi
2. If the book is translated into Russian, then nor German, then which one of the
what is the maximum number of following could be an accurate, partial list of
languages into which the book could be the languages into which it is translated?
translated? a. Chinese, French
a. One b. Spanish, Chinese
b. Two c. Russian, Arabic
c. Three d. Spanish, Chinese, Russian
d. Four e. Spanish, Arabic, French
e. Five

3. Which two languages CANNOT be a pair


into which the book is translated?
a. Spanish and French
b. Russian and French
c. German and Hindi
d. Spanish and Hindi
e. Arabic and Chinese

84
selectioN games reView

setup, answers, and


explanations Now look for connections among the restric-
tions. The third restriction and the contrapositive
Practice Game 1 of the first restriction link up:
This selection game uses a simple domain and
doesn’t have number restrictions, so a simple list of ≠ p S ≠ s Sg
the entities will suffice for the diagram:
The contrapositives:
spcbgo
≠gS sS p
The first restriction can be
Looking at the restrictions, the salt seems to
symbolized: s S p (≠ p S ≠ s) play an important role (at least it appears in two
rules—al- though the chili and garlic do so as
The second restriction: well). It’s also the case that both the salt being in
the dish has implica- tions (that the pepper is
c S ≠ b (bS ≠ c; either ≠ b or ≠c) selected) and the salt not being in the dish has
implications (that the garlic is selected). So to try to
This can also be symbolized: get a handle on the action of the game, ex-
periment with dividing it up into two scenarios—
bc one in which salt is chosen, and one in which it
is not.
Note that this puts either b or c in the out If the salt is chosen, then according to the
group, so you could also write: first restriction, pepper is also chosen. If pepper is
chosen, then according to the fourth restriction,
OUT: b or c garlic is also chosen. So we have:

The third restriction: S1


s pcbg o
≠ s S g (≠ g S s; g or s)
If the salt is not chosen, we know that
Note that this is a disguised disjunction. This garlic is chosen. So we have:
re- striction says that either g or s must be
selected. S2
The fourth restriction: spcbgo

(c and p) S g Notice what we have done with the


restrictions. The first restriction, third restriction,
The contrapositive is: and fourth re- striction are all satisfied in these
two scenarios (the fourth restriction is satisfied
≠ g S (≠ c and ≠ p) because in both scenar- ios, garlic is chosen, and
so the conditional represent- ing the fourth
restriction is true).
We have effectively made it such that only the
sec- ond restriction is relevant to further game play
(i.e., we only need to consider the second restriction
now), given

85
selectioN games reView

the two scenarios. As we proceed, we only need to Question 3


worry about the fact that c and b cannot both be The correct answer is choice a. Since we can’t
chosen. have both b and c, at least one of these two
must not be se- lected. But if we are choosing five
tIP of six spices, then there can only be one spice that
Folding up rules into your diagrams is an
is not selected, and so that spice must be either b
essen- tial strategy. Try to diagram scenarios
or c, and the rest must be in the dish.
(without dividing up into too many scenarios)
If you are using the scenario diagrams,
such that as many restrictions as possible are
you should note that if the dish contains exactly
satisfied. Then you can focus on the
five spices, we must be in scenario 1 since scenario
remaining restric- tions while resting assured
2 contains at most four spices. Since there are five
that your diagrams are taking care of the rest.
spices, and we can select at most one of c or b, we
must have the oregano. So our diagram is:

Now we can turn to our questions. s pcbg o

Question 1 Now check each answer choice against the


The correct answer is choice d. This is a “Test- dia- gram. Salt, pepper, garlic, and oregano must
the- Rules” question. The first restriction rules out all be se- lected. But basil could be selected, and
e, since salt is chosen but pepper is not. The chili not. So the correct answer is a.
second restriction rules out a, since both chili and
basil are chosen. The third restriction rules out c, Question 4
since neither salt nor gar- lic is chosen. The fourth The correct answer is choice c. If the dish does
restriction rules out b, since pepper is chosen but not contain pepper, we must be in scenario S2,
garlic is not. since in S1, the dish contains pepper. Our
diagram is now:
Question 2
The correct answer is choice b. This is where spcbgo
diagram- ming ahead of time really pays off. In
both possible scenarios, we see that g is selected. Now we see what the greatest number of
Circle this answer choice, and move on. ingre- dients we could select is (we circle as many
If you had not diagrammed, you could also as we can). In addition to g (already selected), we
get the answer by noting that the answer to can select one of c and b, and we can select o.
question 1 rules out c, and that d can’t be the That leaves us with a maximum of three spices,
answer because if it were true, e would be true as answer choice c.
well. You are left with a, b, and e, which could
be tested one at a time by seeing if the statement Question 5
could be made false without violating any of the The correct answer is choice c. If the dish
restrictions. When you get to b, you will see that contains salt, we must be in scenario 1, since
you can’t put garlic in the out group without scenario 2 has the dish lacking salt. In scenario 1,
violating a restriction: if g is out, then by the salt, pepper, and garlic must be selected. As our
contrapositive of the fourth restriction, c and p are diagram indicates, be- yond that, nothing must be
out as well; but if p is out, then by the selected. Therefore, the least number of spices the
contrapositive of the first restriction, s is out; but dish could contain is three, answer choice c.
then the third restriction (saying that either s or g
must be in) must be violated.
86
selectioN games reView

You could also answer this question by three philosophy books chosen. We are left with
just applying the rules one at a time, and keeping two scenarios, one with one philosophy book
track of what must be selected. By the first chosen, and one with two philosophy books
restriction, since salt is selected, pepper must be chosen:
selected. The second restriction doesn’t force us to
select anything. The third is already satisfied S1:
since we have selected salt. The fourth forces us p l/h l/h l h
to also choose garlic. So we have:
S2:
s pcbg o p p l l h

Now, cycle back through the rules to make Now consider the other rules, and start
sure that we don’t have to select anything else making deductions that can be applied to these
(can cross out c, b, and o). diagrams. We could go one of two routes—either
noting a couple of bits of information about these
Practice Game 2 diagrams and moving the questions, or going into
Since we are choosing five books from three a bit more depth with the initial deductions. Which
sub- groups, we want to diagram five spaces, and you ought to do depends on how quickly you are
charac- terize those spaces as either philosophy able to make initial deductions. If you can make
(p), history (h), or literature (l). them quickly, it might pay off to do them up front;
We can also keep track of what’s selected but if they generally take a bit longer for you, cut
and definitely not selected with a list of each off the initial deduction process earlier and just
group: make deductions as the questions require.
The less involved route:
h: A B C
p: D E F The third restriction tells us that one of the
G l: H I his- tory books must be C or B. so we can
write “C/B” into the required history slot for
The first rule is a number rule, so see if the both S1 and S2. We also note that since
rule allows you to create different scenarios. Since there are only two lit- erature books, there
there are five spots, and there is at least one of each must be at least two his- tory books in S1.
genre in the syllabus, there can be one, two, or Then, just keep the other rules handy:
three philosophy books chosen. If there is one
philosophy book cho- sen, the first restriction S1: C/B
does not apply (and we would be left with one p h l/h l h
required spot for literature, one re- quired spot
for history, and two spots that could go either way S2: C/B
(“l/h”)). If there are two chosen, the first p p l l h
restriction applies, and the specific number of
each type of book is fixed: two philosophy, two DI
literature, and one history. If three philosophy
books were to be chosen, then there would be two B S (D or F)
literature books, and no more room for a history (not D and not F) S not B
book—so there can’t be
87
selectioN games reView

The more involved route: immediately deduced from the rules given this
new information. So we turn to the answer choices
We start with the deductions just made. and test each one to see if the two books in the
Then, we look at S2 more carefully. In S2, choice could also be selected; if so, that’s not
there are two literature books required, and the correct answer. Can E, H, and I be selected?
only two to choose from, so those spots can be S2 seems to allow this, so a seems incorrect, and
filled in with H and I. According to the we move on. Can E, G, and B be selected? Since E
second restriction, D and I can- not be chosen and G are selected, there are two philosophy
together. Since we have I, we know that D books chosen, and we must be in S2. Realizing
cannot be one of the philosophy books. So this fixes the entire selection: E, G, H, I, B. Is
we can write E/F/G into the two philosophy that a possible selection? No, the fourth restriction
spots. So we have: is violated, because B is selected, but neither D nor
F is selected.
S2: E/F/G E/F/G H I C/B So G and B cannot be selected, and choice
p p l l h b is correct. Circle this answer and move on to
the next question.
The fourth restriction can’t be easily incorpo-
rated into the diagram, so we just note that: Question 3
The correct answer is choice a. If book A is
B S (D or F) selected, we must be in scenario 1, as a brief
(~not D and ~not F) S ~not B glance at scenario 2 tells us that there is no room
for A there. S1 shows us that there is only one
There is not much else to add to S1, so we philosophy spot, so a must be the correct answer.
leave that as it is:
Question 4
S1: C/B The correct answer is choice c. If book I is the only
p h l/h l h lit- erature book selected, then we must be in
scenario 1, since scenario 2 has two literature
With some visual sense of how the game books. (You could also see this by applying the
works, we can now turn to the questions. contrapositive of the first restriction—since only
one literature book is selected, only one philosophy
Question 1 book can be selected, requiring all three history
The correct answer is choice d. This is a “Test-the- books). Now see if you can fill out the diagram
Rules” question. The first restriction rules out a, for scenario 1:
since there are
two philosophy books, but only one literature book. The I
second restriction rules out c, both D and I have p l hh h
been selected. The third rule rules out b since
neither book C nor book B have been selected. h: A B C
Choice e violates the re- striction in the setup— p: D E F
there is no literature book. G l: H I

Question 2 Since there are three history slots, we know


The correct answer is choice b. We start by that A, B, and C are chosen. Since B is chosen, by
suppos- ing that E is selected. There is nothing the fourth
that can be
88
selectioN games reView

restriction, either D or F must be chosen (for the Since D and F are out, we can apply the
phi- losophy slot). But since I is chosen, D cannot contra- positive of the fourth restriction to rule out
be chosen by the second restriction. So F must be B, leaving A and C as the two history books:
the philosophy book. We now have a full
syllabus: E G A C
p p l h h
F I A B C
p l hhh h: A B C
p: D E F G
h: A B C l: H I
p: D E F G
l: H I The only remaining spot is a literature
book, and we could either have H fill the spot
We can now scan our answer choices for a or I fill the spot without violating the remaining
pair on this syllabus. F and A are on the list, so restrictions; the second restriction is not violated
choice c is the correct answer. because D is not se- lected, and the third restriction
is not violated because C has been selected. So we
Question 5 have:
The correct answer is choice a. Our strategy
will be to start with the new information, that the E G H/I A C
only phi- losophy books chosen are E and G, and p p l h h
then apply the new first restriction along with the
other restrictions. Since E and G are the only h: A B C
philosophy books chosen, D and F are out. p: D E F G
l: H I
E G
p p Now scan the answer choices for a set of
two selections compatible with this diagram. I
h: A B C and C are possible given this diagram, so a is the
p: D E F correct answer. Since B is not chosen, choices b
G l: H I and e are ruled out. Since D is not chosen, c is
ruled out. Since there is no room for both H and
Since there are two philosophy books I, d is ruled out.
chosen, the new first restriction kicks in so that we
know only one literature book is chosen, and Practice Game 3
therefore, those two history books must be This game has a subgroup domain, but does not
chosen: spec- ify the number to be selected (beyond saying
that there must be at least four students). So our
E G diagram is just a list of the entities in the
p p l h h domain:

h: A B C M: A, B, C
p: D E F C F: d, e, f, g
l: H I

89
selectioN games reView

We now turn to the restrictions. The first Question 2


restric- tion says that there must be at least two The correct answer is choice b. Start by seeing
females, so we can write “at least 2” next to the what deductions can be made from the
female group: information that G is selected. We have already
done the work before get- ting to the question
M: A, B, C (with the long conditional chain we constructed).
F: d, e, f, g (at least 2) g is in, A is out, e is out, d is out, and B is in:

The second restriction, which says that if d is M: A, B , C


in the section, e must be in the section, can be F: d, e, f, g (at least 2)
written:
Since we must have at least 2 females, f must
d S e (and not e S not d) be in as well, and since we must have at least
four stu- dents, C must be in the section. Now
The third restriction can be written: check each an- swer choice against your
diagram.
(A or e) S not g and “g S (not A and
not E)” M: A, B , C
F: d, e, f, g (at least 2)
The fourth restriction can be
Choice a is incorrect because A is out. Choice
written: B or d b is correct because both B and C are in the
section. Circle this answer choice and move on to
This can also be written as “not d S B” and the next question.
“not B S d” (disguised disjunctions).
Now look for connections between the Question 3
restrictions. The third restriction can be connected The correct answer is choice b. If A is in the
with the contrapositive of the second restriction, section, then according to the contrapositive of
which can be connected with the disguised the third re- striction, g is not selected.
disjunction version of the fourth restriction:
M: A , B, C
g S not A and not e S not d S B F: d, e, f, g (at least 2)

Question 1 Since we need at least two of d, e, and f, and


The correct answer is choice b. This is a since d cannot be selected without e, we are going
standard “Test-the-Rules” question. The first to need e in the section (the way we might think e
restriction rules out a, since only one female is is not selected is if d and f are selected—but
selected. The second restriction rules out e, selecting d requires se- lecting e). So choice b is
since D is in the section, but e is not. The third the correct answer.
restriction rules out c, since A is in the section If you did not see this deduction right away,
and g is in the section as well. The fourth then you might have considered each answer choice
restriction rules out d, since neither B nor d are in in turn. Is it possible for B to not be in the section?
the section. It seems so, since the only restriction on B is the
fourth, which can be satisfied by making sure d
is in the section:
90
selectioN games reView

Practice Game 4
M: A , B, C This game uses a cross-group domain, so
F: d , e, f, g (at least 2) drawing a grid will clarify the characteristic of
each entity and the mechanics of the game by
Circle e, and you have an acceptable roster. providing a visual framework. In this diagram,
So a “d” is for “deciduous,” “S” for “sapling,” etc.
is incorrect. Write the short form of the re- strictions next to
Now try choice b. If e is not selected, then d the grid. We write “2 of each cate- gory” to
is not selected. So g, d, and e are not selected. remind ourselves that each row (the s row and f
These three are females, and so restriction 1 will row) and each column (the d column and e
be violated, and choice b is the correct answer. column) will have to have exactly two species
underlined (se- lected for the park).
Question 4
The correct answer is choice a. You need to try d e
each answer choice in turn, to see if it could be S A HIF
true. For questions like this, the answer choices F BDG CE
that put the most numerical strain on the
situation should be the first suspects. Can all 2 of each
four females be in the sec- tion? Test each rule category I or C
with d, e, f, and g in the section. According to
the third restriction, if e is in the section, g cannot Remember to leave enough space under the
be—so a violates the third restriction, and is the let- ters representing the species for you to
correct answer. underline and erase as you go through questions
and consider the rel- evant scenarios (this will save
Question 5 you the trouble of recre- ating the grid again and
The correct answer is choice d. If B is not again). You can also write an “in” and “out”
selected, then d must be selected (by the fourth column next to each question to keep information
restriction). If d is selected, then e must be concerning previous selection scenarios you found
selected (by the second restriction). If e is acceptable.
selected, g cannot be selected (by the third It might pay off to do a bit of thinking about
restriction). So we have: how the selection could occur up front.
This grid should help. You want two of
M: A, B, C each column, and two of each row, so consider the
F: d , e , f, g (at least 2) various ways that could happen. You could
have one tree selected from each of the four
Take each answer choice in turn, and test boxes. Or you could have two selected from one
it against this diagram. The diagram says that D box, and two selected from the box diagonal to that
must be in the section, so choice a is incorrect. The box (this could only occur for the lower left and
diagram says that e is in the section, so b and e upper right boxes, as there is only one tree in the
are incorrect. That leaves c and d. Note at this upper left box). Could you have two in one box,
point that the dia- gram says that at least two of and one in a box above it or next to it? No,
the remaining letters, A, C and f, must be because then you would have three in a column or
selected to get the required four students. That row, and that violates our restriction that there are
rules out choice c, so choice d is the correct exactly two in each column and row. For the same
answer. reason, you could not have three selected from a
single box.
91
selectioN games reView

So we are really left with two possibilities: 1) from the


one from each box, or 2) two from the lower left
and two from the upper right. In either scenario,
we must make sure that the restriction that
either I or C is selected is satisfied.

Question 1
The correct answer is choice b. A scan of the
distribu- tion of letters from each answer choice
on your grid should immediately tell you which
work and which don’t. Choice a gives you one in
each box, but neither I nor C are selected, so
choice a is incorrect. Choice b gives you two in
the lower left box and two in the upper right box,
and I is selected, so choice b is correct. Choice c
does gives you three in the right column (meaning
three evergreens are selected), and so is incorrect.
The same goes for choice d. Choice e gives you
four in the lower row (four fully-grown trees), and
so is incorrect.

Question 2
The correct answer is choice b. Take each
answer choice and check it against diagram to see
if it is pos- sible to select those two. That is, mark
the two given in your answer choice (either in
your head or with your pencil on the diagram),
and then see if you can select two others in
keeping with the restriction. Since B and D are in
the lower-left box, if they are selected, we must
also select two from the upper-right box. We could
choose I, to satisfy the requirement that I or C be
selected, and one other species. So it is possible to
choose both B and D and choice a is incorrect.
Since H and F are in the upper-right box, if they
are selected, then we must also select two from the
lower left box. Notice, however, that we need to
select I or C—and neither I nor C are in the
lower-left box.
So H and F cannot both be selected, and b is
the correct answer. Circle this answer choice and
move on to the next question.

Question 3
The correct answer is choice e. If both A and E
are selected, then we have one species selected
upper-left box, and one from the lower-right box. So answer choices to see if they show up.
we must select one from each of the two remaining H appears in choice e, so e is correct.
boxes (lower-left, and upper-right), otherwise we can- Another way to look at this is to see that
not get a distribution that leads to two from each row since E is selected, and either I or C must be
and two from each column (we already figured this selected, we already have two evergreens. So
out before we attacked the questions). Remember that I we cannot have another evergreen, and therefore
or C must be selected—but C is not available, because neither H nor F can be chosen.
it’s in the lower-right box.
We must select I, so choice e is correct. Question 5
If C is selected, then again, we must select one
Question 4 from each of the four boxes. Since A is the only
The correct answer is choice e. If E is selected, then choice avail- able in the upper-left box, A must be
we must select one from each box (as we discovered selected. So a is the correct answer. Note that we
in the set-up, we must either select two each from the don’t need to neces- sarily select I, since the
lower-left and upper-right boxes, or select one from condition that I or C must be selected is already
each of the boxes). Since E is already selected from the satisfied.
lower-right box, C cannot be. So I must be selected, to
satisfy the condition that either I or C is selected. If Practice Game 5
I is selected from the upper-right box, then nothing This game has a cross-group domain, so start by
else from that box can be selected—in particular, sort- ing the entities into a grid. Since the
neither H nor F can be selected. We now have a couple entities aren’t named, just use dots. Write the
of trees we know cannot be selected, so scan the short form of the

92
selectioN games reView

rules next to the grid, or in the grid. The since no Milwaukee band is selected. The third
restrictions that at least two jazz bands must be re- striction rules out choice e, since no Chicago
selected, at least one Chicago band must be band is selected (but that was already ruled out).
selected, and at least one Milwaukee band must The fourth restriction rules out choice a, since the
be selected, can be written di- rectly into the grid. Milwaukee jazz band is selected, but only one
Milwaukee folk band is selected.
C (at least 1) M (at least 1)
f • • • • • Question 2
j (at least 2) • • • The correct answer is choice a. If both Chicago
folk bands are selected, then we select both bands
Select 4–5 in the upper left box.
Mj S2 + Mf [0 -1 Mf S no Mj]
C M (at least 1)
Now consider each rule, and see if any up-
f  •• •
front deductions can be made. The first restriction
j (at least 1 more) •  •
says that at least 2 jazz bands must be selected.
This means that at least one Chicago jazz band Select 4–5
must be selected, since there is only one Mj S 2+ Mf [0–1 Mf S no Mj]
Milwaukee jazz band. So you can go ahead and
underline a Chicago jazz band (and get rid of the We now consider what other selections we
restriction that at least one Chicago band must be might make. We need at least one more jazz band
selected, since that is satisfied, and change the (one more entity from the lower row), so we could
note that at least two jazz bands must be selected select either the other band in the lower-left box,
to a note that at least one more jazz band must or the band in the lower-right (or both). But if we
be selected). select a band on the lower right—that is, an Mj—
then we’ll need to select at least 2 Mf’s (the fourth
C M (at least 1) restriction). So we would end up with six bands
f • • • • • selected, and that would violate the rule that we
j (at least 1 more) •  • can select only 4–5 bands. So we can’t se- lect the
Mj (and we can cross that out). So we need to
Mj S 2+ Mf [0–1 Mf S no Mj] select the other Cj (and can underline that).

The rest of the restrictions don’t seem to


C M (at least 1)
im- mediately yield deductions or concrete
f  •• •
information that can be incorporated into the
j (at least 1 more)   ✗
diagram, so move on to the questions. Consider
keeping a list of what’s selected next to each Select 4–5
question, for reference in subse- quent questions. Mj S 2+ Mf [0–1 Mf S no Mj]

Question 1 Now, we still need to select at least one


The correct answer is choice d. This is a band from Milwaukee (at least one from the right
standard “Test-the-Rules” question. The first column). But we just crossed out Mj, so we need
restriction rules out choices c and e, since only one Mf (and no more, otherwise we would have
one jazz band is se- lected. The second more than five bands).
restriction rules out choice b,
93
selectioN games reView

So we have: numerically implausible, you would just test


each answer choice in turn for compatibility
C M (at least 1) with the diagram; the further the correct answer
f   ✗✗ appears down the list, the longer it will take you to
j (at least 1 more)   ✗ do the problem. So try to see if information from
other questions can save you some time.
Select 4–5
Mj S 2+ Mf [0–1 Mf S no Question 4
Mj] In: Cf Cf Cj Cj Mf The correct answer is choice d. If exactly one
Chicago jazz band is selected, then the other jazz
Now we can test each answer choice against band we need (as per the first restriction) must be
this diagram. Choice a is incorrect because the the Milwaukee jazz band (glancing at the grid
Milwaukee jazz band is not selected. Choice b is makes this immediately obvious). By the fourth
incorrect because two Chicago jazz bands are restriction, we know that two Milwaukee folk
selected. Choice c is in- correct because one bands must be selected. So our dia- gram looks
Milwaukee folk band is selected, which is choice d like:
—circle your answer choice and move on to the
next question.
C M (at least 1)

Question 3
f • • •
The correct answer is choice a. First, see if any j (at least 1 more) ✗  
choices can be eliminated based on other
questions. The ac- ceptable selection list for Select 4–5
Question 2 rules out choices b and d. The answer Mj S 2+ Mf [0–1 Mf S no Mj]
to Question 1 rules out e. You are left with a and
c—take each answer choice and try to determine We can select one more from the
whether it could be accommodated on the remaining bands, if we like, without violating
diagram. If no Milwaukee folk band is selected, any restrictions. Scan the answer choices to see
then it can’t be the case that the Milwaukee jazz if any concrete information shows up. Choice d
band was selected. This is apparent by is correct because the diagram shows that at least
considering the con- trapositive of the fourth three Milwaukee bands must be chosen. Circle
restriction [or the fourth rule diagrammed into this answer choice and move on to the next
the grid]. So no Milwaukee band is selected at all. question.
But this would violate the sec- ond restriction, Choice a is incorrect because according to
that at least one Milwaukee band is selected. the diagram, we could select just one Chicago
So it can’t be the case that no Milwaukee band. This makes choice b incorrect as well.
folk band is selected, and a is the correct choice. Choice c is incorrect because according to the
Circle this answer and move on to the next diagram, we could select just two folk bands.
question. Choice e is incorrect because according to the
If you did not have the information from diagram, we could choose four Milwaukee
the other questions, you would start by looking bands.
for the most numerically implausible answer
choice (the one that says no member of some Question 5
group is selected), and testing it for compatibility The correct answer is choice c. We must figure
with the diagram. And if you don’t have a out how exactly two Milwaukee bands could be
sense of which is the most selected.
94
selectioN games reView

They could either both be Milwaukee folk bands, C S at least 2 lN [if 0–1 lN S not C]
or a combination of one Milwaukee folk band and (E or I) S B [not B S (not E and not I)]
one Milwaukee jazz band. But if we had the
Milwaukee jazz band, we would need two The restrictions do not produce any
Milwaukee folk bands to satisfy the fourth immediate deductions, so proceed to the
restriction, and we would end up with three questions.
Milwaukee bands. So we can’t choose the
Milwaukee jazz band (cross it out), and we need Question 1
to choose two Milwaukee folk bands (circle them). The correct answer is choice c. This is a
Since we need one more jazz band (besides the standard “Test-the-Rules” question. Refer to the
one already selected Chicago jazz band), and the grid as you check each restriction. The first
Milwaukee jazz band is crossed out, we must restriction rules out choice d, as only one nurse is
select the other Chicago jazz band. So our chosen (only one letter from the left column). The
diagram looks like: second restriction rules out choice b, since no
doctor is chosen (no entity from the right
C M (at least 1) column). The third restriction rules out choice a,
f • • ✗ since C is chosen, but two are not chosen from the
j (at least 1 more)  ✗ upper-left box (the local nurses). The fourth
restriction rules out choice e, since E is chosen,
but B is not.
We can select an additional band from the
It might be helpful to note that the correct
up- per-left box if we want. Now scan the answer
answer choice, in which A, B, C and F are selected,
choices to see if any of the concrete information
means that one possible complete selection list
we have in this diagram is mentioned. Choice c
includes 2 lN, 1fN, and 1fD.
says that exactly two Chicago jazz bands are
selected, so that is the cor- rect answer.
Question 2
Choices a and b are incorrect because
The correct answer is choice b. If exactly one
according to the diagram, we could either select
local nurse is selected, then C cannot be selected
no Chicago folk bands or one Chicago folk band.
(this is ap- parent by considering the
Choice d is incorrect because no Milwaukee jazz
contrapositive of the third restriction), so cross C
bands are selected. And choice e is incorrect
out. If C is not selected, then either E or I must
because exactly two Milwaukee folk bands are
be selected, since at least one doc- tor must be
selected.
selected. If E or I is selected, then by the fourth
restriction, B must be selected, so circle B. Scan the
Practice Game 6
answer choices—choice b says that B must be se-
Since this game involves a cross-group
lected, and so is the correct answer.
domain, set up a grid in which you place all the
nine candidates. Symbolize the restrictions, and
N (at least 2) D (at least 1)
either incorporate them into the diagram or
A, F, G, H
write them next to the diagram. l E, I
(exactly 1)
f B,D C
N (at least 2) D (at least 1)
l A, F, G, H E, I
f B, D C C S at least 2 lN [if 0–1 lN S not C]
(E or I) S B [not B S (not E and not I)]
95
selectioN games reView

Question 3 and we need to have at least two other entities: B,


The correct answer is choice d. We want to rule and either E or I. So a is the correct answer.
out the answer choices consistent with a full
selection that does not violate any of the Question 5
restrictions. We saw in Question 1, choice c, that
The correct answer is choice c. Test the most
A, B, C, and F is an ac- ceptable selection, so that
numeri- cally restrictive answer choices first. (If
rules out a, c, and e. E and I can be selected, as
you had an- swered Question 1, where the
long as B is selected and another entity (not C), so
acceptable list was 2 lN, 1fN, and1fD, you could
b is incorrect. That leaves choice d. If you had not
rule out choices b and d be- fore testing the most
answered Question 1, then you would have to
numerically restrictive answer choice). Can exactly
check each answer choice in turn to see if you
three nurses be chosen? It seems we can—for
could make a full selection consistent with that
example, a selection of A, F, G, C does not violate
choice which does not violate any restrictions.
any restrictions. So choice a is incorrect. Now
Choices a through c would give no inconsistencies.
move to choice c. Can exactly three foreign
When you reach choice d, you would reason as
profession- als be chosen? If three foreign
follows. If C and E are both selected, then B must
professionals are chosen, then we choose all three
be selected (by the fourth restriction, because E is
on the bottom row, including
selected), and two local nurses (A, F, G, and H)
C. And if C is chosen, we must choose two from
must be selected (by the third
the upper left box. But that would leave us with a
restriction, because C is selected). total of five selected. So all three foreign
But that would mean that five professionals professionals can’t be chosen, and c is the correct
are selected (C, E, B, and two of A, F, G, and H) answer.
—so it can’t be that C and E are both selected. You might have also realized that since
So d is the correct answer. there are only three foreign professionals
available (vs. six nurses available), the most
Question 4 numerically restrictive choice is really c, and
The correct answer is choice a. If C is not not a—and you would have saved a bit of time.
selected, then since at least one doctor must be
selected, E or I must be selected. If E or I is Practice Game 7
selected, then B must be selected, so circle B. This game has a simple domain type, so list the

N (at least 2) D (at least 1) entities: A B C D E F G H


l A, F, G, H E, I
f B,D C Symbolize the restrictions:

At most 3 of A, B, C,
C S at least 2 lN [if 0–1 lN S not C]
D At least 2 of E, F,
(E or I) S B [not BS (not E and not
G
I)]
C S not D [D S not C]
A S (B and C) [ (not B or not C) S not
So the selection will have to be: E or I, B,
A ] not A S H [not H S A] [A or H]
and 2 more.
Now consider each answer choice, and
Now see if any deductions can be made
see whether it is ruled out by this diagram. Can
from the rules. Not C seems to be a linking
A, F, G be a partial list? No, because there is only
element— combining the third restriction,
one spot left,
contrapositive of the fourth restriction and gives
us:

96
selectioN games reView

D S not C S not A S H Question 1


The correct answer is choice e. The first
Next, look for positive information about restriction does not rule anything out. The
what must be selected. We know that two of E, second restriction rules out choice c, since it
F, and G must be selected. So we can start a includes only one of E, F and G. The third
selection list. restriction rules out choice d, since she applies to
both C and D. The fourth restriction rules out
E/F/G E/F/G (_ _ _) choice a, since A is chosen but C is not. The fifth
restriction rules out choice b, since she applies to
Notice that the fourth restriction is neither A nor H.
numerically restrictive—if A is chosen, then we
have a full selec- tion list: Question 2
The correct answer is choice a. If the student
E/F/G E/F/G A B C applies to college H, then we must be in scenario
2. We know right away that she does apply to
This seems a fairly concrete scenario, so let’s college A in that sce- nario, so choice a is correct.
see if we can set up two scenarios, one in which A Without the scenarios, we could observe that
is cho- sen, and one in which A is not. If A is not H and at least 2 of E, F, and G take up 3 spots,
chosen, then by the fifth restriction, H must be. leaving at most two spots left. Since A has two
So we have: followers (B and C), there’s no room for A.

E/F/G E/F/G H (_ _) A Question 3


The correct answer is choice b. Compare each
We can now choose 0–2 of B, C/D, or answer choice to the scenarios to see if either
E/F/G. We cannot choose both C and D (by the can accom- modate that set of two schools.
third re- striction). We can write “E/F/G” again Choice a is incorrect because Scenario 2 allows an
because two of those three will have been application to both H and B. Choice b is correct
selected, and one will be left to choose from. because A is only chosen in Scenario 1; according
These two scenarios are not as concrete as the scenario 1 diagram, it is not possible to select
we might like them, but they are enough to start D.
attack- ing the questions. We have: Without the aid of the scenarios, you could
also see that b is correct by looking at the
S1 deduction chain we made up front:
E/F/G E/F/G A B C
D S not C S not A S H

S2 Question 4
E/F/G E/F/G H (_ _) A The correct answer is choice e. A quick glance at
0–2 of B, C/D, or E/F/G. our scenarios shows us that if the student applies
to col- lege D, we must be in scenario 2. And in
Since the scenarios are not very concrete, we scenario 2, the only sure piece of information is
will also show how to answer the questions that she applies to H. This is choice e.
without the scenarios. Without the scenarios, you would reason as
follows. Since we have D, we cannot have C
(third
97
selectioN games reView

restriction). Since we do not have C, we cannot FSRHAG


have A (fourth restriction). Since we do not have A, C IN: A or H
we must have H (fifth restriction). Since the OUT: G or S
student applying to H is an answer choice, we
stop here. Now look for connections among rules. The
first and fourth restrictions connect up through
Question 5 G:
The correct answer is choice c. If the student
applies to A, we know we are in scenario 1. S S not G S not F [and F S G S not S]

S1 The second and third rules connect up


E/F/G E/F/G A B C
through A:
Check each answer choice to see whether
it is compatible with this diagram. E and C not H S A S not R [and R S (not A
could be selected, so a is incorrect. C and G could and not C) S H]
be selected, so b is incorrect. E, F, and G could
not be selected— there is room for only two of The other thing to keep in mind is that there
the three—and so c is the correct answer. is a disguised disjunction: any selection of languages
Without the scenarios, you would reason must have either A or H or both.
as follows. If we have A, we must have B and C We now turn to the questions.
(fourth restriction). By the first restriction, D must
be excluded. But more relevantly, this also only leaves Question 1
two extra spots, so any answer choice with three The correct answer is choice c. This is a
letters none of which are A, B, and C will have to standard “Test-the-Rules” question. The first
be incorrect. This is choice c. restriction rules out b, since French is chosen but
German is not. The second restriction rules out
Practice Game 8 choice e, since both Arabic and Russian are
This game has a simple domain, so merely list chosen. The third restriction rules out choice a,
the symbols for the entities: since neither Arabic nor Hindi are present. The
fourth restriction rules out d, since both Spanish
FSRHAGC and German are chosen.

Now, symbolize the rules: Question 2


The correct answer is choice d. If the book is
F SG [not G S not F] trans- lated into Russian, then we know from our
(A or C) S not R [R S (not A and not deduction chain, R S (not A and not C) S H,
C)] not A S H [not H S A] [A or H] that A and C are not selected, and H is selected.
S S not G [G Snot S] [GS] So we have:

The last restriction can also be written FSRHAGC


as disjunction in an “out” column next to your OUT: G or S
entity list. If we do this, we can also write in the
third restriction as a disjunction in an “in” There are now two selected (R, H), and three
column: left (F, S, G) to select from. Can we select all
three? We know from the fourth restriction that G
and S cannot be selected together. So the most we can select is one of

98
selectioN games reView

those two (G or S) and F, in addition to R and H. Question 4


Let’s pick one to see if it works: G, F, R, H. This The correct answer is choice e. Look again at your
selection does not violate any of the deduc- tion chains to see what follows from the
restrictions, so it works. And so the maximum book not being translated into Hindi. According
number we can select is four, choice d. to the chain not H S A S not R, the book must be
Note that if picking one of the two, G or S, translated into Arabic and not translated into
had not worked, we would have had to try the Russian. Scan the answer choices— choice e is that
other before concluding that we can’t have four the book is not translated into Russian.
entities selected.
Question 5
Question 3 The correct answer is choice b. If the book is
The correct answer is choice a. Test each answer not translated into Hindi, then according to the
choice in turn to see if a selection including the two deduc- tion chain not H S A S not R, it is
languages is consistent with all the restrictions. translated into Arabic and not translated into
Suppose Spanish and French are both selected. Go Russian. If the book is not translated into German,
through each restric- tion to see what happens. By then according to the de- duction chain S S not
the first restriction, since French is selected, G S not F, it is also not trans- lated into French.
German must be selected. The sec- ond restriction So we have:
does not impact these selections, since we haven’t
selected Arabic, Chinese, or Russian yet. The third FSRHAGC
restriction simply says that we must add Arabic
or Hindi to this selection. The fourth restric- tion Now check each answer choice against this
says that since Spanish is selected, German cannot be selec- tion diagram. Choices a and e are incorrect
selected. Here is our contradiction; if both Spanish because they include French, which is crossed
and French are selected, we would have to both out. Choices c and d are ruled out because they
select and rule out German, so it can’t be that both include Russian, which is crossed out. Choice b
Spanish and French are selected. The correct includes Spanish and Chinese—two languages
answer is a. that could be selected ac- cording to the selection
We could also very easily arrive at this diagram—and nothing else.
conclusion by glancing at our deduction chain, S So b is the correct answer.
S not G S not F, which immediately tells us
that if Spanish is selected, French cannot be.
Your first instinct for a question like this is to
glance at deductions you have already made to
see if the answer is obvious.
99
5
chapt e r

distribution games
review

In a distribution game, you are given a set of entities that are to be distributed into two or more groups.
The entities can be persons, places, or things—for example, it may be a set of eight movies which are to be
classified into drama, comedy, and thriller genres, or it may be a set of college students who are to be
divided into three sections. The distribution into groups takes place according to certain rules given to you (for
example, that if stu- dent X is in Section 1, student Y must be in Section 2, or that there must be at least two
students in each section).
In some ways, distribution games are similar to selection games—indeed, one can think of a selection
game as a distribution game with just two groups into which to distribute the entities—namely the “in”
group and the “out” group—such that each entity is distributed into exactly one group. For this reason,
many of the game rules in distribution games will be similar to those we have seen in selection games.
But there will be some rule types that are somewhat different.

what’s Being distributed:


domain types and symbolizing the
domain

We saw that in selection games there are three types of domains (or master group of entities that are to
be dis- tributed)—the simple domain, the subgroup domain, and cross-group domain. Fortunately,
distribution games virtually never feature the cross-groups, which are much more complex than simple
groups and subgroups.
101
–distriButioN games reView–

f Nevertheless, recognizing simple domains and sub- perhaps label them as well (e.g., we label the female
group domains when you see them in group ‘F’ and the male group ‘M’ below).
distribution games will help you anticipate and For example, you might write:
understand the kinds of rules and diagramming
that will be at work in the game. F: A B C
M: d e f g
Simple Domain
The simplest domain is a plain list of some
number of entities, with no further distinction understanding types of
between them. For example, you may be asked to distribution
distribute seven students—A, B, C, D, E, F, and G
—into two sections, Section 1 and Section 2. The The matching of entities in the domain to the
list of seven students is your simple domain. groups usually occurs in one of four ways,
Your diagram for the simple domain will depending on whether or not every entity in the
simply be a list of the entities: domain is distrib- uted into a group, as well as on
whether or not the enti- ties can appear in more
ABCDEF than one group. Again, being aware of these
nuances will help you anticipate and understand
Subgroup Domain the unique mechanics of different distribu- tion games
Some domains consist of subgroups—that is, a —in particular, it will draw your attention to
further distinction is made between entities in the whether every entity must find its way into a
domain. For example, you may be asked to group and whether you can use entities more
distribute seven students, some of whom are than once.
female (A, B, and C) and some of whom are male You can visualize these four different kinds
(D, E, F, and G), into two sec- tions. The two of distribution in the following table. For illustration,
subgroups are the females and males. we suppose here that five entities—A, B, C, D, and
The presence of subgroups in the domain E— are to be distributed into two groups,
allows for a greater variety of game rules. The Group 1 and Group 2.
rules govern- ing the distribution of students into The simplest kind of distribution game is
the two sections will not only make reference to rep- resented in the upper left box—for these
the entities (e.g., “if student A is in Section 1, games, once you put an entity into a group, you
student B is in Section 2”), but also make can cross it off your master list. You are told that
reference to the subgroups (e.g., “if there is a male an entity can’t be in a particular group, so you
in Section 1, at least two females must also be in know that it must be in some other group. The
Section 1”). other boxes represent slightly more complex
One way to think about distribution distribution games. When you first assess a
games with subgroup domains is that you are “re- distribution game, pay attention to what distribution
grouping” the entities: for example, a master list type is at work.
of students is grouped into male and female, Also note that a two-group game from the
and you want to re- group those students in upper left box is really a selection game with two
Section 1 and Section 2. groups, the “in” group and the “out” group. This
When symbolizing a domain consisting of does not apply to any two-group game represented
sub- groups, use capitalization to distinguish by the other three boxes, nor to any three-or-more
them, and group game.
102
–distriButioN games reView–

SoMe entItIeS not


eVeRY entItY dIStRIBUted
dIStRIBUted

RePeatS In GRoUPSno RePeatS In GRoUPS


entItIeS GRoUPS entItIeS GRoUPS
A A

B 1 B 1

C C

D 2 D 2

E E (not in any group)

entItIeS GRoUPS entItIeS GRoUPS


A A

B 1 B 1

C C

D 2 D 2

E E (not in any group)

diagramming the distribution


Maximums and Minimums
How you diagram the action of the game will If there is some numerical restriction—that is,
depend on a number of factors. How you set up if a maximum and minimum are given—draw a
the initial dia- gram will depend on the numerical series of slots for each group. First, draw the
restrictions on the groups into which the entities minimum num- ber of slots, and then draw extra
are to be distributed. slots in parentheses up to the maximum.
For example, if you are told that Group 1
No Restrictions has a minimum of three and maximum of four
If there are no numerical restriction—i.e., if there members, and Group 2 has a minimum of four
is no minimum or maximum number of entities in and maximum of six members, you would
each subgroup—then simply make two columns, draw:
labeled with the names of the groups:
1 2
1 |2 _ _ _ (_) _ _ _ _ (_) (_)
|
| Exact Numbers
| If you are given the exact numbers for each
group, simply draw the corresponding number of
slots. For
103
–distriButioN games reView–

example, if you are told that Group 1 has four we know that D and E are definitely in Group 1,
mem- bers and Group 2 has five members, you that B is definitely not in Group 2, and we want to
would draw: remind ourselves that C could be in Group 1, we
might draw:
1 2
____ _____ 1 2
D E A (_) f
Hybrids fm
Of course, you can mix and match, depending on c B
the game. For example, if you are told that Group You should come up with a tracking system
1 has exactly four members, and Group 2 has a that works for you, as you come across different
minimum of three and maximum of four, you rules. For example, if an entity must be in one of
might draw: two groups but cannot be in a third, consider
putting a letter for the entity down and drawing
1 2 two arrows from the entity, one to each of the
____ _ _ _ (_) two groups.

Subgroup Domains
If you have a subgroup domain, you will want how things are distributed:
to leave room under your slots so that you can the rule types and major players
label that spot with a particular subgroup. For
example, if you are told that each of two A typical distribution game will present a set of
sections (Section 1 and Section 2) must have at rules which will determine how the selection
least one female student, and that Adam (a male must, can, or can’t occur. There are four basic
student) is in Section 2, you might draw: types of rules typical to a distribution game.

1 2 Blocks
A (_) A block is a group of entities that must be placed to-
f fm gether. If one member of the block is in the group,
The “f” reminds you that at least one student then the other must be, and vice-versa. Rules that
in each group must be female, and the “m” produce blocks tell you that two entities must be
reminds you that A is male—information that in the same group. For example, you may be told
might prove useful as you work through a game. that “student A is in the same section as student
C.” Blocks are sym- bolized by placing the
Keeping Track of the Distribution symbols for two entities next to one another:
You will want to keep track of which entities are
defi- nitely in a group, which are definitely not in AC
a group, and which could be in a group. Place
letters for entities in a group in the slots. If an Anti-Blocks
entity is definitely not in a group, write the letter An anti-block is a pair of entities that cannot be in
for that entity next to or under that group and the same group. For example, you may be told that
cross it out. If an entity could be in a group, “student A is not in the same section as student C,”
write the letter for that entity next to or under the or “if student A is in a Section 1, student C is not
group and put it in a parentheses. For example, if in that section.”
104
–distriButioN games reView–

Note that this negative conditional works the same “m” and “a”, so we can write:
way as it did in the selection games—the
contrapositive is that if student C is in a section,
student A is not in that section. Anti-blocks are
rarely presented as condition- als in distribution
games; usually, the wording is fairly
straightforward (“X and Y are not in the same
group”). Anti-blocks are symbolized by placing the
symbols for the entities next to one another and
crossing them out:

AC
Anti-blocks sometimes make reference
to domain subgroups, if those are present. For
example, you may be told that “the two males
are not in the same section,” or that “student A
is not in the same section as either of the female
students.”

Conditionals
We saw in selection games that conditional rules
were rampant and took on many different forms.
In con- trast, distribution games usually have
fairly straight- forward positive conditionals, such
as “if student A is in Section 1, then student C is
in Section 2.”
Some conditionals will be about the
same group (as in selection games)—e.g., “if
student A is in Section 1, then student B will
also be in Section 1.” Occasionally, the
conditional will be slightly more complex—e.g.,
“if student A is in Section 1, then stu- dent D is
also in Section 1 and student C is in Section 2.”
We can symbolize these conditional rules as
follows:

A1 S C2
A1 S (C2 and
D1) A1 S B1

If you are symbolizing both the entities


and groups with letters, it sometimes helps to use
parenthe- ses. For example, we might be given the
restriction that “if student X is in the morning
section, then student Y is in the afternoon section.”
We might symbolize the sections with a lower-case
X(m) S Y(a) notation often makes such distribution games
If the distribution game you are dealing with is really a much easier to handle for many test-takers.
selection game (that is, if it is a two-group game from the To illustrate how this notation works, some of the
upper-left box, as discussed above), then you can change answer explanations to the practice games in this
your symbolizations to make them much simpler. For chapter will use it. You will have to see which
example, suppose that the condition just given—“if student X method makes more sense to you for such two-
is in the morning section, then student Y is in the afternoon group/upper-left-box games.
section”— applies to a game in which there are just two
sections, and every student must be in exactly one section. Number Rules
Then we consider this a selection game, in which the morning As in selection games, number rules are often the
section is the “in” group and the afternoon section is the most important rules, playing a large role in driving
“out” group. Then if a student is in the morning section, we the me- chanics of a game. Number rules always
can consider her selected, and if a student is in the afternoon tell us some- thing about the number, or relative
section, we can consider her not selected. So we can rewrite our number, of entities that comprise a group. These
symbolization: rules often inform your initial game set-up and
the associated diagram.
X(m) S Y(a)
as Section 2 has at most four students.
XS≠Y There is at least one female in every section.
There is at least one male in the same section
This means that if X is selected, Y is not selected. This as student A.

105
–distriButioN games reView–

practice game 1

Seven census workers—Patrick, Quincy, Ramesh, Shawn, Terry, Unger, and Victoria—are each assigned to
survey either the West side or East side of town, but not both. Their assignment is governed by the
following restrictions:

If Patrick surveys the West side, then Shawn also surveys the West
side. If Terry surveys the East side, then Unger surveys the West
side.
If Ramesh surveys the East side, then both Shawn and Unger survey the East side.
If Victoria surveys the West side, then Patrick surveys the East side and Quincy surveys the
West side. Unger surveys the East side if Patrick surveys the East side.

1. Which one of the following could be a 4. If Patrick and Shawn survey different sides
complete and accurate list of the census of the city, then what is the least number of
workers survey- ing the East side? census workers that could be assigned to
a. Terry, Victoria survey the East side of the city?
b. Patrick, Quincy a. one
c. Patrick, Ramesh, Shawn b. two
d. Ramesh, Shawn, Unger c. three
e. Patrick, Ramesh, Shawn, Victoria d. four
e. five
2. If Ramesh surveys the East side, then
which one of the following statements 5. Which one of the following CANNOT be a
must be true? pair assigned to survey the East side?
a. Shawn surveys the West side. a. Shawn and Terry
b. Unger surveys the West side. b. Patrick and Shawn
c. Patrick surveys the West side. c. Patrick and Unger
d. Terry surveys the West side. d. Terry and Victoria
e. Quincy surveys the West side. e. Ramesh and Shawn

3. If Unger surveys the West side, then each of


the following statements must be true
EXCEPT:
a. Patrick surveys the West side.
b. Shawn surveys the West side.
c. Victoria surveys the East side.
d. Ramesh surveys the West side.
e. Quincy surveys the East side.
106
–distriButioN games reView–

practice game 2

Six journalists—A, B, C, D, E, and F—are assigned to cover the two major events of the day, a political
rally and an air show. Each journalist covers one event or the other, but not both, and the assignment of
journalists to events is governed by the following rules:

If B covers the political rally, then E also covers the political


rally. If G covers the air show, then both E and F also cover the
air show. If F covers the air show, then C also covers the air
show.
If D covers the air show, then F covers the political rally.

1. Which one of the following could be a 4. What is the minimum number of


complete and accurate list of the journalists journalists that could be assigned to cover
covering the air show? the political rally?
a. F, E a. zero
b. E, F, G b. one
c. B, F,G c. two
d. B, D, C d. three
e. B, D, F e. four

2. If F and C cover different events, then each 5. If G covers the air show, then what is
of the following could be true EXCEPT: the minimum number of journalists
a. D covers the air show. that must cover the airshow?
b. G covers the air show. a. one
c. E covers the air show. b. two
d. D covers the political rally. c. three
e. B covers the political rally. d. four
e. five
3. If B covers the political rally, then which one
of the following must be true?
a. G covers the political rally.
b. E covers the air show.
c. F covers the political rally.
d. D covers the political rally.
e. C covers the air show.

107
–distriButioN games reView–

practice game 3

Seven students—H, I , J, K, L, M and N—are to design a product. They are divided into three teams—an
en- gineering team consisting of three members, a development team consisting of two members, and a
branding team consisting of two members. Each student is on one and only one team. The students are
divided accord- ing to the following restrictions:

Student H is on the development team.


Student J and student K are on the same
team.
Student L and student H are on different teams.
If student L is on the engineering team, then student N is on the branding team.

1. Which one of the following could be an 4. If student N and student I are on the
accept- able assignment of students to same team, then each of the following
teams? statements must be true EXCEPT:
a. Student N is on the branding team.
Engineering Development Branding b. Student M is on the development team.
a. M N I HL JK c. Student L is on the engineering team.
b. J K H NI LM d. Student K is on the engineering team.
c. J K L HM NI e. Student I is on the engineering team.
d. J I L HK MN
e. J K L HN IM 5. Which one of the following statements
CAN- NOT be true?
2. If student N is on the engineering team, a. Student I is on the branding team.
then which one of the following statements b. Student K is on the branding team.
must be true? c. Student M is on the development team.
a. Student I is on the branding team. d. Student N is on the engineering team.
b. Student I is on the development team. e. Student L is on the engineering team.
c. Student M is on the branding team.
d. Student L is on the branding team.
e. Student J is on the branding team.

3. Which one of the following is a complete


and accurate list of all the students any one
of which could be on the development
team?
a. H, I, M
b. H, M, N
c. H, I, K, L
d. H, I, M, N
e. H, I, K, M, N
108
–distriButioN games reView–

practice game 4

Three artifacts discovered on an archaeological dig—a pot, a coin, and a mirror—are to be examined by
six archaeologists—Q, R, S, T, U, and V. Each artifact is examined by exactly two archaeologists, and each
archae- ologist examines exactly one artifact, subject to the following constraints:

Q and R do not examine the same


artifact. S and T examine the same
artifact.
If S examines the coin, then U examines the mirror.

1. Which one of the following could be an 4. If V examines the mirror, then each of the
accept- able assignment of archaeologists to fol- lowing statements could be true
artifacts? EXCEPT:
a. Q examines the coin.
Coin Pot Mirror b. U examines the pot.
a. QR ST UV c. U examines the coin.
b. ST UQ RV d. R examines the coin.
c. UV SQ TR e. R examines the mirror.
d. ST QR UR
e. ST RV UQ 5. If Q examines the pot, then each of the
follow- ing archaeologists could examine
2. If T examines the coin, then which one of the mirror EXCEPT:
the following statements must be true? a. V
a. V examines the pot. b. S
b. Q examines the pot. c. T
c. R examines the mirror. d. R
d. R examines the pot. e. U
e. U examines the pot.

3. Each of the following is a pair of


archaeologists that could examine the same
artifact EXCEPT:
a. Q and V
b. R and V
c. Q and U
d. V and U
e. R and U
109
–distriButioN games reView–

practice game 5

Seven adventure scouts—I, J, K, L, M, N, and O—take two rafts for a white-water rafting trip. Each raft
holds at least three and at most four scouts. Every scout boards exactly one of the two rafts, a front raft
and a rear raft, according to the following conditions:

If J boards the front raft, then M boards the rear


raft. M and N do not get on the same raft.
N is on the same raft as
O. K is on the rear raft.
If O is on the rear raft, I is also on the rear raft.

1. Which one of the following is an 4. Which one of the following CANNOT be a


acceptable roster of scouts on rafts? pair of scouts in the rear raft?
a. J and K
Front Rear b. L and I
a. J, M, N, O I, K, L c. I and M
b. N, O, J, I L, K, M d. L and O
c. J, M, N O, I, K, L e. K and L
d. I, J, O, L M, N, K
e. L, M, K N, O, I, J 5. Each of the following could be a true
statement EXCEPT:
2. If J and L take different rafts, then how a. M takes the front boat.
many different possible assignments of scouts b. I takes the front boat.
to rafts are there? c. L takes the rear boat.
a. one d. J takes the front boat.
b. two e. K takes the rear boat.
c. three
d. four
e. five

3. Which one of the following could be a


partial, accurate list of the scouts on the
front raft?
a. J, L, O
b. K, L, O
c. I, J, L, N
d. I, J, L, O
e. J, K, N, O

110
–distriButioN games reView–

practice game 6

Six menu times—burgers, pizza, sandwiches, tacos, nachos, and chili—are to appear on three menus: a
lunch menu, a dinner menu, and a night menu. Each menu item appears on at least one menu. Each
menu has exactly three items. The menus are constructed according the following restrictions:

One item appears on all three menus.


Burgers and sandwiches never appear on the same menu.
Tacos and nachos never appear on the same menu.
Chili appears on the dinner menu.

1. Which one of the following is an acceptable 3. If burgers appear on the lunch and
set of menus? dinner menus and chili appears on the
a. Lunch: pizza, tacos, chili night menu, then which one of the
Dinner: chili, burgers, sandwiches following statements CANNOT be true?
Night: chili, nachos, tacos a. Chili and nachos appear on the lunch menu.
b. Lunch: pizza, tacos, sandwiches b. Pizza and nachos appear on the lunch
Dinner: chili, burgers, nachos menu.
Night: chili, pizza, tacos c. Chili and tacos appear on the night menu.
c. Lunch: pizza, tacos, nachos d. Sandwiches and nachos appear on the
Dinner: chili, pizza, sandwiches night menu.
Night: pizza, burgers, tacos e. Burgers and tacos appear on the
d. Lunch: pizza, burgers, tacos dinner menu.
Dinner: chili, sandwiches, pizza
Night: pizza, nachos, chili 4. Which one of the following statements must
e. Lunch: pizza, chili, tacos be true?
Dinner: pizza, nachos, burgers a. Chili and pizza appear on a menu
Night: pizza, sandwiches, tacos together.
b. Burgers and tacos appear on a
2. If chili appears on at most two menus, then menu together.
which one of the following statements must c. Burgers and nachos appear on a
be true? menu together.
a. Pizza appears on exactly three menus. d. Sandwiches and tacos appear on
b. Tacos appear on exactly three menus. menu together.
c. Sandwiches appear on exactly three e. Sandwiches and nachos appear on a
menus. menu together.
d. Pizza appears on exactly two menus.
e. Tacos appear on exactly two menus. 5. If chili and nachos always appear on a
menu together, then which one of the
following state- ments must be true?
a. Burgers appear on the lunch menu.
b. Sandwiches appear on the lunch menu.
c. Tacos appear on the lunch menu.
d. Burgers appear on the night menu.
e. Tacos appear on the dinner menu.
111
–distriButioN games reView–

practice game 7

NASA is launching two simultaneous shuttle missions, using the shuttles Freedom and Independence.
Four astronauts are assigned to each shuttle, chosen from three veteran astronauts —A, B, and C—and
five novice astronauts—V, W, X, Y, and Z. The astronauts are assigned according to the following
conditions:

Astronauts A and C cannot be on the same


shuttle. Astronaut Y is on the Independence.
If Astronaut B is on the Freedom, then Astronaut X is on the
Independence If Astronaut A is on the Independence, then Astronaut Z
is on the Freedom.

1. Which one of the following could be the 4. If exactly two veteran astronauts are on
roster of astronauts on board the board the Freedom, then which one of the
Freedom? following statements must be true?
a. A, V, Y, Z a. A is aboard the Freedom.
b. A, B, V, X b. X is aboard the Independence.
c. A, B, C, V c. C is aboard the Independence.
d. A, V, W, Z d. Z is aboard the Freedom.
e. B, C, V, W e. B is aboard the Independence.

2. If Astronaut B and Astronaut X are 5. If Astronaut X is aboard the Freedom, then


aboard the same shuttle, then each of the which one of the following statements must
following be true?
could be a pair of astronauts on board the a. A is aboard the Freedom.
same shuttle EXCEPT: b. C is aboard the Freedom.
a. V and W c. Z is aboard the Freedom.
b. A and Z d. Exactly two novices are aboard the Freedom.
c. B and C e. Exactly three novices are aboard
d. C and X the Freedom.
e. B and Z

3. If Astronaut Z is aboard the


Independence, then which one of the
following statements must be true?
a. C is aboard the Independence.
b. A is aboard the Independence.
c. B is aboard the Freedom.
d. X is aboard the Independence.
e. X is aboard the Freedom.
112
–distriButioN games reView–

practice game 8

A joint agency task force is formed to investigate two cases—the Appleby case and the Brentwood case.
The task force is made up of seven officers: three FBI agents (X, Y, and Z) and four detectives (H, I, J, K).
Each case has at least one agent and at least one detective. All officers are assigned to one and only once
case, according to the following conditions:

H and I are not assigned to the same


case. I and J are assigned to the same
case.
If K is assigned to the Appleby case, then X is assigned to the Brentwood
case. If Y is assigned to the Appleby case, then I is also assigned to the
Appleby case. Each case is assigned at least one agent and one
detective.

1. Which one of the following could be a 4. If X and Y are both assigned to the
complete and accurate list of the officers Appleby case, then each of the following
assigned to the Appleby case? must be true EXCEPT:
a. I, J, K, Y a. I is assigned to the Appleby case.
b. H, I , J, K b. J is assigned to the Appleby case.
c. I, K, Y, Z c. H is assigned to the Brentwood case.
d. H, K, X, Z d. Z is assigned to the Brentwood case.
e. K, H, Y, Z e. K is assigned to the Appleby case.

2. If H is assigned to the Appleby case, then 5. If the Brentwood case is assigned the
each of the following could be a pair of fewest number of officers possible, then
officers assigned to the same case which one of the following must be
EXCEPT: assigned to the Brent- wood case?
a. H and Z a. H
b. X and Y b. I
c. H and I c. J
d. J and K d. X
e. H and K e. Y

3. If exactly three detectives are assigned to


the Brentwood case, then which one of the
follow- ing must be true?
a. K is assigned to the Appleby case.
b. H is assigned to the Appleby case.
c. H is assigned to the Brentwood case.
d. X is assigned to the Brentwood case.
e. X is assigned to the Appleby case.
113
–distriButioN games reView–

setup, answers, and


explanations Question 1
The correct answer is choice a. This is a “Test-
Practice Game 1 the- Rules” question. For each answer choice, you are
First, symbolize the restrictions, and their contraposi- given a putative complete and accurate list of the
tives. When finding the contrapositives, note that workers on the East side; which means any worker
if some worker does not survey one side of the not appearing on the list must be assigned to the
city, he or she must survey the other. This is a West side. Keeping this in mind should help you
consequence of the fact that this is a two-group see how the restrictions rule out answer choices.
distribution game from the upper-left box (along The first restriction rules out d, since Patrick
with the fact that if he or she surveys one side of is on the West side, but Shawn is on the East
the city, he or she does not survey the other). side. The second restriction doesn’t rule out any
of the answer choices. The third restriction rules
The first restriction: out e, since Ramesh is on the East side, but Unger
is not on the East side. The fourth restriction
P(w) S S(w) rules out b, since Victoria is on the West side, but
S(e) S P(e) Quincy is not on the West side. The fifth
restriction rules out c, since Patrick is on the East
The second side, but Unger is not.

restriction: T(e) S Question 2


The correct answer is choice d. If Ramesh surveys
U(w) the East side, then by the third restriction, both
U(e) S T(w) Shawn and Unger also survey the East side. If
Shawn surveys the East side, then by the
The third restriction: contrapositive of the first restriction, Patrick also
surveys the East side. And if Unger surveys the
R(e) S (S(e) and U(e)) East side, then by the second re- striction, Terry
S(w) or U(w) S R(w) surveys the West side.
It doesn’t seem like we can make any more
The fourth restriction: di- rect deductions from our conditional
statements, so we have:
V(w) S P(e) and Q(w)
P(w) or Q(e) S V(e) e w
RSUP T
The fifth restriction:
Choice d says that Terry surveys the West
P(e) S U(e) side, which is necessitated by our reasoning and
U(w) S P(w) diagram.

Notice that the V, Q, and T only appear in Question 3


one restriction each, and so will have the greatest The correct answer is choice e. If Unger surveys
likeli- hood of being unrestricted and without a the West side, then by the contrapositive of the
forced as- signment, depending on the situation. third re- striction, Ramesh surveys the West side,
and by the contrapositive of the fifth restriction,
Patrick also sur- veys the West side. If Patrick
surveys the West side,

114
–distriButioN games reView–

then by the first restriction, Shawn also surveys Question 5


the West side; and by the contrapositive of the The correct answer is choice a. First, see if any
fourth re- striction, Victoria surveys the East side. It of the answer choices appear in distribution
doesn’t seem like we can make any more direct scenarios you have already found to be acceptable
deductions from our conditional statements, so from other an- swer choices. Choice d is ruled out
we have: by the answer to Question 1. Choices b, c, and e
are ruled out given the diagram through which we
e w answered question 2. That only leaves choice a.
V URPS Without the aid of previous work, you must
go through each pair, and see if the pair could be
According to the diagram, all four of the assigned to the East side or if it would result in a
state- ments correspond to choices a through d contradiction. If Shawn is assigned to the East
must be the case; there is no deduction placing side, then Patrick is also assigned to East side
Quincy on the East side, so e is the correct (first restriction). If Terry is assigned to the East
answer. side, Unger will be assigned to the West side
(second restriction), meaning that Patrick would
Question 4 have to be assigned to the West side (fifth re-
The correct answer is choice b. First, look for striction)—but we already assigned Patrick to the
the re- striction connecting Patrick and Shawn— East side. Choice a results in a contradiction, and
this is the first restriction, which says that if is thus the correct answer.
Patrick surveys the West side, then so does
Shawn. So Patrick cannot survey the West side, Practice Game 2
lest he and Shawn survey the same side. So Patrick First, symbolize the restrictions and their
surveys the East side, and Shawn surveys the West contraposi- tives. Note that this is a two-group
side. Since Patrick surveys the East side, by the distribution game from the upper-left box, and so
fifth restriction, Unger also surveys the East side. can be considered a se- lection game. For this
Since Unger surveys the East side, then by the game, we will symbolize the re- strictions as
second restriction, Terry surveys the West side. though it were a selection game in which the ‘in’
Since Shawn surveys the West side, by the group is the group covering the air show, and the
contraposi- tive of the third restriction, Ramesh ‘out’ group is the group covering the political rally.
also surveys the West side. Since no more Every journalist must either be selected (covering the
immediate deductions can be made, we have: air show) or not selected (covering the political
rally). With this in mind, we can symbolize the
e w restrictions like this:
PU SRT
First restriction:
It looks like we must have at least two
workers on the East side. But double-check to
≠BS ≠E
make sure that the remaining workers, Q and V,
ES B
can be on the West side without violating any
restrictions. The fourth re- striction, the only one
Second restriction:
involving Q and V, would be satisfied if both
were on the West side. So the correct answer
G S (E and F)
remains two, choice b.
(≠ E or ≠ F) S ≠ G
115
–distriButioN games reView–

Third restriction: In Out


C FG
FSC
≠CS ≠F Check to see if any of the other restrictions
can be applied. To do this, check to see if the
Fourth restriction: anteced- ents of any the restrictions or their
contrapositives are true. None of them are, so
DS≠F these are the only group assignments that are
FS≠D forced. The other journalists could be placed into
either group (although placing one of them into
Notice that A does not appear in any one group may make further restric- tions on the
restric- tion, and so will be a complete floater. placement of some other yet-to-be-placed
Any deduc- tions made will likely be based on a journalists.
chain of inferences using these conditional Essentially, the diagram as it stands is
statements and additional in- formation, so go to compatible with the placement of each of the
the questions. remaining journal- ists in either of the two groups.
In any case, we know that G is not selected (i.e.,
Question 1 that G must cover the politi- cal rally)—so the
The correct answer is choice d. This is a “Test- correct answer is choice b. D could either be in or
the- Rules” question. For each answer choice, you are out (cover either event), so choices a and d are
given a putative complete and accurate list of the incorrect. The same goes for E and B, ruling out
journalists assigned to the air show; which means choices c and e.
any journal- ist not appearing on the list must be
assigned to the political rally. But since we are Question 3
considering this a se- lection game, each answer The correct answer is choice a. By the first
choice gives a putative com- plete and accurate list of restriction, if B is out, then E is out (i.e., if B
the journalists selected; which means that any covers the political rally, then E also covers the
journalist not appearing on the list must not be political rally. By the con- trapositive of the
selected. Keeping this in mind should help you see second restriction, if E is out then G is out (i.e., if
how the restrictions rule out answer choices. The E covers the political rally, then G also covers
first restriction rules out b, since B is out but E the political rally. This is answer choice a.
is in. The second restriction rules out c, since G is
in but E is not. The third restriction rules out a Question 4
since F is in but C is not. The fourth restriction The correct answer is choice b. You want to find
rules out e, since D is in, but so is F. the minimum number of journalists that can be
not se- lected, so start by seeing if it could be
Question 2 that all the journalists are selected without
The correct answer is choice b. First, look for violating any of the re- strictions, and work your way
the restriction involving F and C—the third restriction up from there. Suppose all journalists are selected.
says that if F is selected, C is selected. So let F The first restriction is not violated because B is
and C be at the same event (i.e., either both in or selected, so the restriction does not apply. The
both out), F must be out (at the political rally), and second restriction could not be violated because E
C in (at the air show). If F is out, then G must also and F are selected. Same with the third re-
be out (contrapositive of the second restriction). striction—it can’t be violated because C is
So far we have: selected. The fourth restriction, however, is
violated—D is se- lected, but F is selected as
well. So it can’t be that all

116
–distriButioN games reView–

journalists are selected (there are zero journalists not just one journalist not selected) would have saved
selected), and a is incorrect. Now move on to choice you the work of checking to see if you could make
b. choice b (i.e., that just one journalist is not
Can there be just one journalist not selected? selected) work without violating any of the
Use the work we just did. If D is the only restrictions.
journalist not se- lected, then none of the
restrictions will be violated. So there can be just Question 5
one journalist not selected (i.e., one journalist at The correct answer is choice e. By the second
the political rally), and b is the cor- rect answer. restric- tion, if G is selected (if G covers the air
Since this answer explanation is show), then both E and F are also selected (also
particularly tricky, we rewrite it here in the cover the air show). If F is selected, then C is also
original terms of the distribution game rather selected (third restric- tion). If E is selected, then
than in the terms of the equivalent selection B is also selected (contra- positive of the first
game: restriction). And if F is selected, then D is not
You want to find the minimum number of selected (contrapositive of the fourth
jour- nalists that can be at the political rally, so restriction). So we have a full selection list:
start by see- ing of there could be zero journalists
assigned to the political rally without violating any
In Out
of the restrictions, and work your way up from
GEFCB D
there. If there were no journalists at the political
rally, they would all be at the air show. Does that There are five journalists selected—i.e., covering
violate any of the restrictions? the air show—choice e.
The first restriction is not violated because B
is not at the political rally, so the restriction Practice Game 3
does not apply. The second restriction could not This game has groups with exact numbers, so
be violated because E and F are in fact at the air num- bers will probably drive much of the
show. Same with the third restriction—it can’t be mechanics of the game. Start with a basic
violated because C is at the air show. The fourth diagram:
restriction, however, is violated—D is at the air
show, but by our supposition that all journalists e d b
are at the air show, F is not at the
political rally. So there can’t be zero journalists at the
political rally, and choice a is incorrect. Now move
on to choice b. Can there be just one journalist in Question 5 (showing that there can
at the political rally? Use the work we just did. be
If D is the only journalist at the political rally,
then none of the restrictions will be violated. So
there can be just one journalist at the political
rally, and b is the correct answer.
We should also note at this point that a bit
of strategy might have helped.
Minimum/maximum questions are often time-
intensive, since you have to methodically check
each number against the full set of restrictions. If
you had skipped this question and saved it for the
end, you would have seen that the work you did
Symbolize the restrictions and try to work them into the For the third restriction, draw an anti-
diagram.
For the first restriction, H can be entered in the ‘d’ block: LH
group.

For the second restriction, draw a block: JK

117
–distriButioN games reView–

For the fourth restriction: L(e) S N(b)


≠ N(b) S ≠ L(e)
L(e) S N(b)
≠ N(b) S ≠ L(e) Armed with this, we can turn to the questions.

Now consider the restrictions to see what Question 1


deduc- tions can be made. For now, we note that The correct answer is choice c. This is a “Test-
the second restriction means that the block JK the- Rules” question. The first restriction rules out
must either be in group e or group b, since there is b, since H is not on the development team. The
no room in group d (H is already there). The third second restric- tion rules out d, since J and K are
restriction means that L must be in either group on different teams. The third restriction rules out
e or group b, so make note of this in your a, since L and H are on the same team. The fourth
diagram, either with arrows or crosses/ restriction rules out e, since L is in engineering,
parentheses: but N is not in branding.

e d b Question 2
H The correct answer is choice d. If N is on the engi-
(L) L (L) neering team, then by the contrapositive of the
(JK) (JK) fourth restriction, L cannot be on the engineering
team. So we have:
Since JK seems to be an important player
(it would fill up group B and nearly fill up group e d b
e), see what would happen in the two scenarios J K N H L
possible for JK’s placement (in group e and group Scan the answer choices—choice d is that L is
b). Suppose JK were in group b. Then L would on the branding team, and so is the correct
have to be in group e. According to the fifth answer.
restriction, if L were in group e, then N would
have to be in group b. But group b is already Question 3
filled! So we know that being in group b is not The correct answer is choice d. We want all the
really an option for the block JK, and JK must pos- sible students for the development team.
instead be in group e. So we have: Consider the diagram:

e d b e d b
J K H J K H
(L) L (L) (L) L (L)

We have now captured the first, second, and Any choice with J, L or K is definitely
third restrictions in our diagram. So our game out: choices c and e. We have four students left:
has been consolidated into the diagram plus the H, N, M, and I. Check to make sure each of
fourth restriction: these could be on the development team. H is
definitely on the devel- opment team. If N was on
e d b the development team, no restriction would be
J K H violated as long as L was not on the engineering
(L) L (L) team (and so on the branding team, leaving I for
the engineering team):
118
–distriButioN games reView–

to be acceptable in the course of previous work.


e d b The diagram for Question 4 rules out choices a,
J K H N DL c and e. The answer to Question 2 rules out
(L) L (L) choice d. So we are left with b.
If we had not answered these prior
This violates no restrictions. Note that we questions, we could have referred to our up-front
do not actually have to check if M is on the diagram and looked for something that cannot be
develop- ment team, because each of the three true. Just by look- ing at our diagram, we see
remaining answer choices contains M. So the that K cannot be on the branding team, since it is
correct answer must include H, M, N and I, and on the engineering team. So b is the correct
nothing else—this is choice d. choice. Our upfront deductions di- rectly paid off
If we were to miss the commonality of M to here.
the remaining answer choices, we would check
whether M could be on the development team and e d b
reason as follows. If M went on the development J K H
team, then we could put I on the engineering (L) L (L)
team and N on the branding team (ensuring that
the fourth restriction is met). We also could have taken each answer
If I were on the development team, then choice in turn to see if it could be true. Suppose
again, we could N on the branding team and L I is on the branding team. Are there any
on the en- gineering team, satisfying the deductions to be made, or rules violated?
remaining restriction. e d b
J K H I
Question 4 (L) L (L)
The correct answer is choice e. If student N and Again, as we already saw in Question 4, a
I were on the same team—that is, a block—they com- patible scenario did not violate any of the
would have to be in group b, leaving L for restrictions. So choice a is incorrect and we turn to
group e, and M for group d: choice b. Again, just by looking at our diagram,
we recall that K cannot be on the branding team,
e d b since it is on the engineering team. So b is the
J K L H M NI correct choice.

Now, each of the statements in the first four Practice Game 4


an- swer choices is guaranteed by the diagram, so Our distribution diagram will be straightforward:
those choices must be wrong. Choice e states that
student I c p m
is on the engineering team, which is false here, since
student I is on the branding team.
We first symbolize the restrictions.
Question 5
The correct answer is choice b. We need to check The first restriction:
each answer choice to see if it could be true.
Start by seeing if any of the statements in QR
the choices are true on distribution scenarios you
found
119
–distriButioN games reView–

The second restriction: Now, by the first restriction, Q and R are to


be separated between groups p and m. So one of
ST the two spots in group p will either be Q or R,
and the one spot in group m will be whichever
The third restriction: of Q and R is not in group p. Either way, the one
remaining spot—in group p—will have to be
S(c) S U(m) filled by V. So it must be that V examines the pot
≠ U(m) S ≠ S(c) —choice a.

In digesting the game, notice how the block Question 3


ST must take up one of the three groups, The correct answer is choice d. The answer is
leaving two groups behind. The presence of taken directly from our up-front deductions about
the anti-block QR means that Q and R will be the game. We realized that because Q and R would
distributed between the two remaining groups. have to be separated (in separate groups, i.e.,
And this means that the two remaining entities, V examining differ- ent artifacts), so would V and U
and U, will also be separated between the two have to be separated. This is choice d.
remaining groups. We don’t know which group is Note that if we had not made the up-
which, but the general distribution will look front deductions, we could have used previous
something like: questions. The answer to Question 1 rules out b
and c. The answer to Question 2 rules out
1 2 3 choice a. That just leaves d and e; for each, see if
S T Q/R U/V R/Q V/U the pair could examine the same artifact. We
would not be forced to see what V and U together
Question 1 would do—they would be in one group, forcing S
The correct answer is choice e. This is a “Test- and T into a second group, and Q and R into a
the- Rules” question. The first restriction rules out third, violating the first restriction.
a and d, since Q and R are in the same group
(examining the coin for a, and examining the pot Question 4
for d). The second restriction rules out c, since S The correct answer is choice b. If V examines the
and T are in different groups. The third mir- ror, then we know that U cannot examine the
restriction rules out b, since S is ex- amining the mirror (because V and U in the same group, along
coin but U is not examining the mirror. with S and T in the same group, would force Q and
R into the same group, violating our first restriction.
Question 2 And according to the contrapositive of our first
The correct answer is choice a. If T examines the restriction, if U is not ex- amining the mirror, then
coin, then S and T are both examining the coin (by S is not examining the coin, which means that S
the first restriction). Since S is examining the coin, (and T) are examining the pot.
U is exam- ining the mirror (by the third Using our general diagram, we have:
restriction). So we have:
p c m
c p m S T U R/Q R/Q V
S T U
According to this diagram, it’s definitely not
true that U examines the pot, because U is
examining the coin. So b is the correct answer.
120
–distriButioN games reView–

Question 5 Now symbolize the restrictions:


The correct answer is choice a. If Q examines the
pot, then again using our general diagram, we The first restriction:
have:
J(f) S M(r)
1 2 3 M(f) S J(r)
S T Q U/V R V/U
The second
Since we are interested in who can examine
the mirror, let’s try the two scenarios this diagram restriction: MN
allows for—that Group 1 is the mirror group or
that group 3 is the mirror group.
The third

S1 restriction: NO
m p c
S T Q U/V R V/U The fourth restriction can be written into
the diagram.
S2
c p m Fifth
S T Q U/V R V/U
restriction:
We check each scenario to see if it is
consistent with the restrictions. In S2, since S O(r) S I(r)
examines the coin, U must examine the mirror
I(f) S O(f)
(third restriction). So S2 must be:
We can write K into the diagram. Since M
S2 and N must be distributed separately into the two
c p m groups, and O goes with N, the diagram looks
S T Q V R U like:

S1 doesn’t violate any restrictions as it stands. f r


Now, test each answer choice. V does not M/NO (_) K NO/M (_)
examine the mirror in either scenario, so choice
a is the answer. Since there are two scenarios listed here,
go ahead and separate them to see what
Practice Game 5 happens.
The game specifies a maximum of four and
minimum of three scouts per raft. So we have: S1
f r
f r M (_) K N O (_)

S2
f r
(_) (_) N O (_) K M (_)
121
–distriButioN games reView–

For S1: according to the first restriction, since f r


M is in f, J must be in r, pushing the remaining N O J (_) ( I ) K M L I (_)
entities, I and L, into f:
f r
S1 N O L (_) ( I ) K M J I (_)
f r
M I L (_) KNOJ
I can be distributed to the front or rear raft
Is S1 consistent with the restrictions? for each of these two possibilities, so there are a
According to the fifth restriction, if O is in the total of four possibilities—choice d.
rear boat, I must be in the rear boat—but I is in
the front boat. So S1 cannot be an option, and Question 3
we are left only with S2: The correct answer is choice a. Consider the
diagram:
S2
f r f r
N O (_) K M (_) N O (_) (J, L, I) K M (_)

What about the remaining three entities, J, L, Any list with a selection from N, O, J, L, and
and I? Since the consequent of the first I— so long as no other letters appear, and so
condition is true (M is on the rear boat), and the long as no more than two of J, L, and I appear—
antecedent of the fifth restriction false (O is not on will be a partial, accurate list. If all three of J, L
the rear boat), neither the first nor fifth restrictions and I appear, then there will not be enough room
can affect the game. The sec- ond, third, and fourth on the front raft for N and O. Choice a gives such a
restrictions are already satisfied. So as long as we partial list. Choices b and e in- clude K, and so are
meet our numerical restriction that there is a incorrect. Choices c and d include all three of J, L
minimum of three and maximum of four and I, and so are incorrect.
scouts per raft, J, L, and I can appear on either
raft: Question 4
The correct answer is choice d.
f r
N O (_) (J, L, I) K M (_) (J, L, I) f r
N O (_) (J, L, I) K M (_)
Question 1
The correct answer is choice b. This is a “Test- A quick glance at the diagram shows that O
the- Rules” question. The first restriction rules out can- not be in the rear raft. All the other letters
choices a and c, since J is in the front raft but M —J, K, L, I, and M—can be in rear raft.
is not in the rear raft. The second restriction rules Without the aid of the diagram, you could
out d, since M and N are on a raft together. The also note that you can use the answer to
fourth restriction rules out choice e, since K is question 1 to eliminate choice e and the answer
not on the rear raft. to question 2 to eliminate choices b and c. You
can then try the two remaining answer choices and
Question 2 see that choice d results in a contradiction, since it
The correct answer is choice d. If J and L take places five people on the rear boat.
differ- ent rafts, then they can be distributed in
the follow- ing two ways:
122
–distriButioN games reView–

Question 5 no item appears on all three menus. The second


The correct answer is choice a. Again, a brief re- striction rules out a, since the burgers and
glance at the diagram shows that M cannot be in sandwiches are on the same menu. The third
the front boat. restriction rules out choice c, since the tacos and
nachos appear on the same menu. The fourth
Practice Game 6 restriction rules out e, since chili does not appear
First, the numbers: The three groups or three on the dinner menu.
menus have exactly three items each. But the
items to be dis- tributed, the six menu items, can Question 2
(and will have to) appear in more than one The correct answer is choice a. We know from
group. All menu items will be selected. our up-front deductions that either chili or pizza
The fourth restriction is that chili appears on must ap- pear on all three menus. If chili
the dinner menu. So we can start with a appears on at most two menus, then pizza must
diagram: appear on all three. This is choice a.

l d n
C Question 3
The correct answer is choice b. If burgers appear
on
Let’s consider the other restrictions. the- Rules” question. The first
The first restriction is that one menu restriction rules out b, since
item appears on all three menus. We’ll get back
to this.

The second restriction:

BS

The third restriction:

TN

Now let’s consider the menu item that must


ap- pear on all three menus—this will be a major
player. That item could not be the burger,
because then it would appear on at least one
menu with the sand- wiches, violating the
second restriction. The same reasoning applies to
the sandwich, tacos, and nachos. So the item that
appears on all three menus must ei- ther be the
chili or the pizza (the two remaining menu items).
This will prove to be critical information when
answering the questions.

Question 1
The correct answer is choice d. This is a “Test-
the lunch and dinner menus, and chili on the night menu, Let’s consider our major player now—the
we have: item that appears on all three menus. If it were
pizza, we would have:
l d n
B C B C l d n
B P C B P CS P
Since burgers and sandwiches cannot appear on the menu
together, sandwiches must appear on the night menu. But this would leave no room for T and N.
So chili, not pizza, must appear three times, and
l d n we have:
B CB C S
l d n
B C B C C S

123
–distriButioN games reView–

T and N and P must take up the remaining


three spots, and there are no restrictions on l d n
which item takes which spot. P B/S C N P P S/B
Given this diagram, choice b is the
correct answer because it can’t be the case that These two spaces cannot have S or B in
both pizza and nachos appear on the lunch menu them (lest we violate the second restriction), nor
—there is only space for one of them. can they have C or N in them (since either one
would carry the other—i.e., they exist as a block
Question 4 now); nor P, since P already appears on all three
The correct answer is choice a. Since we know menus. So the two open spaces must both have
that either chili or pizza is the item that appears T.
on every menu, it must be the case that either way
—whether chili appears three times and therefore l d n
at least once with pizza, or whether pizza appears P B/S T C N P P S/B T
three times, at least once with chili—chili and pizza
appear together on a menu. The diagram thus shows that choice c,
We could also arrive at the answer by which says that tacos appear on the lunch
noticing that the answer to Question 1 rules out c, menu, must be true, and is therefore the correct
d, and e, and that the answer to Question 2 rules answer.
out b.
Practice Game 7
Question 5 There are four spots each for the Freedom
The correct answer is choice c. If chili and and Independence, and according to the second
nachos always appear on a menu together, then restric- tion, Y is aboard the Independence.
nachos must appear on the dinner menu: Since the first restriction means that A and C
are to be separated between the Freedom and
l d n Independence, we can sym- bolize that information
directly into the diagram as well:
C N
f i
Now consider the major player again, the item A/C Y A/C
that appears on all three menus. We know that it must v n v
be chili or pizza. If it were chili, then the block CN
would appear on all three menus (since chili We note the subgroups under the
would appear on all three)—leaving three spaces for
the four remaining menu items yet to appear slots. We symbolize the third
(burgers, pizza, sandwiches, tacos). So chili cannot
appear three times, and instead, pizza must: restriction:

l d n B(f) S X(i)
P CNP P or X(f) S B(i)

B and S need to be distributed between the We symbolize the fourth


lunch menu and night menu, leaving two open
spaces. restriction: A(i) S Z(f)
Z(i) S A(f)
124
–distriButioN games reView–

Question 1 Question 5
The correct answer is choice e. This is a “Test- The correct answer is choice e. If X is aboard
the- Rules” question. Remember that the four the Freedom, then by the contrapositive of the
astronauts not given in an answer choice will third restric- tion, B is aboard the Independence. So
be aboard the Independence—a fact that will be the Independence will have exactly two veterans: B,
helpful in detecting rule violations. The first and one of A or C. If the Independence has exactly
restriction rules out c, since A and C are on the two veterans, then the Freedom will have exactly
same shuttle. The second restriction rules out a, one veteran, and therefore have exactly three
since Y is not aboard the Independence. The third novices. This is choice e.
restriction rules out choice b, since B is on the
Freedom, but X is not on the Independence. The Practice Game 8
fourth restriction rules out e, since A is on the We are told that each case is assigned at least one
Independence, but Z is not on the Freedom. agent and one detective:

Question 2 A B
The correct answer is choice e. Start by looking for
the restriction involving both B and X—the third a d ad
re- striction. If B and X are aboard the same
shuttle, that shuttle can’t be the Freedom— The first
otherwise, the third re- striction would mean that
X is on the Independence. So B and X have to both restriction:
be aboard the Independence. The remaining
entities, V, W, and Z, have to go into the left HI
group (the Freedom). So we have:
The second
f i
V W Z A/C Y B X A/C restriction: IJ
v n v
The third restriction:
From consulting the diagram, we see that Z
and B cannot be aboard the same shuttle— K(A) S X(B)
choice e. X(A) S K(B)

Question 3 The fourth restriction:


The correct answer is a. If Z is aboard the
Independence, then by the contrapositive to the Y(A) S I(A)
fourth restriction, A is on the Freedom. And if A I(B) S Y(B)
is on the Freedom, C is on the Independence. This
is choice a. Consider the two scenarios created by the
first restriction:
Question 4
The correct answer is choice b. If there are exactly S1
two veterans on the Freedom, then B must be one A B
of those veterans (since veterans A and C can’t be
on the same
shuttle). And if B is on the freedom, then by the third I H
restriction, X is on the Independence. This is choice b. a d a d

125
–distriButioN games reView–

S2A B the contrapositive of the fourth restriction, Y is


H I also in B. So we have:
a d a d
A B
By the second restriction, we can add in J H JY I
with I. By the fourth restriction, we can put Y This is enough to see that H and I cannot
under case B for scenario 2, indicating that be a pair in the same group.
either X or Z (or both) must be assigned to A (to
maintain at least one agent for each group): Question 3
The correct answer is choice b. If exactly three
S1 detec- tives are assigned to the Brentwood case, then
A B we have exactly one detective assigned to the
I J H Appleby case. Since the IJ block is comprised of
a d d a d two detectives, we know that block has to be
assigned to the Brentwood case. This forces H
S2 over to the Appleby case, by the first restriction.
A B This is choice b.
X/Z H Y I J
a d a d d Question 4
The correct answer is choice e. If XY is assigned to
Question 1 A, then we know we are in S1 (because in S2, Y is
The correct answer is choice a. This is a “Test- assigned to B). Now, since XY is assigned to A, Z
the- Rules” question. Note that any officer not will be assigned to B (since we need at least one
assigned to the Appleby case is assigned to the agent assigned to case B). By the contrapositive of
Brentwood case. The first restriction rules out b, the third restriction, K must be assigned to B,
since H and I are as- signed the same case. The since X is in A, and we have:
second restriction rules out choice c, since I and
J are not assigned the same case. The third S1
restriction rules out d, since K is as- signed to A B
the Appleby case, but X is not assigned to the X Y I J ZH K
Brentwood case. The fourth restriction rules out e, a a d d ad d
since Y is assigned the Appleby case, but I is
not. The only item not true among the answer
choices is that K is assigned to the Appleby case
Question 2 (since it is as- signed to Brentwood case).
The correct answer is choice c. If H is assigned We could also get the answer choice without
to the Appleby case, then we must be in the initial scenario diagrams by seeing what follows
scenario 2. According the diagram, H and I from X and Y being in A. Since X is in A, K is in
cannot be assigned to the same case, as I is B (contra- positive of the third restriction). Since Y
assigned to the Brentwood case. is in A, I is in A (fourth restriction). Since I is in
We could also get this answer without the A, J is in A (second restriction). Since I is in A, H
ini- tial scenario diagrams by seeing what follows is in B (first restriction). Finally, since we need at
from H being assigned to A. By the first least one agent in B (and we only have two
restriction, I must be in B, carrying J with it (by detectives at the moment), Z must go into B:
the second restriction). By
126
–distriButioN games reView–

A B
X Y I J KHZ

Question 5
The correct answer is choice a. First, figure
out the fewest number of officers possible in
Brentwood. Consulting our two scenario
diagrams, the fewest number of officers will be in
scenario 1, since scenario 2 guarantees Brentwood
at least three officers. And in S1, H is the only
officer definitely assigned to Brentwood.

127
6
chapt e r

matching games
review

In a matching game, you are given a group of entities to be matched with members of another group of
entities or with some set of characteristics. The entities can be persons, places, or things, and the
characteristics can be anything from shape to color to location. For example, it may be a league of soccer
teams (a group of entities) assigned to wear different jersey colors (a set of characteristics).
Matching games present us with a lot of information. In sequencing, selection, or distribution games,
there is usually only one set of entities to worry about—what we have been calling the domain—and the
relationships between those entities, be it the order of their placement or how they are separated into
groups). In matching games, however, we determine the relationships between members of two groups (e.g.,
students and advisors), or we determine which possible characteristics apply to each member of a group
(e.g., for a set of flights, which are bound for either Chicago or Boston, and whether it’s a morning,
evening, or late-night departure).
For this reason, the key to mastering games is the organization of the information—and in
particular, how well you can create a visual representation of the possible assignments of entities to
entities or entities to characteristics. The kind of diagram you will use depends on the matching game
you are working with. Think of matching games as providing you with a number of groups. Groups of
two can consist of two entities (e.g., students and advisors) or an entity and a characteristic (e.g., cars and
their colors). The groups of three or more usually consist of an entity and sets of characteristics (e.g.,
flights and their destination city and departure time).

129
matchiNg games reView

matching with two groups


Or, if we were matching cars to colors:
If you have a game with just two groups, there are
two approaches you could take—the column or Car 1 Car 2 Car 3 Car 4
the grid. In the column approach, you simply put Red
each member of a group into a column, under Blue
which you enter the members of the other group Green
or the assigned charac- teristics. For example, say
you are matching students (A, B, C, D) to
advisors (E, F, G, H), and are told that student A Columns vs. Grids
is assigned to advisors F and G, and student C One advantage of the column approach is that
does not have advisor E: you can easily keep track of matching scenarios that
work. You do not have to recreate your diagram
for each new question. You simply draw a line
A B C D
and start a new distribution:
FG E

A B C D
This diagram should remind you of our FG E
distribu- tion diagrams. In a sense, we are G EF HE G
distributing advisors into four groups, each group
corresponding to a stu- dent. Of course, this is a
distribution where there may be repeats in the In addition, the column approach is often
groups (e.g., an advisor might advise more than suf- ficient if each entity of the primary group
one student). can be matched to only one entity in the
In the grid approach, you draw a grid with secondary group: For example, if each student can
all the members of one group across the top have only one advisor. The grid approach does not
and all the members of the other group (or all allow you to easily create fresh game scenarios (e.g.,
the possible characteristics) down the side. Each when a question asks you to make a new
box will either have a check mark, indicating a supposition). You either have to re- draw the grid
match; an ✗, indicating no match; or a blank, for a new game scenario, or very lightly mark the
indicating a match is yet to be or cannot be boxes in your grid and erase them as you
determined. Using the student-advisor matching move to a new question or scenario.
example from above, you could draw: However, the grid is often extremely useful
for visualizing key relationships that drive the
game, especially when each primary entity can be
A B C D
matched to more than one secondary entity.
E ✗ You should experiment with both approaches for
F ✓ various types of questions to see which suits you
G ✓ better. This book usually uses the grid for its ease
H of demonstrating logical relationships and the
application of game rules. But if you find that you
can easily manage the logical relationships, it
might save you more time to use the column
approach.
130
matchiNg games reView

matching with three


or more groups given car. But we take the slashes out to indicate
that any number of the letters s, f, and p could
When there are three or more groups, you again apply to a given car.
have a couple of options. First, you could draw a We might also create a grid in which a
grid with all the entities of one group across the characteristic set (in this case, the custom options)
top, and just one row for each set of is expanded so each member of that set gets a
characteristics down the side. For example, if we dedicated row. In addition, if a question tells us
had four flights, and had to deter- mine for each that Car 1 is a Ford with fog lamps and
whether it was headed for Chicago or Boston, and performance tires and Car 3 is a Dodge that
whether it departed in the morning, eve- ning, or definitely lacks a spoiler, we would have a grid
night: that looks like this:

Flight 1 Flight 2 Flight 3 Flight 4 Car 1 Car 2 Car 3 Car 4


C/B s ✗
m/e/n f ✓
p ✓
Notice that in this game, each box will
F/D/C F D
have only one letter—in the top row, C or B for
Chicago or Boston, and in the bottom row, m, e,
or n for morn- ing, evening, or night. Each flight It is usually a good idea to expand all
can be headed for only one city and can be character- istic sets that can be applied to more
assigned to depart only at one time. This is than once to a given entity in order to better
indicated by the slashes. You should also keep visualize the matching that occurs. When the
track of what assignments aren’t possible by characteristic can apply only once, it is often
entering a letter into a box and crossing it out. enough to use a single row for all (as we did
In other games, we might have a set of with the car make).
charac- teristics that apply to a given group of
entities. For ex- ample, we might have a group of
four cars, and a set of three custom options for the working with matching rules
car—spoiler, fog lamps, and performance tires—
any number of which could apply to any given There are six basic kinds of rules you will
car. The game might specify, in ad- dition, that encounter in matching games. Many of the rules
each of the four cars is a Ford, Dodge, or will allow sym- bolization that works in concert
Chevy. We might draw the following grid: with your diagram.

Concrete Matches
Car 1 Car 2 Car 3 Car 4
The simplest rules tell you that a particular entity
sfp
is matched with another particular entity or
F/D/C
character- istic. For example, “Car 1 is a Ford
with fog lamps and performance tires.” On grids,
Notice that we used slashes to indicate that simply enter a check mark into the appropriate box.
only one of the three letters F, D, and C could If using the column ap- proach, just write in the
apply to a letter for the characteristic(s) into the appropriate
column.
131
matchiNg games reView

Conditional Matches be written next to the appropriate column or rows


Matching games sometimes present rules that take in order to remind you how many in that column
the form of conditional statements—but the or row are to be selected. With these two rules as
components of these conditional statements are examples:
usually concrete matches. For example, you may
be told that “if Car 1 has fog lamps, then Car 2 Car 1 Car 2 Car 3 Car 4
must have performance tires and a spoiler.” These
s
rules are symbolized in the usual way:
f exactly 3
p
1 = f S 2 = p, s
F/D/C

If you have a grid, they can also be


symbolized by drawing directly into the grid: If you are using the column approach, it
would be difficult to visually incorporate that Car
2 has “at least two” custom options. But you could
Car 1 Car 2 Car 3 Car 4
incorporate the information that exactly three cars
s
have the fog lamps by writing 3 fs next to the
f
appropriate row:
p
F/D/C
Car 1 Car 2 Car 3 Car 4
sfp fff
This shows that a check mark in 1-f requires F/D/C
checkmarks in both 2-s and 2-p, and that an ✗ in
ei- ther 2-s or 2-p requires an ✗ in 1-f (the
contrapositive). You would then cross out the f ’s as you
enter them into the grid.
Restricted Matches
Restricted matches are simply negative statements— Similarities
a given entity definitely does NOT have a certain A staple of the matching game is the rule that two
char- acteristic, or that a given entity is definitely en- tities or characteristics are to be similarly
NOT matched with another entity. For example, “Car matched. For example, you might be told that any
3 does not have a spoiler.” For grids, this custom op- tion on Car 1 will appear on Car 2 as
information is cap- tured by an ✗ in the well. Or you may be told that “the custom
appropriate box. For the column approach, the options on Car 1 and Car 3 are the same.” Or
symbolization is a crossed-out letter in the you may be told that “any car that has fog lamps
appropriate column. also has the performance tires.” This is all crucial
information for matching games. There is a
Matching Numbers particularly useful way for symbolizing these
Rules that deal with matching numbers are rules when using a grid: use arrows to indicate the
probably the most important of matching games. direc- tion of similarity. Here are the
These rules take various forms. You might be told symbolizations for the example rules just
that “Car 2 has at least two custom options” or presented:
that “exactly three cars have fog lamps.” This
sort of information can Any custom option on Car 1 will appear on
Car 2 as well.
132
matchiNg games reView

Car 1 S Car 2
The contrapositive: not Car 2 S not Car Again, be sure to remember how the
1 contraposi- tive works as well. If a box at the end
of an arrow gets an ✗, then so does the box at
The custom options on Car 1 and Car 3 the beginning of the arrow. If Car 2 does not
are the same. have performance tires, then it cannot have fog
lamps.
Car 1 · Car 3
Differences
Any car that has fog lamps also has the Another staple of the matching game is the
perfor- mance tires. rules that two entities or characteristics are not
similarly matched. For example, you might be
f told that “Car 2 does not have any custom option
T that Car 1 has.” And you may be told that “no car
p can have both a spoiler and performance tires.” Or
that “any car with a spoiler does not have
The direction of the arrows is performance tires.” Notice that these last two rules
important. Each symbol is meant to are equivalent. They both mean that 1) if a car has
remind us that if a check mark appears in a a spoiler, it does not have performance tires, and
box associated with a letter or symbol (‘Car 1’ 2) if a car has performance tires, it does not have a
or ‘f’), a check mark must appear in the box spoiler.
associated with the other letter or symbol. As You can symbolize these rules
with conditional matches, these as: s S ≠ p
symbolizations are a visual aid for seeing pS ≠s
how filling in one box may affect another box. Sometimes, these “difference rules” can
This is especially apparent if we write the also be symbolized by a double arrow with an
symbolizations directly into the diagram. For ✗ in the middle—any check mark one side of
example, if we put a check mark to indicate the arrow leads to an ✗ on the other side of
that Car 2 has a fog lamp, we are reminded by the arrow. Keep in mind that this does not
the downward arrow to put a check mark mean that an ✗ on one side leads to a check
indicating that Car 2 has performance tires: mark on the other. According to the rule that
“no car can have both a spoiler and
performance tires,” it’s clearly perfectly possible
Car 1 Car 2 Car 3 Car 4
for a car to have neither a spoiler nor
s
performance tires. And keep in mind that this
f ✓ exactly 3
does not mean that an ✗ on one side leads to
p ✓ an ✗ on the other. It’s possible that a car with
F/D/C no performance tires still has a spoiler. These
symbolizations would look like:
If it is not possible to draw arrows directly
into the diagram, then draw the full Car 1 Car 2 Car 3 Car 4
symbolizations (ar- rows plus letters) in such a ✗
way as to best remind you of the way matching
s
flows through the diagram.
f

p
F/D/C
133
matchiNg games reView

So if we are told that Car 1 has a spoiler, and If you do not find this symbolization helpful,
we put a check mark in the appropriate box, the just stick to the regular conditional
difference arrows immediately indicate that we need symbolizations.
to put ✗s in certain boxes—Car 1 does not have
performance tires, and car 2 does not have a
spoiler:

Car 1 Car 2 Car 3 Car 4


s ✗ ✓ ✗
f
p ✗

134
matchiNg games reView

practice game 1

Six philosophy professors—Albert, Friedman, Godfrey-Smith, Lipton, Pauly, and Varzi—each specialize in one
or more areas—logic, metaphysics, and science. They are specialized according to the following
conditions:

Lipton does not specialize in any area in which Varzi


specializes. Varzi specializes in logic and metaphysics.
Varzi specializes in more areas than Pauly.
Exactly three professors specialize in metaphysics.
Albert, Godfrey-Smith, and exactly two other professors specialize in science.
Pauly and Friedman share exactly one area of specialization.

1. If Albert and Friedman specialize in exactly 4. Which one of the following could be a
the same areas, then each of the following partial, accurate list of professors all of whom
state- ments must be true EXCEPT: specialize in metaphysics?
a. Albert specializes in metaphysics. a. Albert, Friedman
b. Godfrey-Smith specializes in logic. b. Albert, Pauly
c. Friedman specializes in science. c. Godfrey-Smith, Pauly
d. Lipton specializes in science. d. Albert, Lipton
e. Pauly specializes in logic. e. Albert, Friedman, Godfrey-Smith

2. Which one of the following statements must 5. If every professor who specializes in logic
be true? also specializes in metaphysics, then for how
a. Pauly specializes in metaphysics. many professors are the areas of
b. Pauly specializes in science. specialization fully determined?
c. Albert specializes in logic. a. Two
d. Friedman specializes in metaphysics. b. Three
e. Friedman specializes in science. c. Four
d. Five
3. If Godfrey-Smith specializes in every area e. Six
in which Pauly specializes, then which one
of the following statements CANNOT be
true?
a. Pauly specializes in metaphysics.
b. Pauly specializes in logic.
c. Godfrey-Smith specializes in metaphysics.
d. Friedman specializes in metaphysics.
e. Albert specializes in metaphysics.
135
matchiNg games reView

practice game 2

A middle school computer lab receives six new computers, labeled 1 through 6, each with at least one of
three programs installed on it—a math program, a reading program, and a writing program. The
programs are in- stalled according to the following conditions:

Computer 2 has the math and reading programs installed.


Computer 4 has neither the math nor writing programs
installed.
Computer 5 has exactly one fewer program installed than computer 6, and does not have the
writing pro- gram installed.
Computer 2 and computer 5 have exactly two programs in common.
Any computer with the writing program installed also has the reading program installed.

1. For exactly how many of the computers can writing program.


it be determined exactly how many programs e. Computer 3 has only the math
are installed? program.
a. One
136
b. Two
c. Three
d. Four
e. Five

2. If the reading program is installed on


exactly four computers, then each of the
following must be true EXCEPT:
a. Computer 1 has the math program.
b. Computer 3 has the math program.
c. Computer 3 does not have the reading
program.
d. Computer 2 does not have the writing
program.
e. Computer 1 does not have the writing
program.

3. Suppose only two computers have exactly


the same set of programs installed. Which
one of the following statements CANNOT
be true?
a. Computer 1 has both the math and
writing program.
b. Computer 1 has both the reading and
writ- ing program.
c. Computer 3 has both the reading and
writ- ing program.
d. Computer 2 has both the math and
4. If at least two computers have the writing pro-
gram installed, then what is the least number of
computers that must have the reading program
installed?
a. One
b. Two
c. Three
d. Four
e. Five

5. Which one of the following statements must be


false?
a. Exactly four computers have the writing
program.
b. Exactly five computers have the writing
program.
c. Exactly four computers have the reading
program.
d. Exactly five computers have the reading
program.
e. Exactly five computers have the math
program.
matchiNg games reView

practice game 3

At a wedding, each of five entree options—chicken, beef, pork, fish, and vegetarian—comes with at least
one side—rice, salad, or bread—according to the following restrictions:

The chicken comes with exactly one more side than the
beef. The fish comes with exactly one fewer side than
the beef.
The pork comes with salad.
The pork and beef share no sides.
At least three and at most four entrees come with bread.

1. For exactly how many of the entrees can it 4. If the fish comes with salad, then which one
be determined exactly how many sides each of the following statements must be true?
entrée comes with? a. The vegetarian entrée comes with salad.
a. One b. The vegetarian entrée comes with bread.
b. Two c. The vegetarian entrée comes with rice.
c. Three d. The vegetarian entrée does not come
d. Four with bread.
e. Five e. The vegetarian entrée does not come
with rice.
2. If the vegetarian entrée comes with fewer
sides than the beef, then how many 5. If every side that comes with the
entrees come with exactly two sides? vegetarian entrée also comes with the fish,
a. One then which one of the followings statements
b. Two CANNOT be true?
c. Three a. The vegetarian entree comes with
d. Four exactly one side.
e. Five b. The beef entree comes with exactly two
sides.
3. If there are more entrees that come with c. Exactly two entrees come with salad.
rice than entrées that come with bread and d. Exactly three entrees come with salad.
entrees that come with salad, then each of e. Exactly two entrees come with rice.
the following statements must be true
EXCEPT:
a. The fish does not come with bread.
b. The fish does not come with salad.
c. The vegetarian entrée comes with salad.
d. The vegetarian entrée comes with rice.
e. The vegetarian entrée comes with bread.
137
matchiNg games reView

practice game 4

Four friends—Ross, Siddarth, Alex, and Josh—go for an adventure hike in the mountains of Scotland.
Each brings a backpack with at least one of the following items: compass, extra socks, knife, and map. At
least one of each item is brought on the trip. The contents of their backpacks are subject to the
following restrictions:

Ross brings exactly one item.


Alex brings exactly two items.
Exactly three of the friends bring a map.
Alex does not bring every item that Siddarth
brings. Josh brings every item that Alex brings.
Anybody who brings a compass brings a map.

1. Which one of the following statements must 4. Which one of the following statements
be true? CAN- NOT be true?
a. Ross does not bring extra socks. a. Siddarth brings exactly one item.
b. Alex does not bring extra socks. b. Siddarth brings exactly two items.
c. Siddarth does not bring extra socks. c. Josh brings exactly one item.
d. Josh does not bring extra socks. d. Josh brings exactly two items.
e. Josh does not bring a knife. e. Josh brings exactly three items.

2. If Siddarth brings a map, then which one of 5. If the number of backpacks containing a
the following statements CANNOT be com- pass exceeds the number of backpacks
true? contain- ing a knife, then each of the
a. Siddarth brings a compass. following could
b. Siddarth brings extra socks. be a complete and accurate list of those
c. Alex brings extra socks. friends bringing extra socks EXCEPT:
d. Alex brings a knife. a. Josh
e. Josh brings a compass. b. Siddarth
c. Alex, Josh
3. If Alex brings a compass, then which one of d. Siddarth, Josh
the following statements could be true? e. Alex, Siddarth
a. Siddarth brings a compass.
b. Siddarth brings a map.
c. Alex brings extra socks.
d. Josh does not bring a map.
e. Josh brings extra socks.
138
matchiNg games reView

practice game 5

A car dealer sells four cars, labeled 1 through 4. Each car is made by one of Ford, Dodge, or Chevy. Each
car comes equipped with at least one of the following three options: spoiler, fog lights, and performance
tires. The cars meet the following restrictions:

Only Ford cars can have fog


lights. Car 2 has more options
than car 3. Exactly two cars
have fog lights.
Car 2 is a Dodge.
Car 4 has no
spoiler.

1. Which one of the following statements 4. If car 4 is a Dodge, then which one of
CAN- NOT be true? the fol- lowing statements could be
a. Every car with a spoiler has fog lights. true?
b. Every car with a spoiler has a. Every car with performance tires has
performance tires. a spoiler.
c. Every car with fog lights has b. Every car with fog lights has a spoiler.
performance tires. c. Every car with a spoiler has fog lights.
d. Every car with performance tires has a d. Every car with performance tires has
spoiler. fog lights.
e. All but one car with performance tires e. Every car with a spoiler has
has fog lights. performance tires.

2. If Car 1 is a Chevy, then which one of 5. If exactly three cars have spoilers, then
the fol- lowing statements could be true? which one of the following statements
a. Car 1 has fog lights. CANNOT be true?
b. Car 3 has performance tires. a. Car 1 has exactly one option.
c. Car 3 does not have fog lights. b. Car 1 has exactly two options.
d. Car 4 has performance tires. c. Car 1 has exactly three options.
e. Car 4 does not have fog lights. d. Car 4 has exactly one option.
e. Car 4 has exactly two options.
3. Which one of the following statements
CAN- NOT be true?
a. There are more Dodges than Chevys.
b. There are an equal number of Dodges
and Chevys.
c. There are more Dodges than Fords.
d. There are more Fords than Dodges.
e. There are an equal number of Dodges
and Fords.
139
matchiNg games reView

practice game 6

A street has six buildings numbered 1 to 6, three on one side and three on the other, in the following arrangement:

1 2 3

4 5 6

Each building contains a law firm, a bank, or both and each building is either a modern or historical
build- ing, but not both. The following conditions apply:

Each historical building is directly next to another historical building on the same side of the
street. Each law firm is directly across the street from a bank.
Building 3 is historical.
Building 2 contains a law
firm. Building 4 is modern.

1. If every bank is located in a historical 3. If every bank is located in a modern


build- ing, then which one of the following building, then what is the maximum
statements MUST be true? number of modern buildings that contain
a. Building 2 contains a bank. only law firms?
b. Building 3 contains a law firm. a. Zero
c. Building 1 is historical. b. One
d. Building 5 is modern. c. Two
e. Building 6 contains a bank. d. Three
e. Four
2. If exactly three of the six buildings are
modern, then which one of the following 4. If every historical building contains only a
statements must be true? law firm, then which one of the following
a. At least one bank is located in a statements must be true?
historical building. a. Building 4 is modern and contains a bank.
b. At least one bank is located in a b. Building 4 is modern and contains a law firm.
modern building. c. Building 5 is modern and contains both
c. At least one law firm is located in a a bank and law firm.
modern building. d. Building 6 is modern and contains a bank.
d. Exactly one bank is located in a e. Building 6 is modern and contains both
historical building. a bank and law firm.
e. Exactly one bank is located in a
modern building. 5. Which one of the following statements
CAN- NOT be true?
a. Building 6 is modern and building 5 is
historical.
b. Both buildings 1 and 4 are modern.
c. Building 3 is historical and building 4 is
modern.
140 d. Building 4 is modern and building 5
is historical.
e. Both buildings 5 and 6 are historical.
matchiNg games reView

practice game 7

Six skydivers—three male (A, B, and C) and three female (D, E, and F)—each deploy a parachute in
exactly one of four colors: red, green, orange, or purple. The parachute colors meet the following
restrictions:

No two males deploy the same color parachute, and no two females deploy the same color
parachute. A has a green parachute.
D’s parachute is neither orange nor purple.
If B has a red or orange parachute, then E has a green
parachute. Exactly two parachutes are orange.

1. Which one of the following statements 4. If there is exactly one green parachute,
CAN- NOT be true? then which one of the following is a
a. Both B and D have a red parachute. complete and accurate list of the colors
b. C has a red parachute and F has a any one of which could be the color of
green parachute. F’s parachute?
c. C has a purple parachute and E has a a. Orange
green parachute. b. Purple
d. E has a purple parachute and F has an c. Red
orange parachute. d. Orange, red
e. E has an orange parachute and F has a e. Orange, purple
purple parachute.
5. If C’s parachute is purple, then for exactly
2. If E has a red parachute, then which one how many of the six skydivers is the color
of the following statements must be true? of his or her parachute determined?
a. There are exactly two purple parachutes. a. Two
b. There are exactly two red parachutes. b. Three
c. There are exactly two green parachutes. c. Four
d. There is exactly one green parachute. d. Five
e. There are no red parachutes. e. Six

3. If B and E have the same color parachute,


then which one of the following statements
CAN- NOT be true?
a. F has a green parachute.
b. D has a red parachute.
c. D has a green parachute.
d. C has an orange parachute.
e. B has a purple parachute.

141
matchiNg games reView

practice game 8

Five whiskey critics—H, I, J, K and L—are each to taste at least one of the following three scotches: Talkisker,
Glenfiddich, and Macallan. They taste according to the following restrictions:

H tastes the Talisker and the


Macallan. H tastes fewer scotches
than K. Exactly three critics taste the
Talisker.
If I tastes the Macallan, then J tastes both the Talisker and the
Glenfiddich. There are no scotches L tastes that I does not taste.

1. For exactly how many of the critics can 4. If I tastes the Talikser, then each of the
it be determined exactly which scotches follow- ing statements could be true
he or she tastes? EXCEPT:
a. One a. I tastes the Macallan.
b. Two b. I tastes the Glednfiddich.
c. Three c. J tastes the Glenfiddich.
d. Four d. J tastes the Macallan.
e. Five e. L tastes the Glenfiddich.

2. If L tastes the Macallan, then each of the 5. If every critic who tastes the Glenfiddich
follow- ing statements could be true also tastes the Talisker, then for exactly how
EXCEPT: many of the critics can it be determined
a. I tastes the Glenfiddich. exactly which scotches he or she tastes?
b. I tastes the Talkisker. a. One
c. J tastes the Talkisker. b. Two
d. J tastes the Macallan. c. Three
e. L tastes the Glenfiddich. d. Four
e. Five
3. If exactly two critics taste the Glenfiddich,
then each of the following must be true
EXCEPT:
a. I tastes the Macallan.
b. J tastes the Glenfiddich.
c. J tastes the Talikser.
d. J tastes the Macallan.
e. L tastes the Macallan.
142
matchiNg games reView

setup, answers, and diagram:


explanations

Practice Game 1
We have six professors—A, F, G, L, P, V—each
of whom will be assigned one to three specialties
—l, m, and s. The matching can be done in a
grid:

A F G L P V
l
m
s

Now symbolize the restrictions and work


them into the diagram.
The first restriction is that L and V share
no specialties:

Ld ✗S
V

(We would draw this into the diagram, but


we are about to see that we can fully determine L
and V.) The second is concrete information
that V specializes in l and m. We can immediately
apply the first restriction to see that L does not
specialize in l or
m. So L must specialize in s; and since s
specializes in s, V cannot specialize in s. So we
have:

A F G L P V
l ✗ ✓
m ✗ ✓
s ✓ ✗
The third restriction is that V has more
special- ties than P. Since V has exactly two
specialties, this means that P has exactly one
specialty. The fourth re- striction is that exactly
three professors specialize in metaphysics. The
fifth is that G and A specialize in science and
that exactly four professors in total spe- cialize in
science. We can work this information into the
A F G L P V
l ✗ ✓ Now, see if any other deductions can be
m (ex. 3) ✗ ✓ made. We know that exactly four professors
specialize in sci- ence and we have identified
s (ex. 4) ✓ ✓ ✓ ✗
three of them. Can we identify the fourth? If we
(ex. 1) check P’s s-box, then we will have to check F’s s-
box as well—that is, if P special- izes in s, then F
must specialize in s. But then we will have exactly
The sixth restriction is that P shares exactly one specialty 5 professors specializing in science. So P can’t
with F. Since P has exactly one specialty, this means that we specialize in s, meaning that F must specialize in
can draw an arrow from P to F: any specialty that P has, F s, yielding:
will have. (Note: this does not mean that F will have only
one specialty).
A F G L P V
So we have: l ✗ ✓
m (ex. 3) ✗ ✓
A F G L P V s ✓ ✓ ✓ ✓ ✗ ✗
l ✗ ✓
m (ex. 3) ✗ ✓ (ex. 1)
s (ex. 4) ✓ ✓ ✓ ✗
No other deductions seem obvious, so we
(ex. 1) can move on to the questions.

143
matchiNg games reView

Question 1 G) specializing in m, or else four professors (A,


The correct answer is choice b. We start by seeing F, V, and G) specializing in m. So G cannot
what deductions follow from the new specialize in m, meaning that A and F must
information, which can be symbolized A 4 F, specialize in m:
and entered into the dia- gram as follows:

A F G L P V
l ✓ ✓ ✗ ✓ ✓
V
A F G L m (ex. 3) ✓ ✗ ✗ ✗ ✓
l P ✗ ✓ ✓ ✓ ✓ ✗ ✗
m (ex. 3) ✗ ✓ ✓
s ✓ ✓ ✓ ✓ ✗ ✗ s

G may or may not specialize in l.


(ex. 1) Now compare the answer choices against
the diagram. All but b must be true (we cannot
By looking at the diagram, the way the arrows deduce that G specializes in logic, nor that G does
work together should help us realize that whatever P not specialize in logic)—so b is the correct
specializes in, F and A will specialize in them as answer.
well. This means that if P specializes in m, so will F
and A. But that would mean that at least four Question 2
professors specialize in m, contra- dicting the fourth The correct answer is choice e. Compare the
restriction (written into our diagram, that row m answer choices to the initial diagram.
has exactly three, not four, check marks). So we
know that P does not specialize in m, meaning that
P specializes in l. According to the arrows, p
specializing in l means s that F and A will A F G L P V
specialize in l. l ✗ ✓
So we have: m (ex. 3) ✗ ✓
s ✓ ✓ ✓ ✓ ✗ ✗
A F V In other words, G can’t specialize in m,
l ✓ ✓ G L✗ P✓ ✓ because then we would either have just two
m (ex. 3) ✗ ✗ ✓ professors (V and
s ✓ ✓ ✓ ✓ ✗ ✗

Now, what further deductions can be made?


We only have three boxes left for row m, from
which two must have check marks (since exactly
three must have check marks, and one check mark
already appears under V). Could G have a check
mark? If it did, then to keep exactly three check
marks for row m, one of A and F must have a
check mark, and the other would have an ✗—but
the bidirectional arrow between A and F would
prevent this.
(ex. 1) G, entered into the diagram as follows:

The only thing appearing on that diagram is that


A F G L P V
F specializes in s.
l ✗ ✓
Question 3 m (ex. 3) ✗ ✓
The correct answer is choice a. Start by seeing what s ✓ ✓ ✓ ✓ ✗ ✗
follows from the new information that G specializes
(ex. 1)
in every area in which P specializes—that is, that P S

144
matchiNg games reView

The arrows taken together mean that


whatever P specializes in, both F and G must Choice c is incorrect for a similar reason—we
specialize in. So if P specializes in m, then F and would have G, P, F, and V specializing in m.
G must, meaning that at least four professors Choice e is incorrect because we would have A,
specialize in m. So P cannot specialize in m, F, G, and V specializing in m, again violating
meaning that P specializes in l and that G and F the fourth restriction.
also, therefore: Choice a is correct because we could have
A, F, and V specializing in m, while G, L, and P
A V do not specialize in m.
F G L P
l ✓ ✓ ✗ ✓ ✓ Question 5
m (ex. 3) ✗ ✗ ✓ The correct answer is choice d. Start by seeing
s ✓ ✓ ✓ ✓ ✗ ✗ what follows from the new information, that if a
professor specializes in l, that professor also
(ex. 1) specializes in m:
Now compare the answer choices to this
diagram. Since P cannot specialize in m, choice a l
must be the correct answer. T
m
Question 4
The correct answer is choice a. Notice that we We could incorporate this into the diagram:
are looking for a partial, accurate list of professors
all of whom specialize in m. In other words, the A
F G L P V
correct an- swer choice will be all or part of a list l ✗ ✓
which could be a list of all and only those m (ex. 3) ✗ ✓
professors who specialize in metaphysics. If an s ✓ ✓ ✓ ✓ ✗ ✗
answer choice includes a professor who could not
specialize in m, it would be incorrect. That rules (ex. 1)
out choice d, since L does not specialize in
metaphysics.
By now, we should have an inkling that
A F G L P V trying to work out P’s specialties will be helpful.
l ✗ ✓ Given the new information, P cannot specialize in
l, since P would then also specialize in m (and P is
m (ex. 3) ✗ ✓
limited to one spe- cialty). So P must specialize in
s ✓ ✓ ✓ ✓ ✗ ✗
m, meaning that F also specializes in m:
(ex. 1)
A F G L P V
And if an answer choice includes a set of
l ✗ ✗ ✓
professors who couldn’t all specialize in metaphysics
m (ex. 3) ✓ ✗ ✓ ✓
at the same time, then that answer choice is
s ✓ ✓ ✓ ✓ ✗ ✗
incorrect. Choice b is incorrect because if P
specializes in m, then so does F. Then we
would have A, P, F, and V specializing in m,
violating the fourth restriction.
145
matchiNg games reView

Note that we now have all three check marks we need in 1 2 3 4 5 6


row m, so we can finish row m: m ✓ ✗
r ✓ ✓
A F G L P V w ✗
l ✗ ✗ ✓
m (ex. 3) ✗ ✓ ✗ ✗ ✓ ✓
s ✓ ✓ ✓ ✓ ✗ ✗ The third restriction tells us that 6 has
Now remember what the ✗ at the end of an exactly one more program than 5, meaning that
arrow means—that there must be an ✗ at the either 5 has one program and 6 has two, or 5 has
beginning of that arrow (since if there was a check two and 6 has three. The fourth restriction settles
mark at the be- ginning of the arrow, there must this for us; since computers 5 and 2 have exactly
be a check mark at the end of the arrow; this is two programs in com- mon, computer 5 must
essentially an application of the contrapositive). have two programs (and so computer 6 has three).
So, A and G must not special- ize in l: Furthermore, since according to the third
restriction, 5 cannot have w, those two
programs must be m and r. So we have:

A F G L P V 1 2 3 4 5 6
l ✗ ✗ ✗ ✗ ✓ m ✓ ✗ ✓ ✓
m (ex. 3)✗ ✓ ✗ ✗ ✓ ✓ s ✓ r ✓ ✓ ✓ ✓
✓ ✓ ✓ ✗ ✗ w ✗ ✗ ✓
F could either specialize in l, or not specialize Finally, the fifth restriction tells us that if a
in l without violating any restrictions. com- puter has the writing program, it also has
This means that five professors—A, G, L, P, the reading program. This can be symbolized as:
and V—have their specialties fully determined.
This is choice d. r
c
Practice Game 2 w
We have six computers: 1, 2, 3, 4, 5, and 6—each
of which will have one to three of the following The contrapositive is:
programs installed: m, r, w. The matching can be
done in a grid: ≠r
T
1 2 3 4 5 6 ≠w
m
r
w

Now symbolize the restrictions and work


them into the diagram.
The first restriction tells us that 2 will have
m and r. The second tells us that 4 will not have m
or w, and must therefore have r:

146
matchiNg games reView

We can draw the arrow directly into the diagram: This leaves us with:

1 2 3 4 5 6 1 2 3 4 5 6
m ✓ ✗ ✓ ✓ m ✓ ✓ ✓ ✗ ✓ ✓
r ✓ ✓ ✓ ✓ r ✗ ✓ ✗ ✓ ✓ ✓
w ✗ ✗ ✓ w ✗ ✗ ✗ ✗ ✓

All the information from the restrictions has The only answer choice not given in this
been captured in this diagram, and we can now diagram is that computer 2 does not have a
just refer to this diagram as we tackle the writing program; it is left undetermined whether
questions. computer 2 has a writ- ing program. So the
correct answer is c.
Question 1
The correct answer is choice c. The work here is Question 3
pretty much done already. Consulting the The correct answer is choice a. Start by consulting
diagram, we see that three computers—3, 4, and 5 the diagram to see what the new information
—are fully deter- mined on the grid, and three—1, means.
2, and 3—are not. This is choice c. To double-
check this, see if you can put checks and ✗’s into
1 2 3 4 5 6
each of the columns for 1, 2, and 3, without
m ✓ ✗ ✓ ✓
violating the restrictions. For example, imagine
r ✓ ✓ ✓ ✓
putting a check into m1, m3, and w2, and then an
w ✗ ✗ ✓
✗ into the same—no matter what, as long as the
arrow from w to r is satisfied, either one (checks
into all three and ✗’s into all three) lead to Since only two computers can have exactly
acceptable tables—note that as long as each the same set of programs, every column must be
computer has at least 1 check mark, and the arrow different from every other column (when fully
is satisfied, all the re- strictions of the game are determined with
satisfied (because they are all built into this ✗’s and check marks), save for one set of two
diagram already). columns. 4, 5, and 6 are set already. What about
column 2? Column 2 can either have an ✗ in the
W-box or a check in the W-box. Either way, it will
Question 2
be identical with either col- umn 5 or column 6.
The correct answer is choice d. Consider the
Therefore, columns 1 and 3 need to be different
dia- gram. If the reading program is installed on
from all of the other columns and also need to be
exactly four computers, then we can cross out the
different from each other. Now we need to figure
r-box for 1 and 3. And if we do this, because of
out what can go into columns 1 and 3. We have
the arrow, we can cross out the w-box for 1 and
three of the seven vertical patterns instantiated in
3 as well. And if we do this, that leaves the m-
4, 5, and 6:
box for 1 and 3 to be checked (since each
computer must have at least one program.
✗ ✓ ✓
✓ ✓ ✓
✗ ✗ ✓
147
matchiNg games reView

The remaining four possible patterns are: Therefore, it is possible to have only four
check marks in row r, and the correct answer is
✗ ✓ ✓ ✗ four, choice d.
✗ ✗ ✗ ✓
✓ ✗ ✓ ✓
Question 5
The correct answer is choice b. Compare each
But notice that the first and third patterns answer choice against our diagram. The diagram
violate the arrow—the w-box would be checked while doesn’t seem to prevent four computers from
the r-box is ✗’d. So we only have the following having the writing program—6 already has it,
patterns which are to describe columns 1 and 3 and it seems 1, 2, and 3 could have it. So a is
(one pattern each): incorrect. But since there are
✗-marks in two of the six w-boxes, it can’t be that
✓ ✗ five computers have the writing program—so
✗ ✓
✗ ✓ choice b is correct.

Now compare each answer choice to this set. Practice Game 3


In either case, computer 1 (as well as computer 3) We have five entrees—C, B, P, F, V—each of which
cannot have both a math and writing program (check can come with one or more of three sides, salad
marks in the top and bottom row). This is choice a. (s), bread (b), or rice (r). The matching can be
Computer 1 could have the right-hand pattern, done in a grid:
making b incorrect. Computer 3 could have the right-
hand pattern, making c incorrect. Computer 2 could
C B P F V
have a check mark in the writing-box, making d
s
incorrect. Computer 3 could have the left-hand
b
configuration, making e incorrect.
r

Question 4 Now symbolize the restrictions and work


The correct answer is choice d. Consult the them into the diagram.
diagram. The first restriction is that C has exactly
one more side than B. This means that either B
has two sides and C has three, or B has one side
1 2 3 4 5 6
and C has two. The second restriction says that F
m ✓ ✗ ✓ ✓
has exactly one less side than B. This means that
r ✓ ✓ ✓ ✓
either F has one side and B has two, or F has two
w ✗ ✗ ✓
sides and B has three. The only way these two
We want to enter at least one more check restrictions can both be true is if B has two sides—
mark into the w-row, and see what the least meaning that C has three sides and F has one. So
number of check marks in the r-row there could we have:
be. If we enter a check mark into w1 or w3, then
the arrow indicates that we would have a check C B P F V
mark in r1 or r3, respectively. This would produce
s ✓
at least five check marks in row r. But if we put a
b ✓
check mark in w2, and no other box in the w-
r ✓
row, then we would only have to produce a check
ex. 2 ex. 1
mark in r2, where there is already a check mark.
148
matchiNg games reView

The third restriction is that P comes with s. Question 1


The fourth restriction is that P and B share no The correct answer is choice d. According to the
sides: dia- gram, C, B, and P are fully determined with
respect to the sides that accompany each. F and V
B d✗S P are not de- termined. For example, F might come
with salad, or not come with salad; and V might
So we have: come with bread or not come with bread—and all
these cases are compat- ible with the restrictions.
✗ So the correct answer is that the exact number of
C B P F V sides can be determined for three entrees (C, B,
s ✓ ✓ and P)—choice c.
b ✓
r ✓ Question 2
ex. 2 ex. 1 The correct answer is choice a. If the vegetarian
entrée comes with fewer sides than the beef, then
since the beef comes with two sides, the vegetarian
This means that an ✗ must appear in the s-
entrée must come with one side. So we have:
row under B: B does not come with s. However,
we know that B must come with two sides, so we
now know which sides B comes with: b and r. C B P F V
And since P and B share no sides, this means that s ✓ ✗ ✓
P cannot come with b or r—so P must only come b (3-4) ✓ ✓ ✗
with s. So we have: r ✓ ✓ ✗
ex. 1 ex. 1
C B P F V We can see that C comes with three sides,
s ✓ ✗ ✓ and each of P, F, and V come with exactly 1
b ✓ ✓ ✗ side. Only B comes with exactly two sides—so the
r ✓ ✓ ✗ correct answer is that one entrée comes with
ex. 1 exactly two sides. This is choice a.
Finally, the fifth restriction says that three
Question 3
or four entrees come with bread:
The correct answer is choice c. If there are more
en- trees that come with rice than entrees that
C B P F V come with bread and entrees that come with
s ✓ ✗ ✓ salad, then since we know that the bread must
b (3-4) ✓ ✓ ✗ come with at least three en- trees, we know that
r ✓ ✓ ✗ the rice must come with four en- trees (it cannot
ex. 1 come with five entrees, since we know it does not
come with the pork). Glancing at the dia- gram,
this means that the rice must come with both the
fish and vegetarian entrée. But then, since the fish
comes with exactly one entrée, we know that the
fish does not come with the salad or bread.
149
matchiNg games reView

So we have: Question 5
The correct answer is choice d. We can symbolize
C B P F V the new information that every side that comes
s ✓ ✗ ✓ ✗ with the vegetarian entrée also comes with the
b (3-4) ✓ ✓ ✗ ✗ fish as:
r ✓ ✓ ✗ ✓ ✓
VSF
Contrapositive: not F S not
Finally, since we must have 3-4 check marks
V Or written into the
in the b-row, we see that the V must come with
diagram:
b:

C B P F V C B P F V
s ✓ ✗ ✓ ✗ s ✓ ✗ ✓
b (3-4) ✓ ✓ ✗ ✗ ✓ b (3-4) ✓ ✓ ✗
r ✓ ✓ ✗ ✓ ✓ r ✓ ✓ ✗
ex. 1
The only paring that is not given in this
The arrow combined with the note that F
diagram is that V comes with s—and that is
has exactly one check mark should give us pause.
choice c.
Since every time we have a check mark under V,
we must have a check mark in F, the arrow means
Question 4
that we can have at most one check mark under V
The correct answer is choice b. If F comes with s, (which means we have exactly one check mark
then we know that F cannot come with b or r: under V, since every entrée has at least one side).
Further, since there must be three to four
C B P F V checkmarks in row b, F must have a checkmark
s ✓ ✗ ✓ ✓ in row b—for if it didn’t, then V couldn’t have a
b (3-4) ✓ ✓ ✗ ✗ checkmark in row b either, and we wouldn’t have
r ✓ ✓ ✗ ✗ 3-4 checkmarks in row b. Since F must have ex-
ex. 1 actly one checkmark, it must have ✗-marks in
row s and row r—meaning that V must have ✗-
But since we know that 3-4 check marks must
marks in row s and row r, leaving a checkmark
ap- pear in the b-row, this means that V must come
for V in row b:
with b:

C B P F V
C B P F V
s ✓ ✗ ✓ ✓
s ✓ ✗ ✓ ✗ ✗
b (3-4) ✓ ✓ ✗ ✗ ✓
b (3-4) ✓ ✓ ✗ ✓ ✓
r ✓ ✓ ✗ ✗
r ✓ ✓ ✗ ✗ ✗
ex. 1
ex. 1 ex. 1
Among the answer choices, the only
information that appears in this diagram is that V
Choice d is the only answer choice
comes with b— answer choice b.
inconsistent with this diagram—exactly two, not
three, entrees come with salad.
150
matchiNg games reView

Practice Game 4 R S A J
We are asked to match each of four people—R, S, A, and J—to one c
or more of four items, c, e, k, m: e
k
R S A J m (ex. 3)
c ex. 1 ex. 2
e
✗ ■■
k
m The fifth restriction can also be symbolized
with a regular conditional:
The first, second, and third restrictions can
AS J
be incorporated into the diagram:
≠JS≠A
[contrapositive] And
R S A J the sixth restriction:
c cS m
e ≠mS ≠c
k
m (ex. 3) Now we make deductions. We see that row
ex. 1 ex. 2 m will contain exactly three check marks. Since S
The fourth restriction can be symbolized and A have a difference arrow, it can’t be that
with a ‘difference arrow’: two of those three checkmarks are in the S-box
and A-box. So at most one of those two will
have a check mark in row m (in fact, exactly one
R S A J of those two will have a check mark in row m).
c This means that both R and J must have a check
e mark in row m. Furthermore, since R has a
k check mark in row m, and since R only has one
m (ex. 3) item, all the other boxes in R’s column must be
ex. 1 ex. 2 ✗’s:

It can also be symbolized with regular R S A J
conditionals: c

e ✗
SS ≠A
k ✗
A S ≠ S [the contrapositive]
m (ex. 3) ✓ ✓
ex. 2
The fifth and sixth restrictions can be
✗ ■■
symbol- ized with one-way similarity arrows:

151
matchiNg games reView

Question 1
R S A J
The correct answer is choice a. This is directly
given in the diagram—according to the diagram, c ✗ ✗
Ross does not bring extra socks. e ✗ ✗ ✓ ✓
k ✗ ✗ ✓ ✓
Question 2 m (ex. 3) ✓ ✓ ✗ ✓
The correct answer is choice b. If S brings m, ex. 2
then since row m must have exactly three ✗ ■■
checkmarks, A does not bring m (this is also
apparent from the dif- ference arrow: A does Now we compare the answer choices against
not bring any item S brings): this diagram. According to the diagram, S may or
may not bring c, so a is incorrect. S definitely
does not bring e, so b is the correct answer. A in
R S A J
fact brings e and k, so c and d are incorrect, and
c ✗ J may or may not bring c, so e is incorrect.
e ✗
k ✗ Question 3
m (ex. 3) ✓ ✓ ✗ ✓ The correct answer is choice e. If A brings c then as
ex. 2 the one-way arrows indicate, A brings m and J
✗ ■■ brings c:

Glancing at the arrows, we see that since A


R S A J
does not bring m, he cannot have brought c either.
c
This leaves A to bring e and k (since he must ✗ ✓ ✓
bring two items). e ✗
At this point, we have: k ✗
m (ex. 3) ✓ ✓ ✓
ex. 2
R S A J
c ✗ ■■
✗ ✗
Since A must bring exactly two items, we have:
e ✗ ✓
k ✗ ✓
m (ex. 3) ✓ ✓ ✗ ✓ R S A J
c
ex. 2 ✗ ✓ ✓
✗ e ✗ ✗
■■
k ✗ ✗
Glancing at the arrows again, we see that m (ex. 3) ✓ ✓ ✓
these two checkmarks, in kA and eA, lead us to ex. 2
put check- marks in kJ and eJ, and ✗-marks in kS ✗ ■■
and eS. That is, we see that J must bring e and k,
and S cannot bring either e or k:

152
matchiNg games reView

And since whatever A has, S cannot (the be constructed consistent both with that answer
differ- ence arrow), we have: and the restrictions. Use prior work. The diagram
from question 2 shows that at most likely, S
R S A J could either bring exactly one item or exactly
c ✗ ✗ ✓ ✓ two items.
e ✗ ✗
k ✗ ✗ R S A J
m (ex. 3) ✓ ✗ ✓ ✓ c ✗ ✗
ex. 2 e ✗ ✗ ✓ ✓
✗ ■■ k ✗ ✗ ✓ ✓
m (ex. 3) ✓ ✓ ✗ ✓
Now compare the answer choices against
ex. 2
this diagram. S brings neither c nor m, so choices
✗ ■■
a and b are incorrect. A does not bring e, so c is
incorrect. J brings m, so d is incorrect. But J may Using this table, we can test two scenarios—
or may not bring e, so choice e is correct. one in which S brings c (a total of exactly two
items) and one in which S does not bring c (a total
Question 4 of exactly one item). Both seem consistent with the
The correct answer is choice c. Consult the restrictions, so we move on to answer choice c. If J
diagram: brings exactly one item, then since we already
know he brings m, he can- not bring c, e, or k:
R S A J
c
✗ R S A J
e ✗ c ✗ ✗
k ✗ e ✗ ✗
m (ex. 3) ✓ ✓ k ✗ ✗
ex. 2 m (ex. 3) ✓ ✓
✗ ■■ ex. 2
✗ ■■
Glance at each answer choice and compare it
to the diagram to see if anything quickly strikes The one-directional arrow tells us that A
you as impossible. Looking at S’s column, you cannot bring c, e, or k either:
don’t imme- diately see that he can’t bring one or
can’t bring two items. So choices a and b can be
R S A J
set aside for the mo- ment. Looking at J’s column,
you might immediately see that J brings anything c ✗ ✗ ✗
A brings, and that A brings exactly two items—so e ✗ ✗ ✗
J can’t bring just one item. So choice c is correct. k ✗ ✗ ✗
If you missed this and your quick glance m (ex. 3) ✓ ✓
at the answer choices didn’t yield the answer, ex. 2
then go through each choice more systematically. ✗ ■■
Take each answer and check if it seems that a
scenario could 153
matchiNg games reView

But then there is no way that A can bring S2


ex- actly two items. So it can’t be that J brings
R S A J
exactly one item—choice c. c
✗ ✓ ✓
e ✗
Question 5
The correct answer is choice e. If the number of k ✗
c checkmarks is greater than the number of k m (ex. 3) ✓ ✓
check- marks, then there must be either two or ex. 2
three c check- marks (we already know that R does ✗ ■■
not have a c). If it is three, then each of S, A and
J will bring c. But following the arrow, this will For S1, we can follow the arrows to fill in mA
mean that each of S, A, and J will also bring m— and cross out cS and mS. To keep column A at
and this will exceed the cap of three checkmarks exactly two items, we also cross out eA and kA:
for row m.
S1

R S A J R S A J
c c
✗ ✓ ✓ ✓ ✗ ✗ ✓ ✓
e e ✗ ✗

k k ✗ ✗

m (ex. 3) m (ex. 3) ✓ ✗ ✓ ✓
✓ ✓ ✓ ✓
ex. 2 ex. 2
✗ ■■ ✗ ■■
For S2, we follow the arrows to put a
So this diagram is not possible, and there checkmark in mS and ✗-mark in cA. To keep row
must be exactly two checkmarks in row c. The two m at exactly three checkmarks, we put an ✗ in
check- marks cannot be under S and A, since mA.
again, that would result in four checkmarks for
row m. So one must be under J, and the other S2
must be under one of S or A. We can see what R S A J
happens for both scenarios: c ✗ ✓ ✗ ✓
e ✗
S1
k ✗
R S A J m (ex. 3) ✓ ✓ ✗ ✓
c ✗ ✓ ✓ ex. 2
e ✗ ✗ ■■
k ✗
m (ex. 3) ✓ ✓
ex. 2
✗ ■■

154
matchiNg games reView

Then, to keep column A at exactly two The first restriction says that only Fords can
check- marks, we fill in the remaining spots in have fog lights. We symbolize this as:
that column, eA and kA. This leads to checkmarks
in eJ and kJ, and fS F
✗-marks in eS and kS:
This indicates that if a car has fog lights, it
S2 must be a Ford.
R S A J And the contrapositive:
c ✗ ✓ ✗ ✓
e ✗ ✗ ✓ ✓ ≠FS ≠f
k ✗ ✗ ✓ ✓
m (ex. 3) ✓ ✓ ✗ ✓ The second restriction says that car 2 has
ex. 2 more options than car 3. So car 2 must have
✗ either two or three options, and car 3 must have
■■
either one or two options. We can write that
This determines the entire diagram. We information into the dia- gram, along with the
now compare each answer choice against the two information that one is greater than the other (see
scenarios to see if that choice could be a list of all below).
those bringing e’s. Choices a, b, and d are made The third restriction says that exactly two
possible in S1. Choice c is made possible in S2. cars have fog lights. Note that in the diagram, the
Choice e is not possible in ei- ther scenario: in S1, fourth restriction is that car 2 is a Dodge and the
Alex does not have extra socks and in S2, fifth that car 4 has no spoiler—this can all be
Siddarth does not have extra socks. directly incorporated into the diagram, which
So e is the correct answer. now looks like:

Practice Game 5 1 2 3 4
We have four cars, labeled 1 to 4, and two sets of s ✗
char- acteristics to be assigned to each. The first f (ex. 2)
set is op- tions—each car has one or more the p
following options: spoiler (s), fog lights (f), and F/D/C D
performance tires (p). The second is each car is one 2 or 3 > 1 or 2
of Ford, Dodge, or Chevy. We diagram the
matching with a grid:
f S F [≠ F S ≠ f]

1 2 3 4
Now for the deductions: Since car 2 is a
s
Dodge and only Fords can have fog lamps, we
f
know that car 2 cannot have f. Since car 2 cannot
p
have fog lamps, it must have two options rather
F/D/C
than three (which must be s and p). This means
that car 3 must have exactly one option.
155
matchiNg games reView

The diagram is Since car 3 does have fog lights, the statement in
now: choice c can’t be true, making c incorrect. Since,
1 2 3 4 according to the diagram, car 4 may or may not
s ✓ ✗ have performance tires, choice d is the correct
answer. Since car 4 has fog lights, the statement in
f (ex. 2) ✗
choice e can’t be true, making e incorrect.
p ✓
F/D/C D
Question 3
ex. 1 The correct answer is choice c. We realized in our de-
We also note that since there are exactly two ductions that since there are exactly two cars with
fog lamps and only Fords can have fog lamps, fog lamps, and only Fords have fog lamps, there
there must be at least two Fords. must be at least two Fords. We already know one
car is a Dodge. There could be more Dodges than
Question 1 Chevys if there were no Chevys, so choice a is
The correct answer is choice a. Compare each incorrect. There could be an equal number of
an- swer choice to the diagram. Since car 2 has a Dodges and Chevys if there were one Chevy in
spoiler but no fog lights, it can’t be that every addition to the Dodge, making choice b incorrect
car with a spoiler has fog lights, and choice a is (we saw this in the diagram for Question 2). There
the correct an- swer. The rest of the answer could not be more Dodges than Fords, because
choices are each com- patible with this diagram. that would require three Dodges (since there are
In particular, choice e is incorrect because it two Fords), meaning there would be five cars, not
could be the case that every car with four— so c is the correct answer. There could be
performance tires with the exception of car 2 also more Fords than Dodges if there were one Dodge
has fog lights. and one Chevy, so d is incorrect (again, we saw
this in the diagram for Question 2). And there
Question 2 could be an equal number of Dodges and Fords if
The correct answer is choice d. If car 1 is a Chevy, there were an additional Dodge and no Chevy’s,
then it cannot have f, and so cars 3 and 4 must so choice e is incorrect.
have f (and be Fords). Since car 3 has f, it cannot
have s or p: Question 4
The correct answer is choice e. If car 4 is a Dodge,
1 2 3 4
then it cannot have fog lights and thus must have
s
perfor- mance tires:
✓ ✗ ✗
f (ex. 2) ✗ ✗ ✓ ✓
p ✓ ✗ 1 2 3 4
F/D/C C D F F s ✓ ✗
ex. 1 f (ex. 2) ✗ ✗
p ✓ ✓
F/D/C D D
Now compare each answer choice to the
ex. 1
dia- gram. Since car 1 does not have fog lights, the
state- ment in choice a can’t be true, making a
incorrect. Since car 3 does not have performance Since there are exactly two cars with fog
tires, the state- ment in choice b can’t be true, lights, we now know that these two must be
making b incorrect. cars 1 and 3
156
matchiNg games reView

(which must be Fords). Since car 3 has exactly one could be true). Depending
op- tion, it cannot have a spoiler or performance
tires:

1 2 3 4
s ✓ ✗ ✗
f (ex. 2) ✓ ✗ ✓ ✗
p ✓ ✗ ✓
F/D/C F D F D
ex. 1
Now compare each answer choice to the
dia- gram. Choice a is incorrect because car 4
has per- formance tires but no spoiler. Choice b
is incorrect because car 3 has fog lights but no
spoiler. Choice c is incorrect because car 2 has a
spoiler but no fog lights. Choice d is incorrect
because cars 2 and 4 have per- formance tires
but no fog lights. Choice e is correct because
there is no car that definitely has a spoiler but no
performance tires: car 2 has both, cars 3 and 4 lack
spoilers, and car 1 could have both a spoiler and
per- formance tires (or performance tires and no
spoiler, or neither performance tires nor a
spoiler).

Question 5
The correct answer is choice a. If exactly three
cars have spoilers, then those three must be cars 1,
2, and 3. If car 3 has a spoiler, it does not have fog
lights or per- formance tires. And if car 3 does not
have fog lights, then cars 1 and 4 must have fog
lights and be Fords:

1 2 3 4
S ✓ ✓ ✓ ✗
f (ex. 2) ✓ ✗ ✗ ✓
P ✓ ✗
F/D/C F D F
ex. 1
Now compare the answer choices to this
dia- gram. Car 1 has at least two options (s and f),
so it can’t have exactly one option and a is the
correct answer. The other choices are incorrect
since they are compat- ible with the diagram (they
on whether it has performance tires, car 1 could have two or Symbolize the first restriction
three options, making choices b and c incor- rect. And
depending on whether it has performance tires, car 4 could as: H next to H
have two or three options, making choices d and e incorrect.
Symbolize the second restriction as:
Practice Game 6
Since this game uses a set of entities (buildings) that have a l b
particular spatial configuration, and since their spatial T c
relationships will figure into the game rules, the diagram b l
should use this spatial configura- tion. For each building, 1
through 6, it is to be deter- mined whether it is historical (H) This reminds us that if we have a law firm
or modern (M), and whether it contains a law firm (l), bank (l), there must be a bank (b) across the street.
(b), or both. Write the letters under each number as matches Keep in mind that this does not mean that
are determined. The third, fourth, and fifth restrictions all every bank has a law firm across the street from
give specific information, so write this in: it.
Also, keep the contrapositives in mind:
1 2 3
l H b l
4 5 6 T c
M l b

157
matchiNg games reView

Now make deductions by applying the first Now compare the answer choices to this
and second restrictions to the diagram. Since 3 is dia- gram. Choice c, that building 1 is historical, is
H, 2 must be H. Since 2 is l, 5 must be b. Also, the only choice given in this diagram. The other
since 4 is M, 5 and 6 are either both H (to satisfy choices give statements that could be false without
the ‘H next to H’ rule) or both M. These seem to contradicting this diagram. Note that choice e is
exhaust the immediate de- ductions that can be incorrect because building 6 could contain a law
made. firm or bank. The ques- tion stem says that every
The diagram is now: bank is located in a historical building, not that
every historical building contains a bank (the two
1 2 3 are not the same). This goes for choice a as well.
lH H
4 5 6 Question 2
M b The correct answer is choice b. Since we already
have one modern building, we must place two
Question 1 more M’s into the diagram. If one of those M’s
The correct answer is choice c. If every bank is were placed under building 1, then we would
located in a historical building, then building 5 have an M under one of buildings 5 and 6, and
must be H. But we must also consider the an H under the other. But either way, we would
contrapositive, that if a building is not historical violate the restriction that ‘H next to H’—the H on
(i.e., is modern), then it can- not have a bank (and the 4-5-6 side of the street would not be adjacent
therefore must have a law firm). This means that to any other H. So both M’s must be placed
building 4 has an l: under buildings 5 and 6, making building 1 H:
1 2 3
lH H 1 2 3
4 5 6 H lH H
bl M bH 4 5 6
M bM M

Since building 4 has an l, building 1 must


have a b, and therefore—according to the Now scan the answer choices for something
supposition of this question—be located in a given in this diagram. Choice b says that one
historical building: bank must be in a modern building, and we know
this must be true since building 5 is a modern
1 2 3 building contain- ing a bank. The other choices all
bH lH H show statements that could be false. In particular,
4 5 6 choice e is incorrect be- cause it could be that there
bl M bH is another modern building containing a bank.

Finally, since 5 is H, 6 must also be H (to Question 3


satisfy the ‘H next to H’ rule): The correct answer is choice b. This question
requires playing with the diagram to see how to
1 2 3 maximize the number of modern buildings
bH lH H containing law firms. The first step, however, is to
4 5 6 see what follows from the fact that every bank is
bl M bH H located in a modern building
158
matchiNg games reView

(and the contrapositive that every historical 3 contain a law firm and no bank. Further, the
building lacks a bank and contains a law firm). con- trapositive of this new information—that if a
Given this in- formation, building 5 must be building contains a bank, it is modern—tells us
modern, and buildings 2 and 3 must contain a that building 5 is modern:
law firm and no bank (since they are historical):
1 2 3
1 2 3 bl H bl H
bl H bl H 4 5 6
4 5 6 M bM
M bM

The second restriction gives us that building


Since building 3 contains a law firm, 6 contains a bank—and again, by the
build- ing 6 must contain a bank (by the contrapositive of the new supposition, is modern
second restric- tion), and therefore, by the new (or since 5 is modern, 6 must be as well, to satisfy
supposition, must be modern (you could also the ‘H next to H’ rule):
note that building 6 has to be modern because
there is no historical build- ing next to it): 1 2 3
bl H bl H
1 2 3 4 5 6
bl H bl H M bM bM
4 5 6
M bM bM Finally, by the contrapositive of the second
re- striction, we know that 5 and 6 don’t contain l’s
Now, we are looking to maximize the (since 2 and 3 don’t contain b’s).
number of modern buildings containing only a
law firm (no bank). So buildings 2 and 3 are out 1 2 3
because they are historical, and buildings 5 and 6 bl H bl H
are out because they contain banks. So we must 4 5 6
focus on buildings 1 and 4. If building 4 contain a M blM blM
law firm, then building 1 con- tains a bank (by the
second restriction), leaving build- ing 4 as the only Scan the answer choices for information
possible modern building containing only a law given in this diagram. Choice d, that building 6 is
firm. And if building 1 contains a law firm, then modern and contains a bank, is given in this
building 4 would contain a bank (by the second diagram; therefore, that is the correct answer.
restriction), leaving building 1 to be the only
possible modern building containing only a law Question 5
firm. So the maximum number of modern The correct answer is choice a. Consider each
buildings containing only a law firm is one— answer choice in turn. If building 6 were modern
choice b. and build- ing 5 historical, then the first
restriction would be violated: building 5 would
Question 4 not have any adjacent his- torical building. The
The correct answer is choice d. If every other choices represent possibili- ties that are
historical building contains only a law firm, then consistent with the restrictions.
buildings 2 and
159
matchiNg games reView

Practice Game 7 this information in mind, and note it by writing ‘1


Since the set of characteristics to be assigned to o’ under each group.
the six skydivers are such that only one can be So we have:
assigned to each skydiver, it may be better to use
the column- approach rather than a grid: m f
A B C D E F
m f
r/g/o/p r/g/
g r/g
A B C D E F o/p
r/g/o/p r/g/o/p 1 o 1 o

B = r or o S E = g
The double-line indicates the division E ≠ g S B ≠ r and ≠ o
between male and female. [the contrapositive]
The first restriction tells us that we will
need different colors for each of the group A, B, C, Question 1
and for each of the group D, E, F. We will have to The correct answer is choice b. Take each answer
keep this in mind: to make sure you don’t forget, in turn, and check to see if it can be ruled out by
write this down next to your diagram or circle any de- ductions, or seems to be possible (i.e., can
it. We have included “r/g/o/p” for each of the two be worked into an assignment of colors to
groups, male and female, so that as we assign skydivers consistent with the rules).
colors we can cross the appropriate letter out and Choice a assigns r to B and D:
keep track of what colors are available for that
group. m f
The second restriction tells us that A is
A B C D E F
green, and the third that D is not orange or
r/g/o/p r/g/
purple, meaning that D is red or green. This g r r
o/p
information can be put into the diagram:
1 o 1 o

m f B = r or o S E = g
A B C D E F E ≠ g S B ≠ r and ≠ o
r/g/o/p r/g/ [the contrapositive]
g r/g
o/p
This means that E = g (by the fourth
The fourth restriction can be condition), and that C and F = o (by the fifth):

symbolized: B = r or o S E = g m f
E ≠ g S B ≠ r and ≠ o A B C D E F
[the contrapositive] r/g/o/p r/g/
g r o r g o
o/p
The fifth restriction tells us that there must
1 o 1 o
be exactly one orange parachute in the male
group and exactly one in the female group. We
will have to keep

160
matchiNg games reView

B = r or o S E = g
m f
E ≠ g S B ≠ r and ≠ o [the
contrapositive] A B C D E F
r/g/o/p r/g/
g p g r
This assignment does not seem to contradict o/p
any game rules, so move on to test the next answer 1 o 1 o
choice.
If C = r, then by the fifth restriction, B = o; B = r or o S E = g
and if F = g, then D cannot be G and D must be E ≠ g S B ≠ r and ≠
r, leaving o for E: o[the contrapositive]

m f The fifth restriction puts o in C and F:


A B C D E F
m f
r/g/o/p r/g/
g o r r o g
o/p A B C D E F
1 o 1 o r/g/o/p r/g/
g p o g r o
o/p
B = r or o S E = g 1 o 1 o
E ≠ g S B ≠ r and ≠ o [the
contrapositive] B = r or o S E = g
E ≠ g S B ≠ r and ≠ o
But this contradicts the fourth restriction: B = [the contrapositive]
o, but E is ≠ g. So the statement in b cannot be
true, and b is the correct answer. Now compare the answer choices to this
If you found yourself stuck on this dia- gram. Choice c is correct, as the diagram
question, or find that you take too much time for shows that there are exactly two green
these types of questions in general (in which parachutes.
you need to do a lot of work with each answer
choice to see if it represents an acceptable Question 3
scenario), then it might be a useful strategy to The correct answer is choice a. Look at the fourth
return to this question after you have done the re- striction, which governs the relationship
other questions. In this case, you would have between E and B. If B is r or o, E and B cannot
seen that choice c can be ruled out by Question have the same color. So B cannot be r or o; and
5, and that choices d and e can be ruled out by g is already taken in the male group; so B must
Question 4. be p. This means that E must be p, since B and E
have the same color:
Question 2
The correct answer is choice c. If E has a red m f
parachute, then D must have a green one. Since E A B C D E F
is not green, by the contrapositive of the fourth r/g/o/p r/g/
restriction, B is not red or orange; and since g is g p r/g p
o/p
taken in the male group, B must be p: 1 o 1 o
161
matchiNg games reView

B = r or o S E = g Furthermore, E cannot be green; and so by the


E ≠ g S B ≠ r and ≠ o [the contrapositive of the fourth restriction, B cannot be
contrapositive] red or orange, and so must be purple. This leaves or-
ange for C (by the fifth restriction). E and F are
By the fifth restriction, we can fill in the then left with orange and purple (since red is taken
remain- ing spaces with o: and green is forbidden). The assignment of o and p
to E and F could go either way:
m f
A B C D E F m f
r/g/o/p r/g/ A B C D E F
g p o r/g p o
o/p r/g/o/p r/g/
g p o r o/p o/p
1 o 1 o o/p

B = r or o S E = g 1 o 1 o
E ≠ g S B ≠ r and ≠ o [the B = r or o S E = g
contrapositive] E ≠ g S B ≠ r and ≠ o
[the contrapositive]
Now look at the answer choices to see which
is contradicted by this diagram. Choice a is According to this diagram, then, F could be
contradicted: the diagram shows that F has an ei- ther orange or purple, and could not be any
orange parachute, not a green one. So a is the other color. Therefore, e is the correct answer.
correct answer.
Question 5
Question 4 The correct answer is choice e. If C’s parachute is
The correct answer is choice e. If there is exactly pur- ple, then B must be orange (by the fifth
one green parachute, then the female group restriction). Since B is orange, the fourth restriction
cannot have a green parachute, since the male tells us that E is green. Since E is green, green is not
group already has one (A is green). an option for D, and D must be red. This leaves
So D must be r: orange for F, and we have:

m f m f
A B C D E F A B C D E F
r/g/o/p r/g/ r/g/o/p r/g/
g r g o p r g o
o/p o/p
1 o 1 o 1 o 1 o
B = r or o S E = g B = r or o S E = g
E ≠ g S B ≠ r and ≠ o [the E ≠ g S B ≠ r and ≠ o
contrapositive] [the contrapositive]

162
matchiNg games reView

This shows us that all six skydivers have The fifth restriction essentially says that
their parachute colors determined—choice e. what- ever L tastes, I tastes as well. This can be
symbolized by drawing yet another arrow into
Practice Game 8 the diagram:
This game has 5 critics—H, I, J, K, and L—
matched up to 1–3 of t, g, and m:
H I J K L
t (ex. 3) ✓ ✓
H I J K L g ✗ ✓
t m ✓ ✓
g
m One deduction that can be immediately
made is that since only three critics taste the
The first restriction puts t and m under H. Talisker, L cannot taste the Talisker; for if L did,
The second says that H tastes fewer scotches then I would also taste the Talisker and there
than K— so H must taste two and K must taste would be a total of four critics (H, I, K, L) tasting
all three. The third restriction says that exactly the Talisker.
three critics taste the Talisker—this can be noted So the final diagram looks like:
in the diagram:
H I J K L
H I J K L t (ex. 3) ✓ ✓ ✗
g ✗ ✓
t (ex. 3) ✓ ✓
m ✓ ✓
g ✗ ✓
m ✓ ✓
Question 1
The correct answer is choice b. Consulting the
The fourth restriction can be initial diagram, we see that only H and K are
fully deter- mined with respect to exactly which
symbolized: I = m S J = t and g scotches they do or do not taste—so two, choice b,
J ≠ t or J ≠ g S I ≠ m [the is the correct answer.
contrapositive]
Question 2
This can also be symbolized by drawing The correct answer is choice b. Follow the
arrows directly into the diagram: arrows. If L tastes m, then so does I. If I tastes m,
then J tastes t and g. We have:

H I J K L H I J K L
t (ex. 3) ✓ ✓ ✓ ✗
t (ex. 3) ✓ ✓
g ✗ ✓ ✓
g ✗ ✓
m ✓ ✓ ✓
m ✓ ✓ ✓

163
matchiNg games reView

Since there must be exactly three critics The numerical constraints in row t (exactly
tasting Talisker, we can also put an ✗ for t under three checkmarks) and row g (exactly two
I: checkmarks) are now met, so we can put ✗-marks
in those rows under I:
H I J K L
t (ex. 3) ✓ ✗ ✓ ✓ ✗ H I J K L
g ✗ ✓ ✓ t (ex. 3) ✓ ✗ ✓ ✓ ✗
m ✓ ✓ ✓ ✓ g (ex. 2) ✗ ✗ ✓ ✓ ✗
Now compare the answer choices to this m ✓ ✓ ✓ ✓
dia- gram, looking for one that contradicts the
diagram. Now compare the answer choices to the
Choice b contradicts the diagram—I does not diagram and look for a statement that could be
taste the Talisker—and so choice b is the correct false.
answer. According to the diagram, the statement that
J tastes m, could either be true or false—and so
Question 3 choice d is the correct answer.
The correct answer is choice d. First, note the new
in- formation, that exactly two critics taste g, in Question 4
the dia- gram. Then start by seeing how many The correct answer is choice a. If I tastes t, then
different ways exactly two critics might taste g, and since exactly three critics taste t, J cannot taste t.
see what follows. There is already one checkmark So we have:
in row g (under K), so we must place exactly one
more. If we were to place the checkmark under L, H I J K L
we would also have to place one under I, resulting t (ex. 3) ✓ ✓ ✗ ✓ ✗
in three checkmarks for row g. So we can put an g ✗ ✓
✗ under L: m ✓ ✓

H I J K L Since there is an ✗ at the end of the arrow, in


t (ex. 3) ✓ ✓ ✗ row t under J, there must be an ✗ in row m under
g (ex. 2) ✗ ✓ ✗ I:
m ✓ ✓
H I J K L
Since there are now two ✗-marks under L t (ex. 3) ✓ ✓ ✗ ✓ ✗
(i.e., L tastes neither t nor g), we can put a check g ✗ ✓
mark in row m under L. Following the arrows, m ✓ ✗ ✓
that check mark re- sults in a check mark in row m
under I, which in turn results in check marks
under J for rows t and g:

H I J K L
t (ex. 3) ✓ ✓ ✓ ✗
g (ex. 2) ✗ ✓ ✓ ✗
m ✓ ✓ ✓

164
matchiNg games reView

Further, if I doesn’t taste m, then L cannot taste Since L does not taste t, L cannot taste g (an
m. This means that L must taste g, and so I must taste g: ap- plication of the contrapositive of our new arrow).
This means that L tastes m. Following the arrows, I
H I J K L tastes m, and therefore J tastes t and g:
t (ex. 3) ✓ ✓ ✗ ✓ ✗
g ✗ ✓ ✓ ✓ H I J K L
m ✓ ✗ ✓ ✗ t (ex. 3) ✓ ✓ ✓ ✗
g ✗ ✓ ✓ ✗
m ✓ ✓ ✓
Now compare the answer choices to this ✓
dia- gram and look for one contradicted by the
diagram.
Choice a—that I tastes the Macallan—contradicts We now have three checkmarks in row t,
the diagram, and so is the correct answer. so I does not taste t. Again applying the
contrapositive of our new arrow, we see that I
Question 5 cannot taste g:
The correct answer is choice d. Start by
incorporat- ing the new information into the H I J K L
diagram (which es- sentially says that if a critic t (ex. 3) ✓ ✗ ✓ ✓ ✗
tastes g, that critic also tastes t, along with the g ✗ ✗ ✓ ✓ ✗
contrapositive, that if a critic doesn’t taste t, then m ✓ ✓ ✓ ✓
he doesn’t taste g), and seeing what follows:
The only remaining box concerns
H I J K L whether J tastes m. J could either taste m or not
t (ex. 3) ✓ ✓ ✗ taste m, with- out violating any rules of the
g ✗ ✓ game. So there are exactly four critics—H, I, K,
m ✓ ✓ and L for whom it’s de- termined exactly which
scotches he or she tastes. This is choice d.

165
7
chapt e r

hybrid
games
review

So far, you have looked at sequencing, selection, distribution, and matching games. Each game type has its
own diagramming techniques as well as its own usual set of condition types. Mastering these games has
required learning these techniques and rules and getting a feel for the kind of game mechanics specific to
each game type. But learning each individually is not enough; the makers of the LSAT love to combine
game types to pro-
duce hybrid logic games. Never fear—you already have the tools you need to solve these games. You
just need to practice recognizing what’s familiar in these games, dissecting them into their component
game types, and applying the relevant techniques.
First, recall that each game-type has a certain preferred set of diagramming techniques:

GaMe tYPe dIaGRaMMInG teChnIQUeS

sequencing A sequence of slots


selection List of entities, entities listed on a grid (for cross-group domains), slots
for a group of known size
distribution Columns for each group, sets of slots for groups of known size
matching Grid for matching, column-approach
167
hYBrid games reView

You will want to use one or all of the (entities A, B, C, D, E, and F) to put on a set-list
diagram- ming techniques specific to the game (se- quence) for their next concert:
types that make up the hybrid game you are
considering. Most LSAT
hybrid games include a heavy sequencing component, (A B C D E F)
so you will most often want to start by trying a 1234
se- quencing diagram or technique.
In particular, a high proportion of While the diagram for these games is
sequencing- based hybrids will be fairly straightforward, some of the rules can be a
matching/sequencing hybrids in which you are bit tricky. In particular, beware of rules that
asked to both sequence a group of entities and combine selection and sequencing elements,
match them to characteristics or another group of such as:
entities. For these games, it’s often useful to
modify a matching grid to include a “matching” of If both are selected, X precedes
en- tities to sequencing spots. For example, Y. If both are selected, Y
suppose you are asked to sequence five entities precedes Z.
(A, B, C, D, and E) and match them to one of three
possible characteris- tics (X, Y, and Z). These two rules work a bit differently than
You could set this up by drawing the simi- lar rules in regular sequencing games. In a
following diagram: regular se- quencing game, you might join the
sequence strings X—–Y and Y—–Z into X—–Y—–
1 2 3 4 5 Z. If you do this, you might infer that X—–Z.
A/B/C/D/E Although this would be a valid inference for a
X regular sequencing game, it’s not neces- sarily the
Y case in a selection/sequencing game—if Y is not
selected, Z could come before X.
Z
There are many possible game-type combinations
If a high proportion of the information that produce a hybrid game, but, in general, you
concerns the direct matching of characteristics to should do two things when approaching hybrids.
entities, then you might consider a regular matching First, try to break down the game into its
grid instead of, or in addition to, a sequencing component game types and diagram accordingly.
diagram: Second, keep in mind that although many of the
rules will work the same way as they did for the
A B C D E regular game types, some rules will not, so make sure
X that you fully understand the mechanics of the rule
Y you are considering and be especially wary of
Z hybrid rule types that are thrown your way.

Another popular sequencing-based hybrid


is the selection/sequencing game. In this game,
you are asked to select a group of entities from a
larger domain and sequence them. In a
selection/sequencing hybrid diagram, list the
sequence of spots and write out the symbols for
the entities that will be selected for each spot. For
example, a band must select four of six songs
168
hYBrid games reView

practice game 1

A wine-tasting group tours six wineries—Domaine Chandon, V. Sattui, Piccetti, Mumm, Artesa, and
Black Stallion. At each winery, at least one of three wine types is tasted: white wine, red wine, and
sparkling wine. The tasting is subject to the following restrictions:

The group stops at V. Sattui either immediately before or immediately after Piccetti, and stops at
Domain Chandon before V. Sattui.
The group stops at Domaine Chandon either immediately before or immediately after
Artesa. The group tastes sparkling wine and red wine at Mumm, and stops at
Mumm first.
The group tastes white wine at exactly four wineries.
The group tastes more wine types at V. Sattui than at Piccetti, and more wine types at the last
winery than at the first winery at which they stop.

1. Which one of the following could be the list d. V. Sattui is fifth on the tour.
of wineries in order from the first winery at e. Piccetti is fifth on the tour.
which the group stops to the last?
a. Mumm, Domaine Chandon, Artesa, Black
Stallion, V. Sattui, Piccetti
b. Mumm, Artesa, Domain Chandon, Black
Stallion, Piccetti, V. Sattui
c. Domaine Chandon, Artesa, Mumm, Piccetti,
V. Sattui, Black Stallion
d. Mumm, Piccetti, V. Sattui, Black
Stallion,
Domaine Chandon, Artesa
e. Mumm, Domaine Chandon, Black Stallion,
Artesa, Piccetti, V. Sattui

2. If the group stops at two wineries at which


red wines are tasted before stopping at any
wineries at which white wines are tasted,
then which one of the following statements
must be true?
a. They taste red wine at Black Stallion.
b. They taste white wine at Domaine
Chandon.
c. They taste red wine at Artesa.
d. They taste white wine at Piccetti.
e. They taste red wine at V. Sattui.

3. If the group stops at Black Stallion before


Artesa, then which one of the following
statements must be true?
a. Artesa is third on the tour.
b. Domaine Chandon is third on the tour.
c. Domain Chandon is fourth on the tour.
4. If the group stops at two wineries in between stopping
at Piccetti and stopping at Black Stal- lion, then which
one of the following state- ments must be true?
a. They taste red wine and sparkling wine at V. Sattui.
b. They taste white wine and red wine at Piccetti.
c. They taste red wine and sparkling wine at Artesa.
d. They taste white wine and sparkling wine at Domaine
Chandon.
e. They taste white wine and red wine at Black
Stallion.

5. If the group tastes the exact same types of wine at the


last stop and second to last stop, then each of the
following statements could be true EXCEPT:
a. They taste the same number of wine types at the
first and third stops on the tour.
b. They taste the same number of wine types at the
second and third stops on the tour.
c. They taste the same number of wine types at the
second and fourth stops on the tour.
d. They taste the same number of wine types at the
third and fourth stops on the tour.
e. They taste the same number of wine types at the
fourth and fifth stops on the tour.

169
hYBrid games reView

practice game 2

The top five photos in a photo competition are ranked first through fifth. Each photo is either color or
black and white, and each was submitted by exactly one of five photographers: A, B, C, D, and E. The
highest ranked photo is the first place photo. The ranking meets the following conditions:

D’s photo places second.


The fifth place photo is color.
A’s photo is ranked higher than E’s.
E’s and B’s photos are black and white.

1. Which one of the following could be a list 4. Each of the following could be true EXCEPT:
of photographers in order from the one a. A’s photo is ranked immediately above
whose photo placed first to the one or immediately below B’s.
whose photo placed fifth? b. A’s photo is ranked immediately above
a. A, B, E, D, C or immediately below D’s.
b. B, D, E, A, , C c. D’s photo is ranked immediately above
c. A, D, B, E, C or immediately below B’s.
d. A, D, E, C, B d. B’s photo is ranked immediately above
e. A, D, C, B, E or immediately below C’s.
e. B’s photo is ranked immediately above
2. Which one of the following statements or immediately below E’s.
CAN- NOT be true?
a. The fourth place photo is black and 5. If there is exactly one photo ranked in
white. between B’s photo and C’s photo, then
b. The fourth place photo is color. which one of the following statements
c. The third place photo is black and must be true?
white. a. B’s photo ranks fourth.
d. The third place photo is color. b. A’s photo ranks third.
e. The second place photo is color. c. E’s photo ranks third.
d. B’s photo ranks first.
3. If the third place photo is color, then each e. A’s photo ranks first.
of the following must be true EXCEPT:
a. The first place photo is black and
white.
b. The first place photo is B’s photo.
c. The second place photo is color.
d. The second place photo is D’s photo.
e. The third place photo is color.
170
hYBrid games reView

practice game 3

Five songs are to be selected from eight (H, I, J, K, L, M, N, O) to be burned into tracks on a mix CD.
The tracks are ordered according to the following conditions:

If both are selected for the CD, song H precedes


song J. If both are selected for the CD, song K
precedes song L. If the fourth track is song I, then
the first track is song J. Either song H or song K
must be on the CD.
Song N is the third track.

1. Which one of the following could be the list 4. If song H is on the CD, then each of the
of tracks in order from first to last? follow- ing statements could be true
a. J, L, N, I, K EXCEPT:
b. M, H, N, I, J a. Song H immediately precedes song I.
c. J, O, N, H, I b. Song J immediately precedes song O.
d. M, K, N, L, I c. Song N immediately precedes song I.
e. M, L, N, O, J d. Song K immediately precedes song N.
e. Song L immediately precedes song I.
2. Which pair cannot precede song N on the
CD? 5. If song L precedes song J on the CD, then
a. M and K each of the following statements could be
b. J and L true EXCEPT:
c. O and H a. The fourth track is song K.
d. J and K b. The fourth track is song H.
e. I and L c. The fourth track is song J.
d. The second track is song K.
3. If the fourth track is song I, then each of e. The fifth track is song J.
the following statements could be true
EXCEPT:
a. The second track is song L.
b. The second track is song K.
c. The fifth track is song L.
d. The fifth track is song K.
e. The fifth track is song M.
171
hYBrid games reView

practice game 4

For a local magazine’s article on the best of the seven restaurants in a neighborhood, A, B, C, D, E, F and
G, the top four are chosen by popular vote and then only those four are ranked by a food critic. The
highest ranking is the first place ranking. The final ranking is consistent with the following
conditions:

C is ranked second.
If A or B make the top four, so
does E. If E makes the top four, F
does not.
If both make the top four, D is ranked above
G. If both make the top four, G is ranked
above C.

1. Which one of the following could be the list 4. If A is in the top four, which one of the
of the top four restaurants, in order from follow- ing statements cannot be true?
first to last (top to bottom)? a. E is ranked above G.
a. F, C, A, D b. E is ranked above A.
b. A, C, G, E c. B is ranked above C.
c. E, C, F, B d. A is ranked above D.
d. B, E, A, C e. G is ranked above A.
e. B, C, E, A
5. Which two restaurants cannot both make
2. If C is ranked above D in the top four, the top four?
then which one of the following statements a. D and G
must be true? b. E and A
a. E is not in the top four. c. B and E
b. A is not in the top four. d. G and E
c. G is not in the top four. e. D and B
d. B is in the top four.
e. B is not in the top four.

3. Which one of the following cannot be in


the top four?
a. A
b. B
c. D
d. E
e. F
172
hYBrid games reView

practice game 5

On Halloween, a group of six children approach the scary house at the end of the street. The group
consists of a ghost, witch, zombie, vampire, mummy, and devil. One at a time, each says trick-or-treat to
the kind lady who lives there. Not one for tricks, she gives each one or more of the following types of
treats—chocolate, hard candy, licorice—depending on how much she likes their costumes. The trick-or-
treating proceeds according to the following restrictions:

The devil receives more types of treats than the


ghost. The zombie goes before the vampire, but after
the witch. The ghost goes before the zombie.
The first child to approach the house gets all three treat
types. The devil goes fifth.
The kind lady does not hand out hard candy to two children in a row, nor does she hand out
chocolate to two children in a row.

1. Which one of the following could be a list 3. If the sixth child is the mummy and gets
of the children by costume, in order from chocolate, then each of the following statements
the first to approach the house to the last? could be true EXCEPT:
a. Mummy, ghost, witch, devil, a. The mummy gets hard candy.
zombie, vampire b. The mummy gets licorice.
b. Witch, zombie, mummy, ghost, c. The devil gets hard candy.
devil, vampire d. The vampire gets chocolate.
c. Ghost, zombie, witch, mummy, e. The zombie gets hard candy.
devil, vampire
d. Mummy, ghost, witch, zombie, devil, 4. If the ghost gets chocolate, then which one
vampire of the following statements must be
e. Witch, ghost, mummy, vampire, devil, true?
zombie a. The mummy goes first.
b. The witch goes second.
2. Which one of the following statements c. The ghost goes second.
CAN- NOT be true? d. The vampire goes fourth.
a. The first child gets more types of treats e. The zombie goes fourth.
than the second.
b. The second child gets more types of
5. If the vampire goes fourth, then which one
treats than the third.
of the following statements must be
c. The third child gets more types of treats
true?
than the fourth.
a. The zombie gets licorice.
d. The fifth child gets more types of treats
b. The vampire gets licorice.
than the sixth.
c. The devil gets licorice.
e. The fourth child gets more types of
d. The ghost gets licorice.
treats than the third.
e. The mummy gets licorice.

173
hYBrid games reView

practice game 6

An architectural tour of a college grounds will include exactly five of seven major landmarks: the
chapel, the library, the dining hall, the fountain, the bridge, the porters lodge, and the senior common
room. The tour is subject to the following conditions:

If the dining hall is on the tour, then the fountain and bridge are on the
tour. The second stop on the tour is either the porters lodge or the
bridge.
If both are selected for the tour, the library comes before the dining
hall. The chapel is fourth on the tour.
If all three are selected for the tour, both the senior common room and the fountain
come before the bridge.

1. Which one of the following could be a 3. If the bridge is toured before the senior
list of the landmarks toured, from first common room, then which of the following is
to last? a complete and accurate list of the
a. Library, bridge, dining hall, chapel, porters landmarks any one of which could be the
lodge third landmark on the tour?
b. Dining hall, porters lodge, fountain, a. Library, porters lodge
chapel, bridge b. Library, porters lodge, bridge
c. Bridge, porters lodge, chapel, senior c. Library, bridge, senior common room
com- mon room, library d. Library, bridge, porters lodge, senior
d. Dining hall, bridge, fountain, chapel, com- mon room
library e. Library, bridge, dining hall, senior
e. Porters lodge, senior common room, common room
foun- tain, chapel, bridge
4. If the library and dining hall are both on
2. If the dining hall is on the tour, then each of the tour, then which one of the following
the following statements could be true state- ments must be true?
EXCEPT: a. The fountain is toured before the chapel.
a. The senior common room is on the tour. b. The bridge is toured before the dining hall.
b. The fountain is on the tour. c. The bridge is toured before the fountain.
c. The bridge is on the tour. d. The library is toured before the bridge.
d. The library is on the tour. e. The senior common room is toured
e. The porters lodge is on the tour. before the dining hall.

5. If the library is not on the tour, then each of


the following must be on the tour
EXCEPT:
a. Porters lodge
b. Bridge
c. Chapel
d. Fountain
e. Senior common room
174
hYBrid games reView

practice game 7

Eight friends—Bock, Karla, Michael, Smita, Praveen, Gary, Lauren, and Cynthia—are planning a weekend trip
to the lake. They carpool using three cars, each departing at a different time, and each carrying at least
one and at most three people. The carpool meets the following restrictions:

Michael must ride with Karla, and Praveen must ride with
Smita. Gary departs first and Michael departs last.
Smita and Lauren do not ride together.
Bock either rides with Karla or with Praveen.
If Lauren departs first, then Cynthia departs last.
Bock does not depart immediately after Smita.

1. Each of the following statements could be 4. Which one of the following is a complete
true EXCEPT: and accurate list of the people any one of
a. Smita departs before Bock. which could be in the car that departs
b. Cynthia departs before Lauren. first?
c. Bock departs before Praveen. a. Gary, Smita, Cynthia
d. Gary departs before Karla. b. Gary, Praveen, Smita, Cynthia
e. Cynthia departs before Bock. c. Gary, Praveen, Smita, Lauren, Cynthia
d. Gary, Praveen, Smita, Cynthia, Bock
2. If Smita departs neither first nor last, then e. Gary, Praveen, Smita, Lauren, Cynthia, Bock
which one of the following statements must
be true? 5. If the carpool does not have to meet the
a. Michael rides with Lauren. restric- tion that that Smita and Lauren do
b. Smita rides with Bock. not ride together, then each of the following
c. Lauren rides with Gary. statements could be true EXCEPT:
d. Cynthia rides with Karla. a. Cynthia departs first.
e. Cynthia rides with Bock. b. Bock departs second.
c. Bock departs third.
3. If Lauren and Cynthia do not ride d. Lauren departs third.
together, then each of the following e. Smita departs third.
statements could be true EXCEPT:
a. Bock departs before Cynthia.
b. Smita departs before Karla.
c. Smita departs before Cynthia.
d. Praveen departs before Bock.
e. Praveen departs before Lauren.

175
hYBrid games reView

practice game 8

A circus consists of two tents, A and B, each with three rings, numbered 1 to 3. Five animals—a horse, an
el- ephant, a tiger, a lion, and a monkey—perform simultaneously. Each animal performs in exactly one
of the six rings, and each ring contains at most one animal. Consecutively-numbered rings are adjacent
to one another. The performances proceed according to the following conditions:

The monkey performs in a ring numbered


1. The lion and tiger both perform in
tent B.
The elephant performs in a lower-numbered ring than both the lion and the
horse. The elephant and the tiger perform in the same tent.

1. Each of the following statements could be e. Tiger, horse, lion


true EXCEPT:
a. The monkey performs in tent A.
b. The horse performs in tent B.
c. The elephant performs in a ring numbered
2.
d. The lion performs in a ring numbered 2.
e. The horse performs in a ring numbered
3.

2. If the tiger is in a ring numbered 3, then


each of the following statements must be
true EXCEPT:
a. The monkey performs in tent A, ring 1.
b. The elephant performs in tent B, ring 1.
c. The ring in which the lion performs is
ad- jacent to the ring in which the
elephant performs.
d. The ring in which the tiger performs is
adja- cent to the ring in which the lion
performs.
e. The ring in which the horse performs
is adjacent to the ring in which the
monkey performs.

3. Which one of the following is a complete


and accurate list of the animals any one of
which could perform in tent B, ring 2?
a. Elephant
b. Elephant, lion
c. Elephant, tiger, horse
d. Elephant, tiger, lion
4. Which one of the following statements could be 5. If the tiger performs in a lowered-number
true? ring than the elephant, then which one of
a. There is no performance in tent A, ring 3. the fol- lowing rings must be empty?
b. There is no performance in tent A, ring 1. a. Tent A, ring 1
c. There is no performance in tent B, ring 3. b. Tent A, ring 2
d. The elephant performs in tent A. c. Tent A, ring 3
e. The horse performs in tent B. d. Tent B, ring 2
e. Tent B, ring 3

176
hYBrid games reView

setup, answers, and


explanations 1 2 3 4 5 6
D/V/P/M/A/B M P
Practice Game 1 w (ex. 4) ✗ ✓
This game has elements of sequencing and r ✓ ✓
matching. The first, second, and third rules s ✓ ✓
provide fairly infor- mative sequencing
information, so make the primary diagram a
sequencing diagram, with a grid for match- ing D/A—–V/P
wine-types to the sequence spots (1–6) and the #V > #P
wineries that will be matched to those spots:
Also note that given the sequence string D/A
—– V/P, neither V nor P can be in spots 2 or 3 (they
1 2 3 4 5 6
would leave no room for D/A to their left), and
D/V/P/M/A/B
neither D nor A could be in spots 5 or 6 (they
w
would leave no room for V/P to their right):
r
s
1 2 3 4 5 6
The first and second restrictions give you the D/V/P/M/A/B DA
fol- lowing joint sequence string: M VP VP DA
P
w (ex. 4) ✗ ✓
D/A—–V/P r ✓ ✓
s ✓ ✓
The third restriction gives you concrete
infor- mation, which can be entered into the
diagram and you can enter the fifth restriction D/A—–V/P
into the diagram as a note: #V > #P

Can we make any further deductions? The


1 2 3 4 5 6
se- quencing information is fairly rich, so see what
D/V/P/M/A/B M
op- tions you have for adding the remaining
w(ex. 4)
winery B into the sequence. Since spot 1 has M,
r ✓ we just con- sider spots 2–6. If B comes before
s ✓ D/A—–V/P, then we know that P comes before V,
so that P is not in spot 6: B—–D/A—–PV. If B
The sixth restriction tells you that the comes in between D/A and V/P, then we again
number of checkmarks in column 6 will be greater we know that P comes before V, so that P is not in
than col- umn 1—so there must be three under 6 spot 6: D/A—–B—–PV. If B comes after D/A—–
and exactly two under 1. It also tells you that the V/P, then we have D/A—–V/P—–B.
number of check- marks under V will be greater So we have three options:
than the number under
P. This tells us that spot 6 cannot be P, but we still B—–D/A—–PV
need to keep this information noted next to the D/A—–B—–PV
diagram: D/A—–V/P—–B

177
hYBrid games reView

Question 1 Again, the three strings are:


The correct answer is choice b. This is a “Test-
the- Rules” question. The first restriction B—–D/A—–PV
rules out choice d, since Picceti and V. Sattui D/A—–B—–PV
precede Domain Chandon. The second restriction
D/A—–V/P—–B
rules out choice e, since Domaine Chandon and
Artesa are not imme- diately adjacent on the
These three possible sequence strings can
itinerary. The third restriction rules out choice c,
be placed into spots 2–6. If we use D/A—–B—–PV,
since Mumm is not the first winery. The fifth
Black Stallion will be in spot 4, and there is no
restriction rules out choice a, since it would mean
requirement that red wine is tasted in spot 4—
that three wine-types are tasted at Piccetti (this
so choice a is in- correct. If we use D/A—–B—–PV,
requires the deductions we already made—that since
Domaine Chandon could be in the spot 2, where
two wine-types are tasted in spot 1 as per the
no white wine is tasted— so choice b is incorrect.
third restriction, three are tasted in spot 6).
If we use D/A—–B—–PV, then Artesa could be in
spot 3, and there is no requirement that red wine
Question 2
is tasted in spot 4—so c is incorrect. If we use
The correct answer is choice d. The new
D/A—–V/P—–B, then V. Sattui could be in spot 4
information means that we need to fit two
and there is no requirement that red wine is tasted
checkmarks in the r-row before any checkmarks
in spot 4—so choice e is incorrect.
occur in the w-row. Since ex- actly four spots will
have checkmarks in the w-row (by the fourth
Question 3
restriction), we need to fit two r-row
The correct answer is choice e. If B—–A holds,
checkmarks under spots 1 and 2:
then we must be using the sequence string B—–
D/A—–PV, and our diagram will look like:
1 2 3 4 5 6
D/V/P/M/A/B DA
M VP VP DA 1 2 3 4 5 6
P
D/V/P/M/A/B M B D/A A/D P V
w (ex. 4) ✗ ✗ ✓ ✓ ✓ ✓
w (ex. 4) ✗ ✓
r ✓ ✓ ✓
r ✓ ✓
s ✓ ✓
s ✓ ✓

D/A—–V/P
#V > #P
#V > #P

Scan the answer choices to see which is given


Now scan the answer choices to see if you
in the diagram—choice e, that P is in spot 5, is
can find a statement that must be true. Since P
given by in diagram.
cannot ap- pear in spots 1, 2, or 3, it must appear
in 4, 5, or 6— and each of 4, 5, and 6 has a
Question 4
checkmark in the w-row. So choice d is the correct
The correct answer is choice a. Check each of the
answer, that the group must taste white wine at
three possible sequence strings to see which one
Piccetti.
allows there to be two letters in between P and B:
We could also use the three sequence strings
to systematically check each answer choice to see
if the statement in that answer choice could be
false.
178
hYBrid games reView

B—–D/A—–PV
D/A—–B—–PV Finally, we have noted that P must have one
D/A—–V/P—–B or two checkmarks, since it must have fewer
checkmarks than V.
Only B—–D/A—–PV allows there to be two Now scan the answer choices to see which
let- ters between P and B. So again, we have the one is inconsistent with this diagram. Choice e is
following diagram: inconsis- tent with the diagram, since the fifth
spot has three checkmarks, and the fourth can
have at most two checkmarks—so choice e is
1 2 3 4 5 6
the correct answer.
D/V/P/M/A/B M B D/A A/D P V
w (ex. 4) ✗ ✓ Practice Game 2
r ✓ ✓ This game has a sequencing element (each photo
s ✓ ✓ is ranked first through fifth), and matching
element (each photo is either black and white or
D/A—–V/P color). Three of the four restrictions give
#V > #P sequencing information, so make the primary
diagram a sequencing diagram, with a row for the
Only the information in choice a—r and s sequence spots (1–5) and the pho- tographers (A
are tasted at V—is given in this diagram, so a is through E) will be matched to those spots. Since
the cor- rect answer. there are only two options for the match- ing
(color or black and white), and each photo/spot can
Question 5 only be matched to one of the two options (each
The correct answer is choice e. The number of photo is only color or only black and white, not
check- marks under spot 5 is the same as the both), we can use an unexpanded grid:
number under spot 6—namely, all three of w, r,
and s are checked. If spot 5 has three checkmarks, P 1 2 3 4 5
can’t be in spot 5, because by the fifth restriction, it A/B/C/D/E
must have less checkmarks than V. So, two of our c/bw
three possible sequence strings are ruled out
(namely B—–D/A—–PV and D/A—–B—–PV, which The first and second restrictions can be
both have P in spot 5), and we have the sequence incor- porated into the diagram:
string D/A—–V/P—–B. But again, since P is not
spot 5, we can further determine this: D/A—–PV 1 2 3 4 5
—–B. A/B/C/D/E D
c/bw c
1 2 3 4 5 6 The third restriction can be symbolized: A—–
D/V/P/M/A/B M D/A A/D P V B E The fourth restriction can be
w (ex. 4) ✗ ✓ ✓ symbolized: E =
r ✓ ✓ ✓ bw, B = bw.
s ✓ ✓ ✓ Notice that since spot 5 must contain a
1 or 2 color photo, it cannot contain E or B—that leaves
#V > #P C and A. But now consider the sequencing
information: E must come after A, so A can’t be
last.
Therefore, C is fifth:

179
hYBrid games reView

1 2 3 4 5 must be color, but B is black and white (and so


A/B/C/D/E D C can’t be fifth). Similarly, for choice e, the fifth spot
c/bw c must be color, but E is black and white (and so
can’t be fifth). Finally, the third restriction rules
out choice b, since A is not ranked higher than E.
A—–E
E = bw, B = bw Question 2
The correct answer is choice b. Scan the choices
We need to place A—–E and B in spots 1, 3, for something that contradicts our diagram. Since
and spot 4 must be black and white, choice b, which
4. There are three ways to do this: says that the fourth place photo must be color,
is the correct answer.
B—–A—–E
A—–B—–E Question 3
A—–E—–B The correct answer is choice c. If the third
place photo is color, the third place photo cannot
Notice that whatever the case, spot 4 will be E or B. Therefore, it must be A and the
contain either E or B, and therefore be a black and sequence scenario in play here is B—–A—–E):
white pho- tograph. All in all, we have:
1 2 3 4 5
1 2 3 4 5 A/B/C/D/E B D A E C
A/B/C/D/E D C c/bw bw c bw c
c/bw bw c

B—–A—–E
B—–A—–E A—–B—–E
A—–B—–E A—–E—–B
A—–E—–B E = bw, B = bw
E = bw, B = bw
Compare the answer choices against the
All three possible scenarios for A, B, and E diagram to see which one is not given by the
were placed under the grid to show how you diagram. The sec- ond place photo, D, could be
might be flex- ible with your notations, but if it is either color or black and white, so c is the correct
confusing, just keep in mind that the real sequence answer. The other choices all give statements that
string is A—–E, and B must float somewhere on are in fact true in this diagram and so are
this string. incorrect.

Question 1 Question 4
The correct answer is choice c. This is a “Test- The correct answer is choice a. Check each answer
the- Rules” question. The first restriction rules out to see if it could be accommodated by the initial
choice a, since D is not second. The second diagram and the restrictions (in particular, since
restriction does not rule out anything by itself, the answer choices deal with sequencing, the
but in combination with the fourth restriction, it three sequencing scenarios).
does. Choice d violates a combination of these two
restrictions: the fifth spot
180
hYBrid games reView

1 2 3 4 5 choices for a statement guaranteed by this


A/B/C/D/E D C diagram. According to the diagram, A ranks first,
c/bw bw c so choice e is correct. The other choices all present
statements that are false according to this
diagram, and so are incorrect.
B—–A—–E
A—–B—–E Practice Game 3
A—–E—–B This game combines an element of selection (five
E = bw, B = bw of eight songs) and sequencing (ordering those five
songs into tracks on a CD). Given the mix of
In all three of the sequencing scenarios, A sequencing and selection rules, be prepared to
can- not be immediately adjacent to B. When we use both diagrams:
plug B—–
A—–E into the diagram, we see that B is isolated from (H I J K L M N O)
all other letters (including A) by D. When we plug 12345
ei- ther A—–B—–E or A—–E—–B into the
diagram, we see that A is isolated from all other Now symbolize the restrictions:
letters (including
B) by D. So, A cannot be immediately next to B, If selected, H—–J
and choice a is the correct answer. The other If selected, K—–L
choices rep- resent sequencing statements that are I=4S J=1
made possible by at least one of the three Contrapositive: J ≠ 1 S I ≠
sequencing scenarios. 4 H or K

Question 5 The firth restriction can be entered into the


The correct answer is choice e. See which of the diagram:
three sequencing scenarios is compatible with
there being exactly one letter in between B and
C. Only the sec- ond, A—–B—–E, allows this:
N (H I J K L M N O)
1 2 3 4 5 12345
A/B/C/D/E A D B E C
c/bw bw bw c
There are no obvious upfront deductions to
be made, so move on to the questions. Note,
however, that H and K appear in the first, fourth,
B—–A—–E and fifth restric- tions, so they might be key
A—–B—–E players in the game.
A—–E—–B
E = bw, B = bw Question 1
The correct answer is choice d. This is a “Test-
You could also produce this sequence just the- Rules” question. The first restriction rules out
choice c, since H does not precede J. The second
restriction
by noting that if C is fifth, then B has to be into fourth spot (to keep A ranked
third (since there is one space in between higher than E). Now scan the answer
them), which forces A into the first spot and E
rules out choice a, since K does not precede L. The third The fourth restriction rules out choice e, since
restriction rules out choice b, since I is fourth but J is not first. neither H nor K are selected.

181
hYBrid games reView

Question 2 S2
The correct answer is choice b. If both J and L LNJ
preceded N, then all the other songs go after N. 12345
Since H or K must be selected, either the first or
the second restriction will have to be in effect. S3
Either way, H or
K must be placed after J and N, and so at least one of NL J
these two restrictions would be violated. 12345

Question 3 Since K must precede L if it is on the CD,


The correct answer is choice a. If the fourth these three scenarios show that K cannot be
track is song I, then by the third restriction, the fourth (the latest it could appear would be the
first track must be song J. This means that H second track). So choice a is correct.
cannot be on the CD, because if it were, it
would have to precede J (the first restriction), Practice Game 4
but J is the first track. Since H is not on the CD, This game combines an element of selection (four
K must be on the CD (the fourth restriction). of seven restaurants) and sequencing (ranking those
So we have: four restaurants). Since there are a number of
sequencing rules and a number of selection rules,
J N I (H K L M you will likely have to use both diagrams:
O) 1 2 3 4 5
(A B C D E F G)
Now consider each answer choice. If L is 1 2 3
second, then K must be fifth—but this would 4
violate the sec-
ond restriction. So K cannot be second—choice The first restriction can be entered into the
a. diagram:

Question 4
The correct answer is choice c. If H is selected, then J C (A B C D E F G)
could not be first, since that would violate the first
re- striction. But if J is not first, then by the
182
contrapositive of the third restriction, I is not
fourth. Since I cannot be fourth, N—which is
third—cannot immediately precede I.

Question 5
The correct answer is choice a. We are supposing
that L and J are selected and that L—–J. We
know from Question 2 that L and J cannot both
precede N.
Therefore, we get three possibilities:

S1
L N J
1234 5
1234

Now symbolize the other restrictions:

A or B S E
Contrapositive: ≠E –›≠A and ≠B

E S ≠F
Contrapositive: F S ≠E
EF
If ranked, D—–G If
ranked, G—–C

Notice that these last two are not regular sequence


strings—they each require that the given elements are
selected first. For this reason, it doesn’t make sense to
hYBrid games reView

combine them into a joint sequence string (D—–G


—–C), unless you keep in mind that all three Question 1
elements are re- quired for this string to hold. This The correct answer is choice e. This is a “Test-
is because you would lose the information that the- Rules” question. The first restriction rules out
each one separately requires both elements to first choice d, since C is not ranked second. The second
be selected (e.g., if you write D—– G—–C, you restriction rules out choice a, since A is chosen for
might think that D—–C is a restriction, but this is the top four, but E is not. The third restriction rules
not required as long as G is not selected). out choice c, since both E and F are chosen for
Now try to digest the action of this game. the top four. The fifth re- striction rules out choice
The first thing to do is consider the conditional b, since G is not ranked above C (given that both
selection statements in relation to each other. are chosen for the top four).
The two state- ments (the second and third
restrictions) can be strung together via E: Question 2
The correct answer is choice c. This question
F S ≠E S ≠A and ≠B requires either remembering our up-front
deductions or look- ing for rules that involve C
So if F is chosen, A, E, and B are not. A lot and D. The last two restric- tions together imply
has been excluded from the top four here, so let’s that if all three of D, G, and C are selected for the
see where this goes. C is chosen regardless, so that top four, then their relative ordering must be D—–
only leaves D and G, and we have: G—–C. This ordering is contradicted by the
ordering given in the question stem, C—–D, and so
C (A B D E F all three of D, G, and C cannot be chosen—and
G) 1 2 3 4 since C and D are chosen, that means that G
cannot be chosen. At this point, you might note
A or B S E that what is really going on here is that D and G
≠E—›≠A and cannot both be chosen,
≠B If ranked, D since C must be chosen.
—–G If ranked,
G—–C Question 3
The correct answer is choice e. We already figured
But consider the last two restrictions. G out in our initial deductions that F cannot be
cannot be first, since D must go before G. chosen for the top four.
However, G cannot be third or fourth because G
must go before C. F can- not be first because then Question 4
D and G would be third or fourth, but this The correct answer is choice a. You could take
cannot be, since G must go before C. And D each answer choice in turn to see if the
cannot be first, since that would force G to go after accompanying statement is possible or not.
C. So we must conclude that F cannot be chosen Consider the first answer choice: if G is chosen,
no matter what! then given the fifth restriction, it must be ranked
Therefore, our initial diagram looks like this: first, since it is ranked above C, and C is ranked
second. But if G is first, then nothing can be
C (A B D E F ranked above it, including E, so a is the correct
G) 1 2 3 4 answer.

Move on to the questions. Question 5


The correct answer is choice a. We already saw
in Question 2 that D and G cannot both be selected

183
hYBrid games reView

together, since then the sequence must be D—–G


—–C, which would not be possible given that C is So cross out c and h for column 2, leaving l
second. for column 2. The diagram and notations now
look like:
Practice Game 5
This game has a sequencing element relating to 1 2 3 4 5 6
the order in which the children (G, W, Z, V, M, and G/W/Z/V/M/D D
D) ap- proach the kind lady and a matching c ✓ ✗
element—each child gets between one and three h ✓ ✗
treats (c, h, and l). Since most of the restrictions l ✓ ✓
give sequencing informa- tion, make the primary
diagram a sequencing diagram for the order of
W—–Z—–V
approach (1–6) and rows for matching treat types
to the children (or rather, costumes): ‹—G–|
No hh or
cc D>G
1 2 3 4 5 6
G/W/Z/V/ Question 1
M/D The correct answer is choice d. This is a “Test-
c the- Rules” question. The second restriction rules out
h choice c, since the witch does not come before
l the zombie. The second restriction also rules out
The first restriction can be choice e, since the zombie does not come before the
vampire. The third restriction rules out choice b,
symbolized: D>G since the ghost does not come before the zombie.
The fifth restriction rules out choice a, since the
The second and third restrictions can be devil is not fifth.
joined into the following sequence string:
Question 2
W—–Z—–V The correct answer is choice b. Compare each
‹—G–| an- swer choice to the initial diagram to see
The fourth and fifth restrictions can be entered directly which one is contradicted by the
into the diagram: diagram/notations. Choice b is correct because
according to the diagram, spot 2 has exactly one
checkmark, meaning that for it to have more
1 2 3 4 5 6
checkmarks than spot 3, spot 3 would have to not
G/W/Z/V/M/D D
have any checkmarks—contradicting the game
c ✓ condition that each child gets at least one candy-
h ✓ type. The rest of the answer choices provide
l ✓ statements that are consistent with the diagram.

The sixth restriction can be shortened Question 3


The correct answer is choice a. Start by
to: No hh or cc incorporat- ing the new information, that 6 = M
and gets c, into the diagram:

184
hYBrid games reView

1 2 3 4 5 6 1 2 3 4 5 6
G/W/Z/V/M/D D M G/W/Z/V/M/D W G Z V D M
c ✓ ✗ ✓ c ✓ ✗ ✗ ✓
h ✓ ✗ h ✓ ✗ ✗ ✓ ✗
l ✓ ✓ l ✓ ✓ ✓

W—–Z—–V W—–Z—–V
‹—G–| ‹—G–|
No hh or cc No hh or cc
D>G D>G

Since the last two spots are taken, we can try to fit the Now scan the answer choices for a statement
sequence string into spots 1–4. Z must be in spot 4 and V in that contradicts this diagram. Choice a is the
spot 5. G cannot be in spot 1, because G must have either 1 or 2 correct an- swer because according to the
checkmarks to have fewer checkmarks than D, and spot 1 has diagram, the mummy in fact does not get hard
three checkmarks. So G is in spot 2, leaving W for spot 1. We candy.
also note that the restriction against adjacent h’s or c’s means that
D cannot get c: Question 4
The correct answer is choice e. Start by trying to
place the ghost. We know from previous work that
1 2 3 4 5 6 the ghost cannot be in spot 1, since spot 1 has three
G/W/Z/V/M/D W G Z V D M checkmarks and the ghost can have at most two
c ✓ ✗ ✗ ✓ checkmarks (due to the restriction that D>G). If
h ✓ ✗ the ghost gets choco- late, then according to the
l ✓ ✓ initial diagram, the ghost cannot be in spot 2
either, since the child in spot 2 does not get
W—–Z—–V chocolate. To fit Z and V to the right of G, as
No hh or cc required by the joint sequence string,
D>G
‹—G–| W—–Z—–V
‹—G–|
Since we know that D has more checkmarks
than G, it must have two checkmarks in the G must be in spot 3. This pushes Z into
remaining two rows in that column, rows h and spot 4 and V into spot 6, leaving M and W to be
l. Since there is a checkmark in row h under distributed into spots 1 and 2:
column 5 now, columns 4 and 6 cannot have a
checkmark in that row (by the sixth restriction):
1 2 3 4 5 6
G/W/Z/V/M/D M/W W/M G Z D V
c ✓ ✗ ✓ ✗
h ✓ ✗
l ✓ ✓
185
hYBrid games reView

W—–Z—–V
‹—G–| selection rule (the first one), so go ahead and start
No hh or cc with a sequencing diagram and use a selection list
D>G when necessary.

We have added the information that the


ghost gets chocolate, and what follows from that 1 2 3 4 5
(by the sixth restriction), that Z cannot get
chocolate. Now, scan the answer choices for a Using abbreviations for the landmarks, the selec-
statement guaranteed by this dia- gram. Choice e— tion list is: C L D F B P S
that the zombie goes fourth—is given in the Now symbolize the restrictions:
diagram, and so choice e is the correct answer.
D S F and B
Question 5 Contrapositive: ≠F or ≠B S
The correct answer is choice d. If the vampire
goes fourth, then given the sequence string W, ≠D The second restriction:
X, and G must go into spots 1–3. From previous
work we know that G cannot go into spot 1. To 2 = P or 2 = B
keep Z to the right of G, G must therefore go into If selected, L—–D
spot 2. This pushes Z into spot 3 and W into spot If selected, S//F—–B
1. The remaining spot, spot 6, goes to M:
The fourth restriction can be written into
1 2 3 4 5 6 the diagram:
G/W/Z/V/M/D W G Z V D M
C c ✓ ✗
12345
h ✓ ✗
l ✓ ✓
At this point, there are a couple of routes to
take. The second restriction provides two distinct
W—–Z—–V scenarios, so we could see what follows from
‹—G–| each:
No hh or
cc S1
D>G PC
There do not seem to be any other 12345
immediate further deductions to be made, so
see if there is an answer choice that is S2
guaranteed by this diagram.
Choice d—that the ghost gets licorice—is given in the BC
diagram. 12345

Practice Game 6 For the second scenario, we note that the


This game combines an element of selection (five restric- tion S//F—–B means that we can’t have
of seven landmarks) and sequencing (ordering both F and S on the tour—there wouldn’t be
those five landmarks into a tour). There is only enough room to the left of
one pure B. We can note this next to the scenario:
186
hYBrid games reView

S1 selected D, F, B, and C. Consider how this


P C selection plays out in our two scenarios.
12345 In S1, since P is selected, we have all five
letters selected:
S2
B C S1
12345 P C (D, F, B)
(≠S or ≠F) 1234 5

Unfortunately, not much else seems to In S2, B is already selected, so we have room
follow from the restrictions. At this point, you for one more:
could either make two sub-scenarios for each of S1
and S2, depend- ing on whether D is selected, or S2
you could move to the questions. B C (D, F, one more)
1234 5
Question 1 (≠S or ≠F)
The correct answer is choice b. This is a “Test-
the- Rules” question. The first restriction rules out Notice, however, that we have noted that at
choice a, since the dining hall is on the tour, but least one of S or F can’t be on the tour—and
the fountain is not. The second restriction rules since F is on the tour, S cannot be:
out choice e, since neither the porters lodge nor
the bridge are second. The third restriction rules S2
out choice d, since the li-
brary is not before the chapel. The fourth restriction B C (D, F, one more)
rules out choice c, since the chapel is not fourth 123 4 5
in the tour. (≠S)

Question 2 Now compare each answer choice against


The correct answer is choice a. If D is selected, the two scenarios to see which statement is
then according to the first restriction, F and B are forbidden by both. S is not chosen on either
both se- lected. So we have selected D, F, B, and scenario, so the correct answer is a.
C.
At this point, you could either try to fill out Question 3
the scenarios based on this information, or try The correct answer is choice d. If the bridge is
out the answer choices. If we try out the answer toured before the senior common room (B—–S),
choices, we would start with the first answer then we can- not have S//F—–B. So, F cannot be
choice. If we try to select S for the tour, then we on the tour (be- cause by the fifth restriction, S//F
have filled our five slots with D, F, B, C, and S, —–B would have to hold). Now, scanning the
and therefore by the second re- striction, B would restrictions, we see that by the contrapositive of
have to be second on the tour. But this is not the first restriction, since F is not on the tour, D is
possible since according to the fifth restric- tion, S not on the tour.
and F need to precede B. So S cannot be selected
for the tour and a is the correct answer. CLDFBPS
If we had instead tried to use the
scenarios, we could have reasoned as follows.
Again, we have
187
hYBrid games reView

This leaves L and P to be selected—so C, L, selected, D cannot be selected either (by the
B, P, and S are on the tour. Only C is restricted to contra- positive of the first restriction), and that
the fourth spot, so the list of letters that could be would mean a total of three landmarks are not
in spot 4 is: L, B, P, S. To double-check this, see selected for the tour. If the other landmark not
if the other restric- tions actually restrict the selected is P, then we would have C, D, F, B, and
ordering. The first and third restrictions are S on the tour; and since P would not be selected,
irrelevant because D is not selected. The second B must be second; but then we not have room for S
restriction can be satisfied whether B or P is in and F before B, as would be required by the fifth
spot 4 (since the other can be in spot 2). The restriction, so P must be selected.
fourth restriction is taken into account, and the So we have:
fifth restriction is irrelevant because F is not
selected. So, the correct answer is choice d. CLDFBPS

Question 4 This means that either D or S must be the


The correct answer is choice b. We know that both other landmark (besides L) left off the tour. Since
L and D are selected for the tour, and that L—–D. landmark D is not an answer choice, landmark
Since D is selected, we know that F and B are S must be the other one left off the tour—this is
selected. That gives us our five landmarks: L, D, answer choice e.
F, B, and C. This means we are in scenario 2: You could also try to make further
deductions using the scenarios. If L is not selected,
S2 we have two scenarios:
B C (L —–D, F
1234 5 S1
P C (D F B S)
Compare each answer choice to see which 1234 5
state- ment must be true on this diagram. F could
go in spot S2
5, with L in spot 1 and D in spot 3—so choice a is in- B C (D P S/F)
correct. The furthest left D could go in spot 3, 12345
since there must be room for L to its left and spot (≠S or ≠F)
2 is taken. So, B must come before D—this is
choice b, the cor- rect answer. (We wrote S/F in our list of letters to be
selected from in S2 to remind us that both cannot
Question 5 be selected).
The correct answer is choice e. This question We recall from the initial digestion of the
requires either testing each answer choice in game that it seemed that the presence or absence
turn to see if it could be off the tour if the of D might play a major role in the mechanics of
library is also off the tour, or pushing our the game. So, let’s see what D’s presence or
deductions a bit further. absence means for each of S1 and S2.
Let’s first see if we can make some more In S1, if D is present, F and B must be (by
deduc- tions. We are looking for the landmark that the first restriction). So we would have:
could be off the tour. Since five of seven landmarks
are selected, two will be left out. The question S1.1
already tells us that L
is left out, so that accounts for one of the two. So, both P C (D, F, B)
F and B must be selected, because if either one is not 12345
188
hYBrid games reView

If D was absent, the remaining three letters The first restriction says that M and K
would have to be F, B, and S: form a block and P and S form a block:

S1.2 G M K
P C (F B S) 1 2 3
1234 5 PS

In S2, D and P and exactly one of S or F The third restriction says that L and S form
must be selected. By the first restriction, since D is an anti-block:
selected, F must be selected (rather than S).
So we have: LS

S2 The fourth restriction says that B is either


B C (D P F) with K or P:
1234 5
(≠S or ≠F) BK or BP

Now see which of the landmarks in the The fifth restriction


answer choices fails to appear on at least one of
the three sce- narios, S1.1, S1.2, and S2. Only S fails says: L = 1 S C = 3
to appear (in S1.1 and S2), and so choice e is the C≠3S L≠1
correct answer. P, B, C, and F all appear on all
three scenarios and so choices a through d are The sixth can just be
incorrect.
noted: B not
Practice Game 7
This game combines elements of distribution and immediately after S.
se- quencing. We are to distribute eight friends (B,
K, M, S, P, G, L, and C) into three cars, and then Now try to make some deductions. Since
sequence those groups from first to third. The PS form a block, we can see that the diagram
main action of the game is the distribution—the allows for two scenarios, one in which PS is in
first, second, and fourth restrictions are strictly 1, and one in which PS is in 2:
distribution rules. We can con- sider the whole
game a distribution game in which the groups to S1:
be distributed happen to be labeled first, sec- ond,
and last—or 1, 2, and 3. Essentially, it is a distribu-
G P S M K
tion game in which some of the rules, questions,
1 2 3
and answer choices are couched in sequencing
terms.
S2:
Since the second restriction says that M is
third and G is first, we start our diagram like
G P S M K
this:
1 2 3

G M
1 2 3
189
hYBrid games reView

The fourth restriction tells us that B is be in S2. Scan for an answer choice that is
either with K or P. In S1, this means B is in car guaranteed by S2—S and B ride in the same car,
3, and the re- maining letters, C and L, must be car 2, which his choice b.
in car 2 because of the fifth restriction:
Question 3
S1: The correct answer is choice d. If C and L are
G P S C L MK B not in the same car, we must again be in S2.
1 2 3 Scan for an an- swer choice that contradicts S2.
P and B leave at the same time on S2, so the
In S2, B is either in car 2 or car 3. But if we statement that P departs be- fore B cannot be
con- sider the sixth restriction—that B does not true, and therefore d is the correct answer.
depart im- mediately after S—we realize that B
cannot be in car 3: Question 4
The correct answer is choice c. We are looking for
S2: all the letters that appear in car 1, on either
G P S B M K scenario. In S1, G, P and S appear in car 1. In S2,
1 2 3 G, L and C ap- pear in car 1. Altogether, G, P, S,
L and C can appear in car 1. This is choice c.
Now consider the fifth restriction. If L is in Even if you didn’t see this, you could cross out d
1, then C must be in 3; so we cannot have both and e because you know that B can’t be in car 1,
L and C in 1, which means that L and C must be and you can cross out a since you’ve seen
distributed between 1 and 3: situations where P is in car 1.

S2: Question 5
G L/C P S B M K C/L The correct answer is choice e. Normally, we
1 2 3 would redo the deductions and diagramming, except
without the third restriction. But, you might have
With these scenarios in mind, we can turn noticed that we in fact never used the third
to the questions. restriction. The fact that Smita and Lauren do not
ride together was completely irrelevant to the
Question 1 scenarios generated in the initial dia- gramming.
The correct answer is choice c. Check each So, we just have the same two scenarios:
answer choice against the scenario diagrams. In
neither S1 nor S2 is B in a car that departs before S1:
P, so c is the correct answer. Even if we hadn’t G P S C L MK B
made the scenarios, we can still see that there’s 1 2 3
no room for P and S in car 3, so the latest that P
could be is car 2. But we can also see that B has to S2:
go with either MK of SP, so it can never fit in G L/C P S B M K C/L
car 1. Therefore, B cannot go before P. 1 2 3

Question 2 Each of the statements is true in at least


The correct answer is choice b. If S departs one of the scenarios except for the statement
neither first nor last, then S must depart in car 2, that Smita
and we must
190
hYBrid games reView

departs third—she must either depart first or Now let’s look at the fourth
second. Therefore, the correct answer is e.
restriction: E and T perform in
Practice Game 8
This game combines elements of distribution and the same tent.
se- quencing. The five animals—H, E, T, L, and M
—are to be distributed into two tents, A and B, Since, by way of the second restriction, we al-
and then se- quenced from ring 1 to ring 3 (with ready know that T performs in tent B, we now also
one empty ring). However, we can consider this a know that E performs in tent B. Since each tent has
matching game at core—each of the five animals three rings, and E, L, and T must all perform in
is to be matched to a letter (A or B) and a tent B, the tent is now closed to M and H, who
number (1 to 3), at most one for each letter- must per- form in tent A. Adding this information to
number combination. So we can set up a grid with our grid, we have:
numbers along the top and letters down the side,
and try to place the animals into the grid. That is,
1 2 3
we want to place the letters H, E, T, L, and M
A M, H
into this grid:
B E, L, T
ML E
1 2 3
A Notice that according to the grid, M must be in
B tent A and also in ring 1, so the position of M has
been determined and we now have:
The first restriction says that M performs in ring 1, and the
second that L and T are in tent B:
1 2 3
A M H
1 2 3 B E, L, T
A L E
B L, T
M No other deductions can be made from the
in- formation, so we can turn to the questions.
The third restriction states that E is in a
lower- numbered ring than L and H: Question 1
E—–L//H The correct answer is choice b. We know that each
We start by noting that if E—–L//H, E can’t be in ring 3, and of the three rings in tent B is occupied by one of
neither L nor H can be in ring 1: E, L, or T, so H must perform in tent A, making
choice b false and the correct answer. Again,
1 2 3 since E, L, and T oc- cupy all of tent B, M must
A perform in tent A, making choice a true, and
B incorrect. Looking at ring 2, we see that there are
L, T
no restrictions on E or L performing there, so
MLH E
choices c and d could be true, and so these
answer choices are incorrect. While H is limited
to tent A, it can perform in either ring 2, or ring
3, so choice e can be true, and is incorrect.
191
hYBrid games reView

Question 2 Question 3
The correct answer is choice e. The new information tells us The correct answer is choice d. Looking at our grid
that T performs in a ring numbered three, but we also know again:
that T performs in tent B, so combin- ing these pieces of
information, we know that T must perform in tent B, ring 3.
Let’s add this to the grid: 1 2 3
A M H
B E, L, T
1 2 3
L E
A M H
B T E, L We know that H must perform in tent A.
L E This allows us to eliminate any answer choice that
includes H, so choices c and e are incorrect and
Now, using the third restriction: can be elimi- nated. The three remaining choices
E—–L//H include only E, L, and T. Let’s start with the
We know that E occupies a lower numbered ring than L, choice that includes them all, choice d. Here,
and since they have to occupy the two remain- ing spots in building scenarios, will be an ef- fective way to
tent B, rings 1 and 2, E must occupy ring 1 and L must determine if any one of E, L, or T can be
occupy ring 2. Now we have: eliminated from tent B, ring 2.
The third restriction prevents L from
1 2 3 occupy- ing ring 1, but either T or E may fill that
A M H spot. Let’s try them both, beginning with E:
B E L T
S1

L E
1 2 3
No further deductions can be made, so we turn A M H
to the answer choices. Our grid tells us that M must B E L/T L/T L, T
perform in tent A, ring 1, and so choice a is incorrect. L
E
Similarly, we know that E must perform in tent B, In S1, after E occupies ring 1, L or T can
ring 1 so choice b is incorrect. The grid also occupy ring 2, as there are no restrictions against
makes clear that E and T perform in rings either. Let’s now look at what happens when T
adjacent to L so choices c and d are incorrect. We is in ring 1:
cannot, however, determine whether H performs
in tent A, ring 2 or in tent A, ring 3, so while S2
choice e may be true, it may also be false, and 1 2 3
so choice e is the correct answer.
A M H
B T E L
L E
192
hYBrid games reView

In S2, after T fills ring 1, E and L are left to We can combine this new information with
fill rings 2 and 3, but because of the third what we already have in the third restriction to
restriction, E cannot occupy ring 3 and must get:
then occupy ring 2, leaving ring 3 for L. Taken
together these scenarios show that E, L, or T can T—–E—–L//H
occupy tent B, ring 2, and so choice d is correct.
Choices a and b, while accurate, are only partial There are two key pieces of information to
lists and so are incorrect answer choices. be culled from this: first, it determines the order of
tent B. We know that E, L, and T occupy all the
Question 4 rings in tent B, and now we know that T is in a
The correct answer is choice a. Check each lower-numbered ring than E, and E is in a
statement against the grid: lower-numbered ring than L, so the order must
be, T—–E—–L. Let’s put that into the grid:
1 2 3
H A M 1 2 3
E, L, T B A M H
L E B T E L

Since we know that M occupies tent A, ring 1 L E


and the only other performer in tent A is H, H
The second piece of information relates to
must oc- cupy either ring 2 or ring 3, so choice a
the placement of H. We know that though H is in
could be true, and you could circle it and move
tent A, and E in tent B, but E must be in a lower-
on, but let’s examine the remaining choices.
numbered ring, and since we know that E is in
Choice b is false, because we know that M
ring 2, H must occupy ring 3, in tent A. So now
performs in tent A, ring 1. Since there are three
we have:
rings and three performers for tent B, each ring
must have a performer, so choice c is false and
incorrect. By the fourth restriction we know that 1 2 3
E performs in the same tent as T, and since the A M H
second restriction tells us T performs in tent B, E B T E L
must also perform in tent B, so choice d is false L E
and incorrect. We know that E, L, and T must
Now check the answer choices against the
each occupy one of the three rings in tent B,
grid. Tent A, ring 1 is occupied by M, so choice a
leaving no open rings in tent B for H, so choice
is incor- rect. Tent A, ring 3 is occupied by H, so
e is false and incorrect.
choice c is in- correct. Tent B, ring 2 is occupied
by E, so choice d is incorrect. Tent B, ring 3 is
Question 5
occupied by L, so choice e is incorrect. Only tent
The correct answer is choice b. The new
A, ring 2 remains open, and so choice b is the
information can be noted as:
correct answer.
T—–E
193
chapt e r

8 practice set 1
195
practice set 1

game 1

Seven students—Betty, Carl, Diane, Earl, Fran, Greg, and Hugo—are to be assigned to exactly one of two class
sections, a morning section and an afternoon section, according to the following restrictions:

Each section must have a minimum of three students and a maximum of four

students: If Greg is in the morning section, Carl is in the afternoon section.


Betty and Carl must be in the same section.
Diane is in the afternoon section.
Earl and Fran cannot be in the same section.

1. If exactly four students are in the afternoon 3. Which one of the following could be a
sec- tion, then those four students could complete and accurate list of the students in
be: the morning section?
a. Carl, Betty, Earl, Hugo a. Betty, Carl, Diane, Greg
b. Diane, Greg, Fran, Hugo b. Betty, Diane, Earl, Fran, Greg
c. Diane, Carl, Hugo, Earl c. Diane, Earl, Fran, Greg, Hugo
d. Diane, Greg, Earl, Fran d. Betty, Carl, Earl, Fran, Greg, Hugo
e. Diane, Betty, Fran, Hugo e. Betty, Carl, Diane, Earl, Fran, Greg

2. If Betty is in the morning section, which one 4. If Carl is in the afternoon section, which pair
of the following must be true? of students could be in the morning
a. Exactly three students are in the section?
morning section. a. Fran, Greg
b. Exactly four students are in the b. Diane, Fran
morning section. c. Betty, Earl
c. Hugo is in the morning section. d. Betty, Fran
d. Diane and Greg are in the same section. e. Fran, Earl
e. Earl and Greg are in the same section.
5. If Greg is not in the afternoon section,
which student must be in the morning
section?
a. Earl
b. Betty
c. Carl
d. Hugo
e. Fran
196
practice set 1

game 2

Eight paintings, identified by letter—I, J, K, L, M, N, O, and P—are to be hung in a house. The house has
4 rooms; one painting is to go into room 1, two paintings into room 2, two paintings into room 3, and
three paintings into room 4, according to the following conditions:

Painting J goes in room 3.


Painting M cannot go in the same room as painting
N. Painting K must go in the same room as
painting L.
If Painting O goes in room 2, then painting M goes in room 4.

1. Which one of the following is a complete b. I, P


and accurate distribution of paintings c. P, N
into the rooms of the house? d. I, P, O
a. Room 1: I e. P, K, L
Room 2: K, L
Room 3: O, J
Room 4: M, P,
N
b. Room 1: I
Room 2: O, P
Room 3: M, J
Room 4: N, K,
L
c. Room 1: M
Room 2: N, I
Room 3: J, P
Room 4: O, K,
L
d. Room 1: N
Room 2: M, J
Room 3: O, P
Room 4: K, L,
I
e. Room 1: P
Room 2: K, N
Room 3: M, J
Room 4: O, L,
I

2. If painting M is placed in room 2, then


which of the following is a complete and
accurate list of all the other paintings that
could be placed in room 2?
a. I
3. If painting O is placed in the same room as
painting N, then which of the following is a complete
and accurate list of all the rooms in which M could
be placed?
a. Room 4
b. Rooms 1, 3
c. Rooms 2, 3, 4
d. Rooms 1, 3, 4
e. Rooms 1, 2, 3, 4

4. If painting O is placed in room 2, which one of the


following statements could be true?
a. Painting P is placed in room 4.
b. Painting I is placed in room 3.
c. Painting I is placed in room 4.
d. Painting L is placed in room 2.
e. Painting K is placed in room 1.

5. The placement of all eight paintings can be determined


if which of the following statements is true?
a. Painting K is in room 2, M in room 1, and N in
room 4.
b. Paintings O and P are in room 2, and paint- ing I is
in room 1.
c. Paintings O, N, and I are in room 4.
d. Paintings I and P are in room 2, and painting O in
room 1.
e. Paintings K and L are in room 2, and panting I in
room 4.

197
practice set 1

game 3

An airport is building seven terminals—A, B, C, D, E, F, and G—over the course of seven years.
Due to the airport design, the terminals must be built one at a time, at the rate of one terminal per year
and must be built according to the following restrictions:

Terminal C is built in year 2.


If Terminal F is built in year 4, Terminal B must be built after Terminal
G. Terminal A must be built in year 3 or year 7.
Terminal E must be built in the year immediately before or after the year Terminal D is built.

1. Which one of the following is an 4. Which of the following is a complete and


acceptable schedule for building the accu- rate list of the terminals that could
terminals, starting in year 1 and ending in be built in year 1?
year 7? a. B, F, G
a. B, C, A, F, G, E, D b. A, D, G
b. B, C, F, D, E, G, A c. B, E, F, G
c. G, C, E, D, F, A, B d. B, G
d. F, D, E, C, G, B, A e. B, C, F
e. F, C, D, B, G, E, A
5. If Terminal F is built in year 4, then there
2. If Terminal G is built in year 7, then any are exactly how many acceptable orders in
of the following could be true EXCEPT: which the terminals could be built?
a. Terminal B is built in year 1. a. One
b. Terminal F is built in year 6. b. Two
c. Terminal E is built in year 5. c. Three
d. Terminal F is built in year 4. d. Five
e. Terminal D is built in year 5. e. Six

3. If Terminal F is built in year 4, which of


the following is a complete and accurate
list of the years in which Terminal E could
be built?
a. 5, 6
b. 5, 6, 7
c. 1, 3, 5, 7
d. 2, 5, 6, 7
e. 3, 5, 6, 7
198
practice set 1

game 4

Seven friends—Helen, Ian, Jack, Kate, Lorna, Matt, and Nick—sit in the same row in a movie theater. They
take seats 1 through 7, where seat 1 is the left-most seat, according to the following conditions:

Jack sits to the right of Helen but to the left of


Matt. Kate sits in between Ian and Lorna.
If Jack sits to the left of Nick, then Kate sits to the left of
Jack. If Nick sits to the left of Jack, then Lorna sits to the
left of Kate.

1. Which one of the following could be an 4. Which one of the following is the lowest
accu- rate ordering of the friends, from left num- bered seat in which Matt could sit?
to right? a. One
a. I, N, K, H, L, J, M b. Three
b. H, J, L, M, K, I, N c. Four
c. I, K, H, M, N, J, L d. Five
d. H, L, N, I, J, M, K e. Six
e. I, K, H, L, J, N, M
5. If Kate sits to the left of Lorna, which one of
2. If Jack sits in seat 6, which one of the the following could be true?
following is a complete and accurate list a. Ian sits in seat 3
of the friends who could sit in seat 5? b. Matt sits in seat 4
a. I, H c. Nick sits in seat 3
b. I, N, H d. Helen sits in seat 4
c. K, H, M, N e. Jack sits in seat 3
d. I, N, H, J, L
e. I, K, H, M, N

3. Which one of the following cannot be


true?
a. Kate sits in seat 2
b. Matt sits in seat 5
c. Jack sits in seat 2
d. Lorna sits in seat 7
e. Helen sits in seat 1
199
practice set 1

game 5

Of seven books—A, B, C, D, E, F, and G—at most four will be chosen for a reading list, according to the
follow- ing set of restrictions:

If A is chosen, C will not be


chosen. If B is chosen, E will be
chosen.
If A is not chosen, then either F or G must be chosen.
If both F and G are chosen, then E is not chosen and D is chosen.

1. Which one of the following could be a 4. If C is chosen but F is not chosen, which
complete and accurate list of the books one of following must be chosen?
chosen for the reading list? a. Book B
a. C, D b. Book G
b. F, G, A c. Book E
c. A, B, F, D d. Book D
d. C, A, B, E e. Book A
e. A, B, E, F
5. If B and G are chosen, which one of the
2. Which one of the following could be true? follow- ing must be true?
a. None of A, F, or G is chosen a. A is chosen
b. C is chosen, and neither F nor G is b. C is not chosen
chosen c. F is not chosen
c. F, G, and B are chosen d. A is not chosen
d. B and E are not chosen, and A is chosen e. D is chosen
e. D is not chosen, and F and G are chosen

3. If neither F nor G is chosen, which one


of the following cannot be true?
a. Both B and E are chosen
b. Both C and E are chosen
c. Both B and D are chosen
d. A, D, and E are chosen
e. A, B, D, and E are chosen
200
practice set 1

game 6

At least five and at most seven cars are to be selected for a car show from a set of nine cars, including two
red antique cars, three red modern cars, one white antique car, and three white modern cars. The cars are
to be se- lected according to the following conditions:

Exactly two antique cars are selected.


At least three and at most four red cars are selected.
If the white antique car is selected, at least two white modern cars are
selected. Either the white antique car or at least two red modern cars must
be selected.

1. Which one of the following could be a 4. If the white antique car is not selected,
complete and accurate list of the cars selected then which of the following must be
for the show? true?
a. One white antique car, one white a. Exactly three modern cars are selected.
modern car, one red antique car, two red b. Exactly four red cars are selected.
modern cars c. At most four modern cars are selected.
b. One white antique car, two white d. At most three red cars are selected.
modern cars, three red modern cars e. At least two white cars are selected.
c. One red modern car, two red antique
cars, and two white modern cars 5. If exactly five cars are selected, then which
d. One red antique car, one white antique one of the following is a partial, accurate
car, two white modern cars, three red list of the cars in the show?
modern cars a. White antique car
e. Two red antique cars, three red modern b. Three red modern cars
cars, and one white modern car c. Two red modern cars, one white modern
car
2. If exactly one red antique car is selected,
d. Two white modern cars
then which one of the following must be
e. Two white cars
true?
a. At least three white cars are selected.
b. At most four modern cars are selected.
c. Exactly three red cars are selected.
d. Exactly three modern cars are selected.
e. Exactly four white cars are selected.

3. Each of the following could be true


EXCEPT:
a. Exactly three white modern cars are
selected.
b. Exactly four white cars are selected.
c. Exactly one white car is selected.
d. Exactly one red modern car is selected.
e. Exactly five modern cars are selected.
201
practice set 1

game 7

The five houses in a neighborhood—Q, R, S, T, and V—have each been outfitted with at least one of three
secu- rity features: guard dog, fence, and alarm system. Any house can have more than one feature, and
the following conditions hold:

House Q has a guard dog and


fence. House S has a fence.
House Q and House R have exactly two features in common, and House R does not have an
alarm system.
House V has at least one more feature than
House S. House T has exactly one more feature
than House R.

1. The exact number of features can be 4. If houses R and S have exactly the same
deter- mined for how many of the five features, which of the following is a complete
houses? and accurate list of all houses that could have
a. One an alarm system?
b. Two a. T
c. Three b. T, V
d. Four c. Q, S
e. Five d. Q, T, V
e. Q, T, R, S
2. If exactly four houses have an alarm
system, then how many houses have 5. If House V does not have a guard dog, then
exactly three features? which one of the following statements must
a. One be true?
b. Two a. House S has more features than House R.
c. Three b. House Q has more features than House S.
d. Four c. House Q has more features than House R.
e. Five d. House V has the same number of features
as House T.
3. If every house with a fence but one also has e. House R has more features than House V.
an alarm system, then all of the following
state- ments must be true EXCEPT:
a. House Q has an alarm system.
b. House V has a fence.
c. House S has a guard dog.
d. House R has a guard dog.
e. House T has a fence.
202
practice set 1

game 8

Six ships—D, E, F, G, H, and I—arrive at port, one at a time. Three ships are from Portland, two from
Seattle, and one from Vancouver. The ships are either cargo or military, but not both. They arrive
according to the fol- lowing conditions:

Ships F and H are the last two to arrive, not necessarily in that
order. Ships G, D, and E arrive after ship I.
The second ship to arrive is a cargo ship from
Portland. At least two military ships are from
Portland.
The first ship to arrive is a cargo ship.

1. Which of the following could be an accurate 4. If ship E and ship F are separated by
list of the ships in the order in which they exactly two ships in the arrival sequence
arrive, from first to last? and are both from Seattle, then the
a. I, G, E, F, D, H following must be true EXCEPT:
b. G, I, D, E, H, F a. Ship I is from Vancouver.
c. I, D, G, H, F, E b. Ship H is from Portland.
d. I, E, D, G, F, H c. Ship H is a military ship.
e. E, D, G, I, H, F d. Ship G is a cargo ship.
e. Ship D is from Portland.
2. Which one of the following cannot be true?
a. Ship D is from Portland. 5. If all the ships from any given city arrive
b. Ship I is from Portland. consecutively, then which one of the following
c. Ship H is from Vancouver. cannot be true?
d. Ship F is from Seattle. a. A Seattle ship is cargo.
e. Ship G is from Seattle. b. A Seattle ship is military.
c. A Portland ship is military.
3. If four cargo ships arrive before all d. The Vancouver ship is military.
military ships, then which one of the e. The Vancouver ship is cargo.
following must be true?
a. Ship H is from Portland.
b. Ship F is from Seattle.
c. Ship I is from Vancouver.
d. Ship D is from Portland.
e. Ship D is from Vancouver.

203
practice set 1

answer explanations
The third restriction says that Betty and
Game 1 Carl must be in the same section.
In this distribution game, there are two We can symbolize this:
sections, morning and afternoon; according to the
first restric- tion, each must have three or four cb
students. Note that each of the seven students is
assigned to exactly one of two groups, and so this The fourth restriction says that Diane is in
could be treated like a selec- tion game. However, the afternoon section. We can add that directly
the fourth rule places a student in a specific group, into our diagram:
making this game different from most selection
games. Since there are only seven stu- M A
dents and each is assigned to exactly one section, one (_) d (_)
section must have three students and the other must
have four students. The fifth restriction says that Earl and Fran
We can now set up our basic diagram, can- not be in the same section. We can
labeling the morning section (M) and afternoon symbolize this:
section (A):
ef
M A
(_) (_) We can also symbolize this directly into our
dia- gram, where the slash means “or”:
We draw three slots in each section and the
fourth slot in parentheses to indicate that there may M A
be a fourth student in that section. (_) d (_)
We can symbolize our entities by lowercase e/f e/f
letters: b, c, d, e, f, g, and h.
We now symbolize the restrictions. The Note that Carl appears in both the second
second restriction says that if Greg is in the and third restriction. The second rule says that if
morning section, Carl is in the afternoon section. Greg is in the morning section, Carl is in the
We can symbolize this afternoon section. But since Carl and Betty must
be in the same section, according the third
as: g(M) —–›c(A) restriction, this means that if Greg is in the
morning section, both Betty and Carl must be in
We remember to find the contrapositive: If the afternoon section:
Carl is not in the afternoon section, Greg is not
in the morning section. Since there are only two g(M) —–›c(A) and b(A)
sections and each student must be assigned to one
of them, we can state this more directly: If Carl is This symbolization can replace the original
in morning section, Greg is in the afternoon symbolization of the second restriction. Note the
section. new contrapositive:
We can symbolize this
c(M) or b(M) —–›g(A)
as: c(M) —–›g(A)
204
practice set 1

Now we can start looking at our


(symbolized) restrictions, and apply them directly Scenario 2
to the diagram. For this game, we use scenarios. M A
To begin, we might try to find a strong clue with b c (_) d g (_)
which to divide the game into different possible e/f (h) e/f
scenarios. The third restriction says that Betty and
Carl must be in the same section. Our eye should We should note that instead of creating
gravitate to that symbolization because blocks force scenar- ios, you might also have chosen to proceed
entities together. So, there are two possible directly to the questions after symbolizing the
scenarios, one in which c and b are in A, and restrictions and filling out the initial diagram. We
another in which c and b are in M. provide both meth- ods for each answer explanation
that follows—you will have to see which method
Scenario 1 suits you better and saves you the most time.
M A
(_) d b c (_) Question 1
e/f e/f The correct answer is choice b. This question is a
stan- dard “Test-the-Rules” question. The second
Scenario 2 restriction rules out choice e. It implies that both
M A Greg and Carl must be in the morning section, but
b c (_) d (_) the second restric- tion says that if Greg is in the
e/f e/f morning section, Carl is in the afternoon section.
Choice e is also ruled out by the third restriction,
Now the second restriction comes into play. because Betty is present but Carl is missing. The
The contrapositive version of our revised second third restriction also rules out choice c because it
restric- tion dictates that if b and c are in M, includes Carl, but not Betty. The fourth
then g is in A. Likewise, b and c must be in A if restriction rules out choice a, since it does not
g is in M. Therefore, in scenario 1, where b and include Diane. The fifth restriction rules out
c are in A, g must go into the morning section. choice d, since it includes both Earl and Fran.
And finally, in scenario 1, h must be in M Another method for answering this question
because placing h in A would cause e and f to is to refer to the scenarios diagram. Simply look at
be together in M, which the fifth restric- tion the afternoon section part of the two scenarios,
does not allow. Therefore, M must have three and test each answer choice against it. Choice a
students and A must have four in scenario 1. In cannot work because in the first scenario, c and
sce- nario 2, g is the afternoon section. There are b are in A, while h is in M, and second scenario
no other restrictions in play. We do not know does not have c and b in A. Choice b is correct: in
where to place h, so we can draw h between the scenario 2, the afternoon section contains Diane
two sections. Thus, our scenarios are: and Greg, and can contain Fran and Hugo. You
can immediately stop and move on to the next
Scenario 1 question.
M A
g h d b c Question 2
e/f e/f The correct answer is choice d. This is a
standard “supposition” question. One method
for answering this question is to start making a
chain of inferences. If Betty is in the morning
section, then by the third
205
practice set 1

restriction, Carl is also in the morning section. If A word of caution: you might be tempted
Carl is in the morning section, then by the to think that since Diane is the only student
contrapositive of the second restriction, Greg is directly placed into the afternoon section, this
in the afternoon section. Filling out this answer choice is obvious. But it is entirely possible
information in the diagram, coupled with the that some other entity is also, indirectly, restricted
information in fourth and fifth re- strictions, from the morning section. For example, if there
gives us: was an additional restric- tion that Carl was in the
morning section, then by the second restriction,
M A Greg must be in the afternoon sec- tion and
b c (_) d g (_) would not appear on our list.
e/f (h) e/f We could also answer this question without
the aid of the scenarios, by using information
Note that the scenario we created based on from previous questions. The answer to Question
the information from the question stem is exactly 1 tells us that b, c and e have to be present in
the same as scenario 2 from our pre-question the correct answer. This eliminates answer
diagram- ming. Therefore, you could go straight choices a, b and c. We also know that d has to be
to scenario 2 and use the following reasoning to in the afternoon section, so it can’t show up on
consider each an- swer choice. this list. That eliminates answer choice e and
Choice a is incorrect because the we’re only left with choice d.
morning section could have four students (e.g., b,
c, f, h). Choice b is incorrect for the same reason Question 4
—the morning section could have three students The correct answer is choice a. This is a
(e.g., b, c, e). Choice c is incorrect because Hugo supposition question, so we could start by making
could either be in the morning section or the a chain of in- ferences. If c is in A, then b is also
afternoon section. Choice e is incorrect because in A, by the third restriction. By the fourth
Earl and Greg do not have to be in the same restriction, d is in A. So b, c, and d are in A. By
section—e.g., the afternoon section might contain fifth restriction Earl or Fran must be in the
d, g, and f, while the morning section contains b, afternoon section, and so we have filled our
c, e, and h. Choice d is correct because if Betty is maximum number of slots in that section (which
in the morning section, Diane and Greg must both is four slots, by the first restriction). So the
be in the afternoon section. remaining entities—g and h—must be in the
morning section. Entering this information into a
Question 3 diagram, we have:
The correct answer is choice d. First, we need to
un- derstand the question. We want a list of all M A
the stu- dents who, given the restrictions, g h d b c
might be in the morning section. If any rule e/f e/f
prevents a student from being in the morning
section, that student cannot be on the list. Note that this is just scenario 1 from our
Any entity that appears in the morning pre- question diagramming. We can now test our
section of either of our scenarios must appear on answer choices. Choice b is wrong because d
the list. We see that b, c, e, f, g, and h all appear cannot be in the morning section. Choices c and d
somewhere under M, but d never does. Therefore, are wrong because they separate b from c. Choice
the correct choice is d. e is wrong because it places e and f together. This
leaves choice a as the only possible correct
answer.
206
practice set 1

Question 5 We now symbolize our restrictions and, if


The correct answer is choice d. If Greg is not in the pos- sible, work them into the diagram. We can
af- ternoon section, he must be in the morning write J di- rectly into room 3.
section. The contrapositive version of our revised
second restriction dictates that if g is in the morning 1 2 3 4
section, b and c must
be in the afternoon section. We also know that h must J
be in the morning section because placing h in the af-
ternoon section would put e and f together. We symbolize the second restriction as an
MN block that is crossed out (MN), the third
M A restriction as a KL block, and the fourth
g h d b c restriction and its contrapositive as follows:
e/f e/f
O = 2 —–›M = 4
Note that this is just scenario 1 from our M ≠ 4 —–›O ≠ 2
pre- question diagramming. We can now quickly
and easily test our answer choices. Choices a and e Now we look to see if there are any
are incorrect because e and f could be in either obvious deductions, connections, or insights to be
the morning or af- ternoon sections. Choices b made. Since K and L go together, they cannot fit
and c are incorrect be- cause b and c must be in into room 1 (since there is one space), or room 3
the afternoon section. Hugo is the only other (since one of the two spaces is already filled with
student who must be in the morning section— J). If any other painting is placed in room 2, KL
this is choice d. must appear in room 4.
For this game, we will illustrate the
Also note that if you ran out of time and approach of creating scenarios only when they
couldn’t diagram, you could eliminate two might appear useful for particular questions.
answers because they are interchangeable. Here,
choices a and e must be incorrect. Earl and Fran Question 1
appear only in the fifth restriction. Neither one The correct answer is choice c. This question is a
could be the correct answer, since they would stan- dard “Test-the-Rules” question. The first
both have to be correct. Now you only have three restriction rules out choice d, since J is not in
possible answers, which improves your odds of room 3. The sec- ond restriction rules out choice
guessing correctly. a, since M and N are both in room 4. The third
restriction rules out choice e, since K and L are not
Game 2 in the same room. The fourth restriction rules out
In this grouping game, eight paintings are to be choice b, since O is in room 2, but M is not in
placed in four rooms. So, there are four groups room 4.
(the four rooms) into which eight entities (the
paintings) are to be distributed and the number of Question 2
entities in each group are specified. Since we know The correct answer is choice b. Start by seeing
the specific numbers in- volved, the game should what deductions can be made from the new
be diagrammed as follows: information that M is placed in Room 2. Since M
is in room 2, there is no room for the KL block
1 2 3 4 in room 2, so K and L must be in Room 4.
Further, according to the
207
practice set 1

contrapositive of the fourth restriction, since M is Question 3


not in room 4, O is not in room 2. According to The correct answer is choice d. Start by seeing
the sec- ond restriction, N cannot be in room 2, what deductions can be made from the
because M is in room 2. information that O and N go into the same room.
This leaves us with the following diagram: Since O and N are now a 2-entity block, like K and
L, the same reasoning ap- plies to them as to K
(I, P, O, N) and L: the ON block must go ei- ther in room 2
1 2 3 4 or room 4. Since we have two scenarios, draw
each out and see where they lead.
M J K L
(O, N) Scenario 1
1 2 3 4
The “(I, P, O, N)” note is written to remind us O N J
that these four paintings still need to be distributed
into the four remaining spots. From this diagram, Scenario 2
we can determine which paintings besides M can 1 2 3 4
be in
placed in room 2. We have I, P, O, and N remaining J ON
to be placed, but O and N are banned from room 2. So
I and P are the only paintings which can be placed Notice that the room for the KL block can
in room 2. This is answer choice b. Choice a is now be determined for each scenario. In scenario
incorrect because it does not include P as a 1, KL must go in room 4, and in scenario 2, KL
possibility. You can also tell that choice a is must go in room 2. Further, notice that the fourth
incorrect because it includes only one of two restriction will apply to scenario 1: since O is in
interchangeable elements. Paintings I and P are not room 2, M must be in room 4. And the second
involved in any restrictions, so anything that restriction will give us more informa- tion about
applies to one (including being a candidate for scenario 2: since N is in room 4, M cannot be.
room 2) must apply to the other. Choice c is So our diagram will now look like:
incorrect because it violates the second restriction: if
N appeared in room 2, M and N would both Scenario 1
appear in Room 2. Choice d is incorrect because (I, P)
it violates the fourth re- striction: O is in room 2, 1 2 3 4
but M is not in room 4 (it is
in room 2, as specified by the question stem). Choice O N J KL M
e is incorrect because it violates the fourth restriction.
It may not at first appear that the fourth Scenario 2
restriction is violated, since both K and L appear (I, P, M)
in the answer choice. But note that the answer 1 2 3 4
choice is supposed to
be a list of the possible candidates for room 2—and if K L J O N
K appears in room 2, L cannot (and vice-versa), since (M)
there are only two spots in room 2 and M is filling
one of them. So neither K nor L should appear on Now we want to determine where M can
the list we want. be placed. In scenario 1, M is placed in room 4.
In sce- nario 2, M is free to float into rooms 1 or
room 3. So M can be placed exactly in room 1, 3,
and 4, which is answer choice d.

208
practice set 1

Question 4 No restrictions are made on I and P, so they


The correct answer is choice b. Start by seeing are interchangeable, and the distribution cannot
what deductions follow from the new information be fully determined, so choice a is incorrect.
that O is placed in room 2. If O is in room 2, Let’s turn to choice b. If O is in 2, then M
then the fourth condition kicks in, and M must be must be in 4, by the fourth restriction. Since room
placed in room 4. Since O is in room 2, the KL 2 is filled, K and L also must be in room 4.
block has no place to go but room 4 as well. So we have:
So our diagram now looks like:

(N, I, P)
1 2 3 4
1 2 3 4
I OP J MKL
O J KL M N must then be in Room 3, and we have a
fully determined distribution. So the correct
N, I, and P can f loat into any spot, since answer is b. We can circle the answer and move
the only restriction on N (that it is not in same on to the next question.
room as M) is satisfied no matter what, and I For the sake of illustration, we note that
and P have no restrictions on them. Now, the remaining answer choices have the following
consider each answer choice. deductions/diagrams.
Choice a is incorrect because there is no For choice c, since O and N are in room 4,
room for P in room 4. Choice c is incorrect K and L are in room 2. P and M are left to float
because there is no room for I in room 4. Choice d and so the distribution is not determined.
is incorrect because painting L is in room 4. And
choice e is incorrect not only because we already (P, M)
know that painting K is in room 4, but also 1 2 3 4
because it could never be in room 1—
the KL block would be broken. Therefore, the correct K L J ON I
answer is choice b.
For choice d, since I and P are in room 2, K
Question 5 and L are in room 4. M and N are left to float
The correct answer is choice b. This is a difficult between rooms 3 and 4, so the distribution is not
ques- tion, which must be answered by trying each determined.
statement individually to see which deductions can
be made. For choice a, M is in room 1, N is in (M, N)
room 4, and since K is in room 2, so is L. 1 2 3 4
So we have: O I P J K L

(O, I, P) For choice e, we have K and L in room 2,


1 2 3 4 which means O cannot be in room 2. The
M K L J N contrapositive of the fourth restriction is that if
O is not in room 2, then M is not in room 4. We
also know that M and N are not in the same
room. No further deductions assigning a
painting to a room can be made.

209
practice set 1

(M, N, O, P) Question 1
MN The correct answer choice is b. This question is a
1 2 3 stan- dard “Test-the-Rules” question. The first
4 restriction
K L J I rules out choice d, since C is in year 4. The
(M) second restriction rules out choice a, since F is
in 4, but B is before G instead of after G. The third
Game 3 restriction rules out choice c, since A is in year 6.
The fourth restric-
This is a standard sequencing game, so you tion rules out choice e, since D and E are not
should draw and label seven slots for each built in consecutive years.
terminal:
Question 2
The correct answer is choice d. Start by inputting
1 2 3 4 5 6 7 the new information about G into the diagram.
And since G occupies 7, we know that A occupies
3. The contra- positive of the second restriction
Now symbolize the clues. Since we are told says that if B is before G, F will not be in year 4.
that Terminal C is built in year 2, C can be written Since G is in the last year, B must be before year
directly into the diagram. The other restrictions G, so F cannot be in year 4:
can be noted above the diagram, such as A = 3
or 7; Terminal E must be built in the year D/E
immediately before or after the year Terminal D; F = 4 —–›G–B
and if Terminal F is built in year 4, Terminal B B–G —–›F ≠ 4
must be built after Terminal G (along with the
contrapositive):
C A G
A = 3 or 7 1 2 3 4 5 6 7
D/E D/E F
F = 4 —–›G–B
B–G —–›F ≠ 4 Now that we’ve made all possible deductions,
we
look for answer choices that don’t work in our diagram.

C The only answer choice with an impossible option


is d.
1 2 3 4 5 6 7
D/E Question 3
We look for any deductions or connections to Now is
The correct answer we turn b.
choice to Start
the questions.
by inputting
be made, but the years and terminals mentioned
do not appear in multiple restrictions, so it is
difficult to come up with any immediate insights.
Notice, however, that since E and D must be side by
210
side, neither can be built in year 1 (since C occupies
year 2). You could mark this by crossing out E
and D underneath slot 1.
new information about Terminal F into the diagram.

A = 3 or 7
D/E
F = 4 —–›G–B
B–G —–›F ≠ 4

C F
1 2 3 4 5 6 7
D/E
practice set 1

Now let’s divide our diagram into scenarios, one with A built
third and the other with A built seventh: G C A F B
1 2 3 4 5 6 7
Scenario 1 D/E D/E
D/E
F = 4 —–›G–B G C A F B
B–G —–›F ≠ 4 1 2 3 4 5 6 7
D/E D/E
C A F
1 2 3 4 5 6 7 Now we turn to scenario 2. The D/E block can
D/E also only occupy years 5/6:

Scenario 2 C 3 F A
D/E 1 2 4 5 6 7
F = 4 —–›G–B D/E D/E
B–G —–›F ≠ 4
And again, G must come before B:
C F A
1 2 3 4 5 6 7 G C B F A
D/E 1 2 3 4 5 6 7
D/E D/E
Now we turn to the D/E block. Let’s see how
that block works in our two scenarios. In scenario Note that the placement of G and B in all
1, we see that the D/E block can occupy years three scenarios is superfluous for this question:
5/6 or 6/7: we are only interested in where E can go, but these
scenarios might come in handy for a different
Scenario 1 question, so it can be time well spent.
F = 4 —–›GB We are now in a position to answer the
BG —–›F ≠ 4 question. E can be built in years 5, 6, or 7, which is
answer choice b.
C A F This was admittedly an involved way of
1 2 3 4 5 6 7 finding the answer. Alternatively, you can cross
D/E D/E out choice d since E can’t go in year 2. We also
made an upfront deduction that E can’t go in
C A F year 1 because of the DE block, so answer choice c
1 2 3 4 5 6 7 is out. Years 5 and 6 are in all of the remaining
D/E D/E answer choices, so they don’t need to be tested. E
can’t be in year 3 since the DE block can’t be
And we know that if F is in 4, G must come placed, so choice e is out. The remaining hurdle
before B, so we can finish the diagram with two is then to decide whether E can go in year 7,
possibilities: and it’s straightforward to show a situation where
it can, so the answer has to be b.

211
practice set 1

Question 4
The correct answer choice is a. To answer this ques- G C B F A
tion, start with the original diagram: 1 2 3 4 5 6 7
D/E D/E
A = 3 or 7
D/E For each of the scenarios, the order of D and E can be switched
F = 4 —–›G– (yielding two sub-scenarios for each of our three scenarios). Therefore,
B there are six pos- sible orders, and the answer is e.
B–G —–›F ≠
4 C
1 2 3 4 5 6 7 Game 4
D/E This is a standard sequencing game, so draw and
label
seven slots:
A must be built in years 3 or 7, and C must be
built in year 2, so they are out. As part of a block,
nei-
ther D nor E can fit into year 1. We are left with 1 2 3 4 5 6 7
B, F,
and G. At this point, we can eliminate answer
choices
b, c, and e, since they contain terminals other than Now symbolize the first restriction: H—J—
B, F, and G. We have to choose between a and d. M
To do this, draw a diagram where F is Symbolize the second restriction: I—K—L
built in year 1: or L—K—I

F C A E D B G Symbolize the third restriction and its


1 2 3 4 5 6 7 contrapositive:

J—N —–›K—J
Since none of the rules are violated, F can J—K —–›N—J
be in year 1, and the correct answer is a.
Notice, however, that if Jack sits to the left
Question 5 of Nick, we know that the relative order must be K—J
The correct answer is choice e. To see this, we —N. This information and the conjoined
sim- ply need to go through the same process we contraposi-
did for Question 3. We start with the new tive can be symbolized:
information that F is built in year 4, and end up
with three scenarios: J—N —–›K—J—N
J—K —–›N—J—K
G C A F B
1 2 3 4 5 6 7 And finally, symbolize the fourth restriction
D/E D/E and
its contrapositive:

G C A F B N—J —–›L—K
1 2 3 4 5 6 7 K—L —–›J—N
D/E D/E

212
practice set 1

Now we move on to the questions. there would be no room for I in slot 5. And since
there are no further restrictions on H and N, N, H,
Question 1 and I are our only options for seat 5—answer
The correct answer is choice e. This question is a choice b.
stan- dard “Test-the-Rules” question. The first
restriction rules out choice c, since M is not to Question 3
the right of J. The second restriction rules out The correct answer is choice c. One approach is
also choice d, since K is not in between L and I. to take each answer choice in turn, and see
The third restriction rules out choice b, since J is whether a diagram could be constructed that
to the left of N, but K is not to the left of J. The doesn’t violate the conditions. Choice a was
fourth restriction rules out choice a, since N is to already shown possible in the answer to Question
the left of J, but L is not to the left of K. 1. One possible arrangement accommodating
choice b is: I, N, H, J, M, L, K. When you get to
Question 2 choice c, start the diagram by putting J in the
The correct answer is choice b. Start by filling second spot.
the new information into the diagram, and seeing By the first restriction, the first spot must
what follows: then be occupied by H:

HJ
J 1 2 3 4 5 6 7
1 2 3 4 5 6 7
Now check each restriction to see how to
Looking at the first restriction, we see that continue the diagram, and see if any restriction
M must be in seat 7: can’t be met. Notice that N must now be to the
right of J, so by the third restriction, K must be
to the left of
JM J. However, notice in the diagram that there is no
1234567 room for K to the left of J. Therefore, putting J
in the second spot is not possible. Circle your
Looking at the fourth restriction, and answer and move on.
realizing that N must now be left of J, we see Just for your information, there are
that L must be left of K. This means that, possible diagrams for choices d and e. One
according to the second restriction, we have: possible sequence for choice d is I, K, H, J, M, N, L.
One possible sequence for choice e is H, I, K, J, N,
L—K—I M, L.
(H, N)
Question 4
JM The correct answer is choice c. To see this, start
with
1 2 3 4 5 6 7 the leftmost spot, and check the restrictions to see
if M could sit there. He cannot sit in seats 1 and
Also, H now must be left of J and M, and N 2 be- cause that would not leave room for H
must be to the left of J. and/or J, as required by the first restriction (you
We now consider our list of possible letters should immedi- ately see this by your
for slot 5. L and K could not sit in slot 5, since if symbolic/visual representation of restriction 1).
they did,
213
practice set 1

What about seat 3? Try constructing a


diagram in which Matt sits in seat 3: (H//N—–J is placed into spots 1–3; remember,
this symbolization means that H and N are to the
M left of J, but the relative order of H and N is
1234567 unspecified.)
Restriction 1 is not violated, restriction 3
By restriction 1, H must be in seat 1, and J does not apply (since J is not to the left of N),
in seat 2: and restrictions 2 and 4 (which applies because N
is to the left of J) can be accommodated by filling
H J M in seats 5, 6, and 7 with the order L—K –I.
1234567 So Matt can sit in seat 4, but not in seats 1–
3. Seat 4 is therefore the lowest numbered seat in
Now, look to the other restrictions to see how which he could sit.
the diagram-filling must proceed. The third
restriction says that if J is to the left of N, the order Question 5
must be K—J— The correct answer is choice d. Start by seeing
N. Notice in the diagram that N must be to the right what follows from the new information that Kate
of J—so the order must in fact be K—J—N. But sits to the left of Lorna. According to the
notice again, in the diagram, that there is no room contrapositive of the fourth restriction, Jack must
for K to the left of J, so it’s not possible for Matt to sit to the left of Nick. We also know that according
sit in seat 3. What about seat 4? Again, H and J to the first restriction, Helen must be to the left of
must be somewhere to the left of M, in that order. Jack and Matt to the right of Jack.
There is now We can summarize this deduced
room to place K somewhere to the left of J as
well: information: H—–J—–M//N

M
Remember, this symbolization means that
1234567
Jack is to the left of Nick, and to the left of Matt
H—–J (though the order of Matt and Nick is unknown),
K and Helen is to the left of Jack.
(H—–J and K are placed into spots 1–3.) We also know, from the information that Kate
sits to the left of Lorna and the second restriction,
But now notice a new problem: K must be that I—– K—–L. According to the third restriction,
in between I and L, according to the third since Jack sits to the left of Nick, Kate must sit to
restriction— but there is no room for either I or the left of Jack: K—–J So we have three strings that
L to the left of K, so this won’t work. need to be joined, K—–J, I—–K—–L, and H—–J
But also notice that we don’t need K to the —–M//N. We can join H—–J—–M//N and K—–J
left of J anymore. The only reason we needed that at the common point, J. Since both K and H
before was that N had to be the right of J, since appear to the left of J, we will have to choose
there was no space to J’s left if M was in seat 3. one for the primary string and one for the
But now M is in seat 4, so we have space to secondary string. Since K also appears in I—–
place N to the left of J: —–
K L, we should choose K for the primary string, and
therefore H for the secondary string:
M
1234567 K—–J—–M//N
H//N—–J ‹—–H—|
214
practice set 1

Now we have to join I—–K—–L at the


common element K. We can keep I on the A —–›≠ C
primary string, but have to add L as another
secondary string: This means that if A is chosen, C is not
chosen.
|—L—–› I—– Its equivalent contrapositive is:
K—–J—–M//N
‹—–H—| C —–›≠ A

With this master joint sequence string The second restriction can be symbolized:
put together, we can easily tackle the answer choices.
There must be at least five people to the right of I B —–›E
(namely, K, L J, M and N), so I cannot sit in seat Its contrapositive is: ≠ E —–›≠ B
3 (there are only seven seats), and choice a is
incorrect. There must be at least four people to the The third restriction can be symbolized:
left of M (namely, I, K, J, and H), so M cannot be in
seat 4, and choice b is incorrect. Similar reasoning ≠ A —–›(F or G)
applies to N: there must be at least four people to Its contrapositive is: (≠ F and ≠ G)
the left of N (namely, J, K, I, and H), so N
cannot be in seat 3, and choice c is incorrect. H —–›A The fourth restriction can be
could sit in seat 4 if I, K, and L preceded it—for
example, we could imagine moving the letters on symbolized:
in the following way:
(F and G) —–›(≠ E and D)
|—L I—–K———– Its contrapositive is: (E or ≠ D) —–›(≠ F or ≠
J—–M//N G)
H—|
Now that you’ve collected all the
This gives us the sequence: I-K-L-H-J-M//N symbolizations and contrapositives into one place,
So it is possible for H to be in seat 4, and d look for connec- tions. The most obvious
is the correct answer. connection is that the third restriction and the
You should circle your choice and move on, contrapositive of the first restric- tion join up:
but if you checked e, you would see that J
cannot be in seat 3 because at least three people C —–›≠ A —–›(F or G)
(namely I, K, and
H) must sit to his left. Also notice that the second restriction
connects with the contrapositive of the fourth
Game 5 restriction:
An efficient approach to working through this se-
lection game will make heavy use of formal B —–›E —–›(≠F or ≠ G)
logic. Symbolize the restrictions as conditionals,
and sym- bolize their contrapositives as well. Restrictions three and four both deal with F
Then look for con- nections and further and G, but they cannot be straightforwardly
deductions that can be made. connected since one deals with F or G and one
The first restriction can be symbolized: deals with F and
G. Further connections between the conditionals
will become apparent as more suppositions are introduced in the questions.

215
practice set 1

Question 1 Question 3
The correct answer is choice e. This question is a The correct answer is choice b. The first thing to
stan- dard “Test-the-Rules” question. The first do is to see if the new information provided, that
restriction rules out choice d, since both A and C neither F nor G is chosen, leads to a new
are chosen. The second restriction rules out deduction. According to the contrapositive of the
choice c, since B is cho- sen, but E is not chosen. third restriction, if it’s the case that both F is not
The third restriction rules out choice a, since A is chosen and G is not chosen (i.e., that neither F nor
not chosen, but neither is F nor G chosen. The G is chosen), then A is chosen. Are there any
fourth restriction rules out choice b, since both F further deductions to be made? According to the
and G are chosen, but D is not chosen. first restriction, if A is chosen, then C is not
chosen. Now look for any further deductions—
Question 2 none are readily apparent. Now, scan the answer
The correct answer is choice d. The right choices to see if any of them contradict this newly
approach here, again, is to check each answer deduced in- formation, that C is not chosen, and
choice against the restrictions. This is a more that A is chosen. Choice b says that C is chosen
sophisticated version of the “Test-the-Rules” (as well as E, though that is irrelevant)—and this
question and tests your ability to draw connections contradicts our deduced information, so the
between the rules. statement in choice b cannot be true, and choice
Choice a is incorrect because it violates the b is the correct answer to the ques- tion. If
third restriction: if A is not chosen, F or G (or you’ve approached the question in this way, you
both) must be chosen. Choice b is incorrect should be confident in your answer and move on
because it violates the contrapositive of the third at this point. Of course, you could see if each
restriction combined with the first restriction. That answer choice represents a possible reading list if
is, the first restriction says that if C is chosen, A neither F nor G are chosen by coming up with a
cannot be chosen. But the contrapositive of the list of chosen books and testing them against each
third restriction says that if neither F nor G are restriction to be sure the restrictions are not
chosen, then A is chosen. If, as in this answer violated.
choice, it’s both the case that C is chosen, and that Again, this would be very time-consuming
neither F nor G are chosen, we are led to a and is not recommended.
contradiction concerning A. So this answer
choice cannot be true. Question 4
Choice c is incorrect because according to The correct answer is choice b. Start by seeing if
the fourth restriction, if F and G are chosen, then the new information can set off a chain of
E is not chosen (and D is chosen, but that is deductions. If C is chosen, then according to the
irrelevant here); according to the second first restriction, A is not chosen. And if A is not
restriction, if E is not chosen, then B is not chosen, then according to the third restriction,
chosen. But in choice c, B is chosen even either F or G must be chosen. But we also know
though F and G are also chosen. Choice e is that F is not chosen. So therefore G must be
incorrect because it violates the contrapositive of chosen. Now scan the answer choices to see if any
the fourth restriction. Since D is not chosen, of this information (that A is not chosen and G is
according to that restriction, either F is not chosen) appears—it does, in choice b. Circle your
chosen or G is not chosen. But this answer answer, and move on.
choice has both F and G being chosen.
Choice d is correct—one possible scenario is Question 5
one in which A is chosen and nothing else is—this The correct answer is choice c. Start by seeing if the
would not violate any restrictions. new information can set off a set of deductions.
According

216
practice set 1

to the second restriction, if B is chosen, then E must The third restriction can be
be chosen. According to the contrapositive of the
fourth re- striction, if E is chosen (or if D is not
symbolized: wA —–›+2wM
chosen), then either F will not be chosen or G will
not be chosen, but we also know that G is chosen,
The fourth restriction can be
so we know that F is not chosen. Now, scan the
answer choices to see if any of this infor- mation
symbolized: wA or +2rM
(that E is chosen and F is not chosen) appears— the
fact that F is not chosen appears in choice c.
We also want to add the information that at least
five and at most seven cars must be selected:
Game 6
This selection game is on the difficult side,
A M
since the entities to be selected from are
r • • • • • 3–4
characterized by combinations of characteristics
w • • • •
(whether they are red or white, and whether they
ex. 2
are antique or mod- ern), rather than just being a
list of named entities (A through F). This selection
Select 5–7
game uses a cross-group domain. So, create a two-
wA —–›+2wM
by-two grid and place dots to represent the cars:
wA or +2rM
A M
r •• • • • Now we consider the conditions and start
w • • • •
mak- ing deductions.
The first condition says that exactly two
antiques are selected. Since there is only one white
Now symbolize and incorporate the antique, that must mean that at the very least, one
restrictions. The first restriction says that exactly red antique must be selected (and in addition,
two antique cars are selected: either another red antique or the white antique).
So, we can circle one red antique in our grid to
remember that at least one red antique must be
A M
selected.
r •• • • •
The second condition says that there are
w • • • •
either three or four red cars selected. Looking at
ex. 2
our grid again, we see that there are a total of five
The second restriction says that at least three and at most four red cars, two of them antique and three of them
red cars are selected: modern. Since at least three red cars must be
selected, and at most two of them can be antique
(since there are only two an- tique red cars), that
A M
means that at least one red mod- ern car must be
r • • • • • 3–4
selected . So circle a red modern car as well.
w • • • •
ex. 2

217
practice set 1

The grid now looks like: A M

A M
r •  • •
w  •
r •  • •
ex. 2
w • • • •
ex. 2 select 5–7
Select 5–7 wA —–›+2wM
wA —–›+2wM wA or +2rM
wA or +2rM
We also need at least three red cars, so we
Armed with this diagram, we can turn to the need to select at least one more rM:
questions.
A M
Question 1 r • •
The correct answer is choice d. This question is a w  •
stan- dard “Test-the-Rules” question. The first ex. 2
restriction rules out choice b, since only one
antique car is selected. The second restriction rules select 5–7
out choice e, since five red cars are selected. The wA —–›+2wM
third condition rules out choice a, since the white wA or +2rM
antique car is selected, but only one white modern
car is selected. The fourth condition rules out choice Now, all the restrictions are satisfied.
c, since neither the white antique car is se- lected, We have six cars selected, and the leeway to
nor are two red modern cars selected. select one more car, an rM or a wM. Now look
at the answer choices to see which statement
Question 2 must be true given this diagram.
The correct answer is choice a. Use the diagram to Choice b is incorrect because five modern
see what follows from the new information, that cars could be selected (e.g., three red and 2 white).
exactly one red antique car is selected (which Choice c is incorrect because according to the
means we cross out the second rA car): diagram, a fourth red car could be selected. Choice
d is incorrect because four modern cars must be
A M chosen, and in fact, five could be selected.
r •  • • Choice e is incorrect because the car show
w • • • • selection could include just three white cars.
ex. 2 Therefore, choice a, that at least three white cars
are selected, is correct.
select 5–7
wA —–›+2wM Question 3
wA or +2rM The correct answer is choice d. For this “except”
ques- tion, the incorrect answers will yield an
Exactly two cars must be selected in column acceptable sce- nario or fail to produce a
A, so wA must be selected. But if wA is selected, contradiction. We start with our original diagram,
then at least two wMs are selected: and apply the scenarios of each answer choice.

218
practice set 1

Choice a states that exactly 3 wM’s are


selected: Now we apply the other restrictions to try to
find a contradiction. One more rM must be
A M selected to get three red cars:
r •  • •
w •  A M
ex. 2 r • •
w  •
select 5–7 ex. 2
wA —–›+2wM
wA or +2rM select 5–7
wA —–›+2wM
Now we apply the other restrictions to try to wA or +2rM
find a contradiction. We could select the other rA
(since we need exactly two As) and another rM (so All restrictions are satisfied, so b is
that the “wA or +2rM” restriction is satisfied): definitively incorrect. What about choice c, that
exactly one white car is selected? Keep in mind
A M that if you select a white car and reach a
r  • contradiction, you would need to check if
w •  selecting a different white car would also reach a
ex. 2 contradiction.
In our first scenario, we select the wA and
select 5–7 cross out all other white cars. However, the third
wA —–›+2wM restriction states that if the white antique car is
wA or +2rM selected, at least two white modern cars are
selected. We have a contradiction here. Now we
With these restrictions satisfied, a could be employ a second scenario, where we select a wM
true, and is definitively incorrect. What about and cross out all other white cars. Since wA is
choice b? All four white cars are selected. Since not selected, 2 rAs must be selected to fulfill the
we have two As selected, the second rA cannot be first restriction that there must be exactly two
selected, so we’ll cross that out: antique cars. And since wA is not selected, another
rM must be selected, to satisfy the “wA or
+2rM” restriction:

A M A M
r •  • • r  •
w   w •  • •
ex. 2 ex. 2
select 5–7
select 5–7 wA —–›+2wM
wA wA or +2rM
—–›+2wM
wA or +2rM All other restrictions are satisfied in the
second scenario, so c is incorrect. What about
choice d, where exactly one red modern car is
selected?
219
practice set 1

A M
We start by crossing out the other two rMs
from our original diagram. The first restriction
r  •
states that exactly two antique cars are selected,
w •  
and the second restriction states that at least ex. 2
three red cars must be selected. Therefore, we
must also select the other rA. select 5–7
wA —–›+2wM A M
r   • • wA or +2rM
w • • • •
ex. 2 This scenario also fulfills all restrictions. In
the last scenario, you take the original diagram,
select 5–7 and then select only one other rM, all three wMs,
and the wA:
wA —–›+2wM
wA or +2rM A M
r • •
However, this presents a contradiction with w  
the fourth restriction, which states that either the ex. 2
white antique car or at least two red modern
cars must be selected. So we have found our select 5–
contradiction, and 7
choice d is the correct answer. wA —–›+2wM
You’ve found your correct answer and moved on. For wA or +2rM
your reference, here is how you would rule out choice e, in
which five modern cars are selected— by looking at all Since this scenario also fulfills all
scenarios with five modern cars. In the first scenario, you take restrictions, choice e is incorrect.
the original diagram, then select the other two rMs, two
wMs, and (in order to fulfill the second restriction where at Question 4
most four red cars are selected) the wA: The correct answer is choice b. Place the new
informa- tion that wA is not selected (and
therefore the other rA must be selected) into the
A M original diagram:
r •  A M
w  • r   • •
w • • • •
ex. 2
ex. 2
select 5–7
wA —–›+2wM select 5–7
wA or +2rM wA —–›+2wM
wA or +2rM
This scenario fulfills all restrictions. In
the other scenario, you take the original Since wA is not selected, at least one more
diagram, and then select only one other rM, all rM must be selected; and since there can be no
three wMs, and the other rA: more than four red cars in the show, the third
rM should
220
practice set 1

be crossed out. Finally, between one and three select 5–7


wMs could be selected to fulfill the requirement wA —–›+2wM
that five to seven cars appear in the show, and all wA or +2rM
the restrictions will be satisfied.
In our diagram, we’ll include just one wM: But then we would already have the
maximum five cars selected, and we do not have
A M
at least three red cars as required by the second
r  • restriction. Therefore, the other rA must be
w •  • •
selected:
ex. 2 A M
r   • •
select 5–7 w • • • •
wA ex. 2
—–›+2wM
wA or +2rM select 5–7
Now scan the answer choices for a statement that must be wA —–›+2wM
true given this diagram. The diagram shows that exactly four red wA or +2rM
cars must be selected, so the correct answer is choice b. Circle this
choice and move on to the next question. Since wA is not selected, another rM must
For your reference, choice a—exactly be selected:
three modern cars are selected—is incorrect
because you could have five modern cars in the A M
show. Choice c—at most four modern cars are r  •
selected—is incorrect for the same reason as w • • • •
choice a. Choice d—at most three red cars are ex. 2
selected—is incorrect because in fact four red cars
must be selected. Choice e—at least two white cars select 5–7
are selected—is incorrect because, as our diagram wA —–›+2wM
shows, you could have just one white car in the wA or +2rM
show.
We have our maximum of four red cars, so the
Question 5 fifth car must be a wM:
The correct answer is choice c. Since only five cars
can be selected, wA cannot be selected. This is A M
because if wA were selected, then at least two wM r  •
would need to be selected: w •  • •

A M ex. 2
r •  • •
w  • select 5–7
ex. 2 wA —–›+2wM
wA or +2rM

221
practice set 1

Now we can just check each answer choice to more feature than R, and R has two features, T
see which represents an accurate part of this must have all three features.
selection list. Choice a is incorrect because no wA
is selected. Choice b is incorrect because exactly Q R S T V
two rMs are selected. Choice d is incorrect
g ✓ ✓ ✓
because there is only one wM selected. Choice e
f ✓ ✓ ✓ ✓
is incorrect because there is only one white car
a ✗ ✓
selected (the wM). Choice c is correct, as there are
two rMs and one wM selected.
The fourth restriction states that V must
Game 7 have at least one more feature than S. We can
The first thing to do is to diagram the action of write in “+1” with an arrow to specifically indicate
this matching game. Three features (g, f, and a) that V will have one more feature than S. We also
are to be matched with five houses (Q, R, S, T, therefore know that S must have at most two
and V). Each house must have at least one features, and V must have at least two. We can
feature, but can have up to three features. A grid also write a “1” under the S column, and “2 or 3”
would be helpful here: under the V column to remind us of the number
of features that can be added to V and S.
Q R S T V
g Q R S T V
f g ✓ ✓ ✓
a f ✓ ✓ ✓ ✓
a ✗ ✓
(1)——–-–+1——–– (2 or 3)
Now consider and symbolize the
restrictions with checks and ✗s. The information Our diagram and deductions are now
from the first two restrictions can be written complete, and we move on to the questions.
directly into the dia- gram: Q has features g and f,
and S has feature f. Also, some of the information Question 1
from the third restriction can be added, namely The correct answer is choice b. To answer this
that R does not have feature a. ques- tion, we need to refer to our finished
diagram. Only two houses have no blanks: R
Q R S T V and T. So the correct answer is two. Because you
g ✓ took the time upfront to make your deductions
f ✓ ✓ and diagram the results, you can be confident in
a ✗ your answer and move on to the next question.

Question 2
The rest of the restrictions cannot be written The correct answer is choice c. To answer this
di- rectly into the diagram without making some ques- tion, first see how the new information—
further deductions (by looking for connections that exactly four houses have alarm systems—fits
between the restrictions). The third restriction into the dia- gram. Since there are only five
says that Q and R share exactly two features, so R houses, and R cannot have an alarm system, all
must have both g and the other houses must have
f. Now consider the fifth restriction. Since T has
one
222
practice set 1

an alarm system. Now see what other deductions Now see what further information can
can be made. be deduced. Just as before, S is now maxed out
S has now maxed out its number of at two features, and V must have all three
features, and the fourth restriction kicks so that V features:
has all three features (one more than S’s two
features). Q R S T V
The entire diagram is now filled out.
g ✓ ✓ ✗ ✓ ✓
f ✓ ✓ ✓ ✓ ✓
Q R S T V
a ✓ ✗ ✓ ✓ ✓
g ✓ ✓ ✗ ✓ ✓
(1)——–-–+1——–– (2 or 3)
f ✓ ✓ ✓ ✓ ✓
a ✓ ✗ ✓ ✓ ✓ Now compare each answer choice to our
(1)——–-–+1——–– (2 or 3) filled- out grid. We are looking for a false
statement, so the incorrect answers will be
We just count now. There are three houses statements that must be true. Choice a is incorrect
with exactly three features, which is answer because Q has an alarm system. Choice b is
choice c. incorrect because V has a fence. Choice d is
incorrect because R has a guard dog.
Question 3 Choice e is incorrect because T has a fence. Choice c
The correct answer is choice c. First, see what is correct because S does not have a guard dog.
deduc- tions can be made from the new
information that every house with a fence but Question 4
one also has an alarm sys- tem (note that this does The correct answer is choice d. Start by seeing
NOT mean that every house with an alarm system what deductions can be made from the new
also has a fence—the direction of the conditional information, that Houses R and S have exactly the
is one way: if a house has a fence, then that same features. We already know R’s features, so
house also has an alarm system). mirror those in S. Since S now has its max of two
First, what is the “but one” business? See if features, we know that V has all three features.
there is any house with a fence that you already
know doesn’t have an alarm system—there is, Q R S T V
house R. So house R is the “one” in “but one,” the
g ✓ ✓ ✓ ✓ ✓
exception to this new rule. Now, simply find any
f ✓ ✓ ✓ ✓ ✓
house that has a fence, and add in the information
a ✗ ✗ ✓ ✓
that it also has an alarm system (if not there
(1)——–-–+1——–– (2 or 3)
already)—these will be houses Q and S. Now look for
any house that does not have an alarm, to ensure Now see which houses could have an
that it doesn’t also have a fence (the alarm system. Q may or may not have an
contrapositive of the new information)—there is alarm system (you could try both scenarios to
one, R, but that is already covered by the see if any restriction is violated, but you can trust
exception in the new rule. the diagram you have created). T and V definitely
have alarm systems, and houses R and S
Q R S T V definitely don’t. So, the houses that could have
g ✓ ✓ ✓ an alarm system are houses Q, T, and V, which
f ✓ ✓ ✓ ✓ is choice d. Choices a and b are incorrect
a ✓ ✗ ✓ ✓ because they do not include Q and V. Choice c
(1)——–-–+1——–– (2 or 3)
223
practice set 1

is incorrect because it includes S, but not T and 1 2 3 4 5 6


V. You also know it’s incorrect because it DEFGHI
includes S but not R, and S and R now PPPSSV
functionally equivalent, or interchangeable, since
cm
according to the question set-up they have exactly
the same features. Choice e is incorrect because it Now symbolize the restrictions and start
includes R and S, but not V. filling information into the diagram where
possible.
Question 5 The first restriction can be symbolized by ‘F/H’
The correct answer is choice b. Start by seeing in boxes 5 and 6. That is, the last two spots, 5 and 6,
what new deductions can be made from the are taken by F or H. The second restriction says that
information that V does not have g. We know that of the four remaining spots (1 through 4), ship I must
what happens in V is connected to what happens be in spot 1, and the rest of the spots are taken up
in S—and since there are two features in V, there by G, D, and E, in some unknown order. The third
must be only one feature in S (which must be f, restriction gives us information for spot 2—that it is
as per the second condition). from Portland and a cargo ship (so we put P and c
under 2). The fifth restriction says that spot 1 is a
Q R S T V cargo ship (so we put a c under 1).
g ✓ ✓ ✗ ✓ ✗
f ✓ ✓ ✓ ✓ ✓ 1 2 3 4 5 6
a ✗ ✗ ✓ ✓ I G/D/E G/D/E G/D/E F/H F/H
(1)——–-–+1——–– (2 or 3) P PPSSV
c c cm
Now check each answer choice against
our filled-out grid. Choice a is incorrect
because R has two features, and S only has one
The fourth restriction says that at least two mili-
feature. Choice c is incorrect because S has two
tary ships are from Portland. We know from the third
features, and Q might have two features as well.
restriction that at least one Portland ship is a cargo
Choice d is incorrect because T has three features
ship (the ship in the second spot). Therefore, we can
while V only has two features. Choice e is
definitively deduce that the remaining two Portland
incorrect because V and R both have two features.
ships are military. This means all other cargo ships
Choice b is the correct answer, since Q has two or
(aside from the one in column 2) cannot be from
more features, while S has only one feature.
Portland—so the cargo ship in column 1 must be from
Vancouver or Seattle:
Game 8
This hybrid game combines elements of matching
1 2 3 4 5 6
(cargo or military to ships), sequencing (the
I G/D/E G/D/E G/D/E F/H F/H
sequence of ship arrival), and grouping (of ships into
V/S P PPSSV
three groups, based on place of origin). To keep
c c cm
track of the volume of information and all the
moving parts, a diagram will be necessary. We’ll use
matching diagram, where ships (D, E, F, G, H, and
2 remaining P’s are m.
I), places of origin (P, S, or V), and ship-type (c or
We can now turn to the questions.
m) are matched to sequence spots (note that the
grouping element and sequencing element have been
subsumed under a matching scheme).
224
practice set 1

Question 1 For your reference, choice b—that F is


The correct answer is choice d. This question is from Seattle—is incorrect because F must be from
a “Test-the-Rules” question. The first restriction Portland. Choice c—that I is from Vancouver—
rules out choice a, since F is not one of the last is incorrect because ship I could be from
two ships, and choice c, since H is not one of Seattle. And choices d and e—that D is from
the last two ships. The second restriction rules Portland or Vancouver, respectively—are
out choice b and choice e, since I is not first. incorrect because according to the information in
our diagram, D could be from Portland (if it is
Question 2 second), Vancouver, or Seattle (if it is third or
The correct answer is choice b. We start by fourth).
consult- ing the diagram we created, and This question is also a good example of
scanning the an- swer choices to see if any, based how important it is to pay close attention to the
on the diagram, must be false. Choice a—that D is wording of each question. You were asked for
from Portland—could be true, so it is incorrect. what must be true, not what could be true or
Choice b—that I is from Portland—cannot be must be false. If you dove into the answer choices
true. Our diagram shows that Ship I must be without taking note of what you were being
from either Vancouver or Seattle. None of the asked for, you could turn an easy,
other answer choices—that H is from straightforward question into a time-consuming
Vancouver, F is from Seattle, and D is from problem.
Portland— can be proved false based on the
information provided. Question 4
The correct answer is choice d. Start by seeing
Question 3 what new deductions can be made from the
The correct answer is choice a. First see what additional in- formation that ship E and ship F
deductions can be made from the new information have two ships in between them, and that both E
that four cargo ships arrive before all military and F are from Seattle. We know from the set-up
ships. We know from the fourth restriction that and diagram that E must be in spot 2, 3, or 4. If
there are at least two military ships—so the first E is in spot 4, to maintain a separation of two
four ships to arrive must be cargo ships, followed ships, F would have to be in spot 1 (since there is
by two military ships. Furthermore, we know that at no spot 7). But we know from the first
least two ships from Portland are mili- tary—so we restriction that F must be in spot 5 or 6, so E
can definitively deduce that these must be the two cannot be in spot 4.
military ships from Portland. This information can What about spot 3? That would seem to
be put into the diagram: work, as F would be in spot 6. What about spot 2?
The third restriction states that the ship in spot 2 is
1 2 3 4 5 6 a cargo ship from Portland. But the additional
I G/D/E G/D/E G/D/E F/H F/H information specifies that E is from Seattle, so E
V/S P V/S V/S P P cannot be in spot 2.
c c c c m m Therefore, E must be in spot 3, forcing F
into spot 6. Fixing F in spot 6 will also fix H into
spot 5, and fixing E in spot 3 leaves G and D
Now scan the answer choices to see which either in 2 or 4. Now consider the fact that E
one of them must be true. Choice a states that H and F are both from Seattle. Since only two ships
is from Portland, which is confirmed by the are from Seattle, this means that no other ships
diagram. Circle your answer choice and move on can be from Seattle, so I must be from Vancouver.
to the next question. And since there is only one ship from Vancouver,
this must meant that the

225
practice set 1

rest of the ships are from Portland. Draw all this in, along Then, consider the ships from Seattle. There must
with the facts that the other Portland ships are military: be two open spots next to one another for them.
Spot 1 is adjacent to a ship from Portland, so
there is no room there—therefore, the two Seattle
1 2 3 4 5 6 ships must be in spots 5 and 6—this leaves the
I G/D E G/D H F Vancouver ship in spot 1. Draw all this out,
V P S P P S cm including the further infor- mation (from the
c c m m fourth restriction) that the remain- ing Portland
ships are military, and that the ships in spots 5 an
Now, look at the answer choices. Any 6 could be military or cargo.
answer choice that must be true is incorrect.
Choice a is incor- rect, since the diagram shows 1 2 3 4 5 6
that I is from Vancouver. Choice b is incorrect, I G/D/E G/D/E G/D/E F/H F/H
since the diagram shows that H is from Portland. V P P P S S
Choice c is incorrect, since the diagram shows that c c m m c/m c/m
H is a military ship. Choice e is incorrect because
regardless of sequence, the diagram shows that D is
from Portland. Choice d is correct because if G in Now consider the answer choices, looking
spot 4, it is a military ship, not a cargo ship. for the answer choice that must be false. Choice
a is in- correct because according to the diagram,
Question 5 the Seattle ships could both be either military or
The correct answer is choice d. Since we know cargo. Choice b is incorrect for the same reason.
that the second ship is from Portland, and the Choice c is incor- rect because there are in fact
first ship is either from Vancouver or Seattle, we two Portland ships that are military. Choice e is
know that the second, third, and fourth ships are incorrect because, again, the Vancouver ship is
all from Portland. cargo. Choice d is correct because the Vancouver
ship is a cargo ship.
226
chapt e r

9 practice set 2
227
practice set 2

game 1

Six animals are selected for a new zoo exhibit. They are selected from four lions (A, B, C, D), four giraffes
(H, I, J, K), and three tigers (X, Y, Z), according to the following conditions:

At least two lions are selected.


If H or J or both are selected, then both X and Y will be
selected. If Y is selected, then A is not selected.
If J is selected, then K is not selected.
If at least two tigers are selected, then exactly three lions will be selected.

1. Which one of the following could be a 4. If exactly one giraffe is selected, then which
complete and accurate list of the animals one of the following CANNOT be a true?
selected for the exhibit? a. Exactly one tiger is selected.
a. A, B, C, J, X, Z b. Exactly two tigers are selected.
b. B, H, I, J, K, Y c. Exactly two lions are selected.
c. A, B, C, I, K, Y d. Exactly three lions are selected.
d. C, D, I, K, X, Y e. Exactly four lions are selected.
e. B, C, D, I, X, Y
5. Which one the following statements CANNOT
2. If J is selected, then each of the following be true?
must be selected EXCEPT: a. Exactly two tigers are selected.
a. B b. Exactly three tigers are selected.
b. C c. Exactly two giraffes are selected.
c. D d. Exactly three giraffes are selected.
d. I e. Exactly three lions are selected.
e. Y

3. If the only tiger selected is Y, then which one


of the following could be a partial, accurate
list of the animals selected?
a. H
b. A, C
c. B, D, I
d. I, J, K
e. B, D, H, K

228
practice set 2

game 2

A textbook is to be translated into at least one of seven languages—Arabic, Chinese, French, German,
Hindi, Russian, and Spanish—according to the following conditions:

If it is translated into French, it is translated into German.


If it translated into Arabic or Chinese or both, it will not be translated into
Russian. If it is not translated into Arabic, it is translated into Hindi.
If it is translated into Spanish, it will not be translated into German.

1. Which one of the following could be a 4. If the book is not translated into Hindi,
complete and accurate list of the languages which one of the following must be true?
into which the book is translated? a. It is translated into Spanish
a. Chinese, German b. It is translated into German
b. Arabic, French, Spanish c. It is not translated into French.
c. Chinese, French, German, Hindi d. It is not translated into German.
d. Arabic, French, German, Spanish e. It is not translated into Russian.
e. Arabic, Chinese, Russian, Spanish
5. If the book is neither translated into Hindi
2. If the book is translated into Russian, then nor German, then which one of the
what is the maximum number of following could be an accurate, partial list of
languages into which the book could be the languages into which it is translated?
translated? a. Chinese, French
a. One b. Spanish, Chinese –This choice does not
b. Two cor- relate with the description of choice
c. Three B in the Answer Key
d. Four c. Russian, Arabic
e. Five d. Spanish, Chinese, Russian
e. Spanish, Arabic, French
3. Which two languages CANNOT be a pair
into both of which the book is translated?
a. Spanish and French
b. Russian and French
c. German and Hindi
d. Spanish and Hindi
e. Arabic and Chinese

229
practice set 2

game 3

Three cheese platters are to be served at a party, each containing three types of cheeses. The cheeses are
to be chosen from nine cheeses—American, Brie, Chevre, Dunlop, Evora, Feta, Gouda, Havarti, and
Jarlsberg—and each of these nine cheeses will be served on exactly one platter, according to the
following conditions:

The American and Chevre are not served on the same


platter. The Dunlop and Havarti are served on the same
platter.
If the Evora and Gouda are served on the same platter, then the Jarlsberg and Havarti will not be
served on the same platter.
The Chevre and Gouda are served on the same platter.

1. Which one of the following is an d. Brie is served on the same


acceptable division of cheeses into platter as American.
platters? e. American is served on the same
a. Platter 1: Dunlop, Havarti, platter as Evora.
Jarlsberg Platter 2: American,
Brie, Evora Platter 3: Chevre,
Feta, Gouda
b. Platter 1: Dunlop, Havarti,
Jarlsberg Platter 2: American,
Brie, Feta Platter 3: Chevre,
Evora, Gouda
c. Platter 1: Evora, Feta, Jarlsberg
Platter 2: American, Chevre Gouda
Platter 3: Brie, Dunlop, Havarti
d. Platter 1: Brie, Dunlop,
Jarlsberg Platter 2: American,
Evora, Feta Platter 3: Chevre,
Gouda, Havarti
e. Platter 1: Brie, Evora, Feta
Platter 2: Chevre, Dunlop, Havarti
Platter 3: American, Gouda, Jarlsberg

2. If the Jarlsberg is served on the same platter


as the Dunlop, then which one of the
following CANNOT be true?
a. Feta is served on the same platter as
Evora.
b. Feta is served on the same platter as
Brie.
c. Brie is served on the same platter as
Gouda.
3. If the American and the Evora are served a. American and Brie
on different platters, then which one of the b. Gouda and Jarlsberg
follow- ing CANNOT be the list of cheeses c. Dunlop and Jarlsberg
on a single platter? d. Dunlop and Evora
a. American, Brie, Jarlsberg e. American and Feta
b. Evora, Feta, Jarlsberg
c. Chevre, Gouda, Jarlsberg 5. If the Evora and Feta are served on the
d. American, Dunlop, Havarti same platter, then how many different
e. Dunlop, Harvarti, Jarlsberg ways can the platters be served?
a. One
4. If the Feta and the Brie are served on the b. Two
same platter, then which one of the c. Three
following CAN- NOT be a pair of cheeses d. Four
served on the same platter? e. Five

230
practice set 2

game 4

Each of eight patients—A, B, C, D, E, F, G, and H—is assigned to exactly one of three wings in a hospital
ward. The west wing is assigned three patients, the north wing is assigned two patients, and the south wing is
assigned three patients, according the following restrictions:

Patient D is assigned to the north wing.


Patients E and G are assigned to the same
wing.
If patient H is assigned to the west wing, then patient B is assigned to the south
wing. Patients B and G are not assigned to the same wing.

1. Which one of the following could be a together.


complete and accurate assignment of e. Patients G and H are not
patients to wings? assigned together.
a. West: A, G,
E North: C,
D South: B,
F, H
b. West: A, C,
F North: D,
G South: B,
E, H
c. West: A, C,
H North: B,
D South: E,
F, G
d. West: D, F,
H North: C,
G South: A,
B, E
e. West: B, E, G
North: D, H
South: A, C,
F

2. Which one of the following statements must


be true?
a. Patients B and E are not assigned together.
b. Patients D and E are not assigned
together.
c. Patients D and H are not assigned
together.
d. Patients B and D are not assigned
3. If patient B is assigned to the west wing, then which c. Patients D and E
one of the following could be true? d. Patients H and B
a. Patient E is assigned to the west wing. e. Patients H and E
b. Patient H is assigned to the west wing.
c. Patient H is assigned to the south wing. 5. If patient G is in the south wing, then
d. Patient G is assigned to the west wing. which one of the following CANNOT
e. Patient G is assigned to the north wing. be true?
a. Patients C and F are assigned together.
4. If patients A, C, and F are assigned to the same wing, b. Patients D and H are assigned together.
then which one of the following pairs must be c. Patients A and B are assigned together.
assigned to the south wing? d. Patients B and H are assigned together.
a. Patients A and C e. Patients G and H are assigned together.
b. Patients B and D

231
practice set 2

game 5

Six horses—A, B, C, D, E, and F—are stabled in six horse stalls, numbered 1 through 6, with one
horse per stall. They are stabled according to the following conditions:

Horse E is in a stall next to


horse D. Horse B is either in
stall 1 or stall 2.
Horse A is in a higher-numbered stall than horse D, but in a lower-numbered stall than horse C.

1. If horse F is stabled in a lower-numbered 4. If there are four stalls in between the stall
stall than horse B, then which one of the in which horse F is stabled and the stall in
following CANNOT be true? which horse C is stabled, then for how many
a. Horse E is in stall 3. stalls is it determined which horse is
b. Horse E is in stall 4. stabled there?
c. Horse A is in stall 4. a. One
d. Horse A is in stall 5. b. Two
e. Horse C is in stall 6. c. Three
d. Four
2. If horse C is in stall 5, then which one e. Five
of the following must be true?
a. Horse B is in stall 2. 5. If horse E is stabled next to horse F, then
b. Horse A is in stall 3. how many different sequences of horses in
c. Horse D is in stall 3. their stalls are possible?
d. Horse E is in stall 2. a. One
e. Horse F is in stall 6. b. Two
c. Three
3. Which one of the following could be the stall d. Four
in which horse A is stabled? e. Five
a. Stall 1
b. Stall 2
c. Stall 3
d. Stall 4
e. Stall 6

232
practice set 2

game 6

A bus has eight seats, numbered 1 through 8, with odd-numbered seats on the left side, and
increasing from 1 to 7 from front to back, and even-numbered seats on the right side, increasing
from 2 to 8 from front to back. Seat 1 is across the aisle from seat 2, seat 3 across from 4, seat 5 across
from 6, and seat 7 from 8. Eight passengers fill the bus, one to each seat. Each passenger is headed to
Philadelphia or Boston (not both), and is a senior, adult, or youth. The passengers take seats according
to the following conditions:

Seat 6 is taken by a youth headed for


Philadelphia. Seat 3 is taken by an adult
headed for Boston.
Seniors only sit across the aisle from other
seniors. Exactly six passengers are headed for
Philadelphia. Every youth sits across from an
adult.

1. Which one of the following could be a d. Three


complete and accurate list of the passengers e. Four
in seats 1, 2, 5, and 8, respectively?
a. A senior headed for Boston, an adult
headed for Philadelphia, an adult headed
for Phila- delphia, a youth headed for
Philadelphia
b. A senior headed for Philadelphia, a
senior headed for Boston, a youth headed
for Phila- delphia, an adult headed for
Philadelphia
c. A senior headed for Boston, a senior
headed for Boston, an adult headed for
Boston, a youth headed for Boston
d. A youth headed for Philadelphia, a youth
headed for Boston, an adult headed for
Bos- ton, an adult headed for
Philadelphia
e. A youth headed for Philadelphia, an adult
headed for Philadelphia, an adult headed for
Boston, a senior headed for Philadelphia.

2. If there are exactly five adults on the bus,


what is the maximum number of youths
that could be on the bus?
a. Zero
b. One
c. Two
3. If the right side of the bus has exactly three youths c. There are at least five seniors on the bus.
headed for Philadelphia, which one of the following d. There are exactly two youths headed for
must be true? Bos- ton on the bus.
a. There are exactly four adults on the bus. e. There are exactly three adults headed
b. There are exactly three adults on the bus. for Philadelphia on the bus.
c. There are at most two seniors on the bus.
d. The left side of the bus has all adults. 5. If there are more youths than seniors on
e. There are exactly two seniors on the bus. the bus, what is the maximum number of
seniors that could be on the bus?
4. Which one of the following statements could be true? a. Zero
a. Seat 5 is taken by a senior headed for Boston. b. One
b. Seat 4 is taken by a senior headed for c. Two
Philadelphia. d. Three
e. Four

233
practice set 2

game 7

A band tours five cities—Honolulu, Indianapolis, Jacksonville, Kansas City, and Las Vegas—one at a
time. In each city, they play a concert which ends either with a new song or a classic song. The band tours
according to the following conditions:

The second city toured is Honolulu.


The band ends with new songs in Las Vegas and Indianapolis (but not necessarily only those
cities). In the third city toured, the concert ends with a classic song.
Kansas City is toured before Jacksonville.

1. Which one of the following could be a 4. If Indianapolis and Las Vegas are adjacent
complete and accurate list of the cities on the tour schedule, then which one of the
toured, from first to last? follow- ing CANNOT be true?
a. Las Vegas, Honolulu, Indianapolis, Kansas a. The concert in the fourth city toured
City, Jacksonville ends with a classic song.
b. Indianapolis, Honolulu, Kansas City, b. The concert in the fifth city toured ends
Jacksonville, Las Vegas with a new song.
c. Las Vegas, Kansas City, Jacksonville, c. The concert in the second city toured
Honolulu, Indianapolis ends with a new song.
d. Las Vegas, Honolulu, Jacksonville, Kansas d. The concert in the second city toured
City, Indianapolis ends with a classic song.
e. Kansas City, Honolulu, Las e. The concert in the first city toured ends
Vegas, Jacksonville, with a new song.
Indianapolis
5. Which one of the following CANNOT be true?
2. If Indianapolis and Kansas City are adjacent a. The concerts in the fourth city and fifth
on the tour schedule, then which one of the city toured both end with new songs.
follow- ing must be true? b. The concerts in the fourth city and fifth
a. Las Vegas is first. city toured both end with classic songs.
b. Las Vegas is second. c. The concerts in the first city and second
c. Kansas City is first. city toured both end with new songs.
d. Kansas City is second. d. The concerts in the first city and third
e. Jacksonville is fourth. city toured both end with classic
songs.
3. If the concert in the first city toured e. The concerts in the second city and
ends with a classic song, then how many fourth city toured both end with
different sequences of cities toured could classic songs.
there be?
a. One
b. Two
c. Three
d. Four
e. Five

234
practice set 2

game 8

Six flights—L, M, N, O, P and Q—depart from an airport at different times. They depart from gates A, B,
or C, according to the following restrictions:

No two consecutive departures are from the same gate.


M departs fifth.
N departs before P and Q.
The departure before flight N is from gate
A. Flight P departs before flight M.

1. Which one of the following could be the 4. If Flight Q departs third from Gate B,
order of departing flights, from first to which one of the following must be
last? true?
a. L, O, N, P, M, Q a. The fourth flight departs from Gate C.
b. N, O, P, L, M, Q b. The fourth flight departs from Gate A.
c. O, M, N, L, P, Q c. The fifth flight departs from Gate A.
d. L, N, O, Q, M, P d. The sixth flight departs from Gate C.
e. O, L, P, N, M, Q e. The second flight departs from Gate C.

2. If the first flight departs from Gate C, 5. If the second departure is from Gate C,
then which one of the following must which one of the following CANNOT be
be true? true?
a. Flight N departs second. a. Flight O departs before flights N and M.
b. Flight O departs first. b. Flight P departs after flight Q.
c. Flight P departs fourth. c. Flight L departs after flight M.
d. Flight L departs second. d. Flights L and O depart before flight N.
e. Flight O departs sixth. e. Flight Q departs after flight P.

3. Which one of the following is a complete


and accurate list of the possible departures for
flight P?
a. Third, fourth
b. First, third, fourth
c. Third, fourth, sixth
d. First, second, fifth
e. First, second, fourth, fifth
235
practice set 2

answer explanations
If we choose J in particular, we
Game 1
This selection game has a sub-group domain, so have: J S X and Y S ≠A
sym- bolize the entities: J S ≠K
Since we must select an exact number of
l: A B C entities (six), numerical rules will likely have
D g: H I J importance here. The first numerical restriction is
K t: X Y that if two or three tigers are chosen, three lions
Z are chosen. There are many particular ways to
select one, two, or three tigers, so we could instead
Since we are selecting exactly six entities, we create three general scenarios.
will use six spaces, with the top-side reserved for As mentioned before, you need to be careful
specific entities (A, B, C, etc.), and the bottom for not to spend too much time on scenarios. They
entity sub- group types (l, g, t). can be time-consuming, and might not even be
fruitful if you can’t determine much. You may
find that using scenarios is intuitive and doesn’t
take much of your time. If not, you could go
Now symbolize the rules. The first and fifth straight to the questions and uncover the
rule can be worked into the entity list: mechanics of the game as you go along.
For this practice game, we will provide both
l: A B C D (2–4) (ex.3) types of answer explanations—with and without the
g: H I J K c use of up-front scenarios.
With that in mind, let’s create some
t: X Y Z (2+)
scenarios Scenario 1: If three tigers are
The second, third, and fourth restrictions chosen, exactly
three lions will be chosen:
are: (H or J) S X and Y
Contrapositive: (≠ X or ≠ Y) S ( ≠ H
t t t l l l
and ≠ J)

YS≠A Can any other deductions be made? Indeed,


the first scenario requires all three tigers to be
Contrapositive: A S ≠ Y
selected, so we can go ahead and fill in X, Y, and Z
there. Since Y is selected, the third restriction
JS ≠K
kicks in, and A (a lion) cannot be selected. So
Contrapositive: K S ≠ J
now we know what will fill the three l’s here: B,
C, and D. So, in this scenario, we have a
Now these restrictions can be combined to
complete list.
form a deduction chain:

S1
(H or J) S X and Y S ≠A
X Y Z B C D
t t t l l l
236
practice set 2

Scenario 2: If two tigers are chosen, exactly Question 1


three lions are chosen, leaving one space for a The correct answer is choice e. This is a
giraffe: standard “Test-the-Rules” question. The first
restriction rules out choice b, since only one of
A, B, C, D is selected. The second restriction
t t l l l g rules out choice a, since J is selected but Y is
not. The third restriction rules out choice c
Scenario 3: If one tiger is chosen, the fifth since both Y and A are chosen. The fifth re-
rule does not kick in, but we can make some striction rules out choice d, since X and Y are
other deductions about this scenario. selected, but only two of A, B, C, D are selected.
First, look for restrictions that deal with
the tigers—X, Y and Z. The second restriction Question 2
says that if H or J or both are selected, then two The correct answer is choice d. We already had
tigers (X and Y) must both be chosen. We note some upfront deductions concerning J:
that since exactly one tiger is chosen, then at
least one of X and Y won’t be chosen. So the J S X and Y S
contrapositive of the second restriction kicks in, ≠A J S ≠K
and we know that neither H nor J can be
selected. Therefore, when choosing giraffes for l: A B C D (2–4) (ex.3)
the remaining five spots, we only have I and K
to select from. And since we have one tiger and
g: H I J K c
t: X Y Z (2+)
two giraffes to choose from, we know that at least
three spaces must be lions, with the remaining
Is there anything else that we can deduce
spot going to either a giraffe or a lion:
here? It should be clear from the diagram that
since two tigers have been selected, exactly three
lions must be selected, and those lions must be
t l l l l/g g (g = I or K)
B, C, and D (since A is out). So, we have a full
selection list: J X Y B C D.
So we have our three scenarios, with varying
Look for an answer choice that doesn’t
levels of concrete information given:
appear on this list—I does not appear on this
selection list, so choice d is correct.
S1
X Y Z B C D
Question 3
t t t l l l
The correct answer is choice c. If the only tiger
is Y, then we must be in the one-tiger scenario,
S2
scenario 3:

t t l l l g S3
Y
t l l l g g/l (g = I or K)
S3
Since Y is selected, we know from the
t l l l l/g g (g = I or K) third restriction that A cannot be selected. We
have seen this logic at work before: since A is
out, the three
237
practice set 2

remaining lions must be selected, B, C, and Question 4


D, meaning the remaining spots must both be The correct answer is choice c. Consult the
giraffes, I and K. scenario diagrams to see which ones have one
giraffe selected. Scenarios 2 and 3 both allow for
So we have: Y B C D I K one giraffe:

S2
Look for an answer choice consistent with this
list. Choices a and e are incorrect because H is not t t l l l g
on this selection list. Choice b is incorrect
because A is not on this selection list. Choice d is S3 (modified)
incorrect because
J is not on this selection list. Choice c works because
B, D, and I are all on the selection list. t l l l l/g g (g = I or K)
To answer this question without the use of
up- front scenarios, make deductions from the Now check each answer choice against these
informa- tion that the only tiger is Y by cycling sce- narios. Choice a is incorrect because exactly
through the rules and applying them. Keep track of one tiger is selected in S3. Choice b is incorrect
what’s selected and what can’t be selected on the because exactly two tigers are selected in S2.
diagram: Choice c is correct be- cause S2 has three lions
Y is the only tiger: selected and S3 has four lions selected; it is not
possible to select just two lions. Circle your answer
l: A B C D (2–4) (ex.3) choice and move on to the next question. To
g: H I J K c answer the question without the use of up- front
t: X Y Z ( 2+) scenarios, eliminate answer choices by looking at the
answers for the other questions, and then test each
Now go through the rules and their remaining answer choice to see if it could be true.
contraposi- tives. The first rule does not apply. The answer to Question 1 shows us that an
The contrapositive of the second rules applies: acceptable se- lection list includes two tigers and
since X is not selected, nei- ther H nor J can be three lions, so you can eliminate choices b and d.
selected: Questions 2 and 3 don’t appear to be much help,
so try each of the remaining answer choices to see
l: A B C D (2–4) (ex.3) if an acceptable scenario could be constructed
g: H I J K c around it. If exactly one tiger is selected, and
t: X Y Z ( 2+) exactly one giraffe is selected, then the remaining
four animals must be the four lions:
The third rule indicates that A is not
selected:
ABCD
l: A B C D (2–4) (ex.3) again, we have the selection list: Y B C D I
g: H I J K c K.
t: X Y Z ( 2+)

This means that to get six animals, the


remaining entities must all be selected. So
t g l l l l contradictions. The first rule is satisfied. The
second rule will be satisfied as long as neither
There don’t seem to be any contradictions, H not J is selected (since two tigers, X and Y,
but see if you can fill it out without violating any cannot both be selected)—so let’s select I for the
giraffe:

238
practice set 2

Z. Since Y is selected, A cannot be—so the three


I A B C D lions must be B, C, and D. This selection does not
t g l l l l violate any of the rules, and so choice b can be
eliminated. That leaves choice d as the correct
The third rule will be satisfied as long as answer. If you tested choice d, you would see that
Y is not selected (since A is selected)—so let’s if three giraffes are selected, they would have to be
select Z for the tiger: H, I, and one of J and K (since by the fourth
restriction, J and K cannot both be selected). This
Z I A B C D means that X and Y must also be selected (by the
t g l l l l second restriction), leaving one spot.
But we need two lions, and have none so far
If you continue checking the rules, you will —so it’s not possible to have three giraffes.
see that none are violated—so having exactly one
tiger is possible, and we can rule out choice a. You Game 2
might also notice that since four lions are selected This selection game has a simple domain, so
here, choice e is also ruled out. That leaves merely list the symbols for the entities:
choice c as the correct answer. One can see that c
is indeed the correct answer, since exactly one FSRHAGC
giraffe and exactly two lions means we need
exactly three tigers—which would violate the fifth Now, symbolize the rules:
rule, since we would have at least two tigers, but
not have exactly three lions. FS G
Contrapositive: ≠G S ≠F
Question 5
The correct answer is choice d. Test each (A v C) S ≠R
answer choice against the scenario diagrams. Contrapositive: R S (≠A and ≠C)
Exactly two ti- gers are selected in S2, so choice a
is incorrect. Exactly three tigers are selected in S1, ≠A S H
so choice b is incorrect. Exactly two giraffes could Contrapositive: ≠H S
be selected in S3, so choice c is incorrect. But in no A Disjunctive form: A
scenario are three giraffes se- lected, so choice d vH
is the correct answer.
To answer the question without the use of S S ≠G
up- front scenarios, eliminate answer choices by Contrapositive: G S ≠S
looking at the answers for the other questions, and Now look for connections among rules. The
then test each remaining answer choice to see if first and fourth restrictions connect through G:
it could be true. The answer to Question 1
eliminates choice a, since exactly two tigers are S S ≠G S ≠F
selected there. The answer to Question 2 eliminates F S G S ≠S
choice e, since exactly three lions are selected
there. The answer to Question 3 eliminates The second and third rules connect up through
choice c, since exactly two giraffes are selected A:
there. So we need to test choices b and d. If
three tigers are selected, then by the fifth ≠H S A S ≠R
restriction, three lions are selected. The three R S (≠A and ≠C) S H
tigers are X, Y, and
239
practice set 2

The other thing to keep in mind is that there Question 4


is a disguised disjunction: any selection of languages The correct answer is choice e. Look again at your
must have either A or H or both. de- duction chains to see what follows from the
We now turn to the questions. book not being translated into Hindi. According to
the ≠H S A S ≠R chain, the book must be
Question 1 translated into Arabic and not into Russian. Scan
The correct answer is choice c. This is a the answer choices—choice e is that the book is
standard “Test-the-Rules” question. The first not translated into Russian.
restriction rules out choice b, since French is
chosen, but German is not. The second Question 5
restriction rules out choice e, since both Arabic The correct answer is choice b. If the book is
and Russian are chosen. The third restric- tion not translated into Hindi, then according to the
rules out choice a, since neither Arabic nor Hindi deduc- tion chain ≠H S A S ≠R, it is translated
are present. The fourth restriction rules out choice into Arabic and not translated into Russian. If
d, since both Spanish and German are chosen. the book is not translated into German, then
according to the de- duction chain S S ≠G S ≠F,
Question 2 it is also not translated into French.
The correct answer is choice d. If the book is So we have:
trans- lated into Russian, then we know from our
deduction chain—R S (≠A and ≠C) S H—that A FSRHAGC
and C are not selected, and H is selected.
So we have: Now check each answer choice against
this selection diagram. Choices a and e are
FSRHAGC incorrect because they include French, which is
crossed out. Choices c and d are ruled out
There are now two selected (R, H), and three because they include Russian, which is crossed
left (F, S, G) to select from. Can we select all out. Choice b includes Spanish and Chinese—
three. We know from the fourth restriction that G two languages that could be selected according to
and S cannot be selected together. So the most we the selection diagram. So b is the correct answer.
can select is one of those two (G or S) and F, in
addition to R and H. Let’s pick one to see if it Game 3
works: G, F, R, H. This selection does not violate For this grouping game, first diagram the game
any of the restrictions, so it works, and the play. We have three groups of three:
maximum number we can select is four, choice d.
1:
Question 3 2:
The correct answer is choice a. Your first instinct 3:
for a question like this should be to review your
deductions to see if the correct answer is Note that we label the groups 1, 2, and 3 for
obvious. The S S ≠G S ease of reference in these answer explanations;
≠F deduction chain immediately tells you that when you are actually diagramming, you
choice a wouldn’t need these labels.
cannot be true. If Spanish is selected, French cannot
be.
None of the other answer choices present
pairs that cannot be together.
240
practice set 2

Next, symbolize the restrictions. What about scenario 2, in which J and H do


not go together? All we know here is that J is
AC not in plat- ter 1 and, by the first restriction, that
DH A does not go in platter 3.
EG S JH
Contrapositive: JH S EG S2
CG 1: D H (J)
2:
Now see what deductions can be made. 3: C G (A)
Since there are only three cheeses per plate, pairs
DH and CG must appear on different plates (they Question 1
are placed on 1 and 3 arbitrarily—remember that The correct answer is choice a. This is a
the numbers here mean nothing): standard “Test-the-Rules” question. The first
restriction rules out choice c, since A and C are
1: D H together. The second restriction rules out choice
2: d, since D and H are not together. The third
3: C G restriction rules out choice b, since E and G are
together, but so are J and H. The fourth
To get a handle on the game play, you might restriction rules out choice e, since C and G are
try to divide the game into scenarios. There may not together.
be a few ways to do this. One way would be to
work with the contrapositive of the third Question 2
restriction, dividing the game into two scenarios The correct answer is choice b. If J and D go
—one on which J and H go together, and one in together, then we must be in S1:
which J and H do not go together.
In Scenario 1, J and H go together, so 1: D H J
platter 1 is set: 2: A E F/B
3: C G F/B
1: D H J
2: To answer this question without the use of
3: C G up- front scenarios, start with the information that
J and D go together:
Now apply the other restrictions. Since A
does not go with C, A must appear in platter 2 1: D H J
According to the contrapositive of the third 2:
restriction, since J goes with H, E cannot go with 3: C G
G, and E must appear in platter 2 as well. The two
remaining cheeses, F and B, are free for the Now apply the rules. The first rule forces
remaining slots. So we have scenario 1: A into platter 2 The second rule is satisfied.
And the contrapositive of the third rule indicates
S1 that E and G cannot go together, which also forces
1: D H J E into platter 2. We don’t know where to place F
2: A E F/B or B yet. They could go in either platter 2 or 3.
3: C G F/B

241
practice set 2

Now we have scenario 1: S2.2


1: D H (J)
1: D H J 2: A (E)
2: A E F/B 3: C G
3: C G F/B
Now test each answer choice against these
F and E could be served on the same dia- grams A, J, B could appear on platter 2 in
platter (platter 2), so a is incorrect B and G could S2.2, so a is incorrect J, E, F could appear on
go together in platter 3, so c is incorrect B and A platter 2 of S2.1, so b is incorrect C, G, J could
could go together in platter 2, so d is incorrect A appear on platter 3 of S2.2, so c is incorrect D,
is served on the same platter as E, so e is H, A appears on platter 1 of S2.1, so d is
incorrect. F and B could not be served on the incorrect. But D, H, J could neither appear in a
same platter, since those two have to be platter in S2.1, nor appear in a platter in S2.2
distributed over platters 2 and 3. So b is the (since J has been ruled out from platter 1, where
correct answer. we already have D and H), so e is the correct
answer.
Question 3 Answering this question without the aid
The correct answer is choice e. If A and E are of scenarios is more time-consuming: you need to
not to- gether, then we must be in scenario 2: try each set of three cheeses to see if they could all
appear together on a platter. Since we start with
1: D H (J) the initial diagram, which incorporates all the
2: information from the second and fourth rules, we
3: C G (A) only need to consider the first and third rules. If
A, J, and B are on the same platter, we have:
Since the new information involves A, let’s
work with A. A must appear in platter 1 or platter 1: D H
2, so draw out both possibilities: 2: A J B
3: C G
S2.1
1: D H A The new restriction (AE) forces E into platter
2: 1, and so F is left for platter 3:
3: C G
1: D H E
S2.2 2: A J B
1: D H (J) 3: C G F
2: A
3: C G We need to check that the first and third
rules are satisfied. A and C are separated, so the
Now add in the information that A and E do first rule is satisfied, and since E and G are
not go together. For S2.1, this means that E will be separated, the third rule does not apply. So
in plat- ter 2 or platter 3, which was known this is an acceptable distribution, and choice a
already, so there are no further restrictions on E. is incorrect.
For S2.2, this means E will not appear on platter
2:

242
practice set 2

What about choice b? If J, E, and F are on


the same platter, we have: Since J and H are together, the contrapositive
of the third rule kicks in, and E and G must be
1: D H sepa- rated—meaning that E is on platter 2:
2: J E F
3: C G 1: D H J
2: E
The first rule forces A into platter 1, leaving B 3: C G
for platter 3 Since E and G are separated, the
third rule does not apply, and this is an acceptable Now look at the first rule (AC) and the new
distribution— meaning that b is incorrect. rule (AE): A can neither go on platter 2 or
If C, G, and J go together, we have: platter 3 (and there is no room on platter 1) So
this distribution is impossible and choice e is
1: D H the correct answer.
2:
3: C G J Question 4
The correct answer is choice c. If F and B go
None of the rules—the first rule, the together, then we cannot be in Scenario 1 (in
third rule, or the new rule—are triggered, so which F and B are split over platters 2 and 3), and
rather than waste time completely filling out the we must be in Scenario
scenario, we can tentatively assume this is an 2. Since F and B go together, there is only room
acceptable distribution, and tentatively rule out for them on platter 2:
choice c.
If D, H, and A go together, then we have: 1: D H _ (J)
2: F B _
1: D H A 3: C G _ (A)
2:
3: C G Since A could go with B in platter 2, a is
incorrect. Since J could go with G in platter 3, b
Again, none of the rules are immediately is incorrect. Since J cannot go with D (which is in
triggered, so we tentatively assume this is an platter 1), choice c is the correct answer. Since E
acceptable distribution, and tentatively rule out could go with D in platter 1, d is incorrect. Since
choice d. A could go with F in platter 2, e is incorrect.
Since we have only tentatively ruled out This question is also easy to answer without
choices c and d, we must test choice e (if we had the use of scenarios. Since F and B go together, we
definitively ruled them out, then we could circle e have:
and move on).
If D, H and J go together, we have: 1: D H
2: F B
1: D H J 3: C G
2:
3: C G
243
practice set 2

Now try to apply the rules. The first means J must be on platter 3, and B must be on platter
that A cannot be on platter 3: 1— so we have 1 possible grouping. The total
possible groupings for S2 are three, combined
1: D H _ with the one possible grouping from S1; therefore,
2: F B _ we have a grand total of four possible groupings,
3: C G _ (A) answer d.
We could also answer this question without
If it’s not immediately clear what other the use of scenarios. Since E and F are together,
deduc- tions can be made from the rules, start we see that we must have:
testing the answer choices. Choices a and b can
be eliminated as above (there are no immediate 1: D H
rules triggered or contradictions). Now we test 2: E F
choice c. If J and D go together, then we have: 3: C G

1: D H J Since E and G are not together, the third


2: F B _ rule does not apply, and so we only need to
3: C G _ (A) consider the first rule (remember, the initial
diagram already incorporates all the information
Since J and H are together, the contrapositive from the second and fourth rules). The first rule
of the third rule kicks in, and E and G are means that A could only be on one of two
separated, forcing E into platter 2, leaving room platters (platter 1 and 2), and there are two
only on platter 3 for A—a situation ruled out by unrestricted cheeses left. Therefore, there are four
the diagram. This contradiction shows us that c is total assignments—two in which A is on platter 1
the correct answer choice. and two in which A is on platter 2.

Question 5 Game 4
The correct answer is choice d. We want to know Start by drawing the diagram for this grouping
how many different groupings of cheeses onto platters game:
there are, so long as E and F are served on the
same plat- ter and other restrictions are being w:
met. Consult the scenarios/diagrams. If in S1, then n:
there is one possible grouping (with E and F both s:
on platter 2). If we are in S2, then we have:
We know that D is in n, so draw that in
1: D H _ (J)
2: E F _ directly. w:
3: C G _ (A) n: D
s:
Since A could be on platter 1 or platter 2,
let’s take each one at a time. If A is on platter 1, No we symbolize the other
then J could either be on platter 2 or platter 3—so
we have two possible groupings. And if A is on restrictions: EG
platter 2, then H=wS B=s
Contrapositive: B ≠ s S H ≠
w BG
244
practice set 2

You could create scenarios (perhaps one with The fourth rule can also be applied: G cannot
EG in the west wing and one with EG in the south be in the west wing because B is:
wing), but these questions can be answered by
just applying the rules. w: B HG
n: D
Question 1 s:
The correct answer is choice a. The first
restriction rules out choice d, since D is not in
the north wing. The second restriction rules out This means that the EG block (given by the
choice b, since E and G are not assigned together. sec- ond rule) must be in the south wing:
The third restriction rules out choice c, since H is
in the west wing, but B is not in the south wing. w: B HG
The fourth restriction rules out choice e, since B n: D
and G are assigned together. s: E G

Question 2 No other deductions are immediately apparent,


The correct answer is choice a. Look for the choice so consider the answer choices. Choice a is
that does not violate any rules, or leads to a incorrect because E is assigned to the south
contradiction. Can E and B be assigned together? wing. Choice b is incorrect because H is barred
By the second rule, E and G must be assigned from the west wing. Choices d and e are incorrect
together; but by the fourth rule, B and G cannot because G is assigned to south wing. Choice c is
be together, and so if B can’t be with G, B can’t correct because H can be assigned to the north
be with E either. So E and B cannot be assigned wing or south wing.
together, and the correct answer is choice a.
Question 4
Question 3 The correct answer is choice e. If A, C, and F are
The correct answer is choice c. We know that as- signed to the same wing, that wing must be
patient B is assigned to the west wing: either the west wing or the south wing. Try each
to see if we can figure out which one it is. If A,
w: B C, and F are assigned to the south wing, we
n: D have:
s:
w:
Now, go through the rules. The n: D
contrapositive of the third rule can be applied, s: A C F
since B is not in the south wing, it means H is
not in the west wing: Now see what follows from the rules. By
the contrapositive of the third rule, since B is
w: B H not in the south wing, H is not in the west
n: D wing—meaning that H is in the north wing,
s: and the remaining three patients are in the west
wing:

w: E G B
n: D H
s: A C F
245
practice set 2

But this violates the fourth rule, which says By the fourth rule, B cannot be in the south
that B and G cannot go together. So A, C and F wing:
must be in the west wing:
w:
w: A C F n: D
n: D s: E G B
s:
By the contrapositive of the third rule, this
This means that the EG block must be in means that H is not in the west wing:
the south wing:
w: H
w: A C F n: D
n: D s: E G B
s: E G
This captures all the information in the rules,
Since B cannot go with G (by the fourth and no further deductions are immediately
restriction, B must be in the north wing), H is apparent, so test each of the answer choices to see
forced into the south wing: if it could be true. As usual, when testing answer
choices for questions like these, check each for a
w: A C F relatively immediate contradiction—and if none
n: D B surfaces, tentatively rule that choice out and move
s: E G H on. There are no immediate contradictions for
choices a, b, and c. It seems that F and C could be
Choice a is incorrect because A and C are assigned together in the west wing, H and D
assigned to the west wing. Choice b is together in the north wing, and B and A together
incorrect because D and B are assigned to the in the west wing, without any immediately
north wing. Choice c is incorrect because D is apparent problems. But H and B seem to
assigned to the north wing and E assigned to the present a problem. There is space for both of
south wing. Choice d is incorrect because H is them only in the west wing, but H cannot be in the
assigned to the south wing and B is assigned to west wing—so H and B can’t be assigned together,
the north wing. Choice e is correct because both and choice d is the correct answer.
H and E are assigned to the south wing.
Game 5
Question 5 We have six stalls for this sequencing game,
The correct answer is choice d. If G is in the numbered 1 through 6:
south wing, E must be in the south wing as
well (because of the EG block given by the second
restriction), and we have: 123456

w: Now symbolize the restrictions:


n: D
s: E G E/D
E/D—A—C
246
practice set 2

This captures the first and third Question 2


restrictions. Since the second restriction—which The correct answer is choice e. If C is in stall 5,
says that B must either be in 1 or 2—presents two then we must be in scenario 2, since in scenario
scenarios, we could see what follows from each 1, C is in stall 6. If C is in stall 5, then E/D—A—
scenario. C must be in between B and F (that is, there is
If B is in stall 2, then we know that the E/D only room for E/D-A to the left of C).
block cannot be to the left of B (since there is So we have:
only one spot to the left of B). So E/D must be to
the right of B, which means that the whole B E/D A C F
sequence E/D—A—C must fill the four spots to 12345 6
the right of B (stalls 3 through 6). And that
means that the remaining horse (F) must be in B is not in stall 2, so a is incorrect. A is not
stall one. So for scenario 1, we have: in stall 3, so b is incorrect. D could be in stall 3, but
could also be in stall 2, so it’s not the case that D
S1 must be in stall 3; so c is incorrect. E could be in
stall 2, but could also be in stall 3, so it’s the case
F B E/D A C that E must be in stall 2; so d is incorrect. F
12345 6 must be in stall 6, so e is correct.

If B is in stall 1, then E/D—A—C must be Question 3


to the right of B, and F floats somewhere to the The correct answer is choice d. Consult the
right of B as well (but not between E and D). So diagrams of the two scenarios to see where A
we have: appears. In S1, A appears in stall 5. But stall 5 is
not one of the answer choices. In S2, A is floating
S2 —but it cannot appear in stalls 2 or 3, because that
would leave no room for E/D to its left. And it
‹——F——› cannot appear in stall 6, because that would leave
B E/D—A—C no room for C to its right. So it must ap- pear in
1 234 5 6 either stall 4 or 5. So d is the correct answer.

Question 4
Question 1 The correct answer is choice d. If there are four
The correct answer is choice c. If F is before B, then stalls between F and C, then we must be in
we must be in scenario 1: scenario 1 (sce- nario 2 does not allow for four
stalls between F and C):
F B E/D A C
12345 6 F B E/D A C
12345 6
E could be in stall 3 or 4, so choices a and b
are incorrect. A must be in stall 5, and cannot be The diagram shows that stalls 1, 2, 5, and 6
in stall 4, so choice c is the correct answer and d are all filled by a particular horse; stalls 3 and 4
incorrect. C must be in stall 6, so e is incorrect. are not set (although we know that E and D fill
the two). So there are four stalls for which it is
determined which horse is stabled there—
answer choice d.
247
practice set 2

Question 5 The third restriction can be


The correct answer is choice b. If E and F are
next to one another, then we must be in scenario 2 symbolized: s
(in scenario 1, B, and maybe D, separates E from
F):
s
‹——F——›
B E/D—A—C
This indicates that if a senior is sitting in
1 234 5 6
any seat, another senior must be sitting right
across from him or her.
But E and F must also be next to one another,
The fourth restriction can be
so we have a new block that includes E, F, and
D:
symbolized: 6 P, 2 B

B E/F/D A C
The fifth restriction can be
12345 6
symbolized: a
But for E to be both next to D and next to
F, it must appear in the middle of the two. There
are two ways to accomplish this, D-E-F and F-E-
y
D:
and
B D E F A C or B F E D A C
y
So there are two possible sequences, choice
b.
a
Game 6
This matching game is particularly spatial in
charac- ter, so a diagram will be a necessity. Draw
the num- bered seats on the bus left to right,
front to back:

7( , ) 5( , ) 3( , ) 1( , ) This indicates that wherever a youth sits,


an
8( , ) 6( , ) 4( , ) 2(,) adult must sit across from him or her.
Finally, see if any obvious deductions can
Each ( , ) notation is there be filled with a be made. Since, according to the fifth restriction,
letter (P or B) for the destination, and a letter (s, every youth sits across from an adult, we see that
a, y) for the age-type. seat 5 must be filled by an adult. This deduction
Now symbolize the first two restrictions: should also be made obvious by comparing our
spatial symbolization of the fifth restriction to the
7( , ) 5( , ) 3(a,B) 1( , ) diagram.
8( , ) 6(y,P) 4( , ) 2 ( , )
7( , ) 5(a, ) 3(a,B) 1( , )
8( , ) 6(y,P) 4( , ) 2 ( , )
248
practice set 2

Question 1 already two adults fixed (in seats 5 and 3). So


The correct answer is choice e. This question is a we have to distribute three adults to maximize
“Test- the-Rules” question, but it requires more space for youths. To do this, put one adult in
processing than usual. We usually approach one of the seat 7/ seat 8 combo, and put another
questions like this rule-by-rule—that is, we check in one of the seat 1/seat 2 combo. At this point, we
to see which answer choice(s) violates the first have space for three more youths (one in the seat
rule, and then which an- swer choice(s) violate the 7/seat 8 combo, one in the seat 1/seat 2 combo,
second rule, etc. But that ap- proach will be and one across from seat 3). But we still have to
difficult here, because understanding each answer distribute one more adult. Since anywhere we put
choice requires assessing how it looks in the that adult will result in him or her sitting across
diagram first. So you want to go choice-by-choice from another adult, we reduce our capacity for
rather than rule-by-rule. youths by one. So there will be two open spots for
Choice a is incorrect because it violates the youths, not three.
third restriction, which says that seniors only sit Add those two spots to the youth already
across from seniors (and a senior in seat 1 would present in seat 6, and we see that the maximum
be sitting across from an adult in seat 2). Choice b number of youth is three, or answer choice d.
is incorrect because it violates a combination of
the fifth restriction, which says that every youth
Question 3
sits across from an adult, and the first restriction,
The correct answer is choice c. First, note that the
which says that there is a youth in seat 6 (since a
right side of the bus will have two more (y,P)’s:
youth in seat 5 would be sitting across from a
youth in seat 6). Choice c is incorrect because it
7( , ) 5(a, ) 3(a,B) 1( , )
violates the fourth restriction, which says that
exactly six passengers are headed for Philadelphia
8( , ) 6(y,P) 4( , ) 2 ( , )
(since the combination of these four passengers
2 more (y,P)’s
headed for Boston, with the passenger in seat 3
headed for Boston, amounts to a total of at least
Scan each answer choice to see if it you
five passengers headed for Boston, leaving only
could make it false without violating a
three or less headed for Philadelphia). Choice d
restriction. Since you don’t want to waste time,
violates the fifth restriction, since it would have
check each choice very briefly, in the hopes that
a youth sitting across from a youth in seats 1
the correct answer will be apparent once you
and 2. Choice e does not violate any restrictions.
try to make it false.
Could there be more than four or less than
Question 2
four adults? It seems we could have five adults if the
The correct answer is choice d. Start with the
left side were all adults and there was one adult on
restric- tion that will play the most important part
the right. So we can tentatively rule out choice a.
given the question: the fifth restriction says that for
This would also tentatively rule out choice b (and,
every youth, an adult sits across the aisle. So we
incidentally, choice d—but we won’t need to get
want to play with our diagram to maximize the
that far). Now, could there be more than two
number of youths, given exactly five adults. Have as
seniors on the bus? Each senior needs to sit across
many adults sitting across from empty seats as
from another senior—and since there are three
possible, rather than sitting across from other
youths on the right side, there is only space for one
adults.
row of seniors (that is, space for only two seniors).
Looking at our diagram, we see that there
So choice c is the correct answer.
are
249
practice set 2

Note that you could tell choice e is


incorrect simply by noticing that it logically 7(a,P) 5(a,P) 3(a,B) 1(s,P)
entails choice c— and so it couldn’t be the correct 8(a,P) 6(y,P) 4(y,B) 2 (s,P)
answer, otherwise there would be two correct
answers. In any case, choice e is incorrect Question 5
because we could fill all the open spots (not The correct answer is choice c. First, note that
taken by the required youths) with adults. the number of seniors must be a multiple of 2
(including 0), since they come in across-the-aisle
Question 4 pairs, as per the fifth restriction. So choices b and
The correct answer is choice e. Take each d are incorrect. Next, since there are a total of
statement, and test it against the diagram, eight people, and more youths than seniors, there
restrictions, and fur- ther deductions to see if it can be at most three seniors (if there were four
could be true. Choice a is incorrect because as seniors, there could only be four youth). So
our diagram shows, seat 5 is taken by an adult. choice e is incorrect. Now we must choose
Choice b is incorrect because according to the between choices a and c. Could there be two
third restriction, every senior must sit across from seniors? Yes, it’s possible for there to be more
another senior, and this choice would have a youth than se- niors when there are two
senior in seat 4 sitting across from an adult in seat seniors.
3. Choice c is incorrect because there is only room You have to arrive at this conclusion by
for, at most, four seniors on the bus—the seat trying to make this possibility work:
5/seat 6 and seat 3/seat 4 across-the-aisle pairs each
contain at least one non-se- nior, leaving just two 7(y, ) 5(a, ) 3(a,B) 1(s, )
fully open across-the-aisle pairs, the seat 7/seat 8 8(a, ) 6(y,P) 4(y, ) 2 (s, )
pair and the seat 1/seat 2 pair. These two fully
open pairs can accommodate at most four seniors. This diagram shows two seniors, and
Choice d is incorrect because if there were two three youth. If you examine the rules, none are
youths headed for Boston, then there would be violated by this arrangement (as long as we assign
three total passengers headed for Boston (including P to exactly five more passengers). So we have
the adult headed for Boston in seat 3), which found our maximum possible number of seniors:
would violate the fourth restriction (which implies two. Choice a is incorrect because this diagram
that there are only two passengers headed for shows that we can have at least two seniors and
Boston). So choice e is the correct answer. still meet the requirement that there be more
Since you have eliminated the other answer youth than seniors.
choices, you should just circle this and move
on. Game 7
If you happened to test choice e before This is a hybrid game—a sequencing game with an
testing all the others, you would see that it is ele- ment of matching—so draw a grid in which
correct because it could be the case that there are we match the cities and song-type to each
exactly three adults headed for Philadelphia, or number, 1 to 5:
three (a,P)s. It’s enough to see that choice e
probably allows for an acceptable scenario by not 1 2 3 4 5
immediately seeing any problems with it (and then H, I , J, K, L
eliminating the other answers), rather than c/n
coming up with a specific example—but here’s one
example of the bus containing exactly three adults Now symbolize the restrictions and start
headed for Philadelphia: fill- ing in the grid. The first restriction can be
written directly into the diagram. The second restriction can

250
practice set 2

by symbolized by writing n next to L and I. The third restriction 1 2 3 4 5


can also be written directly into the dia- gram. The fourth J H IL K I(n) , J, K, L(n)
restriction can be symbolized K—J. c c/n

K—J 1 2 3 4 5
J H K I(n) , J, K, L(n)
c c/n The block clearly cannot go in spots 1 or 2
for lack of room. This means they must take up
K—J spots 3–4 or spots 4–5. But the block I/K couldn’t
take up spots 4–5 because there would be no room
We have also written in that the first city is for J to the right of K (as the string K—J requires).
not J, and the last not K. So they must take up spots 3–4. But I cannot be
See if any further deductions can be made. in spot 3, so spot 3 must be K, spot 4 must be I,
Since the third city has a concert that ends with leaving only spot 5 for J, forcing L into spot 1:
a classic song, that third city cannot be either
Indianapolis or Las Vegas, since they have 1 2 3 4 5
concerts ending with new songs: JL H ILK I KJ I(n) , J, K, L(n)
n c n c/n
1 2 3 4 5
J H IL K I(n) , J, K, L(n) (We have also noted that spots 1 and 4
c c/n have an n, due the second restriction).

K—J K—J

Since Las Vegas is in spot 1 on this grid, a


Question 1 is the correct answer choice. Choice b is incorrect
The correct answer is choice b. This is a because Las Vegas is not second, choice c is
standard “Test-the-Rules” question. The first incorrect because Kansas City is not the first,
restriction rules out choice c, since the second choice d is incorrect because Kansas City is not the
city toured is Kansas City, not Honolulu. The second, and choice e is incorrect because
second and third restric- tions together rule out Jacksonville is not the fourth city toured.
choice a, since the third city, Indianapolis, ends
with a new song, not a classic song. They also rule Question 3
out choice e, since the third city toured is Las The correct answer is choice b. Work the
Vegas, which ends with a new song, not a classic informa- tion that spot 1 has a c into the diagram,
song. The fourth restriction rules out choice d, and see what follows:
since K is not before J.
1 2 3 4 5
Question 2 J H IL K I(n) , J, K, L(n)
The correct answer is choice a. I and K are next to c c c/n
one another, and form a block. Try to see where
you can fit I/K into the diagram: K—J

251
practice set 2

Since spot 1 is a c, neither I nor l can be in spot 1—this 1 2 3 4 5


leaves only K for spot 1: J H IL I/L K I/L I(n) , J, K, L(n)
c c/n
1 2 3 4 5
JILK H IL K I(n) , J, K, L(n) K—J
c c c/n
This means that to preserve the K– J
K—J sequence, K must be in spot 1 and J in spot 3.
And it also means that spots 4 and 5 have an n
Since neither I nor L can be in spot 3, they (because of the second restriction):
must be in spots 4–5 (meaning that spots 4 and
5 have an n, given the second restriction). 1 2 3 4 5
This leaves J for spot 3: JK H ILJ I/L K L/I I(n) , J, K, L(n)
c n n c/n
1 2 3 4 5
JILK H ILJ I/L K I/L I(n) , J, K, L(n) K—J
c c n n c/n
Now turn to the answer choices. The fourth
K—J spot has an n, and so it can’t be the case that the
fourth city toured has a concert that ends with a
There is no restriction on the order of I and classic song, so a is the correct answer. The fifth
L here, so there are two possible different spot has an n, so b is not correct. The second spot
sequences— answer choice b. is open, and so could be filled with either c or n,
and so c and d are incorrect. The first spot is
Question 4 also open, so it could have an n, and therefore
The correct answer is choice a. I and L are next to choice e is incorrect.
one another, and form a block. See where this
block can fit into the diagram: Question 5
The correct answer is choice b. Take each
1 2 3 4 5 answer choice in turn, and quickly test it to see
J H IL K I(n) , J, K, L(n) if any con- tradictions emerge when you apply
c c/n the rules. Keep in mind that information from
previous questions might come in handy—look for
K—J scenarios you have already established to be
acceptable. The diagram created for Question 5
The block clearly cannot go in spots 1 or 2, rules out choice (a), since spots 4 and 5 have n
for lack of room. This means they must take up there. The diagram for Question 4 rules out
spots 3–4 or spots 4–5. But neither I nor L can be choice d. Now test choice b:
in spot 3, so the block must take up spots 4–5:
1 2 3 4 5
J H IL K I(n) , J, K, L(n)
c c c c/n

K—J

252
practice set 2

If spots 4 and 5 have c, then neither of Now look for connections between the
those spots can have I or L (since they both carry restric- tions and make deductions. Since P cannot
an n with them). And since neither I nor L can be after M (in the fifth spot), we can note that P
be in spot 3, that leaves only spots 1 and 2—but does not depart sixth (which captures all the
spot 2 is already taken by H. So it can’t be that
information from the P—–M restriction). And
spots 4 and 5 have c’s, and therefore choice b is
since N is before P, we note that P cannot depart
correct.
first. Similarly, since N is before Q, we note that
Q cannot depart first:
Game 8
This is a hybrid game with an element of
1 2 3 4 5 6
sequenc- ing (putting the six flights in order of
PQ M P L, M, N, O, P, Q
departure) and matching (matching each flight
A/B/C
to a gate). The best way to diagram this game is
with a grid: No consec dep. from same
gate Gate before N is A
1 2 3 4 5 6 N —–P/Q
L, M, N, O, P, Q
A/B/C Since there must be at least one departure
before N (as per the fourth restriction), we
Now symbolize the restrictions and
know that N cannot be first. Since P and Q
add whatever information you can into the
must come after N (as per the third restriction),
diagram. The first restriction is difficult to
we know that N needs two open spaces to its
symbolize, so circle it and make sure to keep it
right. N cannot be in spot 6 (zero spaces to its
in mind. You may also want to write it below
right), or spot 5 (M is already there), or spot 4
the diagram to remember it. The second
(only one open spot to its right):
restriction can be directly coded into the
diagram.
1 2 3 4 5 6
PQ
1 2 3 4 5 6 N M P N L, M, N, O, P, Q
N
M L, M, N, O, P, Q
A/B/C
A/B/C
No consec dep. from same
No consec dep. from same gate
gate Gate before N is A
The third restriction can be
N —–P/Q

symbolized: N —– P/Q Question 1


The correct answer is choice a. This question is a
The fourth restriction is also diffi cult to “Test- the-Rules” question. The second restriction
symbolize, so circle it for now and keep it in rules out choice c, since M is the second departure.
mind. You may also want to write it below the The third re- striction rules out choice e, since N
diagram to remember it. does not depart be- fore P. The fourth restriction
The fifth restriction can be rules out choice b, since N is the first departure on
this answer choice, but the restriction implies that
symbolized: P —– M there is at least one flight be- fore N. The fifth
restriction rules out choice d, since M departs
before P.
253
practice set 2

Question 2 because N departs third, not second. Choice b


The correct answer is choice c. See what is incorrect because it is not specified when O
deductions can be made from the new departs— it could be first or second, according to
information that the first flight departs from this diagram. Choice d is incorrect because L’s
Gate C: departure is not specified either—it could either
be first or second. Choice e is incorrect because O
1 2 3 4 5 6 does not depart sixth; rather, Q departs sixth.
PQN N M P N L, M, N, O, P, Q Choice c is correct because P departs fourth in
C A/B/C this diagram.

No consec dep. from same Question 3


gate Gate before N is A The correct answer is choice a. We’ve already done
N—–P/Q the necessary work, so take a look at the
diagram.
Since there can be no consecutive
departures from the same gate, the second f 1 2 3 4 5 6
light must leave from Gate A or Gate B. Further, PQN N M P N L, M, N, O, P, Q
since the gate for the departure before N is gate A, A/B/C
spot 2 cannot have N— this only leaves spot 3 for
N, which means that spot 2 is gate A: No consec dep. from same
gate Gate before N is A
1 2 3 4 5 6 N—–P/Q
PQN N N M P N L, M, N, O, P, Q
C A A/B/C This tells us that P cannot be first, fifth,
or sixth—this eliminates choices b, c, d, and e.
No consec dep. from same This only leaves answer choice a.
gate Gate before N is A
N—–P/Q Question 4
The correct answer is choice e. See what
Since N is before both P and Q, P and Q deductions can be made from the new
must take spots 4 and 6. But P cannot be in spot 6, information that Q departs third from gate B:
so P must be in spot 4 and Q in spot 6:
1 2 3 4 5 6
1 2 3 4 5 6 PQN Q N M P N L, M, N, O, P, Q
PQN N NP M P N Q L, M, N, O, P, Q B A/B/C
C A A/B/C
No consec dep. from same
No consec dep. from same gate Gate before N is A
gate Gate before N is A N—–P/Q
N—–P/Q
Since N has to come before Q, and N can’t
Now scan the answer choices to see which be in spot 1, N must be in spot 2. Further, since
one is given to us in this diagram. Choice a is the gate for the departure before N is Gate A, the
incorrect first flight must then be from Gate A:

254
practice set 2

1 2 3 4 5 6 1 2 3 4 5 6
PQN N Q N M P N L, M, N, O, P, Q PQN N M P N L, M, N, O, P, Q
A B A/B/C C A/B/C

No consec dep. from same No consec dep. from same


gate Gate before N is A gate Gate before N is A
N—–P/Q N—–P/Q

Now, look at the rules again. Since no Note that flight N has to depart second or
two consecutive departures can be from the same third. But it cannot depart third, because the
gate, the second flight must depart from Gate C gate for the departure just before it is Gate C (and
(since it is flanked by A and B). This rule also the fourth rule tells us that that gate must be A).
indicates that spot 4 must be Gate A or C. So N must depart second, and flight 1 must
depart from Gate A:
1 2 3 4 5 6
PQN N Q N M P N L, M, N, O, P, Q 1 2 3 4 5 6
A C B A/C A/B/C PQN N N M P N L, M, N, O, P, Q
A C A/B/C
No consec dep. from same
gate Gate before N is A No consec dep. from same
N—–P/Q gate Gate before N is A
N—–P/Q
Now turn to the answer choices and see if
any of them are given in this diagram. Choice a is This also means that the gate for the third
incorrect because according to the diagram, the flight must be A or B:
fourth flight can depart from gate A. Choice b is
incorrect because according to the diagram, the 1 2 3 4 5 6
fourth flight can also depart from gate C. Choice PQN N N M P N L, M, N, O, P, Q
c is incorrect because according to the diagram
A C A/B A/B/C
the fifth flight can depart from any gate
(depending on what gates are assigned to the No consec dep. from same
fourth and sixth flights). Choice d is incorrect for gate Gate before N is A
the same reason: the sixth flight can depart from
any gate, depending on what gate is assigned to Now check the answer choices to see which
the fifth flight. could be true (the incorrect answer choices), and
Choice e is correct because according the which can’t be true (the correct answer choice).
diagram, the second flight in fact departs from While you may not be certain about the choices
Gate C. that could be true, you will be certain about the
choices that can’t be true. Choice a is incorrect
Question 5 because O could depart before M and N. Choice b
The correct answer is choice d. See what new is incorrect because P could depart after Q. Choice
deduc- tions can be made from the new c is incorrect because L could depart after M.
information that the second departure is from Choice d is correct because it can’t be the
Gate C:
255
practice set 2

case that both L and O depart before N, since


there is only one space before N. The diagram
makes clear that d cannot be the case, and so d is
the correct an- swer. Choice e is incorrect because,
as with choice b, it seems that the diagram makes
no restrictions on Q departing before or after P.

256
additioNal
oNliNe
practice

w hether you need help building basic skills or preparing for an exam, visit the
LearningExpress Practice Center! On this site, you can access additional practice materials.
Using the code below, you’ll be able to log in and complete additional LSAT Logic Game
practice. This online practice
will also provide you with:

■■ Immediate scoring
■■ Detailed answer explanations
■■ Personalized recommendations for further practice and study

Log in to the LearningExpress Practice Center by using this URL: www.learnatest.com/practice

This is your Access Code: 7793

Follow the steps online to redeem your access code. After you’ve used your access code to register with
the site, you will be prompted to create a username and password. For easy reference, record them
here:

Username: Password:

With your username and password, you can log in and answer these practice questions as many times as
you like.If you have any questions or problems, please contact LearningExpress customer service at 1-
800-295-9556 ext. 2, or e-mail us at [email protected].
257
Notes
Notes
Notes

You might also like